You are on page 1of 320

Chapter 7 Techniques of Integration

7.1 Integration by Parts


Concepts and Vocabulary
1. True.
R
2. uv − v du

Skill Building
3. We use integration by parts with u = x and dv = e2x dx. Then du = dx and v = 21 e2x . We
obtain
  Z
1 2x 1 2x
Z
2x
xe dx = x e − e dx
2 2
1 2x 1
Z
= xe − e2x dx
2 2
 
1 2x 1 1 2x
= xe − e +C
2 2 2
1
= 2 xe
2x
− 41 e2x + C .

4. We use integration by parts with u = x and dv = e−3x dx. Then du = dx and v = − 31 e−3x .
We obtain
  Z  
1 1
Z
xe−3x dx = x − e−3x − − e−3x dx
3 3
1 −3x 1
Z
= − xe + e−3x dx
3 3
 
1 −3x 1 1 −3x
= − xe + − e +C
3 3 3
= − 13 xe−3x − 19 e−3x + C .

5. We use integration by parts with u = x and dv = cos x dx. Then du = dx and v = sin x.
We obtain
Z Z
x cos x dx = x sin x − sin x dx

= x sin x − (− cos x) + C
= x sin x + cos x + C .

7-1
7-2 Chapter 7 Techniques of Integration

6. We use integration by parts with u = x and dv = sin (3x) dx. Then du = dx and v =
1 1
3 (− cos (3x)) = − 3 cos (3x). We obtain
  Z  
1 1
Z
x sin (3x) dx = x − cos (3x) − − cos (3x) dx
3 3
1 1
Z
= − x cos (3x) + cos (3x) dx
3 3
 
1 1 1
= − x cos (3x) + sin 3x + C
3 3 3
= − 31 x cos (3x) + 1
9 sin (3x) + C .
√ 1
7. We use integration by parts with u = ln x and dv = x dx = x1/2 dx. Then du = x dx and
v = 23 x3/2 . We obtain

  Z   
2 3/2 2 3/2 1
Z
x ln x dx = (ln x) x − x dx
3 3 x
2 3/2 2
Z
= x ln x − x1/2 dx
3 3
 
2 3/2 2 2 3/2
= x ln x − x +C
3 3 3
2 3/2
= 3x ln x − 49 x3/2 + C .

8. We use integration by parts with u = ln x and dv = x−2 dx. Then du = x1 dx and v = −x−1 .
We obtain
 
 1
Z Z
x−2 ln x dx = (ln x) −x−1 − −x−1

dx
x
ln x
Z
= − + x−2 dx
x
= − lnxx − 1
x +C .
 
1
9. We use integration by parts with u = cot−1 x and dv = dx. Then du = − 1+x 2 dx and
v = x. We obtain
Z  
1
Z
cot−1 x dx = cot−1 x (x) −

− (x) dx
1 + x2
x
Z
= x cot−1 x + dx.
1 + x2
Let u = 1 + x2 , then du = 2x dx, so x dx = du
2 . We now substitute and obtain
1 du
Z Z
cot−1 x dx = x cot−1 x +
u 2
1 1
Z
−1
= x cot x + du
2 u
1
= x cot−1 x + ln |u| + C
2
1
= x cot−1 x + ln 1 + x2 + C

2
= x cot x + 21 ln 1 + x2 + C .
−1

7.1 Integration by Parts 7-3

10. We use integration by parts with u = sin−1 x and dv = dx. Then du = √1−x 1
2
dx and
v = x. We obtain
Z  
1
Z
−1 −1

sin x dx = sin x (x) − √ (x) dx
1 − x2
x
Z
= x sin−1 x − √ dx.
1 − x2

Let u = 1 − x2 , then du = (−2x) dx, so x dx = − du



2 . We now substitute and obtain
 
1 du
Z Z
−1 −1
sin x dx = x sin x − √ −
u 2
1
Z
= x sin−1 x + u−1/2 du
2
1  1/2 
= x sin−1 x + 2u +C
2

= x sin−1 x + 1 − x2 + C .

11. We use integration by parts with u = (ln x)2 and dv = dx. Then du = 2 ln x
x dx and v = x.
We obtain
Z  
2 ln x
Z
2 2
(ln x) dx = (ln x) x − x dx
x
Z
2
= x(ln x) − 2 ln x dx

1
We use integration by parts again with u = ln x and dv = dx. Then du = x dx and v = x.
We obtain
 Z   
1
Z
2 2
(ln x) dx = x(ln x) − 2 (ln x)x − x dx
x
 Z 
= x(ln x)2 − 2 (ln x)x − 1 dx
2
= x(ln x) − 2 [(ln x)x − x] + C
2
= x(ln x) − 2x ln x + 2x + C .

2 2 ln x
12. We use integration by parts with u = (ln x) and dv = x dx. Then du = x dx and
v = 12 x2 . We obtain
  Z   
1 2 1 2 2 ln x
Z
x(ln x)2 dx = (ln x)2 x − x dx
2 2 x
1 2
Z
= x (ln x)2 − x ln x dx
2
1
We use integration by parts again with u = ln x and dv = x dx. Then du = x dx and
v = 12 x2 . We obtain
7-4 Chapter 7 Techniques of Integration

   Z    
1 2 1 2 1 2 1
Z
2
x(ln x) dx = x (ln x)2 − (ln x) x − x dx
2 2 2 x
1 2 1 2 1
Z
= x (ln x)2 − x (ln x) + x dx
2 2 2
 
1 2 1 2 1 1 2
= x (ln x)2 − x (ln x) + x +C
2 2 2 2
1 2 2
= 2 x (ln x) − 21 x2 (ln x) + 14 x2 + C .

13. We use integration by parts with u = x2 and dv = sin x dx. Then du = 2x dx and
v = − cos x. We obtain
Z Z
x2 sin x dx = x2 (− cos x) − (− cos x)(2x) dx
Z
2
= −x cos x + 2 x cos x dx

We use integration by parts again with u = x and dv = cos x dx. Then du = dx and
v = sin x. We obtain
Z  Z 
2 2
x sin x dx = −x cos x + 2 x sin x − sin x dx
Z
2
= −x cos x + 2x sin x − 2 sin x dx

= −x2 cos x + 2x sin x − 2(− cos x) + C


= −x2 cos x + 2x sin x + 2 cos x + C .

14. We use integration by parts with u = x2 and dv = cos x dx. Then du = 2x dx and v = sin x.
We obtain
Z Z
2 2
x cos x dx = x (sin x) − (sin x)(2x) dx
Z
= x2 sin x − 2 x sin x dx

We use integration by parts again with u = x and dv = sin x dx. Then du = dx and
v = − cos x. We obtain
Z  Z 
2 2
x cos x dx = x sin x − 2 x(− cos x) − (− cos x) dx
Z
2
= x sin x + 2x cos x − 2 cos x dx

= x2 sin x + 2x cos x − 2 sin x + C .


7.1 Integration by Parts 7-5

15. We use integration by parts with u = x cos x and dv = cos x dx. Then du = (−x sin x + cos x) dx
and v = sin x. We obtain
Z Z
x cos2 x dx = (x cos x)(sin x) − (sin x)(−x sin x + cos x) dx
Z Z
= x cos x sin x + x sin2 x dx − cos x sin x dx
Z Z
2

= x cos x sin x + x 1 − cos x dx − cos x sin x dx
Z Z Z
= x cos x sin x + x dx − x cos2 x dx − cos x sin x dx.

We add x cos2 x dx to each side, and then let u = sin x, so du = cos x dx in the remaining
R
integral. Upon substitution, we obtain
1
Z Z
2 x cos2 x dx = x cos x sin x + x2 − u du
2
1 1
= x cos x sin x + x2 − u2 + C
2 2
1 1
= x cos x sin x + x2 − sin2 x + C.
2 2
Finally, we divide by 2 to obtain
Z
x cos2 x dx = 1
2 x cos x sin x + 14 x2 − 1
4 sin2 x + C .

16. We use integration by parts with u = x sin x and dv = sin x dx. Then du = (x cos x + sin x) dx
and v = − cos x. We obtain
Z Z
x sin2 x dx = (x sin x)(− cos x) − (− cos x)(x cos x + sin x) dx
Z Z
2
= −x cos x sin x + x cos x dx + cos x sin x dx
Z Z
= −x cos x sin x + x 1 − sin2 x dx + cos x sin x dx


Z Z Z
2
= −x cos x sin x + x dx − x sin x dx + cos x sin x dx.

We add x sin2 x dx to each side, and then let u = sin x, so du = cos x dx in the remaining
R
integral. Upon substitution, we obtain
1
Z Z
2 x sin2 x dx = −x cos x sin x + x2 + u du
2
1 2 1 2
= −x cos x sin x + x + u
2 2
1 2 1
= −x cos x sin x + x + sin2 x.
2 2
Finally, we divide by 2, add the constant of integration, and obtain
Z
x sin2 x dx = − 12 x cos x sin x + 41 x2 + 41 sin2 x + C .
7-6 Chapter 7 Techniques of Integration

17. We use integration by parts with u = x and dv = sinh x dx. Then du = dx and v = cosh x.
We obtain
Z Z
x sinh x dx = x cosh x − cosh x dx

= x cosh x − sinh x + C .

18. We use integration by parts with u = x and dv = cosh x dx. Then du = dx and v = sinh x.
We obtain
Z Z
x cosh x dx = x sinh x − sinh x dx

= x sinh x − cosh x + C .

19. We use integration by parts with u = cosh−1 x and dv = dx. Then du = √ 1


x2 −1
dx and
v = x. We obtain
Z  
1
Z
cosh−1 x dx = cosh−1 x x − x √

dx.
x2 − 1
Let u = x2 − 1, then du = 2x dx, so x dx = du
2 . We substitute and obtain

du
Z Z
−1 −1
cosh x dx = x cosh x − u−1/2
2
1
Z
= x cosh−1 x − u−1/2 du
2
1 √ 
= x cosh−1 x − 2 u + C
2
−1

= x cosh x − x2 − 1 + C .

20. We use integration by parts with u = sinh−1 x and dv = dx. Then du = √ 1


x2 +1
dx and
v = x. We obtain
Z  
1
Z
sinh−1 x dx = sinh−1 x x − x √

dx.
x2 + 1
Let u = x2 + 1, then du = 2x dx, so x dx = du
2 . We substitute and obtain

du
Z Z
sinh−1 x dx = x sinh−1 x − u−1/2
2
1
Z
= x sinh−1 x − u−1/2 du
2
1 √ 
= x sinh−1 x − 2 u + C
2
−1

= x sinh x − x2 + 1 + C .

cos (ln x)
21. We use integration by parts with u = sin (ln x) and dv = dx. Then du = x dx and
v = x. We obtain
Z  
cos (ln x)
Z
sin (ln x) dx = (sin (ln x))x − x dx
x
Z
= x sin (ln x) − cos (ln x) dx.
7.1 Integration by Parts 7-7

We use integration by parts again with u = cos (ln x) and dv = dx. Then du = − sinx(ln x) dx
and v = x. We obtain
 Z   
− sin (ln x)
Z
sin (ln x) dx = x sin (ln x) − (cos (ln x))x − x dx
x
Z
= x sin (ln x) − x cos (ln x) − sin (ln x) dx.
R
We add sin (ln x) dx to each side, divide by 2, and add the constant of integration to
obtain
Z
2 sin (ln x) dx = x sin (ln x) − x cos (ln x) + C
1 1
Z
sin (ln x) dx = x sin (ln x) − x cos (ln x) + C = x2 [sin (ln x) − cos (ln x)] + C .
2 2

22. We use integration by parts with u = cos (ln x) and dv = dx. Then du = − sinx(ln x) dx and
v = x. We obtain
Z  
− sin (ln x)
Z
cos (ln x) dx = (cos (ln x))x − x dx
x
Z
= x cos (ln x) + sin (ln x) dx.

We use integration by parts again with u = sin (ln x) and dv = dx. Then du = cos (ln
x
x)
dx
and v = x. We obtain
 Z   
cos (ln x)
Z
cos (ln x) dx = x cos (ln x) + (sin (ln x))x − x dx
x
Z
= x cos (ln x) + x sin (ln x) − cos (ln x) dx.
R
We add cos (ln x) dx to each side, divide by 2, and add the constant of integration to
obtain
Z
2 cos (ln x) dx = x cos (ln x) + x sin (ln x) + C
Z
1
cos (ln x) dx = 2 x cos (ln x) + 12 x sin (ln x) + C .

2
3
23. We use integration by parts with u = (ln x) and dv = dx. Then du = 3 (lnxx) dx and
v = x. We obtain
!
2
3(ln x)
Z Z
3 3
(ln x) dx = (ln x) x − x dx
x
Z
3 2
= x(ln x) − 3 (ln x) dx.

We use integration by parts again with u = (ln x)2 and dv = dx. Then du = 2 lnxx dx and
v = x. We obtain
 Z   
ln x
Z
3 3 2
(ln x) dx = x(ln x) − 3 (ln x) (x) − x 2 dx
x
Z
3 2
= x(ln x) − 3x(ln x) + 6 ln x dx.
7-8 Chapter 7 Techniques of Integration

We use integration by parts again with u = ln x and dv = dx. Then du = x1 dx and v = x.


We obtain
 Z   
1
Z
3 3 2
(ln x) dx = x(ln x) − 3x(ln x) + 6 (ln x)x − x dx
x
Z
= x(ln x)3 − 3x(ln x)2 + 6x ln x − 6 dx

= x(ln x)3 − 3x(ln x)2 + 6x ln x − 6x + C .

3
4
24. We use integration by parts with u = (ln x) and dv = dx. Then du = 4 (lnxx) dx and
v = x. We obtain
!
3
4(ln x)
Z Z
4 4
(ln x) dx = (ln x) x − x dx
x
Z
4 3
= x(ln x) − 4 (ln x) dx.

2
3
We use integration by parts again with u = (ln x) and dv = dx. Then du = 3 (lnxx) dx
and v = x. We obtain
" ! #
2
3(ln x)
Z Z
4 4 3
(ln x) dx = x(ln x) − 4 (ln x) x − x dx
x
Z
= x(ln x) − 4x(ln x) + 12 (ln x)2 dx.
4 3

We use integration by parts again with u = (ln x)2 and dv = dx. Then du = 2 lnxx dx and
v = x. We obtain
 Z   
ln x
Z
4 4 3 2
(ln x) dx = x(ln x) − 4x(ln x) + 12 (ln x) (x) − x 2 dx
x
Z
4 3 2
= x(ln x) − 4x(ln x) + 12x(ln x) − 24 ln x dx.

We use integration by parts again with u = ln x and dv = dx. Then du = x1 dx and v = x.


We obtain
 Z   
1
Z
4 4 3 2
(ln x) dx = x(ln x) − 4x(ln x) + 12x(ln x) − 24 (ln x)x − x dx
x
Z
4 3 2
= x(ln x) − 4x(ln x) + 12x(ln x) − 24x ln x + 24 dx

4 3 2
= x(ln x) − 4x(ln x) + 12x(ln x) − 24x ln x + 24x + C .

2
25. We use integration by parts with u = (ln x) and dv = x2 dx. Then du = 2 lnxx dx and
v = 31 x3 . We obtain
  Z   
2 1 3 1 3 ln x
Z
2
x2 (ln x) dx = (ln x) x − x 2 dx
3 3 x
1 3 2
Z
= x (ln x)2 − x2 ln x dx.
3 3
7.1 Integration by Parts 7-9

dx
We use integration by parts again with u = ln x and dv = x2 dx. Then du = x and
v = 13 x3 . We obtain
   Z   
1 3 2 1 1 3 dx
Z
2 2
x2 (ln x) dx = x (ln x) − (ln x) x3 − x
3 3 3 3 x
1 3 2 2
Z
2
= x (ln x) − x3 ln x + x2 dx
3 9 9
 
1 3 2 2 3 2 1 3
= x (ln x) − x ln x + x +C
3 9 9 3
1 3 2
= 3 x (ln x) − 29 x3 ln x + 2 3
27 x +C .

26. We use integration by parts with u = (ln x)2 and dv = x3 dx. Then du = 2 lnxx dx and
v = 14 x4 . We obtain
  Z   
2 1 4 1 4 ln x
Z
3 2
x (ln x) dx = (ln x) x − x 2 dx
4 4 x
1 4 1
Z
2
= x (ln x) − x3 ln x dx.
4 2
dx
We use integration by parts again with u = ln x and dv = x3 dx. Then du = x and
v = 14 x4 . We obtain
   Z   
1 4 1 1 4 1 4 dx
Z
3 2 2
x (ln x) dx = x (ln x) − (ln x) x − x
4 2 4 4 x
1 4 1 1
Z
= x (ln x)2 − x4 ln x + x3 dx
4 8 8
 
1 4 1 1 1 4
= x (ln x)2 − x4 ln x + x +C
4 8 8 4
1 4 2
= 4 x (ln x) − 18 x4 ln x + 1 4
32 x +C .

1
27. We use integration by parts with u = tan−1 x and dv = x2 dx. Then du = 1+x 2 dx and
1 3
v = 3 x . We obtain
  Z   
 1 3 1 3 1
Z
2 −1 −1
x tan x dx = tan x x − x dx
3 3 1 + x2
1 3 1 x3
Z
= x tan−1 x − dx.
3 3 1 + x2
Let u = 1 + x2 , then du = 2x dx, so x dx = du 2 3
2 . Since x = u − 1, we have x dx =
u−1
2 du.
We substitute and obtain
Z u−1
1 3 1
Z
x2 tan−1 x dx = x tan−1 x − 2
du
3 3 u
Z  
1 3 −1 1 1
= x tan x − 1− du
3 6 u
1 3 1
= x tan−1 x − (u − ln |u|) + C
3 6
1 3 −1 1
1 + x2 − ln 1 + x2 + C
 
= x tan x −
3 6
= 3 x tan x − 61 x2 + 61 ln 1 + x2 + C ,
1 3 −1


where we have absorbed any constants with the constant C of integration.


7-10 Chapter 7 Techniques of Integration

1
28. We use integration by parts with u = tan−1 x and dv = x dx. Then du = 1+x2 dx and
v = 21 x2 . We obtain
  Z   
 1 2 1 2 1
Z
x tan−1 x dx = tan−1 x x − x dx
2 2 1 + x2
1 2 1 x2
Z
= x tan−1 x − dx
2 2 1 + x2
1 + x2 − 1

1 2 1
Z
= x tan−1 x − dx
2 2 1 + x2
Z  
1 2 1 1
= x tan−1 x − 1− dx
2 2 1 + x2
1 2 1
x tan−1 x − x − tan−1 x + C

=
2 2
1 2
= 2x tan−1 x − 21 x + 1
2 tan−1 x + C .

7x
29. We use integration by parts with u = x and dv = 7x dx. Then du = dx and v = ln 7 . We
obtain
 x  Z x
7 7
Z
7x x dx = (x) − dx
ln 7 ln 7
7x x 1
Z
= − 7x dx
ln 7 ln 7
7x x
 x 
1 7
= − +C
ln 7 ln 7 ln 7
7x x 7x
= ln 7 − (ln 7)2
+C .

−x
30. We use integration by parts with u = x and dv = 2−x dx. Then du = dx and v = − 2ln 2 .
We obtain
 −x  Z  −x 
2 2
Z
2−x x dx = (x) − − − dx
ln 2 ln 2
2−x x 1
Z
= − + 2−x dx
ln 2 ln 2
2−x x
 −x 
1 2
= − + − +C
ln 2 ln 2 ln 2
−x
2−x
= − 2ln 2x − (ln 2)2
+C .

e−x cos (2x) dx using integration by parts.


R
31. Evaluate
Let u = e−x and dv = cos (2x) dx.
1
Then du = −e−x dx and v = cos (2x) dx =
R
2 sin (2x).
Now
  Z  
1 1
Z
−x −x
sin (2x) −e−x dx

e cos (2x) dx = e sin (2x) −
2 2
1 −x 1
Z
= e sin (2x) + e−x sin (2x) dx.
2 2
7.1 Integration by Parts 7-11

e−x sin (2x) dx using integration by parts again.


R
Integrate
Let u = e−x and dv = sin (2x) dx.
Then du = −e−x dx and v = sin (2x) dx = − 12 cos (2x).
R

Now
  Z  
1 1
Z
−x −x
− cos (2x) −e−x dx

e sin (2x) dx = e − cos (2x) −
2 2
1 −x 1
Z
= − e cos (2x) − e−x cos (2x) dx
2 2
and
 
1 −x 1 1 −x 1
Z Z
−x −x
e cos (2x) dx = e sin (2x) + − e cos (2x) − e cos (2x) dx .
2 2 2 2
Simplifying, we obtain
1 1 1
Z Z
e−x cos (2x) dx = e−x sin (2x) − e−x cos (2x) − e−x cos (2x) dx.
2 4 4
Add 14 e−x cos (2x) dx to both sides of the equation to obtain
R

1 1 1
Z Z
e−x cos (2x) dx + e−x cos (2x) dx = e−x sin (2x) − e−x cos (2x)
4 2 4
5 1
Z
e−x cos (2x) dx = 2e−x sin (2x) − e−x cos (2x)

4 4
1
Z
e−x cos (2x) dx = 2e−x sin (2x) − e−x cos (2x)

5
1 −x
= 5e [2 sin (2x) − cos (2x)] + C

e−2x sin (3x) dx using integration by parts.


R
32. Evaluate
Let u = e−2x and dv = sin (3x) dx.
Then du = −2e−2x dx and v = sin (3x) dx = − 13 cos (3x).
R

Now
  Z  
1 1
Z
e−2x sin (3x) dx = e−2x − cos (3x) − − cos (3x) −2e−2x dx

3 3
1 −2x 2
Z
=− e cos (3x) − e−2x cos (3x) dx.
3 3
e−2x cos (3x) dx using integration by parts again.
R
Integrate
Let u = e−2x and dv = cos (3x) dx.
1
Then du = −2e−2x dx and v = cos (3x) dx =
R
3 sin (3x).
Now
  Z  
1 1
Z
−2x −2x
sin (3x) −2e−2x dx

e cos (3x) dx = e sin (3x) −
3 3
1 2
Z
= e−2x sin (3x) + e−2x sin (3x) dx
3 3
7-12 Chapter 7 Techniques of Integration

and
 
1 2 1 −2x 2
Z Z
e−2x sin (3x) dx = − e−2x cos (3x) − e sin (3x) + e−2x sin (3x) dx .
3 3 3 3
Simplifying, we obtain
1 −2x 2 −2x 4
Z Z
−2x
e sin (3x) dx = − e cos (3x) − e sin (3x) − e−2x sin (3x) dx.
3 9 9
Add 49 e−2x sin (3x) dx to both sides of the equation to obtain
R

4 1 2
Z Z
e−2x sin (3x) dx + e−2x sin (3x) dx = − e−2x cos (3x) − e−2x sin (3x)
9 3 9
13 1
Z
e−2x sin (3x) dx = − 3e−2x cos (3x) + 2e−2x sin (3x)

9 9
1  −2x
Z
−2x
cos (3x) + 2e−2x sin (3x)

e sin (3x) dx = − 3e
13
1 −2x
= − 13 e [3 cos (3x) + 2 sin (3x)] + C

e2x sin x dx using integration by parts.


R
33. Evaluate
Let u = e2x and dv = sin x dx.
Then du = 2e2x dx and v = sin x dx = −cos x.
R

Now
Z Z Z
2x 2x 2x
dx = −e2x cos x + 2 e2x cos x dx.

e sin x dx = e (−cos x) − (−cos x) 2e

e2x cos x dx using integration by parts again.


R
Integrate
Let u = e2x and dv = cos x dx.
Then du = 2e2x dx and v = cos x dx = sin x.
R

Now
Z Z
e cos x dx = e sin x − sin x 2e2x dx
2x 2x


Z
= e2x sin x − 2 e2x sin x dx

and Z  Z 
e2x sin x dx = −e2x cos x + 2 e2x sin x − 2 e2x sin x dx .

Simplifying, we obtain
Z Z
e2x sin x dx = −e2x cos x + 2e2x sin x − 4 e2x sin x dx.

Add 4 e2x sin x dx to both sides of the equation to obtain


R
Z Z
e sin x dx + 4 e2x sin x dx = −e2x cos x + 2e2x sin x
2x

Z
5 e2x sin x dx = −e2x cos x + 2e2x sin x
1
Z
e2x sin x dx = −e2x cos x + 2e2x sin x

5
1 2x
= 5 e (−cos x + 2 sin x) + C
7.1 Integration by Parts 7-13

e3x cos (5x) dx using integration by parts.


R
34. Evaluate
Let u = e3x and dv = cos (5x) dx.
Then du = 3e3x dx and v = cos (5x) dx = 15 sin (5x).
R

Now
  Z  
1 1
Z
e3x cos (5x) dx = e3x sin (5x) − sin (5x) 3e3x dx

5 5
1 3x 3
Z
= e sin (5x) − e3x sin (5x) dx.
5 5
Integrate e3x sin (5x) dx using integration by parts again.
R

Let u = e3x and dv = sin (5x) dx.


Then du = 3e3x dx and v = sin (5x) dx = − 51 cos (5x).
R

Now
  Z  
1 1
Z
e3x sin (5x) dx = e3x − cos (5x) − − cos (5x) 3e3x dx

5 5
1 3x 3
Z
= − e cos (5x) + e3x cos (5x) dx
5 5
and
 
1 3x 3 1 3x 3
Z Z
3x 3x
e cos (5x) dx = e sin (5x) − − e cos (5x) + e cos (5x) dx .
5 5 5 5
Simplifying, we obtain
1 3x 3 3x 9
Z Z
3x
e cos (5x) dx = e sin (5x) + e cos (5x) − e3x cos (5x) dx.
5 25 25
9
R 3x
Add 25 e cos (5x) dx to both sides of the equation to obtain
9 1 3
Z Z
e3x cos (5x) dx + e3x cos (5x) dx = e3x sin (5x) + e3x cos (5x)
25 5 25
34 1  3x
Z
3x
5e sin (5x) + 3e3x cos (5x)

e cos (5x) dx =
25 25
1  3x
Z
e3x cos (5x) dx = 5e sin (5x) + 3e3x cos (5x)

34
1 3x
= 34 e [5 sin (5x) + 3 cos (5x)] + C

35. We use integration by parts with u = ex and dv = cos x dx. Then du = ex dx and v = sin x.
We obtain
Z π Z π
π
ex cos x dx = [ex sin x]0 − ex sin x dx
0 0
Z π
 π 0
ex sin x dx

= e sin π − e sin 0 −
0
Z π
π
= [e (0) − 1(0)] − ex sin x dx
0
Z π
= − ex sin x dx.
0
7-14 Chapter 7 Techniques of Integration

We use integration by parts again with u = ex and dv = sin x dx. Then du = ex dx and
v = − cos x. We obtain
Z π  Z π 
π
ex cos x dx = − [ex (− cos x)]0 − ex (− cos x) dx
0 0
 Z π 
= − e (− cos π) − e0 (− cos 0) +
 π
ex cos x dx

0
 Z π 
= − [eπ (1) − 1(−1)] + ex cos x dx
0
Z π
= −eπ − 1 − ex cos x dx.
0

We add 0
ex cos x dx to each side, and then divide by 2 to obtain
Z π
2 ex cos x dx = −eπ − 1
0
Z π
−eπ − 1 π
ex cos x dx = = − e 2+1 .
0 2

e−x sin x dx using integration by parts.


R
36. Evaluate
Let u = e−x and dv = sin x dx.
Then du = −e−x dx and v = sin x dx = −cos x.
R

Now
Z Z Z
e−x sin x dx = e−x (−cos x) − (−cos x) −e−x dx = −e−x cos x − e−x cos x dx.


e−x cos x dx using integration by parts again.


R
Integrate
Let u = e−x and dv = cos x dx.
Then du = −e−x dx and v = cos x dx = sin x.
R

Now
Z Z
e−x cos x dx = e−x (sin x) − (sin x) −e−x dx


Z
= e−x sin x + e−x sin x dx

and Z  Z 
−x −x −x −x
e sin x dx = −e cos x − e sin x + e sin x dx .

Simplifying, we obtain
Z Z
e−x sin x dx = −e−x cos x − e−x sin x − e−x sin x dx.

e−x sin x dx to both sides of the equation to obtain


R
Add
Z Z
e−x sin x dx + e−x sin x dx = −e−x cos x − e−x sin x
Z
2 e−x sin x dx = −e−x (cos x + sin x)
1
Z
e−x sin x dx = − e−x (cos x + sin x)
2
7.1 Integration by Parts 7-15

So,
π/2
1  −x
Z
π/2
e−x sin x dx = − e (cos x + sin x) 0
0 2
1h i
= − e−π/2 (0 + 1) − e0 (1 + 0)
2
1
= 2 1 − e−π/2


37. We use integration by parts with u = x2 and dv = e−3x dx. Then du = 2x dx and
v = − 31 e−3x . We obtain
2   2 Z 2  
1 −3x 1 −3x
Z
2 −3x 2
x e dx = x − e − − e (2x) dx
0 3 0 0 3
2 2 −3x
    
1 1
Z
= 22 − e−3(2) − 02 − e−3(0) + xe dx
3 3 3 0
4 2 2 −3x
Z
= − e−6 + xe dx.
3 3 0

We use integration by parts again with u = x and dv = e−3x dx. Then du = dx and
v = − 31 e−3x . We obtain
Z 2   2 Z 2   !
2 −3x 4 −6 2 1 −3x 1 −3x
x e dx = − e + x − e − − e dx
0 3 3 3 0 0 3
1 2 −3x
     
4 2 1 1
Z
= − e−6 + 2 − e−3(2) − 0 − e−3(0) + e dx
3 3 3 3 3 0
   2 !
4 2 2 1 1
= − e−6 + − e−6 + − e−3x
3 3 3 3 3 0
    
4 −6 2 2 −6 1 1 −3(2) 1 −3(0)
= − e + − e + − e − − e
3 3 3 3 3 3
   
4 2 2 1 1 1 −6
= − e−6 + − e−6 + − e
3 3 3 3 3 3
2 50 −6
= 27 − 27 e .

38. We use integration by parts with u = x2 and dv = e−x dx. Then du = 2x dx and v = −e−x .
We obtain
Z 1 Z 1
1
x2 e−x dx = x2 −e−x 0 − −e−x (2x) dx
 
0 0
Z 1
12 −e −1
− 02 −e−0 xe−x dx
  
= +2
0
Z 1
−1 −x
= −e +2 xe dx.
0

We use integration by parts again with u = x and dv = e−x dx. Then du = dx and
v = −e−x . We obtain
7-16 Chapter 7 Techniques of Integration

Z 1 
1
Z 1 
x2 e−x dx −e−1 + 2 x −e−x −e−x dx
 
= 0

0 0
 Z 1 
−e−1 + 2 − 0 −e−0 + −1
e−x dx
  
= 1 −e
0
  −x 1 
−1 −1
= −e + 2 −e + −e 0

−e−1 + 2 −e−1 + −e−1 − −e−0


 
=
= 2 − 5e−1 .

R
39. Evaluate x sec x tan x dx using integration by parts.
Let u = x and dv = sec x tan x dx.
R
Then du = dx and v = sec x tan x dx = sec x.
Now Z Z
x sec x tan x dx = x sec x − sec x dx.
R
To integrate sec x dx, multiply sec x by sec x + tan x:
sec x + tan x sec2 x + sec x tan x
Z Z Z
sec x dx = sec x · dx = dx.
sec x + tan x sec x + tan x
Integrate using u-substitution:
Let u = sec x + tan x. Then du = sec x tan x + sec2 x dx.


Now
1 1
Z Z Z
· sec2 x + sec x tan x dx =

sec x dx = du = ln |u| = ln |sec x + tan x| .
sec x + tan x u
(This also could have been found using a Table of Integrals.)
So Z
x sec x tan x = x sec x − ln |sec x + tan x| + C.
Then
Z π/4 π π π π 
x sec x tan x dx = sec
− ln sec + tan − (0 · sec 0 − ln |sec 0 + tan 0|)

0 4 4 4 4
π √ √ 
= · 2 − ln 2 + 1 − (0 · 1 − ln |1 + 0|)

4
√ √
= 42 π − ln 2 + 1


40. We use integration by parts with u = x and dv = tan2 x dx = sec2 x − 1 dx. Then

du = dx and v = tan x − x. We obtain
Z π/4 Z π/4
x tan2 x dx = [x(tan x − x)]π/4
0 − (tan x − x) dx
0 0
hπ  i  π/4
π π 1 2
= tan − − 0(tan 0 − 0) − ln |sec x| − x
4 4 4 2
0
π2
 1  π 2 
π π 1
= − − ln sec − − ln |sec 0| − 02

4 16 4 2 4 2
2 √ 2
 
π π π
= − − ln 2 −
4 16 32
π π2 1
= 4 − 32 − 2 ln 2 .
7.1 Integration by Parts 7-17

R9 √ R9 R9
41. We rewrite 1 ln x dx = 1 ln x1/2 dx = 12 1 ln x dx. We use integration by parts with
u = ln x and dv = dx. Then du = x1 dx and v = x. We obtain
9 √ 9
1
Z Z
ln x dx = ln x dx
1 2 1
 Z 9   
1 9 1
= [(ln x)(x)]1 − x dx
2 1 x
 Z 9 
1
= [(ln 9)(9) − (ln 1)(1)] − dx
2 1
1
= (9 ln 9 − 8)
2
= 9 ln 3 − 4 .

42. We use integration by parts with u = x and dv = csc2 x dx. Then du = dx and v = − cot x.
We obtain
Z 3π/4 Z 3π/4
3π/4
x csc2 x dx = [x(− cot x)]π/4 − (− cot x) dx
π/4 π/4
    Z 3π/4
3π 3π π π
= − cot − − cot + cot x dx
4 4 4 4 π/4
 
3π π 3π/4
= + + [ln |sin x|]π/4
4 4
 
3π π
= π + ln sin − ln

4 4
sin
√ √ !
2 2
= π + ln − ln
2 2
= π.

43. We use integration by parts with u = (ln x)2 and dv = dx. Then du = 2 lnxx dx and v = x.
We obtain
Z e h ie Z e  ln x 
2 2
(ln x) dx = (ln x) (x) − x 2 dx
1 1 1 x
Z e
= (ln e)2 (e) − (ln 1)2 (1) − 2 ln x dx
1
Z e
= e−2 ln x dx.
1

1
We use integration by parts again with u = ln x and dv = dx. Then du = x dx and v = x.
We obtain
Z e  Z e 
2 e 1
(ln x) dx = e − 2 [(ln x) x]1 − x dx
1 1 x
 Z e 
= e − 2 (ln e) e − (ln 1) (1) − dx
1
= e − 2 (e − (e − 1))
= e−2 .
7-18 Chapter 7 Techniques of Integration

44. We use integration by parts with u = x and dv = sec2 x dx. Then du = dx and v = tan x.
We obtain
Z π/4 Z π/4
x sec2 x dx = [x(tan x)]π/4
0 − tan x dx
0 0
hπ π  i
π/4
= tan − 0(tan 0) − [ln |sec x|]0
4 h 4
π π i
= − ln sec − ln |sec 0|

4 4
π √
= − ln 2
4
π 1
= 4 − 2 ln 2 .

45. To determine where the graphs intersect, set 3 ln x = x ln x, and obtain x = 3. Since
ln x ≥ 0 when 1 ≤ x ≤ 3, we have x ln x ≤ 3 ln x on the interval 1 ≤ x ≤ 3 and the area
enclosed by the graphs of f and g is
Z 3 Z 3
(3 ln x − x ln x) dx = (3 − x) ln x dx.
1 1
1
We use integration by parts with u = ln x and dv = (3 − x) dx. Then du = x dx and
v = 3x − 12 x2 . We obtain
3 Z 3
3x − 21 x2
Z 3  
1 2
(3 − x) ln x dx = (ln x) 3x − x − dx
1 2 1 1 x
     Z 3  
1 1 1
= (ln 3) 3(3) − 32 − (ln 1) 3(1) − 12 − 3− x dx
2 2 1 2
 3
9 1 2
= ln 3 − 3x − x
2 4 1
   
9 1 2 1 2
= ln 3 − 3(3) − 3 − 3(1) − 1
2 4 4
9
= 2 ln 3 − 4 .

46. To determine where the graphs intersect, set 4x ln x = x2 ln x, and obtain x = 1 and x = 4.
Since ln x ≥ 0 when 1 ≤ x ≤ 4, we have x2 ln x ≤ 4x ln x on the interval 1 ≤ x ≤ 4, and
the area enclosed by the graphs of f and g is
Z 4 Z 4
4x ln x − x2 ln x dx = 4x − x2 ln x dx.
 
1 1

We use integration by parts with u = ln x and dv = 4x − x2 dx. Then du = x1 dx and




v = 2x2 − 31 x3 . We obtain
Z 4 4 Z 4 2 1 3
2x − 3 x
 
2
 2 1 3
4x − x ln x dx = (ln x) 2x − x − dx
1 3 1 1 x
     Z 4  
1 1 1
= (ln 4) 2(4)2 − 43 − (ln 1) 2(1)2 − 13 − 2x − x2 dx
3 3 1 3
 4
32 1
= ln 4 − x2 − x3
3 9 1
   
32 2 1 3 2 1 3
= ln 4 − 4 − 4 − 1 − 1
3 9 9
32
= 3 ln 4 − 8 .
7.1 Integration by Parts 7-19


47. The area under the graph of y = ex sin x from 0 to π is given by 0 ex sin x dx. We use
integration by parts with u = ex and dv = sin x dx. Then du = ex dx and v = − cos x. We
obtain
Z π Z π
x x π
e sin x dx = [e (− cos x)]0 − ex (− cos x) dx
0 0
Z π
= eπ (− cos π) − e0 (− cos 0) + ex cos x dx
 
0
Z π
π x
= e +1+ e cos x dx.
0

We use integration by parts again with u = ex and dv = cos x dx. Then du = ex dx and
v = sin x. We obtain
Z π Z π
π
ex sin x dx = eπ + 1 + [ex sin x]0 − ex sin x dx
0 0
Z π
π π 0
ex sin x dx

= e + 1 + e sin π − e sin 0 −
0
Z π
= eπ + 1 − ex sin x dx
0
Rπ x
We add 0
e sin x dx to each side, and then divide by 2 to obtain
Z π
2 ex sin x dx = eπ + 1
0
Z π
π
ex sin x dx = e 2+1 .
0
 R π/2
48. Since y = x cos x is nonnegative on 0, π2 , 0 x cos x dx is the area under the graph of

π
y = x cos x from x = 0 to x = 2 .

R
Evaluate x cos x dx using integration by parts.
Let u = x and dv = cos x dx.
R
Then du = dx and v = cos x dx = sin x.
Now
Z Z
x cos x dx = x sin x − sin x dx = x sin x − (−cos x) = x sin x + cos x + C.

So,
Z π/2 hπ i
π/2 π
x cos x dx = [x sin x + cos x]0 = (1) + 0 − [0 + 1] = 2 −1 .
0 2
7-20 Chapter 7 Techniques of Integration

R1
49. Since y = xe−x is nonnegative on [0, 1], 0 xe−x dx is the area under the graph of y = xe−x
from x = 0 to x = 1.

xe−x dx using integration by parts.


R
Evaluate
Let u = x and dv = e−x dx.
Then du = dx and v = e−x dx = −e−x .
R

Now Z Z
−x −x
−e−x dx = −xe−x − e−x = −e−x (x + 1).

xe dx = −xe −

So,
Z 1 1
xe−x dx = −e−x (x + 1) 0 = −2e−1 − (−1) = 1 − 2

e .
0
R2
50. Since y = xe3x is nonnegative on [0, 2], 0 xe3x dx is the area under the graph of y = xe3x
from x = 0 to x = 2.

xe3x dx using integration by parts.


R
Evaluate
Let u = x and dv = e3x dx.
Then du = dx and v = e3x dx = 31 e3x .
R

Now
  Z  
1 1 3x 1 1 1
Z
xe3x dx = x e3x − e dx = xe3x − e3x + C = e3x (3x − 1) + C.
3 3 3 9 9
7.1 Integration by Parts 7-21

So,
2
1  3x 1
Z
2
xe3x dx = e (3x − 1) 0 = e6 (6 − 1) − e0 (0 − 1) = 1
5e6 + 1 .
 
9 9 9
0

51. (a) The velocity of the object at time t is given by


Z t Z t
v(t) = v(0) + a(x) dx = 8 + e−2x sin x dx.
0 0

e−2x sin x dx using integration by parts.


R
Integrate
Let u = e−2x and dv = sin x dx.
Then du = −2e−2x dx and v = sin x dx = −cos x.
R

Now
Z Z
e−2x sin x dx = e−2x (−cos x) − (−cos x) −2e−2x dx


Z
= −e−2x cos x − 2 e−2x cos x dx.

e−2x cos x dx using integration by parts again.


R
Integrate
Let u = e−2x and dv = cos x dx.
Then du = −2e−2x dx and v = cos x dx = sin x.
R

Now
Z Z
−2x −2x
sin x − sin x −2e−2x dx

e cos x dx = e
Z
= e−2x sin x + 2 e−2x sin x dx

so Z  Z 
e−2x sin x dx = −e−2x cos x − 2 e−2x sin x + 2 e−2x sin x dx .

Simplifying, we obtain
Z Z
e−2x sin x dx = −e−2x cos x − 2e−2x sin x − 4 e−2x sin x dx.

e−2x sin x dx to both sides of the equation to obtain


R
Add 4
Z Z
e−2x sin x dx + 4 e−2x sin x dx = −e−2x cos x − 2e−2x sin x
Z
5 e−2x sin x dx = −e−2x cos x − 2e−2x sin x
1
Z
e−2x sin x dx = −e−2x cos x − 2e−2x sin x

5
1 −2x
= e (−cos x − 2 sin x) + C
5
7-22 Chapter 7 Techniques of Integration

Therefore,
Z t
v(t) = 8 + e−2x sin x dx
0
 t
1 −2x
=8+ e (−cos x − 2 sin x)
5 0
 
1 −2t 1
= 8 + e (−cos t − 2 sin t) − e0 (−cos 0 − 2 sin 0)
5 5
1  −2t 
= 8 + e (−cos t − 2 sin t) + 1
5
1  −2t 
= 8 − e (cos t + 2 sin t) − 1
5
41
= 5 − 15 [e−2t (cos t + 2 sin t)] .

(b) The position of the object at time t is given by


Z t Z t 
41 1  −2x 
s(t) = s(0) + v(x) dx = 0 + − e (cos x + 2 sin x) dx
0 0 5 5

Recall e−2x sin x dx = 15 e−2x (−cos x − 2 sin x) + C.


R

Similarly, e−2x cos x dx = 15 e−2x (sin x − 2 cos x) + C.


R

Z Z Z
e−2x (cos x + 2 sin x) dx = e−2x cos x dx + 2 e−2x sin x dx
 
1 −2x 1 −2x
= e (sin x − 2 cos x) + 2 e (−cos x − 2 sin x)
5 5
1
= e−2x (sin x − 2 cos x − 2 cos x − 4 sin x)
5
1
= − e−2x (3 sin x + 4 cos x)
5
So,
Z t 
41 1  −2x 
s(t) = − e (cos x + 2 sin x) dx
0 5 5
  t
41 1 1
= x − − e−2x (3 sin x + 4 cos x)
5 5 5
   0
41 1 1 −2t 4
= t − − e (3 sin t + 4 cos t) −
5 5 5 25
41 1 −2t 4
= 5 t − 25 e (3 sin t + 4 cos t) − 25 .

Rt
52. (a) The velocity of the object at time t is given by v(t) = v(0) + 0
a(x) dx = 5 +
R t 2 −x
0
x e dx.
Integrate x2 e−x dx using integration by parts.
R

Let u = x2 and dv = e−x dx.


Then du = 2x dx and v = e−x dx = −e−x.
R
7.1 Integration by Parts 7-23

Now
Z Z Z
x2 e−x dx = x2 −e−x − −e−x (2x dx) = −x2 e−x + 2 xe−x dx.
 

xe−x dx using integration by parts.


R
Integrate
Let u = x and dv = e−x dx.
Then du = dx and v = e−x dx = −e−x .
R

Now
Z Z Z
−x −x −x
(dx) = −xe + e−x dx = −xe−x − e−x + C.
−x
 
xe dx = x −e − −e

So,
Z
x2 e−x dx = −x2 e−x + 2 −xe−x − e−x = −x2 e−x − 2xe−x − 2e−x + C.


Therefore,
Z t
v(t) = 5 + x2 e−x dx
0
t
= 5 + −x2 e−x − 2xe−x − 2e−x 0


= 5 + −t2 e−t − 2te−t − 2e−t − (0 − 0 − 2)


  

= 7 − t2 e−t − 2te−t − 2e−t


= 7 − e−t (t2 + 2t + 2) .

(b) The position of the object at time t is given by


Z t Z t
7 − x2 e−x − 2xe−x − 2e−x dx

s(t) = s(0) + v(x) dx = 0 +
0 0

Using the previous results,


Z
7 − x2 e−x − 2xe−x − 2e−x dx


Z Z Z Z
= 7 dx − x e dx − 2 xe dx − 2 e−x dx
2 −x −x

= 7x − −x2 e−x − 2xe−x − 2e−x − 2 −xe−x − e−x − 2 −e−x + C


  

= 7x + x2 e−x + 4xe−x + 6e−x + C.

So,
Z t
7 − x2 e−x − 2xe−x − 2e−x dx

s(t) =
0
t
= 7x + x2 e−x + 4xe−x + 6e−x 0


= 7t + t2 e−t + 4te−t + 6e−t − (0 + 0 + 0 + 6)




= 7t + t2 e−t + 4te−t + 6e−t − 6


= e−t (t2 + 4t + 6) + 7t − 6 .
7-24 Chapter 7 Techniques of Integration

R π/2
53. The volume is given by the integral 0 2πx sin x dx. We use integration by parts with
u = 2πx and dv = sin x dx. Then du = 2π dx and v = − cos x. We obtain
Z π/2 Z π/2
π/2
2πx sin x dx = [(2πx)(− cos x)]0 − (− cos x)(2π) dx
0 0
 π  Z π/2
h π i
= 2π − cos − (2π(0))(− cos 0) + 2π cos x dx
2 2 0
Z π/2
= 2π cos x dx
0
π/2
= 2π [sin x]0
h π i
= 2π sin − sin 0
2
= 2π .

R π/2
54. The volume is given by the integral 0 2πx cos x dx. We use integration by parts with
u = 2πx and dv = cos x dx. Then du = 2π dx and v = sin x. We obtain
Z π/2 Z π/2
π/2
2πx cos x dx = [(2πx)(sin x)]0 − sin x(2π) dx
0 0
 π  Z π/2
h π i
= 2π sin − (2π(0))(sin 0) − 2π sin x dx
2 2 0
2 π/2
= π − 2π [− cos x]0
h π i
= π 2 − 2π − cos − (− cos 0)
2
= π 2 − 2π .

Re 2
55. The volume is given by the integral 1
π(ln x) dx. We use integration by parts with
2 2 ln x
u = (ln x) and dv = π dx. Then du = xdx and v = πx. We obtain
e ie Z e  
2 ln x
Z h
2 2
π(ln x) dx = (ln x) (πx) − (πx) dx
1 1 1 x
h i Z e
2 2
= (ln e) (πe) − (ln 1) (π1) − 2π ln x dx
1
Z e
= πe − 2π ln x dx.
1

We use integration by parts again with u = ln x and dv = dx. Then du = x1 dx and v = x.


We obtain
Z e  Z e   
2 e 1
π(ln x) dx = πe − 2π [(ln x)x]1 − x dx
1 1 x
 Z e 
= πe − 2π [(ln e)e − (ln 1)1] − dx
1
= πe − 2π(e − (e − 1))
= πe − 2π .
7.1 Integration by Parts 7-25

R π/2  √ 2 R π/2
56. The volume is given by the integral 0
π x sin x dx = 0 πx2 sin x dx. We use
integration by parts with u = πx2 and dv = sin x dx. Then du = 2πx dx and v = − cos x.
We obtain
Z π/2 Z π/2
π/2
πx2 sin x dx = πx2 (− cos x) 0 −
 
(− cos x)(2πx) dx
0 0
     Z π/2
π 2  π 
− π02 (− cos 0) +

= π − cos 2πx cos x dx
2 2 0
Z π/2
= 2πx cos x dx.
0

We use integration by parts again with u = 2πx and dv = cos x dx. Then du = 2π dx and
v = sin x. We obtain
Z π/2 Z π/2
2 π/2
πx sin x dx = [(2πx)(sin x)]0 − (sin x)(2π) dx
0 0
h  π   π  i Z π/2
= 2π sin − (2π(0))(sin 0) − 2π sin x dx
2 2 0
π/2
= π 2 − 2π [− cos x]0
h π i
= π 2 − 2π − cos − (− cos 0)
2
= π 2 − 2π .

57. Using the method of disks, the volume is given by


Z e2 Z e2  √ 2 Z e2
2
V =π [f (x)] dx = π x ln x dx = π x2 ln x dx.
1 1 1

x2 ln x dx using integration by parts.


R
Integrate

Let u = ln x and dv = x2 dx.


Then du = x1 dx and v = x2 dx = 13 x3 .
R

Now
 
1 3
1 3 1 1 1
Z Z Z
2
x ln x dx = ln x − x · dx = x3 ln x −
x x2 dx
3 3x 3 3
1 1 1
= x3 ln x − x3 + C = x3 (3 ln x − 1) + C.
3 9 9
The volume of the solid is
Z e2
V =π x2 ln x dx
1
1 e2
= π · x3 (3 ln x − 1) 1
9
π 6 
= 5e − (−1)
9
= π9 5e6 + 1 .

7-26 Chapter 7 Techniques of Integration

R2 R2 R2
58. Using the Shell Method, the volume is 2π 0 x · f (x) dx = 2π 0 x · xe3x dx = 2π 0 x2 e3x dx.

Integrate x2 e3x dx using integration by parts.


R

Let u = x2 and dv = e3x dx.


Then du = 2x dx and v = 13 e3x as before.
Now
1 1 3x 1 2
Z Z Z
x2 e3x dx = x2 · e3x − e · 2x dx = x2 e3x − xe3x dx.
3 3 3 3
xe3x dx using integration by parts again.
R
Integrate

Let u = x and dv = e3x dx.


Then du = dx and v = 31 e3x as before.
Now
1 1 3x 1 1 1 1 1
Z Z
3x
xe dx = x · e3x − e dx = xe3x − · e3x = xe3x − e3x .
3 3 3 3 3 3 9
So
 
1 2 3x 2 1 3x 1 3x
Z
2 3x
x e dx = x e − xe − e +C
3 3 3 9
1 2 2
= x2 e3x − xe3x + e3x
3 9 27
1 3x
e 9x2 − 6x + 2 + C

=
27
Therefore
2  2
 
1 3x
Z
2π x3 e3x dx = 2π · e 9x2 − 6x + 2
0 27 0
2π  3·2
9 · 2 − 6 · 2 + 2 − e3·0 9 · 02 − 6 · 0 + 2
2
 
= e
27

26e6 − 2

=
27
= 4π 6

27 13e − 1

xf ′ (x) dx using integration by parts.


R
59. Integrate
Let u = x and dv = f ′ (x) dx.
Then du = dx and v = f ′ (x) dx = f (x).
R

Now Z Z Z
xf ′ (x) dx = u · v − v · du = x f (x) − f (x) dx.

So,
Z 5 Z 5
5
x f ′ (x) dx = [x f (x)]1 − f (x) dx = 5f (5) − f (1) − 10 = 5(−5) − 2 − 10 = −37 .
1 1
7.1 Integration by Parts 7-27

60. We use integration by parts, with u = x and dv = f ′ (x) dx. Then du = dx and v = f (x).
We obtain
Z 5 Z 5
xf ′ (x) dx = [xf (x)]53 − f (x) dx
3 3
= 5f (5) − 3f (3) − (18)
= 5(11) − 3(8) − 18
= 13 .
Re
61. (a) Since y = ln x is nonnegative on [1, e], A = 1 ln x dx is the area under the graph of
y = ln x from x = 1 to x = e.

R
Evaluate ln x dx using integration by parts.
Let u = ln x and dv = dx.R
Then du = x1 dx and v = dx = x.
Now
Z  
1
Z Z
ln x dx = (ln x)(x) − x dx = x ln x − dx = x ln x − x = x (ln x − 1) + C.
x
So, Z e
e
A= ln x dx = [x (ln x − 1)]1 = e(1 − 1) − 1(0 − 1) = 1 .
1

(b) Using the method of shells, the volume is given by


Z e Z e
V = 2π x f (x) dx = 2π x ln x dx.
1 1
R
Integrate x ln x dx using integration by parts.
Let u = ln x and dv = x dx.
Then du = x1 dx and v = x dx = 12 x2 .
R

Now
  Z   
1 1 2 1 1 1
Z Z
x ln x dx = ln x x2 − x dx = x2 ln x − x dx
2 2 x 2 2
1 1
= x2 ln x − x2 + C
2 4
1
= x2 (2 ln x − 1) + C.
4
7-28 Chapter 7 Techniques of Integration

The volume of the solid is


e
1 2
Z
e
V = 2π x2 ln x dx = 2π · x (2 ln x − 1) 1
1 4
π 2 
= e − (−1)
2
= π2 e2 + 1 .


f (x)f ′ (x) dx using integration by parts.


R
62. Integrate
Let u = f (x) and dv = f ′ (x) dx.
Then du = f ′ (x) dx and v = f ′ (x) dx = f (x).
R

Now Z Z Z

f (x)f (x) dx = u · v − v · du = f (x) f (x) − f (x) f ′ (x) dx.

f (x) f ′ (x) dx to both sides of the equation.


R
Add
Z
2 f (x)f ′ (x) dx = [f (x)]2 .

So,
b ob
1n
Z
f (x)f ′ (x) dx = [f (x)]2 = 1
[f (b)]2 − [f (a)]2 .

2 a 2
a
√ 1
63. Let w = x, then dw = √
2 x
dx, so dx = 2w dw. We substitute and obtain


Z Z
sin x dx = 2w sin w dw.

We use integration by parts, with u = 2w and dv = sin w dw. Then du = 2 dw and


v = − cos w. We obtain
Z Z
2w sin w dw = (2w)(− cos w) − (− cos w)(2) dw
Z
= −2w cos w + 2 cos w dw

= −2w cos w + 2(sin w) + C


√ √ √
= −2 x cos x + 2 sin x + C .
√ 1
64. Let w = x, then dw = √
2 x
dx, so dx = 2w dw. We substitute and obtain
Z √ Z
e dx = 2wew dw.
x

We use integration by parts, with u = 2w and dv = ew dw. Then du = 2 dw and v = ew .


We obtain
Z Z
2wew dw = (2w)(ew ) − (ew )(2) dw
Z
= 2wew − 2 ew dw

= 2wew − 2(ew ) + C
√ √ √
= 2 xe x − 2e x + C .
7.1 Integration by Parts 7-29

65. Let w = sin x, then dw = cos x dx. We substitute and obtain


Z Z
cos x ln (sin x) dx = ln w dw.

1
We use integration by parts, with u = ln w and dv = dw. Then du = w dw and v = w. We
obtain
 
1
Z Z
ln w dw = (ln w)(w) − (w) dw
w
Z
= w ln w − dw
= w ln w − w + C
= sin x ln (sin x) − sin x + C .

66. Let w = 2 + ex , then dw = ex dx. We substitute and obtain


Z Z
x x
e ln (2 + e ) dx = ln w dw.

1
We use integration by parts, with u = ln w and dv = dw. Then du = w dw and v = w. We
obtain
 
1
Z Z
ln w dw = (ln w)(w) − (w) dw
w
Z
= w ln w − dw
= w ln w − w + C
= (2 + ex ) ln (2 + ex ) − 2 − ex + C
= (2 + ex ) ln (2 + ex ) − ex + C ,

where we have absorbed any constants with the constant C of integration.


1
67. Let w = e2x , then dw = 2e2x dx, so e2x dx =
dw. We substitute and obtain
2
  Z
1 1
Z Z Z
e4x cos e2x dx = e2x cos e2x e2x dx = w cos w
 
dw = w cos w dw.
2 2

We use integration by parts, with u = 12 w and dv = cos w dw. Then du = 1


2 dw and
v = sin w. We obtain
   
1 1 1
Z Z
w cos w dw = w (sin w) − (sin w) dw
2 2 2
1 1
Z
= w sin w − sin w dw
2 2
1 1
= w sin w − (− cos w) + C
2 2
1 1
= w sin w + cos w + C
2 2
1 2x 2x
 1
+ 2 cos e2x + C .

= 2 e sin e
7-30 Chapter 7 Techniques of Integration

68. Let w = sin x, then dw = cos x dx. We substitute and obtain


Z Z
cos x tan−1 (sin x) dx = tan−1 w dw.

1
We use integration by parts with u = tan−1 w and dv = dw. Then du = 1+w 2 dw and
v = w. We obtain
 
1
Z Z
tan−1 w dw = tan−1 w (w) − (w)

dw
1 + w2
w
Z
= w tan−1 w − dw.
1 + w2

Let y = 1 + w2 , then dy = 2w dw, so w dw = 21 dy. We substitute and obtain


 
1 1
Z Z
−1 −1
tan w dw = w tan w − dy
y 2
1 1
Z
= w tan−1 w − dy
2 y
1
= w tan−1 w − ln |y| + C
2
1
= w tan−1 w − ln 1 + w2 + C

2
= sin x tan (sin x) − 21 ln 1 + sin2 x + C .
−1


2 2
69. We use integration by parts with u = x2 and dv = xex dx. Then du = 2x dx and v = 21 ex .
We obtain
  Z  
 1 x2 1 x2
Z
3 x2 2
x e dx = x e − e (2x) dx
2 2
1 2 x2
Z
2
= x e − xex dx
2
1 2 x2 2
= 2x e − 12 ex + C .

70. We use integration by parts with u = ln x and dv = xn dx. Then du = x1 dx and v =


1 n+1
n+1 x . We obtain
  Z   
1 1 1
Z
n n+1 n+1
x ln x dx = (ln x) x − x dx
n+1 n+1 x
1 1
Z
= xn+1 ln x − xn dx
n+1 n+1
 n+1 
1 1 x
= xn+1 ln x − +C
n+1 n+1 n+1
1 n+1 1
= n+1 x ln x − (n+1)2
xn+1 +C .

71. We use integration by parts with u = x cos x and dv = ex dx. Then du = (−x sin x + cos x) dx
and v = ex . We obtain
Z Z
xe cos x dx = (x cos x)e − (−x sin x + cos x)ex dx.
x x
7.1 Integration by Parts 7-31

We use integration by parts again with u = −x sin x + cos x and dv = ex dx. Then
du = (−2 sin x − x cos x) dx and v = ex . We obtain
Z  Z 
x x x x
xe cos x dx = (x cos x)e − (−x sin x + cos x)e − (−2 sin x − x cos x)e dx
Z Z
= x cos x e + x sin x e − cos x e − 2 sin x e dx − x cos x ex dx.
x x x x

xex cos x dx to each side and divide by 2 to obtain


R
We add
Z Z
2 xex cos x dx = x cos x ex + x sin x ex − cos x ex − 2 sin x ex dx
1
Z Z
xex cos x dx = (x cos x ex + x sin x ex − cos x ex ) − sin x ex dx.
2
We determine sin x ex dx using integration by parts. Let u = sin x and dv = ex dx. Then
R
du = cos x dx and v = ex . We obtain
Z Z
sin x e dx = (sin x)e − (cos x)ex dx.
x x

We integrate by parts again with u = cos x and dv = ex dx. Then du = − sin x dx and
v = ex . We obtain
Z  Z 
sin x ex dx = sin x ex − (cos x)ex − (− sin x)ex dx
Z
= sin x ex − cos x ex − sin xex dx.

(sin x)ex dx to each side and divide by 2 to obtain


R
We add
Z
2 sin x ex dx = sin x ex − cos x ex
1 1
Z
sin x ex dx = sin x ex − cos x ex + C.
2 2
We substitute into the above to obtain
 
1 1 1
Z
x x x x x x
xe cos x dx = (x cos x e + x sin x e − cos x e ) − sin x e − cos x e + C
2 2 2
1 1 1
= x cos x ex + x sin x ex − sin x ex + C
2 2 2
1 x
= 2 e (x sin x + x cos x − sin x ) + C .

72. We use integration by parts with u = x sin x and dv = ex dx. Then du = (x cos x + sin x) dx
and v = ex . We obtain
Z Z
xe sin x dx = (x sin x)e − (x cos x + sin x)ex dx.
x x

We use integration by parts again with u = x cos x + sin x and dv = ex dx. Then du =
(−x sin x + 2 cos x) dx and v = ex . We obtain
Z  Z 
x x x x
xe sin x dx = x sin x e − (x cos x + sin x)e − (−x sin x + 2 cos x)e dx
Z Z
= x sin x e − x cos x e − sin x e − x sin x e dx + 2 cos x ex dx.
x x x x
7-32 Chapter 7 Techniques of Integration

xex sin x dx to each side and divide by 2 to obtain


R
We add
Z Z
2 xe sin x dx = x sin x e − x cos x e − sin x e + 2 cos x ex dx
x x x x

1
Z Z
xex sin x dx = (x sin x ex − x cos x ex − sin x ex ) + cos x ex dx.
2

We determine cos x ex dx using integration by parts. Let u = cos x and dv = ex dx. Then
R
du = − sin x dx and v = ex . We obtain
Z Z Z
cos x e dx = (cos x)e − (− sin x )e dx = cos x e + sin x ex dx.
x x x x

We integrate by parts again with u = sin x and dv = ex dx. Then du = cos x dx and v = ex .
We obtain
Z  Z 
x x x x
cos x e dx = cos x e + (sin x)e − (cos x)e dx
Z
= cos x e + sin x e − cos xex dx.
x x

cos x ex dx to each side and divide by 2 to obtain


R
We add
Z
2 cos x ex dx = cos x ex + sin x ex
1 1
Z
cos x ex dx = cos x ex + sin x ex + C.
2 2
We substitute into the above to obtain
 
1 1 1
Z
xex cos x dx = (x sin x ex − x cos x ex − sin x ex ) + cos x ex + sin x ex + C
2 2 2
1
= 2 x sin x e
x
− 21 x cos x ex + 1
2 cos x ex + C .

73. We use integration by parts with u = sin−1 x and dv = xn dx. Then du = √ 1


1−x2
dx and
n+1
x
v= n+1 . We obtain

xn+1 1 xn+1
Z Z
xn sin−1 x dx = sin−1 x − √ dx.
n+1 n+1 1 − x2
1 1
R
74. We consider the integral (x2 +1) n dx and use integration by parts with u =
(x2 +1)n and
−2nx
dv = dx. Then du = (x2 +1)n+1 dx and v = x. We obtain
!
1 x −2nx
Z Z
n dx = n − (x) n+1 dx
(x2 + 1) (x2 + 1) (x2 + 1)
x x2
Z
= + 2n n+1 dx
(x + 1)n
2
(x2 + 1)
x2 + 1 − 1

x
Z
= + 2n n+1 dx
(x2 + 1)n (x2 + 1)
x 1 1
Z Z
= 2 n + 2n 2 n dx − 2n n+1 dx.
(x + 1) (x + 1) 2
(x + 1)
7.1 Integration by Parts 7-33

1
R
We solve for (x2 +1) n+1 dx, and obtain
1 x 1 1
Z Z Z
2n n+1 dx = + 2n dx − dx
(x2 + 1) (x2 + 1)n (x2 + 1)n (x2 + 1)n
1 x 1
Z Z
2n n+1 dx = + (2n − 1) dx
2
(x + 1) (x2 + 1)n (x2 + 1)n
 Z
1 x 1 1
Z
n+1 dx = 2 n + 1−
2n 2 n dx.
2
(x + 1) 2n(x + 1) (x + 1)

75. We use integration by parts with u = sinn−1 x and dv = sin x dx. Then du =
(n − 1) sinn−2 x cos x dx and v = − cos x. We obtain
Z Z
sinn x dx = sinn−1 x(− cos x) − (− cos x)(n − 1) sinn−2 x cos x dx
Z
n−1
= − sin x cos x + (n − 1) cos2 x sinn−2 x dx
Z
= − sinn−1 x cos x + (n − 1) 1 − sin2 x sinn−2 x dx


Z Z 
= − sinn−1 x cos x + (n − 1) sinn−2 x dx − sinn x dx
Z Z
= − sinn−1 x cos x + (n − 1) sinn−2 x dx − (n − 1) sinn x dx.

We add (n − 1) sinn x dx to each side, and then divide by n to obtain


R
Z Z
(1 + n − 1) sinn x dx = − sinn−1 x cos x + (n − 1) sinn−2 x dx

sinn−1 x cos x n − 1
Z Z
sinn x dx = − + sinn−2 x dx.
n n
76. We use integration by parts with u = sinn−1 x and dv = sin x cosm x dx. Then du =
m+1
(n − 1) sinn−2 x cos x dx and v = − cosm+1 x . We obtain
cosm+1 x cosm+1 x
Z   Z  
sinn x cosm x dx = sinn−1 x − − − (n − 1) sinn−2 x cos x dx
m+1 m+1
sinn−1 x cosm+1 x n−1
Z
= − + sinn−2 x cosm+2 x dx
m+1 m+1
sinn−1 x cosm+1 x n−1
Z
sinn−2 x 1 − sin2 x cosm x dx

= − +
m+1 m+1
sinn−1 x cosm+1 x n−1
Z
= − + sinn−2 x cosm x dx
m+1 m+1
n−1
Z
− sinn x cosm x dx.
m+1
n−1
sinn x cosm x dx to each side, and then divide by n+m
R
We add m+1 m+1 to obtain

sinn−1 x cosm+1 x
 Z
n−1 n−1
Z
1+ sinn x cosm x dx = − + sinn−2 x cosm x dx
m+1 m+1 m+1
n+m sinn−1 x cosm+1 x n−1
Z Z
n
sin x dx = − + sinn−2 x cosm x dx
m+1 m+1 m+1
sinn−1 x cosm+1 x n−1
Z Z
n
sin x dx = − + sinn−2 x cosm x dx.
n+m n+m
7-34 Chapter 7 Techniques of Integration

77. (a) We use integration by parts with u = x2 and dv = e5x dx. Then du = 2x dx and
v = 15 e5x . We obtain
  Z  
1 5x 1 5x
Z
x2 e5x dx = x2 e − e (2x) dx
5 5
1 2 5x 2
Z
= x e − xe5x dx.
5 5
We use integration by parts again with u = x and dv = e5x dx. Then du = dx and
v = 15 e5x . We obtain
   Z 
1 2 5x 2 1 5x 1 5x
Z
2 5x
x e dx = x e − x e − e dx
5 5 5 5
1 2 5x 2 2
Z
= x e − xe5x + e5x dx
5 25 25
 
1 2 5x 2 2 1 5x
= x e − xe5x + e +C
5 25 25 5
1 2 5x 2 5x 2 5x
= 5x e − 25 xe + 125 e +C .

(b) We use integration by parts with u = xn and dv = ekx dx. Then du = nxn−1 dx and
v = k1 ekx . We obtain
  Z  
1 kx 1 kx
Z
xn ekx dx = xn nxn−1 dx

e − e
k k
= k1 xn ekx − nk xn−1 ekx dx .
R

d
x3 ex dx = p(x)ex , then x
= x3 ex . So
R
78. (a) If dx (p(x)e )
d
(p(x)ex ) = p(x)ex + p′ (x)ex = x3 ex .
dx
Divide by ex to obtain p(x) + p′ (x) = x3 .
(b) We use integration by parts with u = x3 and dv = ex dx. Then du = 3x2 dx and
v = ex . We obtain
Z Z
x3 ex dx = x3 ex − (ex ) 3x2 dx


Z
= x e − 3 x2 ex dx.
3 x

We use integration by parts again with u = x2 and dv = ex dx. Then du = 2x dx and


v = ex . We obtain
Z  Z 
x3 ex dx = x3 ex − 3 x2 ex − (ex )(2x) dx
Z
= x3 ex − 3x2 ex + 6 xex dx.

We use integration by parts again with u = x and dv = ex dx. Then du = dx and


v = ex . We obtain
Z  Z 
x3 ex dx = x3 ex − 3x2 ex + 6 xex − ex dx

= x3 ex − 3x2 ex + 6xex − 6ex + C


= x3 − 3x2 + 6x − 6 ex + C.

We have found the polynomial p(x) = x3 − 3x2 + 6x − 6 .


7.1 Integration by Parts 7-35

79. (a) We use integration by parts with u = sin x and dv = cos x dx. Then du = cos x dx
and v = sin x. We obtain
Z Z
sin x cos x dx = (sin x)(sin x) − (sin x)(cos x) dx
Z
2
= sin x − sin x cos x dx.
R
We add sin x cos x dx to each side, and then divide by 2 to obtain
Z
2 sin x cos x dx = sin2 x
1
Z
sin x cos x dx = sin2 x + C1 .
2

So we obtain f (x) = 1
2 sin2 x .
(b) We use integration by parts with u = cos x and dv = sin x dx. Then du = − sin x dx
and v = − cos x. We obtain
Z Z
sin x cos x dx = (cos x)(− cos x) − (− cos x)(− sin x) dx
Z
2
= − cos x − sin x cos x dx.
R
We add sin x cos x dx to each side, and then divide by 2 to obtain
Z
2 sin x cos x dx = − cos2 x
1
Z
sin x cos x dx = − cos2 x + C2 .
2

So we obtain g(x) = − 21 cos2 x .


the identity sin (2x) = 2 sin x cos x we have sin x cos x = 12 sin (2x). So sin x cos x dx =
R
(c) From
1
sin (2x) dx. Let u = 2x, then du = 2 dx, so dx = 21 du. We substitute and obtain
R
2

1
Z Z
sin x cos x dx = sin (2x) dx
2
 
1 1
Z
= sin u du
2 2
1
Z
= sin u du
4
1
= (− cos u) + C3
4
1
= − cos (2x) + C3 .
4

So we obtain h(x) = − 14 cos (2x) .

(d) We have f (x) + C1 = g (x) + C2 , so 1


2 sin2 x + C1 = − 21 cos2 x + C2 . We obtain
C2 = 1
2 cos2 x + 1
2 sin2 x + C1 = 1
2 + C1 , so C2 = 1
2 + C1 .

(e) We have f (x) + C 1 = h (x)+C3 , so 12 sin2 x + C 1 = − 14 cos (2x)+C3 . We obtain C3 =


2 2
1 1 1
 1 2 1 1
4 cos (2x)+ 2 sin x + C 1 = 4 1 − 2 sin x + 2 sin x + C 1 = 4 +C1 . So C3 = 4 + C1 .
7-36 Chapter 7 Techniques of Integration

80. (a)
0.1

g(x)
f(x)

-0.2 0.2 0.4 0.6 0.8 1.0 1.2


x

-0.1
R1
(b) The area enclosed by the graphs of f and g is given by 0 x2 e−3x − x3 e−3x dx =

R1 2 3 −3x
dx. We use integration by parts with u = x2 − x3 and dv = e−3x dx.

0 x −x e
Then du = 2x − 3x2 dx and v = − 31 e−3x . We obtain
Z 1   1 Z 1  
2 3 −3x 2 3 1 −3x 1 −3x
2x − 3x2 dx
  
x −x e dx = x −x − e − − e
0 3 0 3
  0  
1 1
12 − 13 − e−3(1) − 02 − 03 − e−3(0)
 
=
3 3
Z 1
1
2x − 3x2 e−3x dx

+
3 0
1 1
Z
2x − 3x2 e−3x dx.

=
3 0
We use integration by parts again with u = 2x − 3x2 and dv = e−3x dx. Then
du = (2 − 6x) dx and v = − 31 e−3x . We obtain
Z 1   1 Z 1   !
1 1 1
x2 − x3 e−3x dx = 2x − 3x2 − e−3x − e−3x (2 − 6x) dx
 

0 3 3 0 0 3
    1 
1 2
 1 
2
= 2(1) − 3(1) − e−3(1) − 2(0) − 3(0) − e−3(0)
3 3 3
Z 1
2
+ (1 − 3x)e−3x dx
9 0
1 −3 2 1
Z
= e + (1 − 3x)e−3x dx.
9 9 0
We use integration by parts again with u = 1−3x and dv = e−3x dx. Then du = −3 dx
and v = − 13 e−3x . We obtain
Z 1   −3x 1 Z 1  −3x  !
2 3 −3x
 1 −3 2 e e
x −x e dx = e + (1 − 3x) − − − (−3) dx
0 9 9 3 0 0 3
  −3(1)   −3(0) 
1 −3 2 e e
= e + (1 − 3(1)) − − (1 − 3(0)) −
9 9 3 3
Z 1
2
− e−3x dx
9 0
 1
7 −3 2 2 1 −3x
= e + − − e
27 27 9 3 0
  
7 −3 2 2 1 −3(1) 1 −3(0)
= e + − − e − − e
27 27 9 3 3
1 −3
= 3e .
7.1 Integration by Parts 7-37

81. (a)
x
4

0
1 2 3 4 5t
-2

-4

(b) Set 3e−t cos (2t)+ 2e−t sin (2t) = 0 and solve for t. We obtain tan (2t) = − 23 , so
2t = tan−1 − 23 + kπ, where k is any integer. So the least positive number t such
that x(t) = 0 is t = π2 + 12 tan−1 − 23 .


π 1
2 −2 tan−1 ( 32 )
3e−t cos (2t) + 2e−t sin (2t) dt using a Computer Algebra
R
(c) We integrate 0
tan−1 ( 32 )
π 1
2 −2
(3e−t cos (2t) + 2e−t sin (2t)) dt ≈ 1.890 .
R
System, and we obtain 0
√ 
82. We use integration by parts with u = ln x + x2 + a2 and dv = dx. Then

1
−1/2
1+ x2 + a2
2 (2x)
du = √ dx
x+ x +a2 2
−1/2 √
1 + x x2 + a2 x2 + a2
= √ √ dx
x + x2 + a2 x2 + a2

x2 + a2 + x
= √ √ dx
x + x2 + a2 x2 + a2
1
= √ dx
x2 + a2
and v = x. We obtain
x
Z  p   p  Z
ln x + x2 + a2 dx = x ln x + x2 + a2 − √ dx.
x + a2
2

Let u = x2 + a2 , then du = 2x dx, so x dx = du


2 . We substitute and obtain

1
Z  p   p  Z
ln x + x2 + a2 dx = x ln x + x2 + a2 − u−1/2 du
2
 p  1  1/2 
= x ln x + x2 + a2 − 2u +C
  p 2
p
= x ln x + x2 + a2 − x2 + a2 + C.

83. We use integration by parts with u = sin (bx) and dv = eax dx. Then du = b cos (bx) dx
and v = a1 eax . We obtain
  Z  
1 ax 1 ax
Z
ax
e sin (bx) dx = (sin (bx)) e − e (b cos (bx)) dx
a a
1 ax b
Z
= e sin (bx) − eax cos (bx) dx
a a
7-38 Chapter 7 Techniques of Integration

We integrate by parts again with u = cos bx and dv = eax dx. Then du = −b sin bx dx and
v = a1 eax . We obtain
   Z   
1 ax b 1 ax 1 ax
Z
ax
e sin (bx) dx = e sin (bx) − (cos (bx)) e − e (−b sin (bx)) dx
a a a a
1 ax b b2
Z
= e sin (bx) − 2 eax cos (bx) − 2 eax sin (bx) dx
a a a
2 R 2 2
We add ab 2 eax sin (bx) dx from each side and divide by a a+b 2 to obtain

b2
 Z
1 ax b
1+ 2 eax sin (bx) dx = e sin (bx) − 2 eax cos (bx)
a a a
a b
Z
eax sin (bx) dx = eax sin (bx) − 2 eax cos (bx) + C
a2 + b 2 a + b2
eax [a sin (bx) − b cos (bx)]
= + C.
a2 + b 2

84. We use integration by parts with u = t and dv = g ′ (t) dt. Then du = dt and v = g(t). We
obtain
Z x Z x
x
F (x) = tg ′ (t) dt = [tg(t)]0 − g(t) dt
0 0
Z x
= xg(x) − 0g(0) − g(t) dt
0
Z x
= xg(x) − g(t) dt.
0

R π/2 (5)(3)(1) π 
85. (a) 0
sin6 x dx = (6)(4)(2) 2 = 5π
32 .
R π/2 (4)(2)
(b) 0 sin5 x dx = (5)(3)(1) = 8
15 .
R π/2 R π/2 (7)(5)(3)(1) π 
(c) 0 cos8 x dx = 0 sin8 x dx = (8)(6)(4)(2) 2 = 35π
256 .
R π/2 R π/2 (5)(3)(1) π 
(d) 0
cos6 x dx = 0
sin6 x dx = (6)(4)(2) 2 = 5π
32 .

Challenge Problems
86. Suppose first that n = 2m + 1 > 1 is odd, so that m is any positive integer. By Problem
75, we have
π/2 π/2
sinn−1 x cos x n − 1 π/2 n−2
Z  Z
n
sin x dx = − + sin x dx
0 n 0 n 0
" #
sinn−1 π2 cos π2 sinn−1 0 cos 0 n − 1 π/2 n−2
  Z
= − − − + sin x dx
n n n 0

n − 1 π/2 n−2
Z
= sin x dx.
n 0
7.1 Integration by Parts 7-39

We continue, and obtain


π/2 π/2
n−1
Z Z
sinn x dx = sinn−2 x dx
0 n 0
!
π/2
n−1 n−3
Z
n−4
= sin x dx
n n−2 0
!
n − 1 n − 3 n − 5 π/2 n−6
Z
= sin x dx
n n−2 n−4 0
= ···
n−1n−3n−5 2 π/2
Z
= ··· sin x dx
n n−2n−4 3 0
n−1n−3n−5 2 π/2
= · · · [− cos x]0
n n−2n−4 3
n−1n−3n−5 2
= · · · (1)
n n−2n−4 3
(2m) (2m − 2) · · · (2)
=
(2m + 1) (2m − 1) · · · (3) (1)

So the formula holds. Now suppose that n = 2m > 1 is even, so that m is any positive
integer. By the above, we have
π/2 π/2
n−1
Z Z
sinn x dx = sinn−2 x dx
0 n 0
π/2
n−1n−3n−5 3
Z
= ··· sin2 x dx
n n−2n−4 4 0
n−1n−3n−5 3 1 π/2
Z
= ··· dx
n n−2n−4 42 0
n−1n−3n−5 31π
= ···
n n−2n−4 422
(2m − 1) (2m − 3) · · · (3) (1)  π 
= ,
(2m) (2m − 2) · · · (4) (2) 2

and the formula holds.


87. (a) We use integration by parts with u = xn and dv = ex dx. Then du = nxn−1 dx and
v = ex . We have Z Z
x e dx = x e − n xn−1 ex dx
n x n x

Repeating, we obtain for some constants c and k


Z  Z 
xn ex dx = xn ex − n xn−1 ex − (n − 1) xn−2 ex dx
Z
= xn − nxn−1 ex + n (n − 1) xn−2 ex dx


= ··· Z
n n−1
+ · · · + kx e + c ex dx
 x
= x − nx

xn − nxn−1 + · · · + kx + c ex + C

=
= p (x) ex + C.
7-40 Chapter 7 Techniques of Integration

d
(b) Since dx (p(x)ex ) = xn ex , we have p(x)ex + p′ (x)ex = xn ex . Divide by ex to obtain
p(x) + p (x) = xn .

Pn k n!
(c) We show p (x) = k=0 (−1) (n−k)! xn−k satisfies p (x) + p′ (x) = xn . We have
n n
!
X n!
k d X
k n!
p (x) + p (x)′
= (−1) xn−k + (−1) xn−k
k=0
(n − k)! dx k=0
(n − k)!
n−1 n−1
X n! X n!
= xn + (−1)k xn−k + (−1)k (n − k) xn−k−1
k=0
(n − k)! k=0
(n − k)!
n−1 n
X n! X n!
= xn + (−1)k xn−k + (−1)k−1 (n − (k − 1)) xn−(k−1)−1
k=0
(n − k)! k=1
(n − (k − 1))!
n−1 n
X n! X n!
= xn + (−1)k xn−k + (−1)k−1 (n − k + 1) xn−k
k=0
(n − k)! k=1
(n − k + 1)!
n−1 n
X n! X n!
= xn + (−1)k xn−k + (−1)k (−1)−1 (n − k + 1) xn−k
k=0
(n − k)! k=1
(n − k + 1) (n − k)!
n−1 n
X n! X n!
= xn + (−1)k xn−k − (−1)k xn−k
k=0
(n − k)! k=1
(n − k)!
= xn .
2 2
88. Use integration by parts with u = ex and dv = x2n dx. Then du = 2xex dx and
1
v = 2n+1 x2n+1 . We obtain

1  1 Z 1  
1 1
Z 
2 2 2
x2n ex dx = x2n+1 ex − x2n+1 2xex dx
0 2n + 1 0 0 2n + 1
  Z 1
1 2 2
= e − x2n+2 ex dx.
2n + 1 2n + 1 0

Starting with the n = 0 case, we repeat the above formula, and obtain
Z 1
2 1 2 x2
 
1
Z
2
ex dx = e − x e dx
0 1 1 0
2 1 4 x2
   
2 1
Z
= e (1) − e − x e dx
1 3 3 0
Z 1
22
 
2 2
= e 1− + x6 ex dx
3 3 (1) 0
22 2 1 8 x2
     
2 1
Z
= e 1− + e − x e dx
3 3 (1) 5 5 0
 2
 3 Z 1
2 2 2 2
= e 1− + + x8 ex dx
3 5 (3) (1) 5 (3) (1) 0
= ···
(−1)n 2n (−1)n+1 2n+1
  Z 1
2 2
= e 1 − + ···+ + x2n+2 ex dx.
3 (2n + 1) · · · (3)(1) (2n + 1) · · · (3)(1) 0

So the formula holds for all nonnegative integers n.


89. We use integration by parts with u = f (x) and dv = ex dx. Then du = f ′ (x) dx and
v = ex . We have Z Z
f (x)ex dx = f (x)ex − f ′ (x)ex dx
7.1 Integration by Parts 7-41

We continue, using u = f ′ (x) , dv = ex and obtain


Z  Z 
x x ′ x ′′ x
f (x)e dx = f (x)e − f (x)e − f (x)e dx
Z
= (f (x) − f (x)) e + f ′′ (x)ex dx
′ x

= ··· Z
 
= f (x) − f ′ (x) + · · · ± f (n−1) (x) ex ± f (n) (x)ex dx

Since f (x) is a polynomial of degree n, f (n) (x) is a constant, k, and we have


Z   Z
f (x)ex dx = f (x) − f ′ (x) + · · · ± f (n−1) (x) ex ± kex dx
 
= f (x) − f ′ (x) + · · · ± f (n−1) (x) ex ± kex + C
= (f (x) − f ′ (x) + · · · ± k) ex + C.
We nowhave f (x) = g (x) ex + C, where g (x) = f (x) − f ′ (x) + · · · ± k is a polynomial of
degree n.
90. (a) We use integration by parts with u = f ′ (t) and dv = dt. Then du = f ′′ (t) dt and take
v = t − b. We obtain
Z b
f (b) − f (a) = f ′ (t) dt
a
Z b
b
= [(t − b)f ′ (t)]a − f ′′ (t)(t − b) dt
a
Z b
= (b − b)f ′ (b) − (a − b)f ′ (a) − f ′′ (t)(t − b) dt
a
Z b
= f ′ (a)(b − a) − f ′′ (t)(t − b) dt.
a
Rb
(b) We use integration by parts on a f ′′ (t)(t − b) dt with u = f ′′ (t) and dv = (t − b) dt.
Then du = f ′′′ (t) dt and v = 12 (t − b)2 . We obtain
Z b

f (b) − f (a) = f (a)(b − a) − f ′′ (t)(t − b) dt
a
 b Z b !
′ 1 2 ′′ ′′′ 1 2
= f (a)(b − a) − (t − b) f (t) − f (t) (t − b) dt
2 a a 2
= f ′ (a)(b − a)
  Z b ′′′ !
1 2 ′′ 1 2 ′′ f (t) 2
− (b − b) f (b) − (a − b) f (a) − (t − b) dt
2 2 a 2
Z b ′′′
f ′′ (a) f (t)
= f ′ (a)(b − a) + (b − a)2 + (t − b)2 dt.
2 a 2
91. We use integration by parts with u = f (x) and dv = dx. Then du = f ′ (x) dx and take
v = x. We obtain
Z b Z b
f (x) dx = [xf (x)]ba − xf ′ (x) dx
a a
Z b
= bf (b) − af (a) − xf ′ (x) dx.
a
7-42 Chapter 7 Techniques of Integration

Let x = f −1 (y), then f (x) = y, so f ′ (x) dx = dy. The limits of integration become
y = f (a) and y = f (b). We substitute and obtain
Z b Z f (b)
f (x) dx = bf (b) − af (a) − f −1 (y) dy,
a f (a)

so Z b Z f (b)
f (x) dx + f −1 (y) dy = bf (b) − af (a).
a f (a)

92. (a) We use integration by parts with u = cosh x and dv = ex dx. Then du = sinh x dx
and take v = ex . We obtain
Z Z
ex cosh x dx = ex cosh x − ex sinh x dx.

We use integration by parts again with u = sinh x and dv = ex dx. Then du =


cosh x dx and take v = ex . We obtain
Z  Z 
ex cosh x dx = ex cosh x − ex sinh x − ex cosh x dx
Z
= ex cosh x − ex sinh x + ex cosh x dx.

The integrals cancel, and this will not allow us to determine the integral. Note that
ex cosh x − ex sinh x = ex (cosh x − sinh x)
= ex e−x


= 1.
This is expected, as any two antiderivatives of ex cosh x, as denoted by ex cosh x dx
R
on each side of the equation
Z Z
ex cosh x dx = ex cosh x − ex sinh x + ex cosh x dx,

must differ by a constant. So their difference, ex cosh x − ex sinh x, must be constant.


ex +e−x
(b) We rewrite cosh x = 2 , and obtain
ex + e−x
Z Z  
ex cosh x dx = ex dx
2
1
Z
e2x + 1 dx

=
2
 
1 1 2x
= e +x +C
2 2
1 2x
= 4e + 12 x + C .

AP
R
Practice Problems
1. The velocity of the object at time t is given by
Z t Z t
v(t) = v(1) + a(x) dx = 2 + ln (x + 1) dx.
1 1
R
Use integration by parts to find ln (x + 1) dx.
Let u = ln (x + 1) and dv = dx.
1
R
Then du = x+1 dx and v = dx = x.
7.1 Integration by Parts 7-43

1
Z Z
Now ln (x + 1) dx = x ln (x + 1) − x· dx
x+1
Z  
1
= x ln (x + 1) − 1− dx
x+1
= x ln (x + 1) − x + ln (x + 1) + C
= (x + 1) ln (x + 1) − x + C.

Z t
Therefore, v(t) = 2 + ln (x + 1) dx
1
t
= 2 + [(x + 1) ln (x + 1) − x]1
= 2 + [(t + 1) ln (t + 1) − t] − (2 ln 2 − 1), and

v(3) = 2 + [4 ln 4 − 3] − (2 ln 2 − 1) = 2 + 4 ln (4) − ln 4 − 3 + 1 = 3 ln 4 .
The answer is B.
2. Integrate x2 e2x dx using integration by parts.
R

Let u = x2 and dv = e2x dx.


Then du = 2x dx and v = e2x dx = 12 e2x .
R

  Z  
2 1 2x 1 2x 1
Z Z
2 2x
Now x e dx = x e − e (2x dx) = x2 e2x − xe2x dx.
2 2 2
R 2x
Integrate xe dx using integration by parts.

Let u = x and dv = e2x dx.


Then du = dx and v = e2x dx = 12 e2x .
R

  Z
1 1 2x
Z
Now xe2x dx = x e2x − e dx
2 2
1 1
Z
= xe2x − e2x dx
2 2
1 1
= xe2x − e2x + C.
2 4

 
1 2 2x 1 2x 1 2x
Z
2 2x
So, x e dx = x e − xe − e + C
2 2 4
1 2 2x 1 2x 1 2x
= x e − xe + e + C
2  2 4

1 1 1
= e2x x2 − x + + C and f (x) = 12 x2 − 21 x + 1
4 .
2 2 4

The answer is D.
cos−1 x dx.
R
3. Use integration by parts to find

Let u = cos−1 x and dv = dx.


1
R
Then du = − √1−x 2
dx and v = dx = x.
7-44 Chapter 7 Techniques of Integration

 
1 x
Z Z Z
Now cos−1 x dx = x cos−1 x − x · −√ dx = x cos−1 x + √ dx.
1−x2 1 − x2
x
dx, let w = 1 − x2 . Then dw = −2x dx, x dx = − dw
R
To evaluate √1−x 2 2 , and

x 1
Z Z
√ dx = √ x dx
1 − x2 1 − x2
 
dw
Z
= w−1/2 −
2
1
Z
=− w−1/2 dw
2
1 w1/2
 
=− 1 +C
2 2

=− w+C
p
= − 1 − x2 + C.

x √
Z Z
So, cos−1 x dx = x cos−1 x + √ dx = x cos−1 x − 1 − x2 + C .
1 − x2

The answer is C.
4. Integrate xe−x dx using integration by parts.
R

Let u = x and dv = e−x dx.


Then du = dx and v = e−x dx = −e−x .
R
Z Z
−x −x
−e−x dx
 
Now xe dx = x −e −
Z
= −xe−x + e−x dx

= −xe−x − e−x + C
= −(x + 1)e−x + C
Z 2 2
xe−x dx = −(x + 1)e−x 0 = −3e−2 − (−1) = 1 − 3e−2 .

So,
0

The answer is A.
3x2 + 2 sin x dx using integration by parts.
R 
5. Integrate
Let u = 3x2 + 2 and dv = sin x dx.
R
Then du = 6x dx and v = sin x dx = −cos x.
Z Z
2 2
 
So 3x + 2 sin x dx = 3x + 2 (−cos x) − (−cos x)(6x dx)

= −(3x2 + 2) cos x + 6
R
x cos x dx .

The answer is B.
6. Integrate xf ′ (x) dx using integration by parts.
R

Let u = x and dv = f ′ (x) dx.


Then du = dx and v = f ′ (x) dx = f (x).
R
7.2 Integrals Containing Trigonometric Functions 7-45

Z
xf ′ (x) dx = x f (x) −
R
Now f (x) dx + C .

The answer is C.
√ √
7. The diameter of each semicircular slice is ln x. The radius is 21 ln x.
 √ 2
The volume of each semicircular slice of thickness dx is dV = 12 · π · 12 ln x dx = π
8 ln x.
R e2 π
R e2
The volume of the solid is V = 1
dV = 8 1
ln x dx.
R
Use integration by parts to find ln x dx.
Let u = ln x and dv = dx.
Then du = x1 dx and v = dx = x.
R

1
Z Z Z
Now ln x dx = x ln x − x · dx = x ln x − dx = x ln x − x + C.
x
Z 2
π e π 2 π 2
ln x dx = [x ln x − x]e1 = e ln e2 − e2 − (0 − 1) = π8 (e2 + 1) .
 
So,
8 1 8 8
The answer is A.
dy dy
8. Rewrite the differential equation dx = (1 + ln x)y as y = (1 + ln x) dx.
R dy R R
Integrate both sides to obtain y dy = (1 + ln x) dx or ln |y| = x + ln x dx.
R
Use integration by parts to find ln x dx.
Let u = ln x and dv = dx.
Then du = x1 dx and v = dx = x.
R

1
Z Z Z
Now ln x dx = x ln x − x · dx = x ln x − dx = x ln x − x + C.
x
R
Then ln |y| = x + ln x dx = x + x ln x − x + C = x ln x + C.
Applying the condition that y = 1 when x = 1 yields ln 1 = 1 · ln 1 + C.
Thus, C = 0 and ln |y| = x ln x = ln xx .
Solving for y, |y| = xx and y = ±xx . The condition y = 1 when x = 1 requires the positive
sign.

So, y = xx .

The answer is D.

7.2 Integrals Containing Trigonometric Functions


Concepts and Vocabulary
1. True.
2. True.
7-46 Chapter 7 Techniques of Integration

Skill Building
3. Factor out cos x and use the identity cos2 x = 1 − sin2 x.
Z Z
cos5 x dx = cos4 x cos x dx
Z
2
= cos2 x cos x dx
Z
2
= 1 − sin2 x cos x dx.

Let u = sin x, then du = cos x dx. We substitute and obtain


Z Z
5
2
cos x dx = 1 − u2 du
Z
u4 − 2u2 + 1 du

=
1 5 2 3
= u − u +u+C
5 3
= 1
5 sin5 x − 2
3 sin3 x + sin x + C .

4. Factor out sin x and use the identity sin2 x = 1 − cos2 x.


Z Z
3
sin x dx = sin2 x sin x dx
Z
1 − cos2 x sin x dx.

=

Let u = cos x, then du = − sin x dx, so sin x dx = −du. We substitute and obtain
Z Z
sin3 x dx = 1 − u2 (−du)


Z
u2 − 1 du

=
1 3
= u −u+C
3
1
= 3 cos3 x − cos x + C .

5. Use the identity sin2 x = 1−cos2 (2x) and obtain


Z Z
6
3
sin x dx = sin2 x dx
Z  3
1 − cos (2x)
= dx
2
1
Z
1 − 3 cos (2x) + 3 cos2 (2x) − cos3 (2x) dx

=
8
1 3 3 1
Z Z Z Z
= dx − cos (2x) dx + cos2 (2x) dx − cos3 (2x) dx
8 8 8 8
1 3 3 1
Z Z
= x− sin (2x) + cos2 (2x) dx − cos3 (2x) dx.
8 16 8 8
7.2 Integrals Containing Trigonometric Functions 7-47

We evaluate
1 + cos (4x)
Z Z
cos2 (2x) dx = dx
2
1
Z
= (1 + cos (4x)) dx
2
x sin (4x)
= + + C.
2 8
And also
Z Z
cos3 (2x) dx = cos2 (2x) cos (2x) dx
Z
1 − sin2 (2x) cos (2x) dx.

=

Let u = sin (2x), then du = 2 cos (2x) dx, so cos (2x) dx = du


2 . We substitute and obtain

 du
Z Z
cos3 (2x) dx = 1 − u2
2
3
 
1 u
= u− +C
2 3
sin (2x) sin3 (2x)
= − + C.
2 6
We now obtain
1 sin (2x) sin3 (2x)
   
1 3 3 x sin (4x)
Z
sin6 x dx = x− sin (2x) + + − − +C
8 16 8 2 8 8 2 6
= 5
16 x − 1
4 sin (2x) + 3
64 sin (4x) + 1
48 sin3 (2x) + C .

6. Use the identity cos2 x = 1+cos2 (2x) and obtain


Z Z
2
cos4 x dx = cos2 x dx
Z  2
1 + cos (2x)
= dx
2
1
Z
1 + 2 cos (2x) + cos2 (2x) dx

=
4
1 1 1
Z Z Z
= dx + cos (2x) dx + cos2 (2x) dx
4 2 4
1 1 1
Z
= x + sin (2x) + cos2 (2x) dx.
4 4 4
We evaluate
1 + cos (4x)
Z Z
2
cos (2x) dx = dx
2
1
Z
= (1 + cos (4x)) dx
2
x sin (4x)
= + + C.
2 8
7-48 Chapter 7 Techniques of Integration

We now obtain
 
1 1 1 x sin (4x)
Z
cos4 x dx = x + sin (2x) + + +C
4 4 4 2 8
3 1 1
= 8x + 4 sin (2x) + 32 sin (4x) + C .

7. Use the identity sin2 (πx) = 1−cos2(2πx) and obtain


1 − cos (2πx)
Z Z
2
sin (πx) dx = dx
2
1
Z
= (1 − cos (2πx)) dx
2
 
1 1
= x− sin (2πx) + C
2 2π
x 1
= 2 − 4π sin (2πx) + C .

8. Use the identity cos2 (2x) = 1+cos2 (4x) and obtain


Z Z
2
cos4 (2x) dx = cos2 (2x) dx
Z  2
1 + cos (4x)
= dx
2
1
Z
1 + 2 cos (4x) + cos2 (4x) dx

=
4
1 1 1
Z Z Z
= dx + cos (4x) dx + cos2 (4x) dx
4 2 4
1 1 1
Z
= x + sin (4x) + cos2 (4x) dx.
4 8 4
We evaluate
1 + cos (8x)
Z Z
cos2 (4x) dx = dx
2
1
Z
= (1 + cos (8x)) dx
2
x sin (8x)
= + + C.
2 16
We now obtain
 
1 1 1 x sin (8x)
Z
cos4 (2x) dx = x + sin (4x) + + +C
4 8 4 2 16
3 1 1
= 8x + 8 sin (4x) + 64 sin (8x) + C .

9. Factor out cos x and use the identity cos2 x = 1 − sin2 x.


Z π Z π
5
cos x dx = cos4 x cos x dx
0 0
Z π
2
= cos2 x cos x dx
Z0 π
2
= 1 − sin2 x cos x dx.
0
7.2 Integrals Containing Trigonometric Functions 7-49

Let u = sin x, then du = cos x dx. The lower limit of integration is u = sin 0 = 0 and the
upper limit of integration is u = sin π = 0. We substitute and obtain
Z π Z 0
5 2
cos x dx = (1 − u) du = 0 .
0 0

10. Since sin3 x is an odd function on a symmetric interval − π3 , π3 about 0, we have


 

Z π/3
sin3 x dx = 0 .
−π/3

11. Factor out sin x and use the identity sin2 x = 1 − cos2 x.
Z Z
sin3 x cos2 x dx = sin2 x cos2 x sin x dx
Z
1 − cos2 x cos2 x sin x dx.

=

Let u = cos x, then du = − sin x dx, so sin x dx = −du. We substitute and obtain
Z Z
sin3 x cos2 x dx = 1 − u2 u2 (−du)
 

Z
u4 − u2 du

=
1 5 1 3
= u − u +C
5 3
1 1
= 5 cos5 x − 3 cos3 x + C .

12. Factor out cos x and use the identity cos2 x = 1 − sin2 x.
Z Z
sin4 x cos3 x dx = sin4 x cos2 x cos x dx
Z
sin4 x 1 − sin2 x cos x dx.

=

Let u = sin x, then du = cos x dx. We substitute and obtain


Z Z
sin4 x cos3 x dx = u4 1 − u2 du


Z
u4 − u6 du

=
1 5 1 7
= u − u +C
5 7
= 1
5 sin5 x − 1
7 sin7 x + C .

R π/2
13. Integrate 0 sin3 x(cos x)3/2 dx using trigonometric identities.
The exponent of sin x is 3, a positive, odd integer. Factor sin x from sin3 x and write the
rest of the integrand in terms of cosines.
Z π/2 Z π/2
3 3/2
sin x(cos x) dx = sin2 x(cos x)3/2 sin x dx
0 0
Z π/2
1 − cos2 x (cos x)3/2 sin x dx.

=
0
7-50 Chapter 7 Techniques of Integration

Now use the substitution u = cos x and du = − sin x dx. Then sin x dx = −du. The lower
limit of integration becomes u = cos 0 = 1 and the upper limit of integration becomes
u = cos π2 = 0. Therefore,
Z π/2 Z π/2
sin3 x(cos x)3/2 dx = 1 − cos2 x (cos x)3/2 sin x dx

0 0
Z 0 Z 1
1 − u2 u3/2 (−du) = 1 − u2 u3/2 du.
 
=
1 0

Use algebraic manipulation to rewrite 1 − u u3/2 in a form whose antiderivative is


2


recognizable: 1 − u2 u3/2 = u3/2 − u7/2 .




Then
Z 1 Z 1  
1 − u2 u3/2 du = u3/2 − u7/2 du

0 0
 1  
2 5/2 2 9/2 2 2 8
= u − u = − − (0 − 0) = 45 .
5 9 0 5 9
R π/2 √
14. Integrate 0 cos3 x sin x dx using trigonometric identities.
The exponent of cos x is 3, a positive, odd integer. Factor cos x from cos3 x and write the
rest of the integrand in terms of sines.
Z π/2 √ Z π/2 √ Z π/2
√
3
cos x sin x dx = 2
cos x sin x cos x dx = 1 − sin2 x sin x cos x dx.
0 0 0

Now use the substitution u = sin x and du = cos x dx. The lower limit of integration
becomes u = sin 0 = 0 and the upper limit of integration becomes u = sin π2 = 1. Therefore,
Z π/2 √ Z π/2 Z 1
3 2
√
1 − u2 u du.
√
cos x sin x dx = 1 − sin x sin x cos x dx =
0 0 0
√ 2
Use algebraic manipulation to rewrite 1 − u u in a form whose antiderivative is recog-

nizable: 1 − u2 u = 1 − u2 u1/2 = u1/2 − u5/2 .
 

Then
Z 1 1  1  
2 3/2 2 7/2 2 2
Z  
1 − u2 u du = u1/2 − u5/2 du = 8
√
u − u = − − (0 − 0) = 21 .
0 0 3 7 0 3 7

sin3 x(cos x)1/3 dx using trigonometric identities.


R
15. Integrate
The exponent of sin x is 3, a positive, odd integer. Factor sin x from sin3 x and write the
rest of the integrand in terms of cosines.
Z Z Z
3 1/3 2 1/3
1 − cos2 x (cos x)1/3 sin x dx.

sin x(cos x) dx = sin x(cos x) sin x dx =

Now use the substitution u = cos x and du = − sin x dx. So, sin x dx = −du and
Z Z
sin3 x(cos x)1/3 dx = 1 − cos2 x (cos x)1/3 sin x dx


Z Z
2
 1/3
1 − u2 u1/3 du.

= 1 − u u (−du) = −

Use algebraic manipulation to rewrite 1 − u2 u1/3 in a form whose antiderivative is




recognizable: 1 − u2 u1/3 = u1/3 − u7/3 .



7.2 Integrals Containing Trigonometric Functions 7-51

Then
 
3 3
Z Z  
sin3 x(cos x)1/3 dx = − u1/3 − u7/3 du = − u4/3 − u10/3 + C
4 10
 
3 3
= − u4/3 − u10/3 + C
4 10
3 10/3 4/3
= 10 (cos x) − 43 (cos x) +C .

16. Factor out cos x and use the identity cos2 x = 1 − sin2 x.
Z Z
cos3 x sin1/2 x dx = cos2 x sin1/2 x cos x dx
Z
1 − sin2 x sin1/2 x cos x dx.

=

Let u = sin x, then du = cos x dx. We substitute and obtain


Z Z
cos3 x sin1/2 x dx = 1 − u2 u1/2 du


Z  
= u1/2 − u5/2 du
2 3/2 2 7/2
= u − u +C
3 7
= 2
3 sin3/2 x − 2
7 sin7/2 x + C .

17. Factor out cos x2 and use the identity cos2 x2 = 1 − sin2 x2 .
  

2 x 3 x
Z     Z x x x
sin cos dx = cos2 sin2 cos dx
2 2 2 2 2
Z   x  x x
= 1 − sin2 sin2 cos dx.
2 2 2

Let u = sin x2 , then du = 21 cos x2 dx, so cos x2 dx = 2 du. We substitute and obtain
  

Z x x Z
sin2 cos3 1 − u2 u2 (2 du)
 
dx =
2 2
Z
u2 − u4 du

= 2
 
1 1
= 2 u3 − u5 + C
3 5
= 23 sin3 x2 − 52 sin5 x2 + C .
 

18. Factor out cos (4x) and use the identity cos2 (4x) = 1 − sin2 (4x).
Z Z
sin3 (4x) cos3 (4x) dx = cos2 (4x) sin3 (4x) cos (4x) dx
Z
1 − sin2 (4x) sin3 (4x) cos (4x) dx.

=
7-52 Chapter 7 Techniques of Integration

Let u = sin (4x), then du = 4 cos (4x) dx, so cos (4x) dx = du


4 . We substitute and obtain

  du
Z Z
sin3 (4x) cos3 (4x) dx = 1 − u2 u3
4
1
Z
u3 − u5 du

=
4
 
1 1 4 1 6
= u − u +C
4 4 6
= 1
16 sin4 (4x) − 1
24 sin6 (4x) + C .

tan3 x sec3 x dx using trigonometric identities.


R
19. Integrate
The exponent of tan x is 3, a positive, odd integer ≥ 3. Factor sec x tan x from tan3 x sec3 x
and write the rest of the integrand in terms of secants. Use the identity tan2 x = sec2 x − 1.
Z Z Z
3 3 2 2
sec2 x − 1 sec2 x · sec x tan x dx.

tan x sec x dx = tan x sec x · sec x tan x dx =

Now use the substitution u = sec x and du = sec x tan x dx.


Z Z Z
3 3 2
 2
u2 − 1 u2 du.

tan x sec x dx = sec x − 1 sec x · sec x tan x dx =

to rewrite u2 − 1 u2 in a form whose antiderivative is recog-



Use algebraic manipulation
nizable: u2 − 1 u2 = u4 − u2 .


Then
1 1
Z Z Z
3 3 sec5 x sec3 x
u − 1 u2 du =
2
u4 − u3 du = u5 − u3 + C =
 
tan x sec x dx = 5 − 3 +C .
5 3

20. Factor out sec x tan x. Then let u = sec x, so du = sec x tan x dx. We substitute and obtain
Z Z
5
tan x sec x dx = sec4 x sec x tan x dx
Z
= u4 du
1 5
= u +C
5
1
= 5 sec5 x + C .

tan3/2 x sec4 x dx using trigonometric identities.


R
21. Integrate
The exponent of sec x is 4, a positive, even integer. Factor sec2 x from sec4 x and write the
rest of the integrand in terms of tangents. Use the identity sec2 x = 1 + tan2 x.
Z Z Z
tan3/2 x sec4 x dx = tan3/2 x sec2 x sec2 x dx = tan3/2 x 1 + tan2 x sec2 x dx.


Now use the substitution u = tan x and du = sec2 x dx.


Z Z Z
tan3/2 x sec4 x dx = tan3/2 x 1 + tan2 x sec2 x dx = u3/2 1 + u2 du.
 

Use algebraic manipulation to rewrite u3/2 1 + u2



in a form whose antiderivative is
recognizable: u3/2 1 + u2 = u3/2 + u7/2 .

7.2 Integrals Containing Trigonometric Functions 7-53

Then
2 2
Z Z  
tan3/2 x sec4 x dx = u3/2 + u7/2 du = u5/2 + u9/2 + C
5 9
2 2
= 5 tan5/2 x + 9 tan9/2 x + C .

tan5 x sec x dx using trigonometric identities.


R
22. Integrate
The exponent of tan x is 5, a positive, odd integer. Factor tan x sec x from tan5 x sec x and
write the rest of the integrand in terms of secants. Use the identity tan2 x = sec2 x − 1.
Z Z Z
5 4
2
tan x sec x dx = tan x tan x sec x dx = sec2 x − 1 tan x sec x dx.

Now use the substitution u = sec x and du = tan x sec x dx.


Z Z Z
2 2
tan5 x sec x dx = sec2 x − 1 tan x sec x dx = u2 − 1 du.
2
Use algebraic manipulation to rewrite u2 − 1 in a form whose antiderivative is recog-
2
nizable: u2 − 1 = u4 − 2u2 + 1.
Then
Z Z
5
2
tan x sec x dx = u2 − 1 du
Z
u4 − 2u2 + 1 du

=
1 5 2 3
= u − u +u+C
5 3
1
= 5 sec
5
x − 32 sec3 x + sec x + C .

3/2
tan3 x (sec x)
R
23. Integrate dx using trigonometric identities.
3/2
The exponent of tan x is 3, a positive, odd integer. Factor tan x sec x from tan3 x (sec x)
and write the rest of the integrand in terms of secants. Use the identity tan2 x = sec2 x − 1.
Z Z
3/2 1/2
tan x (sec x) dx = tan2 x (sec x) tan x sec x dx
3

Z
1/2
sec2 x − 1 (sec x) tan x sec x dx.

=

Now use the substitution u = sec x and du = tan x sec x dx.


Z Z Z
3 3/2 2 1/2
u2 − 1 u1/2 du.
 
tan x (sec x) dx = sec x − 1 (sec x) tan x sec x dx =

Use algebraic manipulation to rewrite u2 − 1 u1/2 in a form whose antiderivative is rec-




ognizable: u2 − 1 u1/2 = u5/2 − u1/2 .




Then
Z Z  
3/2
tan3 x (sec x) dx = u5/2 − u1/2 du
2 7/2 2 3/2
= u − u +C
7 3
2 7/2 3/2
= 7 (sec x) − 32 (sec x) +C .
7-54 Chapter 7 Techniques of Integration


sec4 x tan x dx using trigonometric identities.
R
24. Integrate
The exponent of sec x is 4, a positive, even integer. Factor sec2 x from sec4 x and write the
rest of the integrand in terms of tangents. Use the identity sec2 x = 1 + tan2 x.
√ √
Z Z Z
sec4 x tan x dx = sec2 x tan x sec2 x dx = 1 + tan2 x tan x sec2 x dx.
√

Now use the substitution u = tan x and du = sec2 x dx.



Z Z Z
sec4 x tan x dx = 1 + tan2 x tan x sec2 x dx = 1 + u2
√ √
u du.
√
Use algebraic manipulation to rewrite 1 + u2 u in a form whose antiderivative is recog-
√
nizable: 1 + u2 u = 1 + u2 u1/2 = u1/2 + u5/2 .


Then
√ 2 2
Z Z  
4
sec x tan x dx = u1/2 + u5/2 du = u3/2 + u7/2 + C
3 7
3/2
= 2
3 (tan x) + 72 (tan x)7/2 + C .

25. Factor out csc x cot x, and use the identity cot2 x = csc2 x − 1.
Z Z
cot3 x csc x dx = cot2 x csc x cot x dx
Z
csc2 x − 1 csc x cot x dx.

=

Let u = csc x, then du = − csc x cot x dx, so csc x cot x dx = −du. We substitute and
obtain
Z Z
cot3 x csc x dx = csc2 x − 1 csc x cot x dx


Z
u2 − 1 (−du)

=
Z
1 − u2 du

=
1
= u − u3 + C
3
1
= csc x − 3 csc3 x + C .

26. Let u = cot x, then du = − csc2 x dx, so csc2 x dx = −du. We substitute and obtain
Z Z
cot3 x csc2 x dx = u3 (−du)
Z
= − u3 du
1
= − u4 + C
4
= − 41 cot4 x + C .
7.2 Integrals Containing Trigonometric Functions 7-55

27. We use the identity sin A cos B = 21 (sin (A + B) + sin (A − B)) to obtain

1
Z Z
sin (3x) cos x dx = (sin (4x) + sin (2x)) dx
2
 
1 1 1
= − cos (4x) − cos (2x) + C
2 4 2
= − 18 cos (4x) − 1
4 cos (2x) + C .

28. We use the identity sin A cos B = 21 (sin (A + B) + sin (A − B)) to obtain

1
Z Z
sin x cos (3x) dx = (sin (4x) + sin (−2x)) dx
2
1
Z
= (sin (4x) − sin (2x)) dx
2
  
1 1 1
= − cos (4x) − − cos (2x) +C
2 4 2
= − 18 cos (4x) + 1
4 cos (2x) + C .

29. We use the identity cos A cos B = 21 (cos (A + B) + cos (A − B)) to obtain

1
Z Z
cos x cos (3x) dx = (cos (4x) + cos (−2x)) dx
2
1
Z
= (cos (4x) + cos (2x)) dx
2
 
1 1 1
= sin (4x) + sin (2x) + C
2 4 2
1 1
= 8 sin (4x) + 4 sin (2x) + C .

30. We use the identity cos A cos B = 21 (cos (A + B) + cos (A − B)) to obtain

1
Z Z
cos (2x) cos x dx = (cos (3x) + cos (x)) dx
2
 
1 1
= sin (3x) + sin x + C
2 3
1 1
= 6 sin (3x) + 2 sin x + C .

31. We use the identity sin A sin B = 21 (cos (A − B) − cos (A + B)) to obtain

1
Z Z
sin (2x) sin (4x) dx = (cos (−2x) − cos (6x)) dx
2
1
Z
= (cos (2x) − cos (6x)) dx
2
 
1 1 1
= sin (2x) − sin (6x) + C
2 2 6
1 1
= 4 sin (2x) − 12 sin (6x) + C .
7-56 Chapter 7 Techniques of Integration

32. We use the identity sin A sin B = 21 (cos (A − B) − cos (A + B)) to obtain
1
Z Z
sin (3x) sin (x) dx = (cos (3x − x) − cos (3x+x)) dx
2
1
Z
= (cos (2x) − cos (4x)) dx
2
 
1 1 1
= sin (2x) − sin (4x) + C
2 2 4
1 1
= 4 sin (2x) − 8 sin (4x) + C .

33. We use the identity sin A sin B = 21 (cos (A − B) − cos (A + B)) to obtain
Z π/2
1 π/2
Z
sin (2x) sin x dx = (cos (x) − cos (3x)) dx
0 2 0
 π/2
1 1
= sin x − sin (3x)
2 3
  0   
1 π 1 3π 1
= sin − sin − sin 0 − sin (3(0))
2 2 3 2 3
 
1 4
= −0
2 3
2
= 3 .

34. We use the identity cos A cos B = 12 (cos (A + B) + cos (A − B)) to obtain
Z π
1 π
Z
cos x cos (4x) dx = (cos (5x) + cos (−3x)) dx
0 2 0
Z π
1
= (cos (5x) + cos (3x)) dx
2 0
 π
1 1 1
= sin (5x) + sin (3x)
2 5 3 0
  
1 1 1 1 1
= sin (5π) + sin (3π) − sin (5(0)) + sin (3(0))
2 5 3 5 3
= 0.
R π/2
35. Integrate 0 sin2 x cos5 x dx using trigonometric identities.
The exponent of cos x is 5, a positive, odd integer. Factor cos x from sin2 x cos5 x and write
the rest of the integrand in terms of sines. Use the identity sin2 x = 1 − cos2 x.
Z π/2 Z π/2 Z π/2
2 5 2 4
2
sin x cos x dx = sin x cos x cos x dx = sin2 x 1 − sin2 x cos x dx.
0 0 0

Now use the substitution u = sin x and du = cos x dx. The lower limit of integration
becomes u = sin 0 = 0 and the upper limit of integration becomes u = sin π2 = 1. Therefore,
Z π/2 Z π/2 Z 1
2 5 2 2
2 2
sin x cos x dx = sin x 1 − sin x cos x dx = u2 1 − u2 du.
0 0 0

2 2
Use algebraic manipulation to rewrite u2 1 − u

in a form whose antiderivative is recog-
2
nizable: u2 1 − u2 = u2 − 2u4 + u6 .
7.2 Integrals Containing Trigonometric Functions 7-57

Then
Z 1 2
Z 1
u2 1 − u2 u2 − 2u4 + u6 du

du =
0 0
 1
1 3 1 5 1 7
= u −2· u + u
3 5 7 0
 
1 2 1 8
= − + − 0 = 105 .
3 5 7
R π/2
36. Integrate 0 sin3 x cos1/2 x dx using trigonometric identities.
The exponent of sin x is 3, a positive, odd integer. Factor sin x from sin3 x and write the
rest of the integrand in terms of cosines. Use the identity sin2 x = 1 − cos2 x.
Z π/2 Z π/2 Z π/2
3 1/2 2 1/2
1 − cos2 x cos1/2 x sin x dx.

sin x cos x dx = sin x cos x sin x dx =
0 0 0

Now use the substitution u = cos x and du = − sin x dx. Then sin x dx = −du. The lower
limit of integration becomes u = cos 0 = 1 and the upper limit of integration becomes
u = cos π2 = 0. Therefore,
Z π/2 Z π/2
sin3 x cos1/2 x dx = 1 − cos2 x cos1/2 x sin x dx

0 0
Z 0 Z 0
1 − u2 u1/2 (−du) = − 1 − u2 u1/2 du.
 
=
1 1
R0
R1
Using a property of integrals, − 1 1 − u2 u1/2 du = 0 1 − u2 u1/2 du.
 

Use algebraic manipulation to rewrite 1 − u2 u1/2 in a form whose antiderivative is rec-




ognizable: 1 − u2 u1/2 = u1/2 − u5/2 .




Then
Z 1 Z 1  
1 − u u1/2 du =
2
u1/2 − u5/2 du

0 0
 1
2 3/2 2 7/2
= u − u
3 7 0
 
2 2 8
= − − (0 − 0) = 21 .
3 7

sin3 x
R
37. Integrate cos2 x dx using trigonometric identities.
The exponent of sin x is 3, a positive, odd integer. Factor sin x from sin3 x and write the
rest of the integrand in terms of cosines.
sin3 x sin2 x 1 − cos2 x
Z Z Z
2
dx = 2
sin x dx = sin x dx.
cos x cos x cos2 x
Now use the substitution u = cos x and du = − sin x dx. Then sin x dx = −du and
sin3 x 1 − cos2 x 1 − u2 1 − u2
Z Z Z Z
dx = sin x dx = (−du) = − du.
cos2 x cos2 x u2 u2
1−u2
Use algebraic manipulation to rewrite u2 in a form whose antiderivative is recognizable:
1−u2 1 u2 −2
u2 = u2 − u2 = u − 1.
7-58 Chapter 7 Techniques of Integration

Then
sin3 x 1
Z Z
u−2 − 1 du = − −u−1 − u + C = + u + C
 
dx = −
cos2 x u
1
= cos x + cos x = sec x + cos x + C .

38. We rewrite the integral, and obtain


cos x 1 cos x
Z Z
dx = dx
sin4 x sin3 x sin x
Z
= csc3 x cot x dx
Z
= csc2 x (csc x cot x) dx.

Let u = csc x, then du = − csc x cot x dx, so csc x cot x dx = −du. We substitute and
obtain
cos x
Z Z
dx = u2 (−du)
sin4 x Z
= − u2 du
1
= − u3 + C
3
= − 31 csc3 x + C .

R π/3
39. Integrate 0 cos3 (3x) dx using trigonometric identities.
The exponent of cos (3x) is 3, a positive, odd integer. Factor cos (3x) from cos3 (3x) and
write the rest of the integrand in terms of sines. Use the identity cos2 (3x) = 1 − sin2 (3x).
Z π/3 Z π/3 Z π/3
cos3 (3x) dx = cos2 (3x) cos (3x) dx = 1 − sin2 (3x) cos (3x) dx.
 
0 0 0

du
Now use the substitution u = sin (3x) and du = 3 cos (3x) dx. Then cos (3x) dx = 3 .
The lower limit of integration becomes u = sin 0 = 0 and the upper limit of integration
becomes u = sin 3 π3 = sin (π) = 0.


Therefore,
π/3 π/3 0  du 0
1
Z Z Z Z
3 2 2
1 − u2 du.
  
cos (3x) dx = 1 − sin (3x) cos (3x) dx = 1−u =
0 0 0 3 3 0
R π/3 1 0
cos3 (3x) dx = 2
R 
Using a property of integrals, 0 3 0
1−u du = 0 .
R π/3
40. Integrate 0 sin5 (3x) dx using trigonometric identities.
The exponent of sin (3x) is 5, a positive, odd integer. Factor sin (3x) from sin5 (3x) and
write the rest of the integrand in terms of cosines. Use the identity sin2 (3x) = 1−cos2 (3x).
Z π/3 Z π/3 Z π/3 2
5 4
1 − cos2 (3x) sin (3x) dx.

sin (3x) dx = sin (3x) sin (3x) dx =
0 0 0
7.2 Integrals Containing Trigonometric Functions 7-59

Now use the substitution u = cos (3x) and du = −3 sin (3x) dx. Then sin x dx = − du
3 .
The lower limit of integration becomes u = cos 0 = 1 and the upper limit of integration
becomes u = cos 3 · π3 = cos (π) = −1.


Therefore,
Z π/3 Z π/3 2
5
1 − cos2 (3x) [sin (3x) dx]

sin (3x) dx =
0 0
−1
1 −1
 
du
Z Z
2 2
2
1 − u2 du.

= 1−u − =−
1 3 3 1
R −1 2 R1 2
Using a property of integrals, − 31 1 1 − u2 du = 1
3 −1 1 − u2 du.
2 2

Use algebraic manipulation to rewrite 1 − u in a form whose antiderivative is recog-
2
nizable: 1 − u2 = 1 − 2u2 + u4 .
Then
π/3 1
1
Z Z
5
2
sin (3x) dx = 1 − u2 du
0 3 −1
Z 1
1
1 − 2u2 + u4 du

=
3 −1
 1
1 2 3 1 5
= u− u + u
3 3 5 −1
   
1 2 1 2 1 16
= 1− + − −1 + − = 45 .
3 3 5 3 5

41. Factor out sin x and use the identity sin2 x = 1 − cos2 x.
Z π Z π
3 5
sin x cos x dx = sin2 x cos5 x sin x dx
0 0
Z π
1 − cos2 x cos5 x sin x dx.

=
0

Let u = cos x, then du = − sin x dx, so sin x dx = −du. The lower limit of integration is
u = cos 0 = 1, and the upper limit of integration is u = cos π = −1. We substitute and
obtain
Z π Z −1
3 5
1 − u2 u5 (−du)

sin x cos x dx =
0 1
Z 1
u5 − u7 du

=
−1

= 0,

since we are integrating an odd function on a symmetric interval about 0.


42. Factor out sin x and use the identity sin2 x = 1 − cos2 x.
Z π/2 Z π/2
sin3 x cos3 x dx = sin2 x cos3 x sin x dx
0 0
Z π/2
1 − cos2 x cos3 x sin x dx.

=
0
7-60 Chapter 7 Techniques of Integration

Let u = cos x, then du = − sin x dx, so sin x dx = −du. The lower limit of integration is
u = cos 0 = 1, and the upper limit of integration is u = cos π2 = 0. We substitute and
obtain
Z π/2 Z 0
sin3 x cos3 x dx = 1 − u2 u3 (−du)

0 1
Z 1
u3 − u5 du

=
0
 1
1 4 1 6
= u − u
4 6
 0  
1 4 1 6 1 4 1 6
= 1 − 1 − 0 − 0
4 6 4 6
1
= 12 .

43. Rewrite the integral, factor out sin x, and use the identity sin2 x = 1 − cos2 x to obtain

sin3 x
Z Z
3
tan x dx = dx
cos3 x
sin2 x
Z
= sin x dx
cos3 x
1 − cos2 x
Z
= sin x dx.
cos3 x
Let u = cos x, then du = − sin x dx, so sin x dx = −du. We substitute and obtain

1 − u2
Z Z
tan3 x dx = (−du)
u3
Z  
1 −3
= −u du
u
u−2
= ln |u| − +C
−2
1
= ln |cos x| + 2 sec2 x + C .

44. Rewrite the integral, factor out cos x, and use the identity cos2 x = 1 − sin2 x to obtain

cos5 x
Z Z
cot5 x dx = dx
sin5 x
cos4 x
Z
= cos x dx
sin5 x
2
cos2 x
Z
= cos x dx
sin5 x
2
1 − sin2 x
Z
= cos x dx
sin5 x
7.2 Integrals Containing Trigonometric Functions 7-61

Let u = sin x, then du = cos x dx. We substitute and obtain


2
1 − u2
Z Z
5
cot x dx = du
u5
1 − 2u2 + u4
Z
= du
u5
Z  
1
= u−5 − 2u−3 + du
u
1
= − u−4 + u−2 + ln |u| + C
4
= − 41 csc4 x + csc2 x + ln |sin x| + C .

45. Factor out sec2 x and use the identity sec2 x = 1 + tan2 x.
sec6 x sec4 x
Z Z
3 dx = sec2 x dx
tan x tan3 x
2
sec2 x
Z
= sec2 x dx
tan3 x
2
1 + tan2 x
Z
= sec2 x dx.
tan3 x
Let u = tan x, then du = sec2 x dx. We substitute and obtain
2
sec6 x 1 + u2
Z Z
dx = du
tan3 x u3
1 + 2u2 + u4
Z
= du
u3
Z  
−3 2
= u + + u du
u
u−2 1
= + 2 ln |u| + u2 + C
−2 2
= − 12 cot2 x + 2 ln |tan x| + 1
2 tan2 x + C .

46. Let u = tan x, then du = sec2 x dx. We substitute and obtain


Z Z
tan1/2 x sec2 x dx = u1/2 du
2 3/2
= u +C
3
2
= 3 tan3/2 x + C .

47. Integrate csc4 x cot3 x dx using trigonometric identities.


R

Notice that the exponent of csc x is 4 and the exponent of cot x is 3. So this integral can
be found in two different ways, which will give two different (but equivalent) answers.
One solution:
The exponent of csc x is 4, a positive, even integer. Factor csc2 x from csc4 x cot3 x and
write the rest of the integrand in terms of cotangents. Use the identity csc2 x = 1 + cot2 x.
Z Z Z
4 3 2 3 2
1 + cot2 x cot3 x csc2 x dx.

csc x cot x dx = csc x cot x csc x dx =
7-62 Chapter 7 Techniques of Integration

Now use the substitution u = cot x and du = − csc2 x dx. Then csc2 x dx = −du and
Z Z
csc4 x cot3 x dx = 1 + cot2 x cot3 x csc2 x dx
 

Z Z
2
 3
1 + u2 u3 du.

= 1 + u u (−du) = −

to rewrite 1 + u2 u3 in a form whose antiderivative is recog-



Use algebraic manipulation
nizable: 1 + u2 u3 = u3 + u5 .


Then
 
1 1
Z Z
csc4 x cot3 x dx = − u3 + u5 du = − u4 + u6 + C = − 41 cot4 x − 61 cot6 x + C .

4 6

Another solution:
The exponent of cot x is 3, a positive, odd integer. Factor csc x cot x from csc4 x cot3 x and
write the rest of the integrand in terms of cosecants. Use the identity cot2 x = csc2 x − 1.
Z Z Z
csc x cot x dx = csc x cot x csc x cot x dx = csc3 x csc2 x − 1 csc x cot x dx.
4 3 3 2


Now use the substitution u = csc x and du = − csc x cot x dx. Then csc x cot x dx = −du
and
Z Z Z Z
csc x cot x dx = csc x csc x − 1 csc x cot x dx = u u − 1 (−du) = u3 1 − u2 du.
4 3 3 2 3 2
  

to rewrite u3 1 − u2 in a form whose antiderivative is recog-



Use algebraic manipulation
nizable: u3 1 − u2 = u3 − u5 .


Then
1 1
Z Z
csc4 cot3 x dx = u3 − u5 du = u4 − u6 + C = 1 4
x − 61 csc6 x + C .

4 6 4 csc

The two answers are equivalent, because


1 1 1 2 1 3
csc4 x − csc6 x + C1 = csc2 x − csc2 x + C1
4 6 4 6
1 2 1 3
= 1 + cot x − 1 + cot2 x + C1
2
4 6
1  1
= 1 + 2 cot x + cot4 x − 1 + 3 cot2 x + 3 cot4 x + cot6 x + C1
2

4 6
1 1 1 1 1 1 1
= + cot x + cot x − − cot2 x − cot4 x − cot6 x + C1
2 4
4 2
 4  6 2 2  6  
1 2 1 2 1 1 1 1 1
= cot x − cot x + cot x − cot x − cot6 x +
4 4
− + C1
2 2 4 2 6 4 6
1 1
= − cot4 x − cot6 x + C2 .
4 6

cot3 x csc2 x dx using trigonometric identities.


R
48. Integrate
Notice that the exponent of cot x is 3 and the exponent of csc x is 2. So this integral can
be found in two different ways, which will give two different (but equivalent) answers.
One solution:
The exponent of csc x is 2, a positive, even integer.
7.2 Integrals Containing Trigonometric Functions 7-63

Use the substitution u = cot x and du = − csc2 x dx. Then csc2 x dx = −du and
1
Z Z Z
cot3 x csc2 x dx = u3 (−du) = − u3 du = − u4 + C = − 14 cot4 x + C .
4
Another solution:
The exponent of cot x is 3, a positive, odd integer.
Factor csc x cot x from cot3 x csc2 x and write the rest of the integrand in terms of cosecants.
Use the identity cot2 x = csc2 x − 1.
Z Z Z
3 2 2
csc2 x − 1 csc x cot x dx.

cot x csc x dx = cot x csc x · csc x cot x dx =

Now use the substitution u = csc x and du = − csc x tan x dx. Then csc x tan x dx = −du
and
Z Z Z Z
cot3 x csc2 x dx = csc2 −1 csc x · csc x cot x dx = u2 − 1 u (−du) = 1 − u2 u du
  

to rewrite 1 − u2 u in a form whose antiderivative is recog-



Use algebraic manipulation
nizable: 1 − u2 u = u − u3 .


Then
1 1
Z Z
3 2
u − u3 du = u2 − u4 + C = 1 2
x − 41 csc4 x + C .

cot x csc x dx = 2 csc
2 4
The two answers are equivalent, because
1 1 2 1 2 1
− cot4 x + C1 = − cot2 x + C1 = − csc2 x − 1 + C1 = − csc4 x − 2 csc2 x + 1 + C1

4 4 4
  4
1 1 1 1 1
= − csc4 x + csc2 x + C1 − = csc2 x − csc2 x + C2 .
4 2 4 2 4

49. Factor out csc (2x) cot (2x) to obtain


Z Z
cot (2x) csc (2x) dx = csc3 (2x) csc (2x) cot (2x) dx.
4

Let u = csc (2x), then du = −2 csc (2x) cot (2x) dx, so csc x cot x dx = − du
2 We substitute
and obtain
 
du
Z Z
4 3
cot (2x) csc (2x) dx = u −
2
1
Z
= − u3 du
2
 
1 1 4
= − u +C
2 4
= − 81 csc4 (2x) + C .

50. We first use the identity cot2 (2x) = csc2 (2x) − 1.


Z Z
cot2 (2x) csc3 (2x) dx = csc2 (2x) − 1 csc3 (2x) dx


Z Z
= csc (2x) dx − csc3 (2x) dx.
5
7-64 Chapter 7 Techniques of Integration

We use integration by parts for csc5 (2x) dx with u = csc3 (2x) and dv = csc2 (2x) dx.
R

Then du = −6 csc3 (2x) cot (2x) dx and v = − 12 cot (2x). We obtain


1
Z Z
csc5 (2x) dx = − csc3 (2x) cot (2x) − 3 csc3 (2x) cot (2x) cot (2x) dx

2
1
Z
= − csc3 (2x) cot (2x) − 3 csc3 (2x) cot2 (2x) dx
2
1
Z
= − csc (2x) cot (2x) − 3 csc3 (2x) csc2 (2x) − 1 dx
3

2
1
Z Z
= − csc3 (2x) cot (2x) − 3 csc5 (2x) dx + 3 csc3 (2x) dx.
2
5
R
We add 3 csc (2x) dx to both sides, and divide by 4 and obtain
1
Z Z
4 csc (2x) dx = − csc (2x) cot (2x) + 3 csc3 (2x) dx
5 3
2
1 3
Z Z
csc5 (2x) dx = − csc3 (2x) cot (2x) + csc3 (2x) dx.
8 4
We now have
Z Z Z
cot2 (2x) csc3 (2x) dx = 5
csc (2x) dx − csc3 (2x) dx
1 3
Z Z
= − csc3 (2x) cot (2x) + csc3 (2x) dx − csc3 (2x) dx
8 4
1 1
Z
3
= − csc (2x) cot (2x) − csc3 (2x) dx.
8 4
We use integration by parts for csc3 (2x) dx with u = csc (2x) and dv = csc2 (2x) dx.
R

Then du = −2 csc (2x) cot (2x) dx and v = − 21 cot (2x). We obtain


1
Z Z
csc3 (2x) dx = − csc (2x) cot (2x) − (csc (2x) cot (2x)) cot (2x) dx
2
1
Z
= − csc (2x) cot (2x) − csc (2x) cot2 (2x) dx
2
1
Z
= − csc (2x) cot (2x) − csc (2x) csc2 (2x) − 1 dx

2
1
Z Z
= − csc (2x) cot (2x) − csc3 (2x) dx + csc (2x) dx
2
1 1
Z
= − csc (2x) cot (2x) − csc3 (2x) dx − ln |csc (2x) + cot (2x)| .
2 2
We add csc3 (2x) dx to both sides, and divide by 2 and obtain
R

1 1
Z
2 csc3 (2x) dx = − csc (2x) cot (2x) − ln |csc (2x) + cot (2x)|
2 2
1 1
Z
csc3 (2x) dx = − csc (2x) cot (2x) − ln |csc (2x) + cot (2x)| + C.
4 4
We now have
1  − csc (2x) cot (2x)
Z 
− csc3 (2x) cot (2x) ln|csc (2x)+cot (2x)|
cot2 (2x) csc3 (2x) dx = 8 − 4 − 4 +C
4
3
= − csc (2x) cot (2x)
8 + csc (2x) cot (2x)
16 + ln|csc (2x)+cot (2x)|
16 +C .
7.2 Integrals Containing Trigonometric Functions 7-65

51. We rewrite the integral using the identity tan2 x = sec2 x − 1.


Z π/4 Z π/4 2
tan4 x sec3 x dx = tan2 x sec3 x dx
0 0
Z π/4 2
= sec2 x − 1 sec3 x dx
0
Z π/4
sec4 x − 2 sec2 x + 1 sec3 x dx

=
0
Z π/4 Z π/4 Z π/4
= sec7 x dx − 2 sec5 x dx + sec3 x dx.
0 0 0

3 1
R
From Example 7 we have sec x dx = 2 [sec x tan x + ln |sec x + tan x|] + C, so
π/4
1
Z
sec3 x dx [sec x tan x + ln |sec x + tan x|]π/4
= 0
0 2
1 h π π π π i
= sec tan + ln sec + tan − (sec 0 tan 0 + ln |sec 0 + tan 0|)

2 4 4 √ 4 4
1  √  2
= ln 2+1 + .
2 2
R π/4
We use integration by parts for 0 sec5 x dx with u = sec3 x and dv = sec2 x dx. Then
du = 3 sec3 x tan x dx and v = tan x. We obtain
Z π/4 Z π/4
π/4
sec5 x dx = sec3 x tan x 0 − 3 sec3 x tan x tan x dx
 
0 0
Z π/4
3 π π 3
sec3 x tan2 x dx

= sec tan − sec 0 tan 0 − 3
4 4 0
√ Z π/4
sec3 x sec2 x − 1 dx

= 2 2−3
0
Z π/4 Z π/4 !
√ 5 3
= 2 2−3 sec x dx − sec x dx
0 0
 √2
!
√ π/4
1 √
Z
5
= 2 2−3 sec x dx + 3 ln 2+1 +
0 2 2
π/4
3 √  7√ Z
= ln 2+1 + 2−3 sec5 x dx.
2 2 0
R π/4
We add 3 0 sec5 x dx to each side and divide by 4 to obtain
π/4
3 √ 7√
Z 
4 sec5 x dx = ln 2+1 + 2
0 2 2
π/4
3 √ 7√
Z 
sec5 x dx = ln 2+1 + 2.
0 8 8
7-66 Chapter 7 Techniques of Integration

R π/4
We use integration by parts for 0 sec7 x dx with u = sec5 x and dv = sec2 x dx. Then
du = 5 sec5 x tan x dx and v = tan x. We obtain
Z π/4 Z π/4
π/4
sec7 x dx = sec5 x tan x 0 − 5 sec5 x tan x tan x dx
 
0 0
Z π/4
π5 π 5
sec5 x tan2 x dx

= sec tan − sec 0 tan 0 − 5
4 4 0
√ Z π/4
sec5 x sec2 x − 1 dx

= 4 2−5
0
Z π/4 Z π/4 !
√ 7 5
= 4 2−5 sec x dx − sec x dx
0 0

√ π/4
3 √  7√ 
Z 
7
= 4 2−5 sec x dx + 5 ln 2+1 + 2
0 8 8
Z π/4
15 √  67 √
= ln 2+1 + 2−5 sec7 x dx.
8 8 0
R π/4
We add 5 0 sec7 x dx to each side and divide by 6 to obtain
π/4
15 √ 67 √
Z 
6 sec7 x dx = ln 2+1 + 2
0 8 8
π/4 √
5 67 √
Z 
sec7 x dx = ln 2+1 + 2.
0 16 48

We now obtain
Z π/4 Z π/4 Z π/4
Z π/4
tan4 x sec3 x dx = sec7 x dx − 2
sec5 x dx + sec3 x dx
0
0 √
0
 67 √ 
0
3 √  7√ 

5
= ln 2+1 + 2 −2 ln 2+1 + 2
16 48 8 8
 √2
!
1 √
+ ln 2+1 +
2 2
7
√ 1
√ 
= 48 2 + 16 ln 2+1 .

52. We rewrite the integral using the identity tan2 x = sec2 x − 1.


Z π/4 Z π/4
tan2 x sec x dx sec2 x − 1 sec x dx

=
0 0
Z π/4 Z π/4
= sec3 x dx − sec x dx
0 0
Z π/4
π/4
= sec3 x dx − [ln |sec x + tan x|]0
0
π/4
π π
Z h i
= sec3 x dx − ln sec + tan − ln |sec 0 + tan 0|

0 4 4
Z π/4 √ 
= sec3 x dx − ln 2+1
0
7.2 Integrals Containing Trigonometric Functions 7-67

1
sec3 x dx =
R
From Example 8 we have 2 [sec x tan x + ln |sec x + tan x|] + C, so
π/4
1
Z
π/4
sec3 x dx = [sec x tan x + ln |sec x + tan x|]0
0 2
1h π π π π i
= sec tan + ln sec + tan − (sec 0 tan 0 + ln |sec 0 + tan 0|)

2 4 4 4 4
1 √  √2
= ln 2+1 + .
2 2
We now obtain
Z π/4 Z π/4 √ 
2
tan x sec x dx = sec3 x dx − ln
2+1
0 0
 √2
!
1 √ √ 
= ln 2+1 + − ln 2+1
2 2
1√ 1 √ 
= 2 − ln 2+1 .
2 2
R π/2 x
 3x

53. Integrate 0 sin 2 cos 2 dx using the product-to-sum identity

2 sin A cos B = sin (A + B) + sin (A − B).


Then
π/2 x   π/2     
3x 1 x 3x x 3x
Z Z
sin cos dx = sin + + sin − dx
0 2 2 2 0 2 2 2 2
π/2
1
Z
= [sin (2x) + sin (−x)] dx
2 0
π/2
1
Z
= [sin (2x) − sin x] dx since sin (−x) = − sin x
2 0
π/2 π/2
1 1
Z Z
= sin (2x) dx − sin x dx
2 0 2 0
 π/2
1 1 1 π/2
= − cos (2x) − [− cos (x)]0
2 2 0 2
1 1h π  i
= − [cos (π) − cos (0)] + cos − cos (0)
4 2 2
1 1
= − (−1 − 1) + (0 − 1) = 0 .
4 2
7-68 Chapter 7 Techniques of Integration

R π/4 R π/4
54. Integrate 0 cos (−x) sin (4x) dx = 0 sin (4x) cos (−x) dx using the product-to-sum
identity 2 sin A cos B = sin (A + B) + sin (A − B).
Then
Z π/4 π/4
1
Z
sin (4x) cos (−x) dx = sin [4x + (−x)] cos [4x − (−x)] dx
0 2 0
π/4
1
Z
= [sin (3x) + sin (5x)] dx
2 0
π/4 π/4
1 1
Z Z
= sin (3x) dx + sin (5x) dx
2 0 2 0
 π/4  π/4
1 1 1 1
= − cos (3x) + − cos (5x)
2 3 2 5
   0   0  
1 3π 1 5π
= − cos − cos (0) − cos − cos (0)
6 4 10 4
" √ ! # " √ ! #
1 2 1 2 4 2

=− − −1 − − − 1 = 15 + 15 2.
6 2 10 2

55. We use the identity sin A sin B = 21 (cos (A − B) − cos (A + B)) to obtain
x  
3x 1
Z Z
sin sin dx = (cos (−x) − cos (2x)) dx
2 2 2
1
Z
= (cos (x) − cos (2x)) dx
2
 
1 1
= sin x − sin (2x) + C
2 2
1 1
= 2 sin x − 4 sin (2x) + C .

56. We use the identity cos A cos B = 12 (cos (A + B) + cos (A − B)) to obtain

1
Z Z
cos (πx) cos (3πx) dx = (cos (4πx) + cos (−2πx)) dx
2
1
Z
= (cos (4πx) + cos (2πx)) dx
2
 
1 1 1
= sin (4πx) + sin (2πx) + C
2 4π 2π
1 1
= 8π sin (4πx) + 4π sin (2πx) + C .
7.2 Integrals Containing Trigonometric Functions 7-69

Rπ 2
57. The volume is given by the integral 0 π(sin x) dx. We write sin2 x = 1−cos2 (2x) and obtain
Z π Z π  
2 1 − cos (2x)
π(sin x) dx = π dx
0 0 2
Z π
π
= (1 − cos (2x)) dx
2 0
 π
π 1
= x − sin (2x)
2 2 0
  
π 1 1
= π − sin (2π) − 0 − sin (2(0))
2 2 2
1 2
= 2π .

R π/4 h 2 2
i
58. The volume is given by the integral 0 π (cos x) − (sin x) dx. We write cos2 x −
sin2 x = cos (2x) and obtain
Z π/4 h i Z π/4
π (cos x)2 − (sin x)2 dx = π cos (2x) dx
0 0
Z π/4
= π cos (2x) dx
0
 π/4
1
= π sin (2x)
2 0
 
1   π  1
= π sin 2 − sin (2(0))
2 4 2
1
= 2π .

59. Using the method of disks, the volume is given by


Z π/2 Z π/2 h i2 Z π/2
2 3/2
V =π [f (x)] dx = π sin x(cos x) dx = π sin2 x cos3 x dx.
0 0 0

The exponent of cos x is 3, a positive, odd integer. Factor cos x from cos3 x and write the
rest of the integrand in terms of sines. Use the identity cos2 x = 1 − sin2 x.
Z π/2 Z π/2 Z π/2
sin2 x cos3 x dx = sin2 x cos2 x cos x dx = sin2 x 1 − sin2 x cos x dx.

0 0 0

Now use the substitution u = sin x and du = cos x dx. The lower limit of integration
becomes u = sin 0 = 0 and the upper limit of integration becomes u = sin π2 = 1. Therefore,
Z π/2 Z π/2 Z 1
2 3 2 2
u2 1 − u2 du.
 
sin x cos x dx = sin x 1 − sin x cos x dx =
0 0 0

to rewrite u2 1 − u 2

Use algebraic manipulation in a form whose antiderivative is recog-
nizable: u 1 − u = u − u4 .
2 2 2


Then
Z 1 1  1  
1 3 1 5 1 1 2
Z
2 2 2 4
 
u 1−u du = u −u du = u − u = − − (0 − 0) = .
0 0 3 5 0 3 5 15
7-70 Chapter 7 Techniques of Integration

The volume of the solid is


Z π/2 Z 1
2 3
u2 1 − u2 du = 2π

V =π sin x cos x dx = π 15 .
0 0

60. Using the method of disks, the volume is given by


Z π/2 Z π/2 h i2 Z π/2
2
V =π [f (x)] dx = π (sin x)3/2 cos2 x dx = π sin3 x cos4 x dx.
0 0 0

The exponent of sin x is 3, a positive, odd integer. Factor sin x from sin3 x and write the
rest of the integrand in terms of cosines. Use the identity sin2 x = 1 − cos2 x.
Z π/2 Z π/2 Z π/2
3 4 2 4
1 − cos2 x cos4 x sin x dx.

sin x cos x dx = sin x cos x sin x dx =
0 0 0
Now use the substitution u = cos x and du = − sin x dx. Then sin x dx = −du. The lower
limit of integration becomes u = cos 0 = 1 and the upper limit of integration becomes
u = cos π2 = 0. Therefore,
Z π/2 Z π/2
3 4
1 − cos2 x cos4 x sin x dx

sin x cos x dx =
0 0
Z 0 Z 0
1 − u u4 (−du) = −
2
1 − u2 u4 du.
 
=
1 1
R0 R1
Using a property of integrals, − 1 1 − u u4 du = 0 1 − u2 u4 du.
2
 

2
 4
Use algebraic manipulation to rewrite 1 − u u in a form whose antiderivative is recog-
2
 4 4 6
nizable: 1 − u u = u − u .
Then
Z 1 1  1  
1 5 1 7 1 1 2
Z
1 − u u4 du =
2 4 6
 
u −u du = u − u = − − (0 − 0) = .
0 0 5 7 0 5 7 35
The volume of the solid is
Z π/2 Z 1
sin3 x cos4 x dx = π u2 − u6 du = 2π

V =π 35 .
0 0

1
Rπ 4
61. (a) The average value is given by π−0 0 sin x cos x dx. Let u = cos x, then du =
− sin x dx, so sin x dx = −du. The lower limit of integration is u = cos 0 = 1, and the
upper limit of integration is u = cos π = −1. We substitute and obtain
Z π
1 1 −1 4
Z
sin x cos4 x dx = u (−du)
π−0 0 π 1
1 1 4
Z
= u du
π −1
2 1 4
Z
= u du
π 0
using that we have an even function on an interval symmetric about 0. So
Z π  1
1 4 2 1 5
sin x cos x dx = u
π−0 0 π 5 0
 
2 1 5 1 5
= 1 − 0
π 5 5
2
= 5π .
7.2 Integrals Containing Trigonometric Functions 7-71

(b) The area under the graph of the function f is the same as the area of the rectangle
2
with height 5π ≈ 0.1273 and width π − 0 = π.
(c)

0.3

0.2 f(x)

0.1 f,average

0.0
0 1 2 3
x

62. (a) The average value of f (x) = sin (4x) cos (2x) over the interval 0, π2 is
 

π/2
1
Z
f¯ = π sin (4x) cos (2x) dx.
2 −0 0

Using the product-to-sum identity 2 sin A cos B = sin (A + B) + sin (A − B),


Z π/2
1
f¯ = π sin (4x) cos (2x) dx
2 −0 0
2 1 π/2
Z
= · [sin (4x + 2x) + sin (4x − 2x)] dx
π 2 0
2 1 π/2
Z
= · [sin (6x) + sin (2x)] dx
π 2 0
"Z #
π/2 Z π/2
1
= sin (6x) dx + sin (2x) dx
π 0 0
( π/2  π/2 )
1 1 1
= − cos (6x) + − cos (2x)
π 6 0 2 0
 i
1 1 h  π  i 1 h  π
= − cos 6 · − cos (0) − cos 2 · − cos (0)
π 6 2 2 2
 
1 1 1 4
= − [−1 − 1] − [−1 − 1] = 3π .
π 6 2

(b) The area enclosed by a rectangle of height f¯ = 4


3π and width π
2 equals the area under
the graph of f from 0 to π2 .
(c) The graph of f (x) = sin (4x) cos (2x) and the average value, f¯ = 4
3π , are shown below.
7-72 Chapter 7 Techniques of Integration

63. The net displacement of the object from t = 0 to t = 2π seconds is given by


Z 2π Z 2π
v(t) dt = sin2 t cos2 t dt.
0 0

Use the identity sin2 θ = 12 [1 − cos (2θ)] and cos2 θ = 21 [1 + cos (2θ)].
Then
2π 2π
1 1
Z Z
v (t) dt = [1 − cos (2t)] · [1 + cos (2t)] dt
0 0 2 2
Z 2π
1
1 − cos2 (2t) dt
 
=
4 0
1 2π
 
1
Z
= 1 − [1 + cos (4t)] dt
4 0 2
Z 2π
1
= [1 − cos (4t)] dt
8 0
 2π
1 1
= t − sin (4t)
8 4 0
   
1 1 1
= (2π) − sin (8π) − (0) − sin (0)
8 4 4
π
= 4 ≈ 0.785 .

After moving from t = 0 to t = 2π seconds, the object is approximately 0.785 meters to


the right of where it was at t = 0.
64. The velocity is given by
Z t
v(t) = v(0) + a(w) dw
0
Z t
= 5+ cos2 w sin w dw.
0
7.2 Integrals Containing Trigonometric Functions 7-73

Let u = cos w, then du = − sin w dw, so sin w dw = −du. The lower limit of integration is
u = cos 0 = 1, and the upper limit of integration is u = cos t. We substitute and obtain
Z cos t
v(t) = 5 + u2 (−du)
1
Z cos t
= 5− u2 du
1
cos t
1 3
= 5− u
3 0
 
1 3 1 3
= 5− cos t − cos 0
3 3
16 1
= − cos3 t.
3 3
The distance is given by
Z t
s(t) = s(0) + v(w) dw
0
Z t  
16 1 3
= 0+ − cos w dw
0 3 3
16 1 t
Z
= t− cos3 w dw.
3 3 0

We factor out cos w and use the identity cos2 w = 1 − sin2 w. Let u = sin w, then du =
cos w dw, the lower limit of integration is u = sin 0 = 0, and the upper limit of integration
is u = sin t. We substitute and obtain
16 1 t
Z
s(t) = t− cos2 w cos w dw
3 3 0
16 1 t
Z
1 − sin2 w cos w dw

= t−
3 3 0
16 1 sin t
Z
1 − u2 du

= t−
3 3 0
 sin t
16 1 1
= t− u − u3
3 3 3 0
  
16 1 1 3 1 3
= t− sin t − sin t − sin 0 − sin 0
3 3 3 3
= 16
3 t − 1
3 sin t + 1
9 sin3 t .

65. (a) The region in the first quadrant bounded by the graph of y = tan x and the lines
x = 0 and y = 1 is pictured below. The graph of y = tan x intersects with the line
y = 1 when tan x = 1; that is, when x = π4 . Since the region is above the x-axis,
R π/4
0
(1 − tan x) dx is the area of the desired region.
7-74 Chapter 7 Techniques of Integration

R π/4 R π/4 sin x


To evaluate 0 tan x dx = 0 cos x dx, use the substitution u = cos x and du =
− sin x dx. Then sin x dx = −du. The lower limit of integration √
becomes u = cos 0 = 1
π 2
R π/4
and the upper limit of integration becomes u = cos 4 = 2 . So, 0 tan x dx =
R π/4 1 √
R 2/2 1 √
R 2/2 1
0 cos x sin x dx = 1 u (−du) = − 1 u du.
Using a property of integrals,
Z √2/2 Z 1 √ ! √ ! √ 
1 1 1

2 2
− du = √ du = [ln |u|] 2/2 = ln (1)−ln = − ln = ln 2 .
1 u 2/2 u 2 2

Therefore, the area is


Z π/4 Z π/4 Z π/4 √ 
π
A= (1 − tan x) dx = 1 dx − tan x dx = 4 − ln 2 .
0 0 0

(b) Using the method of washers, the volume is given by


!
Z π/4 n o Z π/4 Z π/4
2 2 2
V =π 1 − [f (x)] dx = π 1 dx − tan x dx .
0 0 0

R π/4
Use the identity tan2 x = sec2 x − 1 to evaluate 0
tan2 x dx.
Z π/4 Z π/4
tan2 x dx = sec2 x − 1 dx

0 0
π/4
= [tan x − x]0
h π π i
= tan − − (tan 0 − 0)
4 4
π
=1−
4
Z π/4 Z π/4 !
hπ  π i 2
So, V = π 1 dx − tan2 x dx = π − 1− = π2 − π .
0 0 4 4
7.2 Integrals Containing Trigonometric Functions 7-75

π π
 R π/2
66. (a) Since y = cot x is nonnegative on 4, 2 , π/4 cot x dx is the area under the graph of
y = cot x from x = π4 to x = π2 .

R π/2 R π/2 cos x


To evaluate π/4
cot x dx = π/4 sin x
dx, use the substitution u = sin x and du =

2
cos x dx. The lower limit of integration becomes u = sin π4 = 2 and the upper limit
of integration becomes u = sin π2 = 1. So,

π/2 π/2 1
√ !
1 1 2
Z Z Z
1
cot x dx = cos x dx = √ du = [ln |u|]√2/2 = ln (1) − ln
π/4 π/4 sin x 2/2 u 2
√ 
= ln 2 = 21 ln 2 .

(b) Using the method of disks, the volume is given by


Z π/2 Z π/2
V =π [f (x)]2 dx = π cot2 x dx.
π/4 π/4

Use the identity cot2 x = csc2 x − 1.


Z π/2 Z π/2
cot2 x dx = π csc2 x − 1 dx

V =π
π/4 π/4
π/2
= π[− cot x − x]π/4
h π π  π π i
= π − cot − − − cot −
2 2  4 4
 π π
=π 0− +1+
2 4
π

= π 1− 4 .

67. (a) Use the identity sin2 x = 1−cos2 (2x) and obtain
Z Z
2
sin4 x dx = sin2 x dx
Z  2
1 − cos (2x)
= dx
2
1
Z
1 − 2 cos (2x) + cos2 (2x) dx

=
4
1 1 1
Z Z Z
= dx − cos (2x) dx + cos2 (2x) dx
4 2 4
1 1 1
Z
= x − sin (2x) + cos2 (2x) dx.
4 4 4
7-76 Chapter 7 Techniques of Integration

We evaluate
1 + cos (4x)
Z Z
cos2 (2x) dx = dx
2
1
Z
= (1 + cos (4x)) dx
2
x sin (4x)
= + + C.
2 8
We now obtain
 
1 1 1 x sin (4x)
Z
4
cos x dx = x − sin (2x) + + +C
4 4 4 2 8
3 1 1
= 8x − 4 sin (2x) + 32 sin (4x) + C .

n−1
sinn x dx = − sin x cos x n−1
sinn−2 x dx with n = 4 to obtain
R R
(b) Use n + n

sin3 x cos x 4 − 1
Z Z
sin4 x dx = − + sin4−2 x dx
4 4
sin3 x cos x 3
Z
= − + sin2 x dx.
4 4
And again with n = 2,

sin3 x cos x 3 sin2−1 x cos x 2 − 1


Z  Z 
4 2−2
sin x dx = − + − + sin x dx
4 4 2 2
sin3 x cos x 3 sin x cos x 3
Z
= − − + dx
4 8 8
3
= − sin x cos x
4 − 3 sin x cos x
8 + 38 x + C .

(c) We have
3 1 1 3 1 1
x − sin (2x) + sin (4x) = x − (2 sin x cos x) + (2 sin (2x) cos (2x))
8 4 32 8 4 32
3 1 1
x − sin x cos x + (2 sin x cos x) 1 − 2 sin2 x

=
8 2 16
3 1 1 1
= x − sin x cos x + sin x cos x − sin3 x cos x
8 2 8 4
sin3 x cos x 3 sin x cos x 3
= − −− + x.
4 8 8
So the two antiderivatives are equal.
(d) Using a Computer Algebra System, we obtain

3 1 1
Z
sin4 x dx = x − sin (2x) + sin (4x) + C,
8 4 32

which agrees with the result obtained in part (a).


7.2 Integrals Containing Trigonometric Functions 7-77

68. (a)
1.0
n=5

0.5

n=50
0.0
0 1 2 3
x
(b) Using a Computer Algebra System, we find
Z π Z π Z π Z π
sin5 x dx ≈ 1. 067, sin10 x dx ≈ 0.773, sin20 x dx ≈ 0.554, and sin50 x dx ≈ 0.353.
0 0 0 0

(c) The shape of the graph appears to contract towards x = π2 , becoming very narrow,
as n → ∞.
Rπ Rπ
(d) We continue to evaluate, and obtain 0 sin100 x dx ≈ 0.250, 0 sin1000 x dx ≈ 0.0792,

and 0 sin10000 x dx ≈ 0.0251. We surmise that
Z π
lim sinn x dx = 0 .
n→∞ 0

69. We use the identity sin A sin B = 21 (cos (A − B) − cos (A + B)) to obtain

1
Z Z
sin (mx) sin (nx) dx = (cos ((m − n)x) − cos ((m + n)x)) dx
2
 
1 sin ((m − n)x) sin ((m + n)x)
= − +C
2 m−n m+n
sin ((m−n)x) sin ((m+n)x)
= 2(m−n) − 2(m+n) +C .

70. We use the identity sin A cos B = 12 (sin (A + B) + sin (A − B)) to obtain

1
Z Z
sin (mx) cos (nx) dx = (sin ((m + n)x) + sin ((m − n)x)) dx
2
 
1 cos ((m + n)x) cos ((m − n)x)
= − − +C
2 m+n m−n

= − cos2(m+n)
((m+n)x)
− cos ((m−n)x)
2(m−n) +C .

71. We use the identity cos A cos B = 21 (cos (A + B) + cos (A − B)) to obtain

1
Z Z
cos (mx) cos (nx) dx = (cos ((m + n)x) + cos ((m − n)x)) dx
2
 
1 sin ((m + n)x) sin ((m − n)x)
= + +C
2 m+n m−n
sin ((m+n)x) sin ((m−n)x)
= 2(m+n) + 2(m−n) +C .
7-78 Chapter 7 Techniques of Integration

72. (a) Let u = sin x, then du = cos x dx. We substitute and obtain
Z Z
sin x cos x dx = u du
1 2
= u + C1
2
1
= sin2 x + C1 .
2
So a function is f (x) = 1
2 sin2 x .
(b) Let u = cos x, then du = − sin x dx, so sin x dx = −du. We substitute and obtain
Z Z
sin x cos x dx = u (−du)
1
= − u2 + C2
2
1
= − cos2 x + C2 .
2
So a function is g(x) = − 21 cos2 x .
the identity sin (2x) = 2 sin x cos x we have sin x cos x = 21 sin (2x). So sin x cos x dx =
R
(c) From
1
sin (2x) dx. Let u = 2x, then du = 2 dx, so dx = 12 du. We substitute and obtain
R
2
1
Z Z
sin x cos x dx = sin (2x) dx
2
 
1 1
Z
= sin u du
2 2
1
Z
= sin u du
4
1
= (− cos u) + C3
4
1
= − cos (2x) + C3 .
4
So the function is h(x) = − 41 cos (2x) .
1
sin2 x − − 21 cos2 x =
sin2 x + cos2 x = 12 .
1
 
(d) We have f (x) − g(x) = 2 2
(e) We have f (x) − h(x) = 21 sin2 x − − 41 cos (2x) = 21 sin2 x + 14 2 cos2 x − 1 =
 
1 2 2 1 1 1 1

2 sin x + cos x − 4 = 2 − 4 = 4 .

Challenge Problems

73. Let x = sin y, so x = sin2 y, and dx = 2 sin y cos y dy. The lower limit of integration is
y = sin−1 0 = 0, and the upper limit of integration is y = sin−1 √12 = π4 . We substitute
and obtain
Z 1/2 √ Z π/4
x sin y
√ dx = p (2 sin y cos y) dy
0 1 − x 0 1 − sin2 y
Z π/4
sin y
= p (2 sin y cos y) dy
0 cos2 y
Z π/4
sin y
= (2 sin y cos y) dy
0 cos y
Z π/4
= 2 sin2 y dy.
0
7.2 Integrals Containing Trigonometric Functions 7-79

1−cos (2y)
Use the identity sin2 y = 2 and obtain
1/2 √ π/4
x 1 − cos (2y)
Z Z
√ dx = 2 dy
0 1−x 0 2
Z π/4
= (1 − cos (2y)) dy
0
 π/4
1
= y − sin (2y)
2 0
   π   
π 1 1
= − sin 2 − 0 − sin (2(0))
4 2 4 2
1 1
= 4π − 2 .

74. Let u = π2 −θ, then du = −dθ, so dθ = −du. Also, θ = π2 −u. The lower limit of integration
is u = π2 − 0 = π2 , and the upper limit of integration is u = π2 − π2 = 0. We substitute and
obtain
Z π/2 Z 0 π 
n
sin θ dθ = sinn − u (−du)
0 π/2 2
Z 0 h 
π in
= − sin −u du
π/2 2
Z π/2
n
= [cos u] du
0
Z π/2
= cosn θ dθ
0

π

where we used the identity sin 2 − x = cos x.
3/2 3 3
75. (a) When writing cos2 x = (cos x) , a mistake is made. When π2 < x ≤ π, (cos x) <
3/2 3/2 3
0, but cos2 x > 0. The correct equality would be cos2 x
 
= |cos x| .
1+cos (2x)
(b) Use the identity cos2 x = 2 and obtain
Z π Z π 2
4
cos x dx = cos2 x dx
0 0
π  2
1 + cos (2x)
Z
= dx
0 2
1 π
Z
1 + 2 cos (2x) + cos2 (2x) dx

=
4 0
1 π 1 π 1 π
Z Z Z
= dx + cos (2x) dx + cos2 (2x) dx
4 0 2 0 4 0
 π  π
1 1 1 π
Z
= x + sin (2x) + cos2 (2x) dx
4 0 4 0 4 0
1 π
   
1 1 1 1
Z
= π − (0) + sin (2π) − sin (2(0)) + cos2 (2x) dx
4 4 4 4 4 0
π 1 π
Z
= + cos2 (2x) dx.
4 4 0
7-80 Chapter 7 Techniques of Integration

We evaluate
π π
1 + cos (4x)
Z Z
cos2 (2x) dx = dx
0 0 2
1 π
Z
= (1 + cos (4x)) dx
2
 0 π
x sin (4x)
= +
2 8 0
 
π sin (4π) 0 sin (4(0))
= + − +
2 8 2 8
π
= .
2
We now obtain π
π 1π 
Z
cos4 x dx = + = 3π
8 .
0 4 4 2

R
AP Practice Problems
1. Integrate sin3 x dx using trigonometric identities.
R

The exponent of sin x is 3, a positive, odd integer. Factor sin x from sin3 x and write the
rest of the integrand in terms of cosines.
Z Z Z
sin3 x dx = sin2 x sin x dx = 1 − cos2 x sin x dx.


Now use the substitution u = cos x and du = − sin x dx. Then sin x dx = −du and
Z Z Z Z
3 2 2
1 − u2 du
  
sin x dx = 1 − cos x sin x dx = 1 − u (−du) = −
   
1 3 1 3 3
= − u − u + C = − cos x − cos x + C = − cos x + cos3 x + C .
3 3

The answer is C.

2. The average value of f (x) = cos2 x
on the interval [0, π] if f¯ = 1
cos2 x
 
2 π−0 0 2 dx.

Use the identity cos2 θ = 12 [1 + cos (2θ)].

1 π 2 x 1 π1 1 1
Z   Z
¯ π 1
f= cos dx = [1 + cos x] dx = [x + sin x]0 = [(π + 0) − (0 + 0)] = 2 .
π 0 2 π 0 2 2π 2π

The answer is D.
R
3. To evaluate sin x cos (2x) dx, use the product-to-sum identity 2 sin A cos B = sin (A + B)+
sin (A − B).
1 1
Z Z Z
sin x cos (2x) dx = [sin (x + 2x) + sin (x − 2x)] dx = [sin (3x) + sin (−x)] dx
2 2
Since sin (−x) = − sin x,
1
Z Z
1
 1 
sin x cos (2x) dx = [sin (3x) − sin x] dx = 2 − 3 cos (3x) + cos x + C .
2

The answer is D.
7.2 Integrals Containing Trigonometric Functions 7-81

4. Using the method of disks, the volume is given by


Z π/4 Z π/4 2
Z π/4
V =π [f (x)]2 dx = π sec2 x dx = π sec4 x dx.
0 0 0

sec4 x dx using trigonometric identities.


R
Integrate

The exponent of sec x is 4, a positive, even integer. Factor sec2 x from sec4 x and write the
rest of the integrand in terms of tangents. Use the identity sec2 x = 1 + tan2 x.
Z π/4 Z π/4 Z π/4
sec4 x dx = π sec2 x sec2 x dx = π 1 + tan2 x sec2 x dx.

Then π
0 0 0

Now use the substitution u = tan x and du = sec2 x dx.


The lower limit of integration becomes u = tan 0 = 0 and the upper limit of integration
becomes u = tan π4 = 1.
Z π/4 Z π/4 Z 1
4
1 + tan x sec2 x dx = π
2
1 + u2 du
 
So, V = π sec x dx = π
0 0 0
3 1
   
u 1 4
=π u+ =π 1+ = 3π .
3 0 3

The answer is C.
R π/4
5. Integrate 0 tan3 x sec x dx using trigonometric identities.

The exponent of tan x is 3, a positive, odd integer. Factor tan x sec x from tan3 x sec x and
write the rest of the integrand in terms of secants. Use the identity tan2 x = sec2 x − 1.
Z π/4 Z π/4 Z π/4
3 2
sec2 x − 1 tan x sec x dx.

Then tan x sec x dx = tan x tan x sec x dx =
0 0 0

Now use the substitution u = sec x and du = tan x sec x dx.


The lower limit of integration
√ becomes u = sec 0 = 1 and the upper limit of integration
becomes u = sec π4 = 2.

Z π/4 Z π/4 Z 2
tan3 x sec x dx = sec2 x − 1 tan x sec x dx = u2 − 1 du
 
So,
0 0 1
√ 2 √ ! 
√ √ 
 3 
u 2 2 1
= −u = − 2 − − 1 = 13 2 − 2 .
3 1 3 3

The answer is C.
6. Integrate sin4 x cos3 x dx using trigonometric identities.
R

The exponent of cos x is 3, a positive, odd integer. Factor cos x from cos3 x and write the
rest of the integrand in terms of sines.
Z Z Z
sin x cos x dx = sin x cos x cos x dx = sin4 x 1 − sin2 x cos x dx.
4 3 4 2

7-82 Chapter 7 Techniques of Integration

Now use the substitution u = sin x and du = cos x dx.


Z Z Z
sin4 x cos3 x dx = sin4 x 1 − sin2 x cos x dx = u4 1 − u2 du.
 

to rewrite u4 1 − u2 in a form whose antiderivative is recog-



Use algebraic manipulation
nizable: u4 1 − u2 = u4 − u6 .


1 1
Z Z
sin4 x cos3 x dx = u4 − u6 du = u5 − u7 + C = 15 sin5 x − 71 sin7 x + C .

Then
5 7

The answer is C.

7.3 Integration Using Trigonometric Substitution


Concepts and Vocabulary
1. True.
2. (d), x = 4 tan θ
3. (c), x = 3 sec θ
5
4. (b), x = 2 sin θ

Skill Building
5. Let x = 2 sin θ, then dx = 2 cos θ dθ. We substitute and obtain
Z p Z q
2
4 − x2 dx = 4 − (2 sin θ) (2 cos θ) dθ
Z p
= 2 4 − 4 sin2 θ cos θ dθ
Z q
4 1 − sin2 θ cos θ dθ

= 2
Z √
= 2 4 cos2 θ cos θ dθ
Z
= 2 (2 cos θ) cos θ dθ
Z
= 4 cos2 θ dθ

1 + cos (2θ)
Z
= 4 dθ
2
Z
= 2 (1 + cos (2θ)) dθ
 
1
= 2 θ + sin (2θ) + C
2
= 2θ + sin (2θ) + C
= 2θ + 2 sin θ cos θ + C.
7.3 Integration Using Trigonometric Substitution 7-83

p q
2 √
We have θ = sin−1 x
, and cos θ = 1 − sin2 θ = 1 − (x/2) = 12 4 − x2 . We obtain

2
Z p x  x  1 p 
4 − x2 dx = 2 sin−1 +2 4 − x2 + C
2 2 2

2 sin−1 x 1

= 2
2 + 2x 4 − x + C .

6. Let x = 4 sin θ, then dx = 4 cos θ dθ. We substitute and obtain


Z p Z q
2
16 − x dx = 16 − (4 sin θ)2 (4 cos θ) dθ
Z p
= 4 16 − 16 sin2 θ cos θ dθ
Z
= 4 (4 cos θ) cos θ dθ
Z
= 16 cos2 θ dθ

1 + cos (2θ)
Z
= 16 dθ
2
Z
= 8 (1 + cos (2θ)) dθ
 
1
= 8 θ + sin (2θ) + C
2
= 8θ + 4 sin (2θ) + C
= 8θ + 8 sin θ cos θ + C.
p q
2 √
We have θ = sin−1 x4 , and cos θ = 1 − sin2 θ = 1 − (x/4) = 14 16 − x2 . We obtain


Z p x  x  1 p 
−1
16 − x2 dx = 8 sin +8 2
16 − x + C
4 4 4
 1 √
= 8 sin−1 x 2
4 + 2 x 16 − x + C .
7-84 Chapter 7 Techniques of Integration

7. Let x = 4 sin θ, then dx = 4 cos θ dθ. We substitute and obtain


2
x2 (4 sin θ)
Z Z
√ dx = q (4 cos θ) dθ
16 − x2 16 − (4 sin θ)
2

sin2 θ
Z
= 64 p cos θ dθ
16 − 16 sin2 θ
sin2 θ
Z
= 64 q  cos θ dθ
16 1 − sin2 θ
sin2 θ
Z
= 64 √ cos θ dθ
16 cos2 θ
sin2 θ
Z
= 16 cos θ dθ
cos θ
Z
= 16 sin2 θ dθ

1 − cos (2θ)
Z
= 16 dθ
2
Z
= 8 (1 − cos (2θ)) dθ
 
1
= 8 θ − sin 2θ + C
2
= 8θ − 8 sin θ cos θ + C.
p q
2 √
We have θ = sin−1 x 2
1 − (x/4) = 14 16 − x2 . We obtain

4 , and cos θ = 1 − sin θ =

x2  x  1 p
Z x 
−1 2
√ dx = 8 sin −8 16 − x + C
16 − x2 4 4 4
 1 √
= 8 sin−1 x 2
4 − 2 x 16 − x + C .
7.3 Integration Using Trigonometric Substitution 7-85

8. Let x = 6 sin θ, then dx = 6 cos θ dθ. We substitute and obtain


2
x2 (6 sin θ)
Z Z
√ dx = q (6 cos θ) dθ
36 − x2 36 − (6 sin θ)
2

sin2 θ
Z
= 216 p cos θ dθ
36 − 36 sin2 θ
sin2 θ
Z
= 216 q  cos θ dθ
36 1 − sin2 θ
sin2 θ
Z
= 216 √ cos θ dθ
36 cos2 θ
sin2 θ
Z
= 36 cos θ dθ
cos θ
Z
= 36 sin2 θ dθ

1 − cos (2θ)
Z
= 36 dθ
2
Z
= 18 (1 − cos (2θ)) dθ
 
1
= 18 θ − sin 2θ + C
2
= 18θ − 18 sin θ cos θ + C.
p q √
−1 x
1 − sin θ = 1 − (x/6)2 = 16 36 − x2 . We obtain
2

We have θ = sin 6 , and cos θ =

x2  x  1 p
Z x 
√ dx = 18 sin−1 − 18 36 − x2 + C
36 − x2 6 6 6

= 18 sin−1 x6 − 21 x 36 − x2 + C .


9. Let x = 2 sin θ, then dx = 2 cos θ dθ. We substitute and obtain


Z √
q
2
4 − x2
Z 4 − (2 sin θ)
dx = 2 (2 cos θ) dθ
x2 (2 sin θ)
Z p
1 4 − 4 sin2 θ
= cos θ dθ
2 sin2 θ
q
4 1 − sin2 θ

1
Z
= cos θ dθ
2 sin2 θ

1 4 cos2 θ
Z
= cos θ dθ
2 sin2 θ
cos θ cos θ
Z
= 2 dθ
Z sin θ
= cot2 θ dθ
Z
csc2 θ − 1 dθ

=
= − cot θ − θ + C.
7-86 Chapter 7 Techniques of Integration

p q
2 √
We have θ = sin−1 x
, and cos θ = 1 − sin2 θ = 1 − (x/2) = 12 4 − x2 . So cot θ =

√ 2
1 √
4−x2 4−x2
2
x/2 = x . We obtain
Z √ √
4 − x2 2
dx = − 4−x − sin−1 x

2 x 2 +C .
x

10. Let x = 3 sin θ, then dx = 3 cos θ dθ. We substitute and obtain

Z √
q
2
9−x 2
Z 9 − (3 sin θ)
dx = 2 (3 cos θ) dθ
x2 (3 sin θ)
Z p
1 9 − 9 sin2 θ
= cos θ dθ
3 sin2 θ
q
9 1 − sin2 θ

1
Z
= cos θ dθ
3 sin2 θ
Z √
1 9 cos2 θ
= cos θ dθ
3 sin2 θ
cos θ cos θ
Z
= dθ
sin2 θ
Z
= cot2 θ dθ
Z
csc2 θ − 1 dθ

=
= − cot θ − θ + C.
p q
2 √
We have θ = sin−1 x 2
1 − (x/3) = 13 9 − x2 . So cot θ =

3 , and cos θ = 1 − sin θ =
1
√ √
9−x2 9−x2
3
x/3 = x . We obtain
Z √
9 − x2 √
9−x2
− sin−1 x

2
dx = − x 3 +C .
x
7.3 Integration Using Trigonometric Substitution 7-87

11. Let x = 2 sin θ, then dx = 2 cos θ dθ. We substitute and obtain


Z p Z q
2 2
x 4 − x dx = (2 sin θ) 4 − (2 sin θ)2 (2 cos θ) dθ
2

Z
p
= 8 sin2 θ 4 − 4 sin2 θ cos θ dθ
Z
q
sin2 θ 4 1 − sin2 θ cos θ dθ

= 8
√
Z
= 8 sin2 θ 4 cos2 θ cos θ dθ
Z
sin2 θ (2 cos θ) cos θ dθ

= 8
Z
= 16 sin2 θ cos2 θ dθ
Z  2
sin 2θ
= 16 dθ
2
Z
= 4 sin2 (2θ) dθ

1 − cos (4θ)
Z
= 4 dθ
2
Z
= 2 (1 − cos (4θ)) dθ
 
1
= 2 θ − sin (4θ) + C
4
1
= 2θ − sin (4θ) + C
2
= 2θ − sin 2θ cos 2θ + C
= 2θ − (2 sin θ cos θ) 1 − 2 sin2 θ + C.


p q
2 √
We have θ = sin−1 x
, and cos θ = 1 − sin2 θ = 1 − (x/2) = 12 4 − x2 . We obtain

2
Z p x  x  1 p   x 2 
x2 4 − x2 dx = 2 sin−1 −2 4 − x2 1−2 +C
2 2 2 2
x(x −2)
2 √
4 − x2 + 2 sin−1 x

= 4 2 +C .
7-88 Chapter 7 Techniques of Integration

12. Let x = 4 sin θ, then dx = 4 cos θ dθ. We substitute and obtain


Z p Z q
2 2
x 16 − x dx = (4 sin θ) 16 − (4 sin θ)2 (4 cos θ) dθ
2

Z
p
= 64 sin2 θ 16 − 16 sin2 θ cos θ dθ
Z
q
sin2 θ 16 1 − sin2 θ cos θ dθ

= 64
√
Z
= 64 sin2 θ 16 cos2 θ cos θ dθ
Z
sin2 θ (4 cos θ) cos θ dθ

= 64
Z
= 256 sin2 θ cos2 θ dθ
Z  2
sin 2θ
= 256 dθ
2
Z
= 64 sin2 (2θ) dθ

1 − cos (4θ)
Z
= 64 dθ
2
Z
= 32 (1 − cos (4θ)) dθ
 
1
= 32 θ − sin (4θ) + C
4
= 32θ − 8 sin (4θ) + C
= 32θ − 16 sin 2θ cos 2θ + C
32θ − 16(2 sin θ cos θ) 1 − 2 sin2 θ + C.

=
p q
2 √
We have θ = sin−1 x
, and cos θ = 1 − sin2 θ = 1 − (x/4) = 14 16 − x2 . We obtain

4
Z p x  x  1 p   x 2 
x2 16 − x2 dx = 32 sin−1 − 32 16 − x2 1−2 +C
4 4 4 4
 1 √
32 sin−1 x 2

= 2
4 − 4 x 16 − x 8 − x +C .

13. Let x = 2 sin θ, then dx = 2 cos θ dθ. We substitute and obtain


1 1
Z Z
3/2
dx = 3/2 (2 cos θ) dθ
2
(4 − x ) 4 − 4 sin2 θ
1
Z
= 2 3/2
cos θ dθ
4 (cos2 θ)
3/2

1 cos θ
Z
= dθ
4 cos3 θ
1
Z
= sec2 θ dθ
4
1
= tan θ + C.
4
7.3 Integration Using Trigonometric Substitution 7-89

p q
2 √
We have sin θ = x
2, and cos θ = 1 − sin2 θ = 1 − (x/2) = 1
2 4 − x2 . So tan θ =
x/2
1
√ = √ x . We obtain
2 4−x2 4−x2

1 1
Z
3/2
dx = tan θ + C
(4 − x2 ) 4
= √x +C .
4 4−x2

14. Let x = sin θ, then dx = cos θ dθ. We substitute and obtain


1 1
Z Z
3/2
dx = 3/2 (cos θ) dθ
2
(1 − x ) 1 − sin2 θ
1
Z
= 3/2
cos θ dθ
2
(cos θ)
cos θ
Z
= dθ
cos3 θ
Z
= sec2 θ dθ
= tan θ + C.
p √
We have sin θ = x, and cos θ = 1 − sin2 θ = 1 − x2 . So tan θ = √ x
1−x2
. We obtain

1
Z
dx = √ x +C .
3/2 1−x2
(1 − x2 )

15. Let x = 2 tan θ, then dx = 2 sec2 θ dθ. We substitute and obtain


Z p Z q
4 + (2 tan θ)2 2 sec2 θ dθ

4 + x2 dx =
Z p
= 2 4 + 4 tan2 θ sec2 θ dθ
Z
= 2 (2 sec θ) sec2 θ dθ
Z
= 4 sec3 θ dθ
 
1 1
= 4 sec θ tan θ + ln |sec θ + tan θ| + C
2 2
= 2 sec θ tan θ + 2 ln |sec θ + tan θ| + C.
√ q
2 √
We have tan θ = x2 , and sec θ = 1 + tan2 θ = 1 + (x/2) = 12 4 + x2 . We obtain
 p   p  x 
1 x 1
Z p
4 + x2 dx = 2 4 + x2 + 2 ln 4 + x2 + +C
2 2 2 2
1
√ √
4+x2 +x

= 2
2 x 4 + x + 2 ln 2 +C
7-90 Chapter 7 Techniques of Integration

16. Let x = tan θ, then dx = sec2 θ dθ. We substitute and obtain


Z p Z q
1 + (tan θ)2 sec2 θ dθ

2
1 + x dx =
Z
= (sec θ) sec2 θ dθ
Z
= sec3 θ dθ
1 1
= sec θ tan θ + ln |sec θ + tan θ| + C.
2 2
√ √
We have tan θ = x, and sec θ = 1 + tan2 θ = 1 + x2 . We obtain
1p 1 p
Z p
1 + x2 dx = 1 + x2 (x) + ln 1 + x2 + x + C

2 2
√ √
= 12 x 1 + x2 + 21 ln 1 + x2 + x + C .

17. Let x = 4 tan θ, then dx = 4 sec2 θ dθ. We substitute and obtain


dx 1
Z Z
4 sec2 θ dθ

√ = q
2
x + 16 (4 tan θ)2 + 16
1
Z
sec2 θ dθ

= 4 √ 2
16 tan θ + 16
1
Z
sec2 θ dθ

= 4 q
16 tan2 θ + 1


1
Z
sec2 θ dθ

= 4 √
2
16 sec θ
1
Z
sec2 θ dθ

=
sec θ
Z
= sec θ dθ

=ln |sec θ + tan θ| + C.


√ q
2 √
We have tan θ = x4 , and sec θ = tan2 θ + 1 = (x/4) + 1 = 14 x2 + 16. We obtain
p  x 
dx 1
Z
√ = ln
2
x + 16 + +C
2
x + 16 4 4

2
= ln x +16+x +C .

4
7.3 Integration Using Trigonometric Substitution 7-91

18. Let x = 5 tan θ, then dx = 5 sec2 θ dθ. We substitute and obtain


dx 1
Z Z
5 sec2 θ dθ

√ = q
2
x + 25 2
(5 tan θ) + 25
1
Z
sec2 θ dθ

= 5 √ 2
25 tan θ + 25
1
Z
sec2 θ dθ

= 5 q
25 tan2 θ + 1


1
Z
sec2 θ dθ

= 5 √
2
25 sec θ
1
Z
sec2 θ dθ

=
sec θ
Z
= sec θ dθ

=
ln |sec θ + tan θ| + C.
√ q √
We have tan θ = 5 , and sec θ = tan θ + 1 = (x/5)2 + 1 = 51 x2 + 25. We obtain
x 2

p  x 
dx 1
Z
√ = ln x2 + 25 + +C
x2 + 25 5 5

2
= ln x +25+x +C .

5

1 1
19. Let x = 3 tan θ, then dx = 3 sec2 θ dθ. We substitute and obtain
s  2  
1 1
Z p Z
1 + 9x2 dx = 1+9 tan θ sec2 θ dθ
3 3
1 p
Z
= 1 + tan2 θ sec2 θ dθ
3
1
Z
= (sec θ) sec2 θ dθ
3
1
Z
= sec3 θ dθ
3
 
1 1 1
= sec θ tan θ + ln |sec θ + tan θ| + C
3 2 2
1 1
= sec θ tan θ + ln |sec θ + tan θ| + C.
6 6
√ q
2 √
We have tan θ = 3x, and sec θ = 1 + tan2 θ = 1 + (3x) = 1 + 9x2 . We obtain

1 p 1 p
Z p 
1 + 9x2 dx = 1 + 9x2 (3x) + ln 1 + 9x2 + (3x) + C

6 6
1
√ 1
√ 
= 2 1 + 9x2 + 3x + C .
2 x 1 + 9x + 6 ln
7-92 Chapter 7 Techniques of Integration

3 3
20. Let x = 2 tan θ, then dx =sec2 θ dθ. We substitute and obtain
2
s  2  
3 3
Z p Z
2 2
9 + 4x dx = 9+4 tan θ sec θ dθ
2 2
3 p
Z
= 9 + 9 tan2 θ sec2 θ dθ
2
3
Z q
9 1 + tan2 θ sec2 θ dθ

=
2
3 √
Z
= 9 sec2 θ sec2 θ dθ
2
3
Z
= (3 sec θ) sec2 θ dθ
2
9
Z
= sec3 θ dθ
2
 
9 1 1
= sec θ tan θ + ln |sec θ + tan θ| + C
2 2 2
9 9
= sec θ tan θ + ln |sec θ + tan θ| + C.
4 4
√ q 2 √
We have tan θ = 2x
3 , and sec θ = 1 + tan2 θ = 1 + 2x 3 = 31 9 + 4x2 . We obtain
  
9 1p 2x 9 1 p 2x
Z p
2
9 + 4x dx = 9 + 4x 2 + ln 2
9 + 4x + +C
4 3 3 4 3 3
√ √
2 +2x

= 12 x 9 + 4x2 + 49 ln 9+4x +C .

3

2 2
21. Let x = 3 tan θ, then dx = sec2 θ dθ. We substitute and obtain
3

2
2
x2 3 tan θ
 
2
Z Z
2
√ dx = q 2 3 sec θ dθ
4 + 9x2 4 + 9 23 tan θ
8 tan2 θ
Z
= √ sec2 θ dθ
27 4 + 4 tan2 θ
8 tan2 θ
Z
2
= q  sec θ dθ
27 4 1 + tan θ 2

8 tan2 θ
Z
= √ sec2 θ dθ
27 2
4 sec θ
4 tan2 θ
Z
= sec2 θ dθ
27 sec θ
4
Z
= tan2 θ sec θ dθ
27
4
Z
sec2 θ − 1 sec θ dθ

=
27
4
Z
sec3 θ − sec θ dθ

=
27
 
4 1 1
= sec θ tan θ + ln |sec θ + tan θ| − ln |sec θ + tan θ| + C
27 2 2
2 2
= sec θ tan θ − ln |sec θ + tan θ| + C.
27 27
7.3 Integration Using Trigonometric Substitution 7-93

3x
√ q
3x 2 1

1 + tan2 θ =

We have tan θ = 2 , and sec θ = 1+ 2 = 2 4 + 9x2 . We obtain

x2
   p
2 1p 3x 2 1 3x
Z
√ dx = 4 + 9x2 − ln 2
4 + 9x + +C
4 + 9x2 27 2 2 27 2 2
1
√ 2
√
4+9x2 +3x

= 2
18 x 4 + 9x − 27 ln 2 +C .

22. Let x = 4 tan θ, then dx = 4 sec2 θ dθ. We substitute and obtain

x2 (4 tan θ)2
Z Z
4 sec2 θ dθ

√ dx = q
x2 + 16 2
(4 tan θ) + 16
tan2 θ
Z
= 64 √ sec2 θ dθ
16 tan2 θ + 16
tan2 θ
Z
2
= 64 q  sec θ dθ
2
16 tan θ + 1
tan2 θ
Z
= 16 sec2 θ dθ
sec θ
Z
= 16 tan2 θ sec θ dθ
Z
sec2 θ − 1 sec θ dθ

= 16
Z
sec3 θ − sec θ dθ

= 16
 
1 1
= 16 sec θ tan θ + ln |sec θ + tan θ| − ln |sec θ + tan θ| + C
2 2
= 8 sec θ tan θ − 8 ln |sec θ + tan θ| + C.
√ q 
x 2

We have tan θ = x4 , and sec θ = tan2 θ + 1 = 4 + 1 = 41 x2 + 16. We obtain

x2
Z
√ dx = 8 sec θ tan θ − 8 ln |sec θ + tan θ| + C
2
x + 16
 p   p
1 2
x 1
2
x
= 8 x + 16 − 8 ln
x + 16 + + C
4 4 4 4
√ √
x2 +16+x

= 12 x x2 + 16 − 8 ln 4 +C .
7-94 Chapter 7 Techniques of Integration

23. Let x = 2 tan θ, then dx = 2 sec2 θ dθ. We substitute and obtain

dx 2 sec2 θ
Z Z
√ = q dθ
x2 x2 + 4 2 2
(2 tan θ) (2 tan θ) + 4
1 sec2 θ
Z
= √ dθ
2 tan2 θ 4 tan2 θ + 4
1 sec2 θ
Z
= q  dθ
2 tan2 θ 4 tan2 θ + 1
1 sec2 θ
Z
= dθ
4 tan2 θ sec θ
1 sec θ
Z
= dθ
4 tan2 θ
1 cos θ
Z
= dθ.
4 sin2 θ
Let u = sin θ, then du = cos θ dθ. We substitute and obtain
dx 1 1
Z Z
√ = du
x2 x2 + 4 4 u2
 
1 1
= − +C
4 u
1
= − csc θ + C.
4
√ q
2 2

We have tan θ = x2 , so cot θ = x2 , and csc θ = 1 + cot2 θ = 1 + 1

x = x x2 + 4. We
obtain
dx 1 1p 2
Z
√ = − x +4+C
2
x x +4 2 4x

x2 +4
= − 4x +C .

1 1
24. Let x = 2 tan θ, then dx = 2 sec2 θ dθ. We substitute and obtain
1 2
dx 2 sec θ
Z Z
√ = q dθ
x 4x2 + 1
2 1
tan θ
2
4 1
tan θ
2
+ 1
2 2
sec2 θ
Z
= 2 √ dθ
tan2 θ tan2 θ + 1
sec2 θ
Z
= 2 dθ
tan2 θ sec θ
sec θ
Z
= 2 dθ
tan2 θ
cos θ
Z
= 2 dθ.
sin2 θ
7.3 Integration Using Trigonometric Substitution 7-95

Let u = sin θ, then du = cos θ dθ. We substitute and obtain


dx 1
Z Z
√ = 2 du
x2 4x2 + 1 u2
 
1
= 2 − +C
u
= −2 csc θ + C.
1
√ q
1 2 1

, and csc θ = 1 + cot2 θ = 1 +

We have tan θ = 2x, so cot θ = 2x 2x = 2x 4x2 + 1.
We obtain
 p 
dx 1
Z
√ = −2 2
4x + 1 + C
x2 4x2 + 1 2x

4x2 +1
= − x +C .

25. Let x = 2 tan θ, then dx = 2 sec2 θ dθ. We substitute and obtain


dx 2 sec2 θ
Z Z
= 3/2 dθ
(x2 + 4)3/2

2
(2 tan θ) + 4
sec2 θ
Z
= 2 3/2 dθ
4 tan2 θ + 4
sec2 θ
Z
= 2 3/2 dθ
4 tan2 θ + 1
1 sec2 θ
Z
= dθ
4 sec3 θ
1
Z
= cos θ dθ
4
1
= sin θ + C.
4
√ q
2 2

We have tan θ = x2 , so cot θ = x2 , and csc θ = 1 + cot2 θ = 1 + 1

x = x x2 + 4. So
sin θ = √xx2 +4 and we obtain

dx 1 x
Z
3/2
= √ +C
(x2 + 4) 4 x2 + 4
= √x +C .
4 x2 +4

26. Let x = tan θ, then dx = sec2 θ dθ. We substitute and obtain


dx sec2 θ
Z Z
3/2
= 3/2 dθ
(x2 + 1)

2
(tan θ) + 1
sec2 θ
Z
= dθ
sec3 θ
Z
= cos θ dθ
= sin θ + C.
7-96 Chapter 7 Techniques of Integration

1
√ q
1 2 1

1 + cot2 θ = 1 +

We have tan θ = x, so cot θ = x, and csc θ = x = x x2 + 1. So
sin θ = √xx2 +1 and we obtain

dx
Z
= √ x +C .
3/2 x2 +1
(x2 + 1)

27. Let x = 5 sec θ, then dx = 5 sec θ tan θ dθ. We substitute and obtain
2
x2 (5 sec θ)
Z Z
√ dx = q (5 sec θ tan θ ) dθ
2
x − 25 2
(5 sec θ) − 25
sec2 θ
Z
= 125 √ sec θ tan θ dθ
25 sec2 θ − 25
sec2 θ
Z
= 25 √ sec θ tan θ dθ
sec2 θ − 1
sec2 θ
Z
= 25 sec θ tan θ dθ
tan θ
Z
= 25 sec3 θ dθ
 
1 1
= 25 sec θ tan θ + ln |sec θ + tan θ| + C
2 2
25 25
= sec θ tan θ + ln |sec θ + tan θ| + C.
2 2
√ q 
x 2

We have sec θ = x5 , so tan θ = sec2 θ − 1 = 5 − 1 = 51 x2 − 25. We obtain

x2
 
25  x  1 p 2 25 x 1 p 2
Z

√ dx = x − 25 + ln + x − 25 + C
x2 − 25 2 5 5 2 5 5
√ √
= 1
x x 2 − 25 + 25 ln x+ x2 −25 + C .
2 2 5

28. Let x = 4 sec θ, then dx = 4 sec θ tan θ dθ. We substitute and obtain

x2 (4 sec θ)2
Z Z
√ dx = q (4 sec θ tan θ ) dθ
x2 − 16 (4 sec θ)2 − 16
sec2 θ
Z
= 64 √ sec θ tan θ dθ
16 sec2 θ − 16
sec2 θ
Z
= 16 √ sec θ tan θ dθ
sec2 θ − 1
sec2 θ
Z
= 16 sec θ tan θ dθ
tan θ
Z
= 16 sec3 θ dθ
 
1 1
= 16 sec θ tan θ + ln |sec θ + tan θ| + C
2 2
= 8 sec θ tan θ + 8 ln |sec θ + tan θ| + C.
7.3 Integration Using Trigonometric Substitution 7-97

√ q
x 2

We have sec θ = x4 , so tan θ = 1

sec2 θ − 1 = 4 −1= 4 x2 − 16. We obtain

x2  x  1 p 
x 1p
Z

√ dx = 8 2
x − 16 + 8 ln + 2
x − 16 + C
x2 − 16 4 4 4 4
√ √
2

= 12 x x2 − 16 + 8 ln x+ x4 −16 + C .

29. Let x = sec θ, then dx = sec θ tan θ dθ. We substitute and obtain

Z √ 2
q
x −1
Z (sec θ)2 − 1
dx = (sec θ tan θ ) dθ
x sec θ
tan θ
Z
= sec θ tan θ dθ
sec θ
Z
= tan2 θ dθ
Z
sec2 θ − 1 dθ

=
= tan θ − θ + C.
√ √
We have θ = sec−1 x, so tan θ = sec2 θ − 1 = x2 − 1. We obtain
Z √ 2
x −1 √
dx = x2 − 1 − sec−1 x + C .
x

30. Let x = sec θ, then dx = sec θ tan θ dθ. We substitute and obtain

Z √ 2
q
x −1
Z (sec θ)2 − 1
dx = (sec θ tan θ ) dθ
x2 sec2 θ
tan θ
Z
= sec θ tan θ dθ
sec2 θ
tan2 θ
Z
= dθ
sec θ
sec2 θ − 1
Z
= dθ
sec θ
Z
= (sec θ − cos θ) dθ

= ln |sec θ + tan θ| − sin θ + C.


√ √ 1
We have sec θ = x, so tan θ = sec2 θ − 1 = x2 − 1. And cos θ = x, so sin θ =
q 2 √
2
1 − x1 = xx−1 . We obtain
Z √ 2
x −1 √ √
x2 −1
dx = ln x + x2 − 1 − x +C .
x
7-98 Chapter 7 Techniques of Integration

31. Let x = 6 sec θ, then dx = 6 sec θ tan θ dθ. We substitute and obtain
dx 6 sec θ tan θ
Z Z
√ = q dθ
2 2
x x − 36 2 2
(6 sec θ) (6 sec θ) − 36
1 sec θ tan θ
Z
= √ dθ
6 sec2 θ 36 sec2 θ − 36
1 tan θ
Z
= √ dθ
36 sec θ sec2 θ − 1
1 tan θ
Z
= dθ
36 sec θ tan θ
1
Z
= cos θ dθ
36
1
= sin θ + C.
36

x
√ q 
x 2

2
We have sec θ = 6 , so tan θ = sec θ − 1 = 6 − 1 = 61 x2 − 36. So sin θ = tan θ
sec θ =
1
√ √
x2 −36 x2 −36
6
x = x . We obtain
6

dx √
Z
1 x2 −36
√ = 36 x +C .
x x2 − 36
2

32. Let x = 3 sec θ, then dx = 3 sec θ tan θ dθ. We substitute and obtain
dx 3 sec θ tan θ
Z Z
√ = q dθ
2
x x −92 2 2
(3 sec θ) (3 sec θ) − 9
1 sec θ tan θ
Z
= √ dθ
3 sec θ 9 sec2 θ − 9
2

1 tan θ
Z
= √ dθ
9 sec θ sec2 θ − 1
1 tan θ
Z
= dθ
9 sec θ tan θ
1
Z
= cos θ dθ
9
1
= sin θ + C.
9
√ q 
x 2

We have sec θ = x3 , so tan θ = sec2 θ − 1 = 3 − 1 = 31 x2 − 9. So sin θ = tan θ
sec θ =
1
√ √
x2 −9 x2 −9
3
x = x . We obtain
3

dx √
Z
1 x2 −9
√ = 9 x +C .
x2 x2 − 9
7.3 Integration Using Trigonometric Substitution 7-99

3 3
33. Let x = 2 sec θ, then dx = 2 sec θ tan θ dθ. We substitute and obtain
3
dx sec θ tan θ
Z Z
√ q2 = dθ
4x2 − 9 3
2
4 2 sec θ − 9
3 sec θ tan θ
Z
= √ dθ
2 9 sec2 θ − 9
3 sec θ tan θ
Z
= = p dθ
2 9(sec2 θ − 1)
1 sec θ tan θ
Z
= dθ
2 tan θ
1
Z
= sec θ dθ
2
1
= ln |sec θ + tan θ| + C.
2
√ q
2x 2

We have sec θ = 2x − 1 = 13 4x2 − 9. We obtain

, so tan θ = sec 2θ−1=
3 3

dx 1 2x 1 p 2
Z

√ = ln + 4x − 9 + C
4x2 − 9 2 3 3

1 2x+ 4x2 −9
= 2 ln 3 +C .

2 2
34. Let x = 3 sec θ, then dx = 3 sec θ tan θ dθ. We substitute and obtain
2
dx sec θ tan θ
Z Z
√ q3 = dθ
9x2 − 4 2
9 32 sec θ − 4
2 sec θ tan θ
Z
= √ dθ
3 4 sec2 θ − 4
2 sec θ tan θ
Z
= p dθ
3 4(sec2 θ − 1)
1 sec θ tan θ
Z
= dθ
3 tan θ
1
Z
= sec θ dθ
3
1
= ln |sec θ + tan θ| + C.
3
√ q
3x 2

We have sec θ = 3x − 1 = 12 9x2 − 4. We obtain

, so tan θ = sec 2θ−1=
2 2

dx 1 3x 1 p 2
Z

√ = ln + 9x − 4 + C
9x2 − 4 3 2 2

1 3x+ 9x2 −4
= 3 ln 2 +C .
7-100 Chapter 7 Techniques of Integration

35. Let x = 3 sec θ, then dx = 3 sec θ tan θ dθ. We substitute and obtain
dx 3 sec θ tan θ
Z Z
3/2
= 3/2 dθ
2

(x − 9) 2
(3 sec θ) − 9
sec θ tan θ
Z
= 3 3/2

(9 sec2 θ − 9)
sec θ tan θ
Z
= 3 3/2

(9(sec2 θ − 1))
1 sec θ tan θ
Z
= dθ
9 tan3 θ
1 sec θ
Z
= dθ
9 tan2 θ
1 cos θ
Z
= dθ.
9 sin2 θ
Let u = sin θ, then du = cos θ dθ. We substitute and obtain
dx 1 du
Z Z
=
2
(x − 9)
3/2 9 u2
 
1 1
= − +C
9 u
1
= − csc θ + C.
9
√ q
x 2

We have sec θ = x3 , so tan θ = sec2 θ − 1 = − 1 = 13 x2 − 9. Then csc θ = sec θ

3 tan θ =
x/3
1
√ = √ x . We obtain
3 x2 −9 x2 −9

dx
Z
3/2
= − 19 √xx2 −9 + C .
(x2 − 9)

1 1
36. Let x = 5 sec θ, then dx = 5 sec θ tan θ dθ. We substitute and obtain
1
dx sec θ tan θ
Z Z
5
3/2
= 3/2 dθ
(25x2 − 1)
 2
1
25 5 sec θ − 1
1 sec θ tan θ
Z
= 3/2

5 (sec2 θ − 1)
1 sec θ tan θ
Z
= dθ
5 tan3 θ
1 sec θ
Z
= dθ
5 tan2 θ
1 cos θ
Z
= dθ.
5 sin2 θ
7.3 Integration Using Trigonometric Substitution 7-101

Let u = sin θ, then du = cos θ dθ. We substitute and obtain


dx 1 du
Z Z
=
2
(25x − 1) 3/2 5 u2
 
1 1
= − +C
5 u
1
= − csc θ + C.
5
√ q
2 √
We have sec θ = 5x, so tan θ = sec2 θ − 1 = (5x) − 1 = 25x2 − 1. Then csc θ =
sec θ √ 5x
tan θ = 25x2 −1
. We obtain

dx 1 5x
Z
3/2
= − √ +C
(25x2 − 1) 5 25x2 − 1
= − √25xx2 −1 + C .

37. Let x = 3 sec θ, then dx = 3 sec θ tan θ dθ. We substitute and obtain
2
x2 dx (3 sec θ) (3 sec θ tan θ)
Z Z
3/2
= 3/2 dθ
(x2 − 9)

2
(3 sec θ) − 9
sec3 θ tan θ
Z
= 27 3/2

(9 sec2 θ − 9)
sec3 θ tan θ
Z
= 27 3/2

(9(sec2 θ − 1))
sec3 θ tan θ
Z
= dθ
tan3 θ
sec3 θ
Z
= dθ
tan2 θ
sec θ tan2 θ + 1
Z 
= dθ
tan2 θ
Z  
sec θ
= sec θ + dθ
tan2 θ
cos θ
Z
= ln |sec θ + tan θ| + dθ.
sin2 θ
Let u = sin θ, then du = cos θ dθ. We substitute and obtain

x2 dx du
Z Z
= ln |sec θ + tan θ| +
(x2 − 9)
3/2 u2
 
1
= ln |sec θ + tan θ| + − +C
u
= ln |sec θ + tan θ| − csc θ + C.
7-102 Chapter 7 Techniques of Integration

√ q
x 2

We have sec θ = x3 , so tan θ = 1 sec θ

sec2 θ − 1 = 3 −1= 3 x2 − 9. Then csc θ = tan θ =
x/3
1
√ = √ x . We obtain
3 x2 −9 x2 −9

x2 dx

x 1p x
Z

= ln +
2
x − 9 − √ +C
3/2 3 3 2
(x2 − 9) x −9

2
= ln x+ 3x −9 − √xx2 −9 + C .

38. Let x = 2 sec θ, then dx = 2 sec θ tan θ dθ. We substitute and obtain

x2 dx (2 sec θ)2 (2 sec θ tan θ)


Z Z
3/2
= 3/2 dθ
(x2 − 4)

2
(2 sec θ) − 4
sec3 θ tan θ
Z
= 8 3/2

(4 sec2 θ − 4)
sec3 θ tan θ
Z
= 8 3/2

(4(sec2 θ − 1))
sec3 θ tan θ
Z
= dθ
tan3 θ
sec3 θ
Z
= dθ
tan2 θ
sec θ tan2 θ + 1
Z 
= dθ
tan2 θ
Z  
sec θ
= sec θ + dθ
tan2 θ
cos θ
Z
= ln |sec θ + tan θ| + dθ.
sin2 θ
Let u = sin θ, then du = cos θ dθ. We substitute and obtain

x2 dx du
Z Z
= ln |sec θ + tan θ| +
2
(x − 4)
3/2 u2
 
1
= ln |sec θ + tan θ| + − +C
u
= ln |sec θ + tan θ| − csc θ + C.
√ q 
x 2

We have sec θ = x2 , so tan θ = sec2 θ − 1 = 2 − 1 = 12 x2 − 4. Then csc θ = sec θ
tan θ =
x/2
1
√ = √ x . We obtain
2 x2 −4 x2 −4

x2 dx

x 1p x
Z

3/2
= ln + x2 − 4 − √ +C
(x2 − 4) 2 2 x2 − 4

2
= ln x+ 2x −4 − √xx2 −4 + C .

7.3 Integration Using Trigonometric Substitution 7-103

39. Let x = 4 tan θ, then dx = 4 sec2 θ dθ. We substitute and obtain


2
x2 (4 tan θ)
Z Z
4 sec2 θ dθ

dx = 2
16 + x2 16 + (4 tan θ)
tan2 θ
Z
= 64 sec2 θ dθ
16 + 16 tan2 θ
tan2 θ
Z
= 4 sec2 θ dθ
sec2 θ
Z
= 4 tan2 θ dθ
Z
sec2 θ − 1 dθ

= 4

= 4(tan θ − θ) + C
x x
= 4 − 4 tan−1 +C
4 4
x − 4 tan−1 x4 + C .

=

1 1
40. Let x = 4 tan θ, then dx = 4 sec2 θ dθ. We substitute and obtain

1
2
x2 tan θ
 
1
Z Z
dx = 4
2 sec2 θ dθ
1 + 16x2 1 + 16 14 tan θ 4
1 tan2 θ
Z
= sec2 θ dθ
64 1 + tan2 θ
1 tan2 θ
Z
= sec2 θ dθ
64 sec2 θ
1
Z
= tan2 θ dθ
64
1
Z
sec2 θ − 1 dθ

=
64
1
= (tan θ − θ) + C
64
1 1
= (4x) − tan−1 (4x) + C
64 64
x 1
= 16 − 64 tan−1 (4x) + C .
7-104 Chapter 7 Techniques of Integration

2 2
41. Let x = 5 sin θ, then dx = 5 cos θ dθ. We substitute and obtain
s  2  
2 2
Z p Z
4 − 25x2 dx = 4 − 25 sin θ cos θ dθ
5 5
2
Z p
= 4 − 4 sin2 θ cos θ dθ
5
2
Z q
4 1 − sin2 θ cos θ dθ

= =
5
4
Z
= cos2 θ dθ
5
4 1 + cos (2θ)
Z
= dθ
5 2
2
Z
= (1 + cos (2θ)) dθ
5
 
2 1
= θ + sin (2θ) + C
5 2
2 1
= θ + sin (2θ) + C
5 5
2 2
= θ + sin θ cos θ + C.
5 5
p q 2 √
We have θ = sin−1 5x 2
1 − 5x = 21 4 − 25x2 . We obtain

2 , and cos θ = 1 − sin θ = 2
    p 
2 5x 2 5x 1
Z p
4 − 25x2 dx = sin−1 + 2
4 − 25x + C
5 2 5 2 2
 1 √
= 2
5 sin−1 5x 2
2 + 2 x 4 − 25x + C .

3 3
42. Let x = 4 sin θ, then dx = 4 cos θ dθ. We substitute and obtain
s  2  
3 3
Z p Z
9 − 16x2 dx = 9 − 16 sin θ cos θ dθ
4 4
3 p
Z
= 9 − 9 sin2 θ cos θ dθ
4
3
Z q
9 1 − sin2 θ cos θ dθ

=
4
9
Z
= cos2 θ dθ
4
9 1 + cos (2θ)
Z
= dθ
4 2
9
Z
= (1 + cos (2θ)) dθ
8
 
9 1
= θ + sin (2θ) + C
8 2
9 9
= θ+ sin (2θ) + C
8 16
9 9
= θ + sin θ cos θ + C.
8 8
7.3 Integration Using Trigonometric Substitution 7-105

p q
4x 2

We have θ = sin−1 4x
, and cos θ = 1 − sin2 θ = 1 − 1
 
3 3 = 3 9 − 16x2 . We obtain
    p 
9 4x 9 4x 1
Z p
9 − 16x2 dx = sin−1 + 9 − 16x2 + C
8 3 8 3 3

9
sin−1 4x 1

= 2
8 3 + 2 x 9 − 16x + C ..

2 2
43. Let x = 5 sin θ, then dx = 5 cos θ dθ. We substitute and obtain
 
1 1 2
Z Z
3/2
dx = cos θ dθ
(4 − 25x2 )

2
2 3/2 5
4 − 25 5 sin θ
2 1
Z
= 3/2
cos θ dθ
43/2 (cos2 θ) 5
1 cos θ
Z
= dθ
20 cos3 θ
1
Z
= sec2 θ dθ
20
1
= tan θ + C.
20
p
2
q
5x 2

We have sin θ = 5x 1

2 , and cos θ = 1 − sin θ = 1− 2 = 2 4 − 25x2 . So tan θ =
5x/2
1
√ = √ 5x . We obtain
2 4−25x2 4−25x2
 
1 1 5x
Z
3/2
dx = √ +C
(4 − 25x2 ) 20 4 − 25x2
= √ x +C .
4 4−25x2

1 1
44. Let x = 3 sin θ, then dx = cos θ dθ. We substitute and obtain
3
 
1 1 1
Z Z
3/2
dx = cos θ dθ
(1 − 9x2 )
 2 3/2 3
1 − 9 13 sin θ
 
1 1
Z
= 3/2 3 cos θ dθ
1 − sin2 θ
1 1
Z
= 3/2
cos θ dθ
3 (cos2 θ)
1 cos θ
Z
= dθ
3 cos3 θ
1
Z
= sec2 θ dθ
3
1
= tan θ + C.
3
7-106 Chapter 7 Techniques of Integration

p q
2 √
We have sin θ = 3x, and cos θ = 1 − sin2 θ = 1 − (3x) = 1 − 9x2 . So tan θ =
√ 3x . We obtain
1−9x2
 
1 1 3x
Z
3/2
dx = √ +C
(1 − 9x2 ) 3 1 − 9x2
= √ x +C .
1−9x2

2 2
45. Let x = 5 tan θ, then dx =sec2 θ dθ. We substitute and obtain
5
s
 2  
2 2
Z p Z
4 + 25x2 dx = 4 + 25 tan θ sec2 θ dθ
5 5
2 p
Z
= 4 + 4 tan2 θ sec2 θ dθ
5
2
Z q
4 1 + tan2 θ sec2 θ dθ

=
5
2
Z
= (2 sec θ) sec2 θ dθ
5
4
Z
= sec3 θ dθ
5
 
4 1 1
= sec θ tan θ + ln |sec θ + tan θ| + C
5 2 2
2 2
= sec θ tan θ + ln |sec θ + tan θ| + C.
5 5
√ q 2 √
We have tan θ = 5x
2 , and sec θ = 1 + tan2 θ = 1 + 5x 2 = 21 4 + 25x2 . We obtain
  
2 1p 5x 2 1 p 5x
Z p
2
4 + 25x dx = 4 + 25x 2 + ln 2
4 + 25x + +C
5 2 2 5 2 2
√ √
4+25x2 +5x

= 12 x 4 + 25x2 + 52 ln 2 +C .

3 3
46. Let x = 4 sec2 θ dθ. We substitute and obtain
tan θ, then dx = 4
s  2  
3 3
Z p Z
2 2
9 + 16x dx = 9 + 16 tan θ sec θ dθ
4 4
3 p
Z
= 9 + 9 tan2 θ sec2 θ dθ
4
3
Z q
9 1 + tan2 θ sec2 θ dθ

=
4
3
Z
= (3 sec θ) sec2 θ dθ
4
9
Z
= sec3 θ dθ
4
 
9 1 1
= sec θ tan θ + ln |sec θ + tan θ| + C
4 2 2
9 9
= sec θ tan θ + ln |sec θ + tan θ| + C.
8 8
7.3 Integration Using Trigonometric Substitution 7-107

4x
√ q
4x 2 1

1 + tan2 θ =

We have tan θ = 3 , and sec θ = 1+ 3 = 3 9 + 16x2 . We obtain
  
9 1p 4x 9 1p 4x
Z p
9 + 16x2 dx = 9 + 16x2 + ln 9 + 16x2 + +C
8 3 3 8 3 3
1
√ 9
√
9+16x2 +4x

= 2
2 x 9 + 16x + 8 ln 3 +C .

47. Let x = 4 sec θ, then dx = 4 sec θ tan θ dθ. We substitute and obtain
dx 4 sec θ tan θ
Z Z
√ = q dθ
3 2
x x − 16 3 2
(4 sec θ) (4 sec θ) − 16
1 sec θ tan θ
Z
= √ dθ
16 sec θ 16 sec2 θ − 16
3

1 tan θ
Z
= √ dθ
64 sec2 θ sec2 θ − 1
1 tan θ
Z
= dθ
64 sec2 θ tan θ
1
Z
= cos2 θ dθ
64
1 1 + cos (2θ)
Z
= dθ
64 2
 
1 1
= θ + sin (2θ) + C
128 2
1 1
= θ+ (2 sin θ cos θ) + C
128 256
1 1
= θ+ sin θ cos θ + C.
128 128

−1 x
√ q 
x 2

− 1 = 41 x2 − 16. So sin θ =

We have θ = sec 2
√ 4 , so tan θ = sec θ − 1 = 4
1 √
tan θ x2 −16 x2 −16
sec θ = 4
x = x , and cos θ = x4 . We obtain
4

√ ! 
dx 1 x 1 x2 − 16 4
Z
√ = sec−1 + +C
x x2 − 16
3 128 4 128 x x

1 x x2 −16
sec−1

= 128 4 + 32x2 +C .
7-108 Chapter 7 Techniques of Integration

48. Let x = sec θ, then dx = sec θ tan θ dθ. We substitute and obtain
dx sec θ tan θ
Z Z
√ = q dθ
3 2
x x −1 3 2
(sec θ) (sec θ) − 1
sec θ tan θ
Z
= √ dθ
sec3 θ sec2 θ − 1
tan θ
Z
= dθ
sec2 θ tan θ
Z
= cos2 θ dθ

1 + cos (2θ)
Z
= dθ
2
 
1 1
= θ + sin (2θ) + C
2 2
1 1
= θ + (2 sin θ cos θ) + C
2 4
1 1
= θ + sin θ cos θ + C.
2 2
√ √ √
x2 −1
We have θ = sec−1 x, so tan θ = sec2 θ − 1 = x2 − 1. So sin θ = tan θ
sec θ = x , and
1
cos θ = x . We obtain
√ ! 
dx 1 1 x2 − 1 1
Z
−1
√ = sec x + +C
3
x x −12 2 2 x x

1 x2 −1
= 2 sec−1 x + 2x2 +C .

49. Let x = sin θ, then dx = cos θ dθ. The lower limit of integration is θ = sin−1 0 = 0, and
the upper limit of integration is θ = sin−1 1 = π2 . We substitute and obtain
Z 1 p Z π/2 p
1 − x2 dx = 1 − sin2 θ (cos θ) dθ
0 0
Z π/2
= cos2 θ dθ
0
π/2
1 + cos (2θ)
Z
= dθ
0 2
 π/2
1 1
= θ + sin (2θ)
2 2 0
   π   
1 π 1 1
= + sin 2 − 0 + sin (2(0))
2 2 2 2 2
1
= 4π .
7.3 Integration Using Trigonometric Substitution 7-109

50. Let x = 12 sin θ, then dx = 21 cos θ dθ. The lower limit of integration is θ = sin−1 (2(0)) = 0,
and the upper limit of integration is θ = sin−1 2 12 = π2 . We substitute and obtain


s 2 
1/2 π/2  
1 1
Z p Z
1 − 4x2 dx = 1−4 sin θ cos θ dθ
0 0 2 2
1 π/2 p
Z
= 1 − sin2 θ (cos θ) dθ
2 0
1 π/2
Z
= cos2 θ dθ
2 0
1 π/2 1 + cos (2θ)
Z
= dθ
2 0 2
 π/2
1 1
= θ + sin (2θ)
4 2 0
   π   
1 π 1 1
= + sin 2 − 0 + sin (2(0))
4 2 2 2 2
1
= 8π .

51. Let x = tan θ, then dx = sec2 θ dθ. The lower limit of integration is θ = tan−1 0 = 0, and
the upper limit of integration is θ = tan−1 1 = π4 . We substitute and obtain
Z 1 p Z π/4 q
1 + (tan θ)2 sec2 θ dθ

1 + x2 dx =
0 0
Z π/4
= (sec θ) sec2 θ dθ
0
Z π/4
= sec3 θ dθ
0
 π/4
1 1
= sec θ tan θ + ln |sec θ + tan θ|
2 2 0
 
1 π π 1 π π 1 1
= sec tan + ln sec + tan − sec 0 tan 0 + ln |sec 0 + tan 0|
2 4 4 2 4 4 2 2
√ √
2+ln ( 2+1)
= 2 .
7-110 Chapter 7 Techniques of Integration

52. We evaluate the corresponding indefinite integral, and then apply the Fundamental Theo-
rem of Calculus. Let x = 3 tan θ, then dx = 3 sec2 θ dθ. We substitute and obtain
2
x2 (3 tan θ)
Z Z
3 sec2 θ dθ

√ dx = q
9 + x2 9 + (3 tan θ)
2

tan2 θ
Z
= 27 √ sec2 θ dθ
9 tan2 θ + 9
tan2 θ
Z
= 9 sec2 θ dθ
sec θ
Z
= 9 tan2 θ sec θ dθ
Z
sec2 θ − 1 sec θ dθ

= 9
Z
sec3 θ − sec θ dθ

= 9
9 9
sec θ tan θ − ln |sec θ + tan θ| + C.
=
2 2
√ q 2 √
Since tan θ = x3 , sec θ = 1 + tan2 θ = 1 + x3 = 13 9 + x2 . So

x2 9 9
Z
√ dx = sec θ tan θ − ln |sec θ + tan θ| + C
9 + x2 2 2
 p  
9 1 2
x 9 1 p 2
x
= 9+x − ln 9+x + +C
2 3 3 2 3 3

1 p 9 1 1 p
= x x2 + 9 − ln x + x2 + 9 + C
2 2 3 3

By the Fundamental Theorem of Calculus,


2 2
x2

1 p 2 9 1 1p 2
Z

√ dx = x x + 9 − ln x + x + 9
0 9 + x2 2 2 3 3
 p 0
1 9 1 1 p
= 2 22 + 9 − ln 2 + 22 + 9
2 2 3 3
 p 
1 2
9 1 1p 2
− 0 0 + 9 − ln 0 + 0 + 9
2 2 3 3
√ 1√
  
9 2
= 13 − ln 13 + −0
2 3 3
√ √
13 − 92 ln 13 13 + 32 .

=
7.3 Integration Using Trigonometric Substitution 7-111

53. We evaluate the corresponding indefinite integral, and then apply the Fundamental Theo-
rem of Calculus. Let x = 3 sec θ, then dx = 3 sec θ tan θ dθ. We substitute and obtain

x2 (3 sec θ)2
Z Z
√ dx = q (3 sec θ tan θ ) dθ
x2 − 9 (3 sec θ)2 − 9
sec2 θ
Z
= 27 √ sec θ tan θ dθ
9 sec2 θ − 9
sec2 θ
Z
= 9 √ sec θ tan θ dθ
sec2 θ − 1
sec2 θ
Z
= 9 sec θ tan θ dθ
tan θ
Z
= 9 sec3 θ dθ
9 9
= sec θ tan θ + ln |sec θ + tan θ| + C.
2 2
√ q
x 2

Since sec θ = x3 , tan θ = sec2 θ − 1 = − 1 = 31 x2 − 9. So

3

x2
 
9 1p 2 x 9 1 p 2 x
Z
√ dx = x −9 + ln x − 9 + + C
x2 − 9 2 3 3 2 3 3

1 p 2 9 1 p 2 x
= x x − 9 + ln x − 9 + + C.
2 2 3 3

By the Fundamental Theorem of Calculus,


Z 5 5
x2
 p
1 9 1p 2 x
√ dx = x x2 − 9 + ln x − 9 +
4 x2 − 9 2 2 3 3 4
 p p   p 
1 9 1 5 1 9 1p 2 4
= 5 52 − 9 + ln 52 − 9 + − 4 42 − 9 + ln 4 −9+
2 2 3 3 2 2 3 3
1√ √
   
9 9 4
= ln 3 + 10 − ln 7+ +2 7
2 2 3 3
9 9 √  9 √
= ln 3 + 10 − ln 7 + 4 + ln 3 − 2 7
2 2 2
√ 9
√ 
= 10 − 2 7 + 9 ln 3 − 2 ln 4 + 7 .
7-112 Chapter 7 Techniques of Integration

54. We evaluate the corresponding indefinite integral, and then apply the Fundamental Theo-
rem of Calculus. Let x = 21 sec θ, then dx = 21 sec θ tan θ dθ. We substitute and obtain
2 1
x2 sec θ
 
1
Z Z
2
√ dx =
q sec θ tan θ dθ
4x2 − 1 2 2
4 12 sec θ − 1
1 sec2 θ
Z
= √ sec θ tan θ dθ
8 sec2 θ − 1
1 sec2 θ
Z
= √ sec θ tan θ dθ
8 sec2 θ − 1
1 sec2 θ
Z
= sec θ tan θ dθ
8 tan θ
1
Z
= sec3 θ dθ
8
1 1
= sec θ tan θ + ln |sec θ + tan θ| + C.
16 16
√ q
2 √
Since sec θ = 2x, tan θ = sec2 θ − 1 = (2x) − 1 = 4x2 − 1. So

x2 1 1
Z p p
√ dx = (2x) 4x2 − 1 + ln 2x + 4x2 − 1 + C

4x2 − 1 16 16
1 p 2 1 p
= x 4x − 1 + ln 2x + 4x2 − 1 + C.

8 16
By the Fundamental Theorem of Calculus,
Z 2
x2

1 p 2 1 p 2
√ dx = x 4x − 1 + ln 2x + 4x2 − 1

1 4x2 − 1 8 16 1
 q 
1 1
q
2 2
= 2 4 (2) − 1 + ln 2 (2) + 4 (2) − 1

8 16
 
1 1
q q
2 2
− (1) 4 (1) − 1 + ln 2 (1) + 4 (1) − 1

8 16
√ √ √  1√ 
  
1   1 1
= ln 15 + 4 + 15 − ln 3+2 + 3
16 4 16 8
1
√ 1
√ √ √
ln 3 + 2 − 81 3 + 41 15 .
 
= 16 ln 15 + 4 − 16
7.3 Integration Using Trigonometric Substitution 7-113

55. We evaluate the corresponding indefinite integral, and then apply the Fundamental Theo-
rem of Calculus. Let x = 4 sin θ, then dx = 4 cos θ dθ. We substitute and obtain
2
x2 (4 sin θ)
Z Z
3/2
dx = 3/2 (4 cos θ) dθ
(16 − x2 )

16 − (4 sin θ)2
sin2 θ
Z
= 64 cos θ dθ
(16 cos2 θ)3/2
sin2 θ cos θ
Z
= dθ
cos3 θ
Z
= tan2 θ dθ
Z
sec2 θ − 1 dθ

=
= tan θ − θ + C
sin θ
= − θ + C.
cos θ
p q 2 √
Since sin θ = x4 , cos θ = 1 − sin2 θ = 1 − x4 = 14 16 − x2 . So we have
x
x2 x
Z
3/2
dx = 1 √ 4 − sin−1 + C
2
(16 − x ) 4 16 − x2 4
x x
= √ − sin−1 + C.
16 − x2 4
By the Fundamental Theorem of Calculus,
Z 2 2
x2

x −1 x
3/2
dx = √ − sin
0 (16 − x2 ) 16 − x2 4 0
   
2 −1 2 0 −1 0
= √ − sin − √ − sin
16 − 22 4 4 − 02 4

3 π
= − −0
3 6

3 π
= 3 − 6 .

56. We evaluate the corresponding indefinite integral, and then apply the Fundamental Theo-
rem of Calculus. Let x = 5 sin θ, then dx = 5 cos θ dθ. We substitute and obtain
x2 (5 sin θ)2
Z Z
3/2
dx = 3/2 (5 cos θ) dθ
(25 − x2 )

2
25 − (5 sin θ)
sin2 θ
Z
= 125 3/2
cos θ dθ
(25 cos2 θ)
sin2 θ cos θ
Z
= dθ
cos3 θ
Z
= tan2 θ dθ
Z
sec2 θ − 1 dθ

=
= tan θ − θ + C.
7-114 Chapter 7 Techniques of Integration

p q
x 2
√ x
x
1 − sin2 θ = 1 − 1

Since sin θ = 5, cos θ = 5 = 5 25 − x2 . So tan θ = 1
√ 5
25−x2
=
5
√ x
25−x2
, and θ = sin−1 x5 . So

x2 x x
Z
3/2
dx = √ − sin−1 + C.
(25 − x2 ) 25 − x2 5

By the Fundamental Theorem of Calculus,


1 1
x2

x x
Z
3/2
dx = √ − sin−1
0 (25 − x2 ) 25 − x2 5 0
   
1 −1 1 0 −1 0
= √ − sin − √ − sin
25 − 12 5 25 − 02 5
1√
 
1
= 6 − sin−1 − (0)
12 5

1
= 12 6 − sin−1 15 .

57. We evaluate the corresponding indefinite integral, and then apply the Fundamental Theo-
rem of Calculus. Let x = 3 tan θ, then dx = 3 sec2 θ dθ. We substitute and obtain

x2 (3 tan θ)2
Z Z
3 sec2 θ dθ

dx = 2
9 + x2 9 + (3 tan θ)
tan2 θ
Z
= 27 sec2 θ dθ
9 tan2 θ + 9
tan2 θ
Z
= 3 sec2 θ dθ
sec2 θ
Z
= 3 tan2 θ dθ
Z
sec2 θ − 1 dθ

= 3

= 3 (tan θ − θ) + C.

Since tan θ = x3 , and θ = tan−1 x3 . So

x2 x x
Z
dx = 3 − tan−1 +C
9 + x2 3 3
x
= x − 3 tan−1 + C.
3
By the Fundamental Theorem of Calculus,
3
x2 x i3
Z h
dx = x − 3 tan−1
0 9 + x2 3 0
   
−1 3 −1 0
= 3 − 3 tan − 0 − 3 tan
3 3
 
3
= 3 − π − (0)
4
= 3 − 34 π .
7.3 Integration Using Trigonometric Substitution 7-115

58. We evaluate the corresponding indefinite integral, and then apply the Fundamental Theo-
rem of Calculus. Let x = 5 tan θ, then dx = 5 sec2 θ dθ. We substitute and obtain
2
x2 (5 tan θ)
Z Z
5 sec2 θ dθ

2
dx = 2
25 + x 25 + (5 tan θ)
tan2 θ
Z
= 125 sec2 θ dθ
25 tan2 θ + 25
tan2 θ
Z
= 5 sec2 θ dθ
sec2 θ
Z
= 5 tan2 θ dθ
Z
sec2 θ − 1 sec θ dθ

= 5

= 5 (tan θ − θ) + C.
x
Since tan θ = 5 and θ = tan−1 x5 , we have
x2 x x
Z
dx = 5 − tan−1 +C
25 + x2 5 5
x
= x − 5 tan−1 + C.
5
So by the Fundamental Theorem of Calculus,
Z 1
x2 h x i1
2
dx = x − 5 tan−1
0 25 + x 5 0
 
1 0
= 1 − 5 tan−1 − 0 − 5 tan−1
5 5
1
= 1 − 5 tan−1 5 .

Applications and Extensions


R1  2
59. The volume is given by 0 π x21+4 dx. We evaluate the corresponding indefinite in-
tegral, and then apply the Fundamental Theorem of Calculus. Let x = 2 tan θ, then
dx = 2 sec2 θ dθ. We substitute and obtain
Z  2 !2
1 1
Z
2 sec2 θ dθ

π 2 dx = π 2
x +4 (2 tan θ) + 4
Z  2
1
= 2π sec2 θ dθ
4 sec2 θ
π sec2 θ
Z
= dθ
8 sec4 θ
π
Z
= cos2 θ dθ
8
π 1 + cos (2θ)
Z
= dθ
8 2
π
Z
= (1 + cos (2θ)) dθ
16
 
π 1
= θ + sin (2θ)
16 2
π
= (θ + sin θ cos θ) + C
16  
π tan θ
= θ+ + C.
16 sec2 θ
7-116 Chapter 7 Techniques of Integration

x
√ q
x 2 1

tan2 θ + 1 = x2 + 4, and θ = tan−1 x2 . We

Since tan θ = 2, sec θ = 2 +1 = 2
have
2 !
 x
1 π x
Z
−1 2
π 2 dx = tan + √ 2 + C
x +4 16 2 1
x2 + 4
2
 
π x 2x
= tan−1 + 2 + C.
16 2 x +4

So by the Fundamental Theorem of Calculus,


1  2   1
1 π x 2x
Z
π dx = tan−1 + 2
0 x2 + 4 16 2 x +4 0
!! !!
π −1 1 2 (1) π −1 0 2 (0)
= tan + − tan +
16 2 (1)2 + 4 16 2 (0)2 + 4
 
π −1 1 2
= tan + −0
16 2 5
π π 1
= 40 + 16 tan−1 2 .

R 2  1 2 R2 1
60. The volume is given by 0 π √9−x 2
dx = π 0 9−x 2 dx. We evaluate the corresponding

indefinite integral, and then apply the Fundamental Theorem of Calculus. Let x = 3 sin θ,
then dx = 3 cos dθ. We substitute and obtain
1 1
Z Z
π dx = π (3 cos θ) dθ
9−x 2
9 − (3 sin θ)2
cos θ
Z
= 3π dθ
9 − 9 sin2 θ
π cos θ
Z
= dθ
3 cos2 θ
π
Z
= sec θ dθ
3
π
= ln |sec θ + tan θ| + C
3
π 1 + sin θ
= ln + C.
3 cos θ
q 2 √
Since sin θ = x3 , cos θ = 1 − x3 = 31 9 − x2 . So

1 π 1 + x3
Z
π dx = ln √ +C
9 − x2 3 13 9 − x2

π x + 3
= ln √ + C.
3 9 − x2
7.3 Integration Using Trigonometric Substitution 7-117

So by the Fundamental Theorem of Calculus,


Z 2  2
1 π x + 3
π 2
dx = ln √
0 9−x 3 9 − x2 0
   
π 2 + 3 π 0 + 3
= ln √ − ln √
3 9 − 22 3 9 − 02
 π √   π 
= ln 5 − ln |1|

3 3
π
= 6 ln 5 .

√ 1 over the interval 0, 12 is


 
61. The average value of f (x) = 9−4x2
1/2 1/2
1 1 1
Z Z
f¯ = 1
√ dx = 2 √ dx.
2 −0 0 9 − 4x2 0 9 − 4x2
3

Use the substitution x = 2 sin θ, − π2
≤ θ ≤ π
Then dx = 23 cos θ dθ and
2. 9 − 4x2 =
p p √
9 − 9 sin2 θ = 3 1 − sin θ = 3 cos2 θ = 3 cos θ since − π2 < θ < π2 .
2

Since θ = sin−1 2x3 , the lower limit of integration becomes θ = sin


−1
0 = 0 and the upper
−1 1
limit of integration becomes u = sin 3 .
The integral becomes
Z 1/2 Z sin−1 13  
1 1 3
2 √ dx = 2 cos θ dθ
0 9 − 4x2 0 3 cos θ 2
Z sin−1 31
= dθ
0
sin−1 31
= [θ]0
= sin−1 13 ≈ 0.340 .

1
R7√ R7√
62. The average value is given by 7−2 2
x2 − 4 dx = 15 2 x2 − 4 dx. We evaluate the
corresponding indefinite integral, and then apply the Fundamental Theorem of Calculus.
Let x = 2 sec θ, then dx = 2 sec θ tan θ dθ. We substitute and obtain
1 p 2 1
Z Z q
2
x − 4 dx = (2 sec θ) − 4 (2 sec θ tan θ) dθ
5 5
2 p
Z
= 4 sec2 θ − 4 sec θ tan θ dθ
5
4
Z
= sec θ tan2 θ dθ
5
4
Z
sec3 θ − sec θ dθ

=
5
 
4 1 1
= sec θ tan θ − ln |sec θ + tan θ| + C.
5 2 2
q 
x 2

Since sec θ = x2 , tan θ = 2 − 1 = 12 x2 − 4. So
   
1 p 2 4 1 x 1p 2 1 x 1p 2
Z

x − 4 dx = x − 4 − ln + x − 4 + C
5 5 2 2 2 2 2 2

1 p 2 1 1 p
= x x2 − 4 − ln x + x2 − 4 + C.
10 5 2 2
7-118 Chapter 7 Techniques of Integration

So by the Fundamental Theorem of Calculus,


7  7
1 1 p 2 2 1 1p 2
Z p
x2 − 4 dx = x x − 4 − ln x + x − 4
5 2 10 5 2 2
 2
1 p 2 1 1 p
= 7 72 − 4 − ln 7 + 72 − 4
10 5 2 2
 
1 p 2 1 1p 2
− 2 22 − 4 − ln 2 + 2 − 4
10 5 2 2
√ √
 
21 2 3 7
= 5 − ln 5+ − (0)
10 5 2 2
21
√ 2 3
√ 7

= 10 5 − 5 ln 2 5 + 2 .

1
R2
63. Since y = √9−x is nonnegative on the interval [0, 2], A = √ 1 dx is the area under
2 0 9−x2
1
the graph of y = √9−x 2
from x = 0 to x = 2.


Use the substitution x = 3 sin θ, − π2 ≤ θ ≤ π2 . Then dx = 3 cos θ dθ and 9 − x2 =
p p √
9 − 9 sin2 θ = 3 1 − sin2 θ = 3 cos2 θ = 3 cos θ since − π2 < θ < π2 .

Since θ = sin−1 x3 , the lower limit of integration becomes θ = sin−1 0 = 0 and the upper
limit of integration becomes u = sin−1 23 .
The integral becomes
2 sin−1 32
1 1
Z Z
A= √ dx = (3 cos θ dθ)
0 9 − x2 0 3 cos θ
Z sin−1 32
= dθ
0
sin−1 23
= [θ]0
= sin−1 23 .

√ R4 √
64. Since y = x2 16 − x2 is nonnegative on the interval [−4, 4], A = −4 x2 16 − x2 dx is the

area under the graph of y = x2 16 − x2 from x = −4 to x = 4.
7.3 Integration Using Trigonometric Substitution 7-119


Use the substitution x = 4 sin θ, − π2 ≤ θ ≤ π2 . Then dx = 4 cos θ dθ and 16 − x2 =
p p √
16 − 16 sin2 θ = 4 1 − sin2 θ = 4 cos2 θ = 4 cos θ since − π2 < θ < π2 .
Since θ = sin−1 x4 , the lower limit of integration becomes θ = sin−1 −4 = − π2 , and the

4
−1 4 π

upper limit of integration becomes u = sin 4 = 2.
The integral becomes
Z 4 p Z π/2
2
16 sin2 θ (4 cos θ)(4 cos θ dθ)

A= 2
x 16 − x dx =
−4 −π/2
Z π/2
= 256 sin2 θ cos2 θ dθ
−π/2
π/2
1 1
Z
= 256 [1 − cos (2θ)] · [1 + cos (2θ)]dθ
−π/2 2 2
Z π/2
1 − cos2 (2θ) dθ
 
= 64
−π/2
π/2  
1
Z
= 64 1 − [1 + cos (4θ)] dθ
−π/2 2
Z π/2
= 32 [1 − cos (4θ)]dθ
−π/2
 π/2
1
= 32 θ − sin (4θ)
4 −π/2
   
π 1 1 1
= 32 − sin (2π) − − − sin (−2π) = 32π .
2 4 2 4
R5 2
65. The area under the graph is given by 3 √xx2 −1 dx. We evaluate the corresponding indefinite
integral, and then apply the Fundamental Theorem of Calculus. Let x = sec θ, then
dx = sec θ tan θ dθ. We substitute and obtain
x2 (sec θ)2
Z Z
√ dx = q (sec θ tan θ ) dθ
x2 − 1 (sec θ)2 − 1
sec2 θ
Z
= sec θ tan θ dθ
tan θ
Z
= sec3 θ dθ
1 1
= sec θ tan θ + ln |sec θ + tan θ| + C.
2 2

Since sec θ = x, tan θ = x2 − 1. So
x2 1 p 1
Z p
√ dx = x x2 − 1 + ln x + x2 − 1 + C.

x2 − 1 2 2
7-120 Chapter 7 Techniques of Integration

So by the Fundamental Theorem of Calculus,


Z 5
x2
 p   1 p 
1 2
1 p
2 2
1 p
2
√ dx = 5 5 − 1 + ln 5 + 5 − 1 − 3 3 − 1 + ln 3 + 3 − 1

3
2
x −1 2 2 2 2
1  √ √ 1  √ √
   
= ln 2 6 + 5 + 5 6 − ln 2 2 + 3 + 3 2
2 2
√ √ √ √
= 12 ln 2 6 + 5 − 21 ln 2 2 + 3 − 3 2 + 5 6 .
 

√ R1√
66. Since y = 16x2 + 9 is nonnegative on [0, 1], A = 0 16x2 + 9 dx is the area under the

graph of y = 16x2 + 9 from x = 0 to x = 1.

3
R√
Use the substitution x = tan θ, − π2 < θ < π2 , to evaluate
4 16x2 + 9 dx.
√ q √ √
Then dx = 43 sec2 θ dθ and 16x2 + 9 = 16 16 9
tan2 θ + 9 = 3 tan2 θ + 1 = 3 sec2 θ =


3 sec θ since − π2 < θ < π2 . Then


 
3 9
Z p Z Z
2
2
16x + 9 dx = (3 sec θ) sec θ dθ = sec3 θ dθ.
4 4

sec3 θ dθ using integration by parts.


R
Evaluate
Let u = sec θ and dv = sec2 θ dθ.
sec2 θ dθ = tan θ.
R
Then du = tan θ sec θ dx and v =
Now
Z Z Z
sec3 θ dθ = sec θ tan θ − (tan θ)(tan θ sec θ dx) = tan θ sec θ − tan2 θ sec θ dθ

Use the identity tan2 θ = sec2 θ − 1.


Z Z Z Z
sec3 θ dθ = tan θ sec θ − sec2 θ − 1 sec θ dθ = tan θ sec θ − sec3 θ dθ + sec θ dθ


sec3 θ dθ to both sides.


R
Add
Z Z
3
2 sec θ dθ = tan θ sec θ + sec θ dθ = tan θ sec θ + ln |sec θ + tan θ| + C
7.3 Integration Using Trigonometric Substitution 7-121

and
1
Z
sec3 θ dθ = [tan θ sec θ + ln |sec θ + tan θ|] + C.
2
So,
9 9
Z
sec3 θ dθ = [tan θ sec θ + ln |sec θ + tan θ|] + C.
4 8
3
From x = 4 tan θ, when x = 0, tan θ = 0 and sec θ = 1.

4
√ q
4 2
tan2 θ + 1 = + 1 = ± 35 .

Also, when x = 1, tan θ = 3 and sec θ = 3

Since − π2 < θ < π


2, sec θ = 35 .
Finally,
1
9
Z
tan−1 (4/3)
p
A= 16x2 + 9 dx = [tan θ sec θ + ln |sec θ + tan θ|]tan−1 0
0 8
   
9 4 5 5 4
= · + ln + − [(0)(1) + ln (1 + 0)]
8 3 3 3 3
5 9
= 2 + 8 ln (3) .

y2
67. (a) The region enclosed by the hyperbola 9 − x2 = 1 and the line y = 4 is pictured
below.

q
y2
p
Solving the equation of the hyperbola for x, x = ± 9 − 1 = ± 31 y 2 − 9.
Using symmetry, the area enclosed by the hyperbola and the line y = 4 is given by
Z 4 p
1 2 4p 2
Z
A=2 2
y − 9 dy = y − 9 dy.
3 3 3 3
Rp
Use the substitution y = 3 sec θ (0 ≤ θ < π2 , π ≤ θ < 3π
2 ) to evaluate y 2 − 9 dy.
Then dy = 3 tan θ sec θ dθ and
p p p p
y 2 − 9 = 9 sec2 θ − 9 = 3 sec2 θ − 1 = 3 tan2 θ = 3 tan θ
π 3π
since 0 ≤ θ < 2 or π ≤ θ < 2 .
7-122 Chapter 7 Techniques of Integration

Then
Z p Z
y 2 − 9 dy = (3 tan θ)(3 tan θ sec θ dθ)
Z
= 9 tan2 θ sec θ dθ
Z
sec2 θ − 1 sec θ dθ

=9
Z
sec3 θ − sec θ dθ.

=9

Evaluate sec3 θ dθ using integration by parts.


R

Let u = sec θ and dv = sec2 θ dθ. R


Then du = tan θ sec θ dx and v = sec2 θ dθ = tan θ.
Now
Z Z Z
sec θ dθ = sec θ tan θ − (tan θ)(tan θ sec θ dx) = tan θ sec θ − tan2 θ sec θ dθ.
3

Use the identity tan2 θ = sec2 θ − 1.


Z Z Z Z
3 2 3

sec θ dθ = tan θ sec θ− sec θ − 1 sec θ dθ = tan θ sec θ− sec θ dθ+ sec θ dθ.

sec3 θ dθ to both sides.


R
Add
Z Z
2 sec3 θ dθ = tan θ sec θ + sec θ dθ = tan θ sec θ + ln |sec θ + tan θ|

and sec3 θ dθ = 12 [tan θ sec θ + ln |sec θ + tan θ|] + C.


R

Therefore,
Z p Z
sec3 θ − sec θ dθ

2
y − 9 dy = 9
 
1
=9 [tan θ sec θ + ln |sec θ + tan θ|] − ln |sec θ + tan θ| + C
2
9
= [tan θ sec θ − ln |sec θ + tan θ|] + C
2" #
p y py 2 − 9
9 y2 − 9 y
= · − ln + +C

2 3 3 3 3
1 h p p i
= y y 2 − 9 − 9 ln y + y 2 − 9 − 9 ln 3 + C

2
1h p 2 p i
= y y − 9 − 9 ln y + y 2 − 9 + C after combining the constants.

2
So,
2 4p 2
Z
A= y − 9 dy
3 3
2 1h p p i4
= · y y 2 − 9 − 9 ln y + y 2 − 9

3 2 3
1 nh √  √ i o
= 4 7 − 9 ln 4 + 7 − [0 − 9 ln (3)]
3
1 √
 
3
= 4 7 + 9 ln √
3 4+ 7
√ √
4 7
= 3 − 3 ln 4+3 7
.
7.3 Integration Using Trigonometric Substitution 7-123

y2
(b) The region enclosed by the hyperbola 9 − x2 = 1 and the line y = −7 is pictured
below.

q
2
Solving the equation of the hyperbola for x, x = ± y9 − 1 = ± 31 y 2 − 9.
p

Using symmetry, the area enclosed by the hyperbola and the line y = −7 is given by
Z −3 p
1 2 −3 p 2
Z
A=2 y 2 − 9 dy = y − 9 dy.
−7 3 3 −7
Using the results from part (a),
So,
2 −3 p 2
Z
A= y − 9 dy
3 −7
2 1h p p i−3
= · y y 2 − 9 − 9 ln y + y 2 − 9

3 2 −7
1n h √ √ io
= [0 − 9 ln |−3|] − −7 40 − 9 ln −7 + 40

3"
√ #
1 √ 7 − 40
= 7 40 + 9 ln
3 3
" √ √ !#
1 √ 7 − 40 7 + 40
= 7 · 2 10 + 9 ln · √
3 3 7 + 40

 
1 9
= 14 10 + 9 ln √
3 3(7 + 40)

14 10 3
= + 3 ln √
3 7 + 40
√ √
14 10
= 3 − 3 ln 7+2310
.
7-124 Chapter 7 Techniques of Integration

68. (a) The region enclosed by the hyperbola and the line x = 6 is pictured below.

q
x2
Solving the equation of the hyperbola for y, y = ±4 9 − 1.
Using symmetry, the area enclosed by the hyperbola and the line x = 6 is given by
Z 6 r ! Z 6r 2
x2 x
A=2 4 − 1 dx = 8 − 1 dx.
3 9 3 9

Use the substitution u = x3 . Then du = dx 3 and dx = 3 du. The lower limit of


integration becomes u = 33 =1 and the upper limit of integration becomes u = 63 = 2.
So,
Z 6r 2 Z 2p Z 2p
x 2
A=8 − 1 dx = 8 u − 1(3 du) = 24 u2 − 1 du.
3 9 1 1
R√
Use the substitution u = sec θ (0 ≤ θ < π2 , π ≤ θ < 3π2 )√to evaluate u2 − 1 du.
√ √
Then du = tan θ sec θ dθ and u2 − 1 = sec2 θ − 1 = tan2 θ = tan θ since 0 ≤ θ <
π 3π
2 or π ≤ θ < 2 . Then
Z p Z
u2 − 1 du = (tan θ)(tan θ sec θ dθ)
Z
= tan2 θ sec θ dθ
Z
sec2 θ − 1 sec θ dθ

=
Z
sec3 θ − sec θ dθ.

=

Evaluate sec3 θ dθ using integration by parts.


R

Let u = sec θ and dv = sec2 θ dθ. R


Then du = tan θ sec θ dx and v = sec2 θ dθ = tan θ.
Now
Z Z Z
sec θ dθ = sec θ tan θ − (tan θ)(tan θ sec θ dθ) = tan θ sec θ − tan2 θ sec θ dθ.
3

Use the identity tan2 θ = sec2 θ − 1.


Z Z Z Z
3 2 3

sec θ dθ = tan θ sec θ− sec θ − 1 sec θ dθ = tan θ sec θ− sec θ dθ+ sec θ dθ.
7.3 Integration Using Trigonometric Substitution 7-125

sec3 θ dθ to both sides.


R
Add
Z Z
3
2 sec θ dθ = tan θ sec θ + sec θ dθ = tan θ sec θ + ln |sec θ + tan θ|

sec3 θ dθ = 12 [tan θ sec θ + ln |sec θ + tan θ|] + C.


R
and
Z p Z
sec3 θ − sec θ dθ

u2 − 1 du =
1
= [tan θ sec θ + ln |sec θ + tan θ|] − ln |sec θ + tan θ| + C
2
1
= [tan θ sec θ − ln |sec θ + tan θ|] + C
2
1 hp 2  p i
= u − 1 (u) − ln u + u2 − 1 + C.

2
So,
2  hp i2 
1
Z p  p
A = 24 2
u − 1 du = 24 2 2
u − 1 (u) − ln u + u − 1

1 2 1
 q  q 
(2)2 − 1 (2) − ln (2) + (2)2 − 1

= 12
q  q 
2 2
− 12 (1) − 1 (1) − ln (1) + (1) − 1

h √ √ i
= 12 2 3 − ln 2 + 3 − 12(0)

√ √
= 24 3 − 12 ln(2 + 3) .

(b) The region enclosed by the hyperbola and the line x = −8 is pictured below.

q
2
Solving the equation of the hyperbola for y, y = ±4 x9 − 1.
Using symmetry, the area enclosed by the hyperbola and the line x = −8 is given by
Z −3 r ! Z −3 r 2
x2 x
A=2 4 − 1 dx = 8 − 1 dx.
−8 9 −8 9
7-126 Chapter 7 Techniques of Integration

Use the substitution u = x3 . Then du = dx 3 and dx = 3 du. The lower limit of


integration becomes u = −8
3 and the upper limit of integration becomes u = −3
3 = −1.
So,
Z −3 r 2 Z −1 p Z −1 p
x
A=8 − 1 dx = 8 u2 − 1 (3 du) = 24 u2 − 1 du.
−8 9 −8/3 −8/3
R√
Use the substitution u = sec θ (0 ≤ θ < π2 , π ≤ θ < 3π2 ) to evaluate u2 − 1 du.
√ √ √
Then du = tan θ sec θ dθ and u2 − 1 = sec2 θ − 1 = tan2 θ = tan θ since 0 ≤ θ <
π 3π
2 or π ≤ θ < 2 . Then
Z p Z
u2 − 1 du = (tan θ)(tan θ sec θ dθ)
Z
= tan2 θ sec θ dθ
Z
sec2 θ − 1 sec θ dθ

=
Z
sec3 θ − sec θ dθ.

=

sec3 θ dθ using integration by parts.


R
Evaluate
Let u = sec θ and dv = sec2 θ dθ.
Then du = tan θ sec θ dx and v = sec2 θ dθ = tan θ.
R

Now
Z Z Z
sec3 θ dθ = sec θ tan θ − (tan θ)(tan θ sec θ dθ) = tan θ sec θ − tan2 θ sec θ dθ.

Use the identity tan2 θ = sec2 θ − 1.


Z Z Z Z
3 2 3

sec θ dθ = tan θ sec θ− sec θ − 1 sec θ dθ = tan θ sec θ− sec θ dθ+ sec θ dθ.

sec3 θ dθ to both sides.


R
Add
Z Z
3
2 sec θ dθ = tan θ sec θ + sec θ dθ = tan θ sec θ + ln |sec θ + tan θ|

sec3 θ dθ = 12 [tan θ sec θ + ln |sec θ + tan θ|] + C.


R
and
Therefore,
Z p Z
sec3 θ − sec θ dθ

u2 − 1 du =
1
= [tan θ sec θ + ln |sec θ + tan θ|] − ln |sec θ + tan θ| + C
2
1
= [tan θ sec θ − ln |sec θ + tan θ|] + C
2
1 hp 2  p i
= u − 1 (u) − ln u + u2 − 1 + C.

2
7.3 Integration Using Trigonometric Substitution 7-127

So,
−1  hp i−1 
1
Z p  p
A = 24 u2 − 1 du = 24 u2 − 1 (u) − ln u + u2 − 1

−8/3 2 −8/3
q  q 
2 2
= 12 (−1) − 1 (−1) − ln −1 + (−1) − 1
s  s 
 2   2
8 8 8 8
− 12  − −1  − − ln − +
− − 1 
3 3 3 3
"  r r #
8 55 8 55
= 12(0) − 12 − − ln − +

3 9 3 9

√ !
32 √ 8 − 55 √ √ 
= 55 + 12 ln = 323 55 + 12 ln 8 − 55 − 12 ln 3 .
3 3

R5q 2 R5q
(5x − x2 ) dx = 0 1 + (5 − 2x)2 dx.
d
69. The length of the graph is given by L = 0 1 + dx
We evaluate the corresponding indefinite integral, and then apply the Fundamental The-
orem of Calculus. Let u = 5 − 2x, so du = −2 dx and dx = − 12 du. We substitute and
obtain
 
1
Z q Z p
2 2
1 + (5 − 2x) dx = 1 + u − du
2
1
Z p
= − 1 + u2 du.
2

Let u = tan θ, then du = sec2 θ dθ. We substitute and obtain

1 p
Z q Z
2
1 + (5 − 2x) dx = − 1 + tan2 θ sec2 θ dθ
2
1
Z
= − sec3 θ dθ
2
1 1
= − sec θ tan θ − ln |sec θ + tan θ| + C.
4 4
√ √
Since tan θ = u, sec θ = 1 + tan2 θ = 1 + u2 , and also substituting u = 5 − 2x, we
obtain
Z q
1p 1 p
1 + (5 − 2x)2 dx = 1 + u2 (u) − ln 1 + u2 + u + C


4 4
q
1 1
q
− (5 − 2x) 1 + (5 − 2x)2 − ln 1 + (5 − 2x)2 + (5 − 2x) + C.

=
4 4
7-128 Chapter 7 Techniques of Integration

By the Fundamental Theorem of Calculus,


5  q 5
1 1
Z q q
1 + (5 − 2x)2 dx − (5 − 2x) 1 + (5 − 2x)2 − ln 1 + (5 − 2x)2 + (5 − 2x)

=
0 4 4 0
 q 
2
q
2 ln 1 + (5 − 2 (5)) + 5 − 2 (5)
 (5 − 2 (5)) 1 + (5 − 2 (5)) 
= − − 
 4 4 

 q 
1 + (5 − 2 (0))2 + 5 − 2 (0)
q
2 ln
 (5 − 2 (0)) 1 + (5 − 2 (0)) 

− − − 
4 4 

5√ 1 √   1 √  5√ 
= 26 − ln 26 − 5 − − ln 26 + 5 − 26
4 4 4 4
√  √  √
1
= 4
ln 26 + 5 − 14 ln 26 − 5 + 52 26 .

70. The parabola x = 6y − 3y 2 = 3y(2 − y) lies to the right of the y-axis for y on the interval
0 ≤ y ≤ 2.

The length of the graph of the part of the parabola that lies in the first quadrant is given by
s  2 Z 2 q
Z 2 Z 2q
dx 2 2
L= 1+ = 1 + (6 − 6y) dy = 36(y − 1) + 1 dy.
0 dy 0 0

du
Begin with the substitution u = 6(y − 1). Then du = 6 dy and dy = 6 .
The lower limit of integration becomes u = 6(0 − 1) = −6 and the upper limit of integration
becomes u = 6(2 − 1) = 6. So,
2 6
1 6p 2
 
du
Z q p Z Z
2
L= 36(y − 1) + 1 dy =u2 + 1 = u + 1 du.
0 −6 6 6 −6
R√
Use the substitution u = tan θ, − π2 < θ < π2 , to evaluate u2 + 1 du.
√ √ √
Then du = sec2 θ dθ and u2 + 1 = tan2 θ + 1 = sec2 θ = sec θ since − π2 < θ < π
2.
Then Z p Z Z
u2 + 1 du = (sec θ) sec2 θ dθ = sec3 θ dθ.

7.3 Integration Using Trigonometric Substitution 7-129

sec3 θ dθ using integration by parts.


R
Evaluate
Let u = sec θ and dv = sec2 θ dθ.
Then du = tan θ sec θ dx and v = sec2 θ dθ = tan θ.
R

Now
Z Z Z
sec3 θ dθ = sec θ tan θ − (tan θ)(tan θ sec θ dx) = tan θ sec θ − tan2 θ sec θ dθ.

Use the identity tan2 θ = sec2 θ − 1.


Z Z Z Z
3 2 3

sec θ dθ = tan θ sec θ − sec θ − 1 sec θ dθ = tan θ sec θ − sec θ dθ + sec θ dθ.

sec3 θ dθ to both sides.


R
Add
Z Z
3
2 sec θ dθ = tan θ sec θ + sec θ dθ = tan θ sec θ + ln |sec θ + tan θ|

and
1
Z
sec3 θ dθ = [tan θ sec θ + ln |sec θ + tan θ|].
2
So,
1
Z p Z
u + 1 du = sec3 θ dθ = [tan θ sec θ + ln |sec θ + tan θ|] + C.
2
2
√ q
2
From x = tan θ, tan θ = −6 when x = −6. Also, sec θ = tan2 θ + 1 = (−6) + 1 =

± 37.

Since − π2 < θ < π2 , sec θ = 37.
√ √ √
From x = tan θ, tan θ = 6 when x = 6. Also, sec θ = tan2 θ + 1 = 62 + 1 = ± 37.

Since − π2 < θ < π2 , sec θ = 37.
Finally,

1 6p 2 1 1
Z
tan−1 6
L= u + 1 du = · [tan θ sec θ + ln |sec θ + tan θ|]tan−1 (−6)
6 −6 6 2
1 nh √  √ i h √  √ io
= (6) 37 + ln 37 + 6 − (−6) 37 + ln 37 − 6
12
√ !
√ 1 37 + 6
= 37 + ln √
12 37 − 6
√ √
√ 1 37 + 6
37 − 6
= 37 + ln √ ·√
12 37 − 6

37 − 6

√ 1 1
= 37 + ln √

12 37 − 6 2

√ 1 √ −2
= 37 + ln 37 − 6
12
√ 1h √ i
= 37 + −2 ln 37 − 6
12
√ √
= 37 − 16 ln 37 − 6 .

7-130 Chapter 7 Techniques of Integration

2
71. (a) We equate x2 = 4 − y 2 with x2 = 1 − (y − 2) to get the points of intersection. We
obtain
4 − y2 = 1 − (y − 2)2
4 − y2 = −y 2 + 4y − 3
4 = 4y − 3
7
= y.
4

7 2 15 15
So x2 = 4 −

4 = 16 , and x = ± 4 . The area is given by
√ √
Z 15/4 hp  p i Z 15/4 hp  p i
A= √ 4 − x2 − 2 − 1 − x2 dx = 2 4 − x2 − 2 − 1 − x2 dx.
− 15/4 0

We expand to obtain
√ √ √
Z 15/4 p Z 15/4 p Z 15/4
A = 2 2
4 − x dx + 2 2
1 − x dx − 2 2 dx
0 0 0
√ √
Z 15/4 p Z 15/4 p √
= 2 4 − x2 dx + 2 1 − x2 dx − 15.
0 0

For the first integral, we evaluate the corresponding indefinite integral, and then
apply the Fundamental Theorem of Calculus. Let x = 2 sin θ, then dx = 2 cos θ dθ.
We substitute and obtain
Z p Z q
2 4 − x2 dx = 2 4 − (2 sin θ)2 (2 cos θ) dθ
Z p
= 4 4 − 4 sin2 θ cos θ dθ
Z
= 8 cos2 θ dθ

1 + cos (2θ)
Z
= 8 dθ
2
Z
= 4 (1 + cos (2θ)) dθ
 
1
= 4 θ + sin 2θ + C
2
= 4θ + 4 cos θ sin θ + C.
p q 2 √
Since sin θ = 2 , cos θ = 1 − sin θ = 1 − x2 = 12 4 − x2 , and θ = sin−1 x2 . We
x 2

obtain
 p  
x 1 x
Z p
2 4 − x2 dx = 4 sin−1 + 4 4 − x2 +C
2 2 2
x p
= 4 sin−1 + x 4 − x2 + C.
2
7.3 Integration Using Trigonometric Substitution 7-131

By the Fundamental Theorem of Calculus,



15/4 i√15/4
−1 x
Z p h p
2 4 − x2 dx = 4 sin +x 4−x 2
0 2 0
 v 
√ !
15
√ u √ !2  
−1 4 15 u 15  −1 0
p
2
= 4 sin + t4 −
 − 4 sin +0 4−0

2 4 4 2
√ ! √ !
15 7 15
= 4 sin−1 + −0
8 16
√ ! √
15 7 15
= 4 sin−1 + .
8 16

For the second integral, we evaluate the corresponding indefinite integral, and then
apply the Fundamental Theorem of Calculus. Let x = sin θ, then dx = cos θ dθ. We
substitute and obtain
Z p Z p
2 2
1 − x dx = 2 1 − sin2 θ(cos θ) dθ
Z
= 2 cos2 θ dθ

1 + cos (2θ)
Z
= 2 dθ
2
Z
= (1 + cos (2θ)) dθ
1
= θ+ sin 2θ + C
2
= θ + cos θ sin θ + C.
p √
Since sin θ = x, cos θ = 1 − sin2 θ = 1 − x2 , and θ = sin−1 x. We obtain
Z p p
2 1 − x2 dx = sin−1 x + 1 − x2 (x) + C
p
= sin−1 x + x 1 − x2 + C.
By the Fundamental Theorem of Calculus,

Z 15/4 p h p i√15/4
−1
2 1 − x2 dx = sin x + x 1 − x2
0 0
 
√ √ u √ !2
v
 −1 15 15 t 15   −1
u p 
= sin + 1−  − sin 0 + 0 1 − 02
4 4 4
√ !

15
−1 15
= sin + −0
4 16
√ √
−1 15 15
= sin + .
4 16
R √15/4 √ R √15/4 √ √  √ √ 
So we obtain A = 2 0 4 − x2 dx+2 0 1 − x2 dx− 15 = 4 sin−1 15
8 + 7 15
16 +
 √ √  √ √ √ √
sin−1 415 + 1615 − 15 = sin−1 415 + 4 sin−1 815 − 21 15 .
7-132 Chapter 7 Techniques of Integration

(b) Here we subtract the area of the smaller lune from the area of the upper circle. We
obtain
√ √ !
2 −1 15 −1 15 1 √
A = π(1) − sin + 4 sin − 15
4 8 2
√ √ √
= π − sin−1 15
4 − 4 sin−1 15
8 + 1
2 15 .
R2  p 
72. The force due to hydrostatic pressure is given by the integral −2 (1000(9.8)(3 − y)) 2 4 − y 2 dy.
We expand, and recognize the final integral as the area of a quarter-circle to obtain
Z 2  p  Z 2p Z 2 p
(9800(3 − y)) 2 4 − y 2 dy = 58800 4 − y 2 dy − 196000 y 4 − y 2 dy
−2 −2 −2
pZ 2
= 117600 4 − y 2 dy − 0
0
 
1 2
= 117600 π(2)
4
= 117600π N .
73. Let u = x − 2, then du = dx. We substitute and obtain
dx du
Z Z
q = √
1 − (x − 2)
2 1 − u2

= sin−1 u + C
= sin−1 (x − 2) + C .

74. Let u = x + 2, then du = dx. We substitute and obtain


Z q Z p
2
4 − (x + 2) dx = 4 − u2 du.

Let u = 2 sin θ, then du = 2 cos θ dθ. We substitute and obtain


Z q Z q
2 2
4 − (x + 2) dx = 4 − (2 sin θ) (2 cos θ) dθ
Z p
= 2 4 − 4 sin2 θ cos θ dθ
Z
= 4 cos2 θ dθ

1 + cos (2θ)
Z
= 4 dθ
2
Z
= 2 (1 + cos (2θ)) dθ
 
1
= 2 θ + sin (2θ) + C
2
= 2θ + 2 sin θ cos θ + C.
p q 2 √
We have θ = sin−1 u2 , so cos θ = 1 − sin2 θ = 1 − u2 = 21 4 − u2 . We obtain

Z q u  u  1 p 
2
4 − (x + 2) dx = 2 sin−1 +2 4 − u2 + C
2 2 2
u 1 p
= 2 sin−1 + u 4 − u2 + C.
2 2
7.3 Integration Using Trigonometric Substitution 7-133

And since u = x + 2, we have


Z q q
2 2
4 − (x + 2) dx = 2 sin−1 x+2
+ 21 (x + 2) 4 − (x + 2) + C .

2

√ dx 1
R
75. To evaluate , use the substitution u = 2x − 1. Then du = 2 dx, dx = 2 du,
(2x−1)2 −4
and  
dx 1 1
1 1
Z Z Z
p = √ du = √ du.
2
(2x − 1) − 4 u −4 2
2 2 2
u −4
Use the substitution u = 2 sec θ (0 ≤ θ < π2 , π ≤ θ < 3π √ 1
R
2 ) to evaluate u2 −4
dx.
Then du = 2 tan θ sec θ dθ and
p p p p
u2 − 4 = 4 sec2 θ − 4 = 2 sec2 θ − 1 = 2 tan2 θ = 2 tan θ
π 3π
since 0 ≤ θ < 2 or π ≤ θ < 2 .

Therefore,
dx 1 1
Z Z
p = √ du
(2x − 1)2 − 4 2 u2 − 4
1 1
Z
= · 2 tan θ sec θ dθ
2 2 tan θ
1
Z
= sec θ dθ
2
1
= ln |sec θ + tan θ| + C
2

1 u u2 − 4
= ln + +C
2 2 2

1 u + u2 − 4
= ln +C
2 2
p
1 (2x − 1) + (2x − 1)2 − 4
= ln + C or, equivalently,
2 2

1 (2x−1)+ 4x2 −4x−3
2 ln 2 +C .

√dx
R
76. To evaluate , use the substitution u = 3x − 2.
(3x−2) (3x−2)2 +9

1 √dx √1 1 1 √1
R R  R
Then du = 3 dx, dx = 3 du, and = u u2 +9 3 du = 3 u u2 +9
du.
(3x−2) (3x−2)2 +9

Use the substitution u = 3 tan θ, − π2 < θ < π2 , to evaluate u√u12 +9 du.


R

√ √ √ √
Then du = 3 sec2 θ dθ and u2 + 9 = 9 tan2 θ + 9 = 3 tan2 θ + 1 = 3 sec2 θ = 3 sec θ since
− π2 < θ < π2 .
Then
1 1 1 1 1 sec θ 1 sec θ 1
Z Z Z Z
2
√ du = 3 sec θ dθ = dθ = dθ.
3 u u2 + 9 3 (3 tan θ)(3 sec θ) 9 tan θ 9 1 tan θ
1 1 cos θ 1 1 1 1
Z Z Z
dθ = dθ = csc θ dθ = − ln |cot θ + csc θ| + C.
9 cos θ sin θ 9 sin θ 9 9
7-134 Chapter 7 Techniques of Integration

√ 1
Since u2 + 9 = 3 sec θ, cos θ = = √ 3 and
sec θ u2 +9
r r
p 9 u2 u
sin θ = 1 − cos2 θ = 1− 2 = = ±√ .
u +9 u2+9 2
u +9

u2 +9
Since − π2 < θ < π
2, sin θ = √ u
u2 +9
and sec θ = u .
1 3
Since u = 3 tan θ, cot θ = tan θ = u .
So,

dx 1 1 1 1 3 u2 + 9
Z Z
p = √ du = − ln |cot θ + csc θ|+C = − ln + + C.
2
(3x − 2) (3x − 2) + 9 3 u u2 + 9 9 9 u u

√dx 1 √u 1 3x−2


R
Using laws of logarithms, = − ln + C = − ln +C=

2
(3x−2) (3x−2) +9 9 3+ u2 +9 9 2
3+ (3x−2) +9

3+ (3x−2)2 +9
− 91 ln 3x−2
+ C by rationalizing the denominator.

77. Let u = ex , then du = ex dx. We substitute and obtain


Z p Z p
ex 25 − e2x dx = 25 − u2 du.

Let u = 5 sin θ, then du = 5 cos θ dθ. We substitute and obtain


Z p Z q
x 2x 2
e 25 − e dx = 25 − (5 sin θ) (5 cos θ) dθ
Z p
= 5 25 − 25 sin2 θ cos θ dθ
Z
= 25 cos2 θ dθ

1 + cos (2θ)
Z
= 25 dθ
2
25
Z
= (1 + cos (2θ)) dθ
2
 
25 1
= θ + sin (2θ) + C
2 2
25 25
= θ+ sin θ cos θ + C.
2 2
p q 2 √
We have θ = sin−1 u5 , so cos θ = 1 − sin2 θ = 1 − u5 = 51 25 − u2 . We obtain


 
25 u 25  u  1 p
Z p
x
e 25 − e2x dx = sin−1 + 2
25 − u + C
2 5 2 5 5
25  u  1 p
= sin−1 + u 25 − u2 + C.
2 5 2
And since u = ex , we have

Z
ex
p
ex 25 − e2x dx = 25
sin−1 + 12 ex 25 − e2x + C.

2 5
7.3 Integration Using Trigonometric Substitution 7-135

78. Let u = ex , then du = ex dx. We substitute and obtain


Z p Z p
ex 4 + e2x dx = 4 + u2 du.

Let u = 2 tan θ, then du = 2 sec2 θ dθ. We substitute and obtain


Z p Z q
x 2
4 + (2 tan θ) 2 sec2 θ dθ

2x
e 4 + e dx =
Z p
= 2 4 + 4 tan2 θ sec2 θ dθ
Z
= 2 (2 sec θ) sec2 θ dθ
Z
= 4 sec3 θ dθ
 
1 1
= 4 sec θ tan θ + ln |sec θ + tan θ| + C
2 2
= 2 sec θ tan θ + 2 ln |sec θ + tan θ| + C.
√ q √
We have tan θ = u2 , and sec θ = 1 + tan2 θ = 1 + (u/2)2 = 12 4 + u2 . We obtain
 p   p  u 
1 u 1
Z p
ex 4 + e2x dx = 2 4 + u2 + 2 ln 4 + u2 + +C
2 2 2 2
1 p p
= u 4 + u2 + 2 ln 4 + u2 + u − 2 ln 2 + C

2
1 p p
= u 4 + u2 + 2 ln 4 + u2 + u + C.

2
And since u = ex , we have
√ √
Z p
ex 4 + e2x dx = 12 ex 4 + e2x + 2 ln 4 + e2x + ex + C .

79. Let u = sin−1 x and dv = x dx. Then du = √1−x


1
2
dx and v = 12 x2 . We use integration by
parts and obtain
Z   
1 2 −1 1 2 1
Z
x sin−1 x dx = x sin x − x √ dx
2 2 1 − x2
1 2 −1 1 x2
Z
= x sin x − √ dx.
2 2 1 − x2
Let x = sin θ, then dx = cos θ dθ. We substitute and obtain
1 2 −1 1 sin2 θ
Z Z
x sin−1 x dx = x sin x − p (cos θ) dθ
2 2 1 − sin2 θ
1 2 −1 1
Z
= x sin x − sin2 θ dθ
2 2
1 2 −1 1 1 − cos (2θ)
Z
= x sin x − dθ
2 2 2
1 2 −1 1
Z
= x sin x − (1 − cos (2θ)) dθ
2 4
 
1 2 −1 1 1
= x sin x − θ − sin 2θ + C
2 4 2
1 2 −1 1 1
= x sin x − θ + sin θ cos θ + C.
2 4 4
7-136 Chapter 7 Techniques of Integration

p √
We have θ = sin−1 x, and cos θ = 1 − sin2 θ = 1 − x2 . We obtain

Z
x sin−1 x dx = 12 x2 sin−1 x − 14 sin−1 x + 14 x 1 − x2 + C .

1
80. Let u = cos−1 x and dv = x dx. Then du = − √1−x 2
dx and v = 21 x2 . We use integration
by parts and obtain
Z   
1 2 1 2 1
Z
−1 −1
x cos x dx = x cos x − x −√ dx
2 2 1 − x2
1 2 1 x2
Z
= x cos−1 x + √ dx.
2 2 1 − x2
Let x = sin θ, then dx = cos θ dθ. We substitute and obtain

1 2 1 sin2 θ
Z Z
−1 −1
x cos x dx = x cos x + p (cos θ) dθ
2 2 1 − sin2 θ
1 2 1
Z
= x cos−1 x + sin2 θ dθ
2 2
1 2 1 1 − cos (2θ)
Z
−1
= x cos x + dθ
2 2 2
1 2 1
Z
= x cos−1 x + (1 − cos (2θ)) dθ
2 4
 
1 2 −1 1 1
= x cos x + θ − sin 2θ + C
2 4 2
1 2 1 1
= x cos−1 x + θ − sin θ cos θ + C.
2 4 4
p √
We have θ = sin−1 x, and cos θ = 1 − sin2 θ = 1 − x2 . We obtain

Z
x cos−1 x dx = 12 x2 cos−1 x + 41 sin−1 x − 41 x 1 − x2 + C .

81. (a) Let x = a tan θ, then dx = a sec2 θ dθ. We substitute and obtain
Z p Z q
(a tan θ)2 + a2 a sec2 θ dθ

x2 + a2 dx =
Z
2
= a sec3 θ dθ
 
2 1 1
= a sec θ tan θ + ln |sec θ + tan θ| + C
2 2
2
a
= (sec θ tan θ + ln |sec θ + tan θ|) + C.
2
√ q 
x 2

We have tan θ = xa , and sec θ = tan2 θ + 1 = a + 1 = a1 x2 + a2 . We obtain

a2
 p p 
1 x 1 x
Z p
x2 + a2 dx = x2 + a2 + ln x2 + a2 + + C
2 a a a a
1 2

x+ a2 +x2 1 √
= 2 a ln a + 2 x a2 + x2 + C .
7.3 Integration Using Trigonometric Substitution 7-137

(b) Let x = a sinh θ, then dx = a cosh θ dθ. We substitute and obtain


Z p Z q
2 2 2
x + a dx = (a sinh θ) + a2 (a cosh θ) dθ
Z p
= a2 sinh2 θ + 1 cosh θ dθ
Z
= a 2
cosh2 θ dθ
2
eθ + e−θ
Z 
= a2 dθ
2
a2
Z
e2θ + 2 + e−2θ dθ

=
4
a2 1 2θ
 
1
= e + 2θ − e−2θ + C
4 2 2
2
1 2 a 2θ
e − e−2θ + C

= a θ+
2 8
a2 eθ − e−θ
 θ
e + e−θ
 
1 2
= a θ+ +C
2 2 2 2
1 2 a2
= a θ+ sinh θ cosh θ + C.
2 2
p
2
q
2 √
x
Since sinh θ = a , cosh θ = sinh θ + 1 = (x/a) + 1 = a1 x2 + a2 . We obtain
Z p
1 2  x  a2  x   1 p 
x2 + a2 dx = a sinh−1 + x2 + a2 + C
2 a 2 a a
1 2 −1 x
 1

= 2 2
2 a sinh a + 2x a + x + C .

82. Since the ellipse is symmetric with respect to both the x-axis and y-axis, the total area A
of the ellipse is four times the area in the first quadrant, where 0 ≤ x ≤ a and 0 ≤ y ≤ b.
7-138 Chapter 7 Techniques of Integration

b

Express y as a function of x. y = a a2 − x2 .

So, the area A of the ellipse is four times the area under the graph of y = ab a2 − x2 on
Ra b√ a √
the interval 0 ≤ x ≤ a. That is, A = 4 0 a a2 − x2 dx = 4b
R
a 0 a2 − x2 dx.
Use the substitution x = a sin θ, − π2 ≤ θ ≤ π2 .
√ p p √
Then dx = a cos θ dθ and a2 − x2 = a2 − a2 sin2 θ = a 1 − sin2 θ = a cos2 θ =
a cos θ since − π2 ≤ θ ≤ π2 .
The lower limit of integration becomes θ = sin−1 (0) = 0, and the upper limit of integration
−1 a −1
(1) = π2 .

becomes u = sin a = sin

Then
a
4b
Z p
A= a2 − x2 dx
a 0
π/2
4b
Z
= (a cos θ)(a cos θ dθ)
a 0
Z π/2
= 4ab cos2 θ dθ
0
π/2
1
Z
= 4ab [1 + cos (2θ)]dθ
0 2
 π/2
1
= 2ab θ + sin (2θ)
2 0
   
π 1 1
= 2ab + sin (π) − 0 + sin (0) = πab .
2 2 2
83. Let x = a sin θ, then dx = a cos θ dθ. We substitute and obtain
dx a cos θ
Z Z
√ = q dθ
a2 − x2 a2 − (a sin θ)
2

a cos θ
Z
= p dθ
a2 − a2 sin2 θ
1 a cos θ
Z
= p dθ
a 1 − sin2 θ
cos θ
Z
= dθ
cos θ
= θ+C
= sin−1 xa + C .


84. Let x = a tan θ, then dx = a sec2 θ dθ. We substitute and obtain


dx a sec2 θ
Z Z
= 2 dθ
a2 + x2 a2 + (a tan θ)
a sec2 θ
Z
= dθ
a2 + a2 tan2 θ
1 sec2 θ
Z
= dθ
a sec2 θ
1
= θ+C
a
= a1 tan−1 xa + C .

7.3 Integration Using Trigonometric Substitution 7-139

85. Let x = a sec θ, then dx = a sec θ tan θ dθ. We substitute and obtain
dx a sec θ tan θ
Z Z
√ = q dθ
2
x x −a 2 2
a sec θ (a sec θ) − a2
1 tan θ
Z
= √ dθ
a sec2 θ − 1
1 tan θ
Z
= dθ
a tan θ
1
= θ+C
a
= a1 sec−1 xa + C .


86. Let x = a sec θ, then dx = a sec θ tan θ dθ. We substitute and obtain
dx 1
Z Z
√ = q (a sec θ tan θ) dθ
x2 − a2 (a sec θ)2 − a2
sec θ tan θ
Z
= dθ
tan θ
Z
= sec θ dθ

=ln |sec θ + tan θ| + C.


√ q 
x 2

We have sec θ = xa , and tan θ = sec2 θ − 1 = a − 1 = a1 x2 − a2 . We obtain

dx x 1p
Z

√ = ln + 2 2
x − a + C
2
x −a 2 a a

2 2
= ln x+ xa −a + C .

87. Let x = a tan θ, then dx = a sec2 θ dθ. We substitute and obtain


1 1
Z Z
a sec2 θ dθ

√ dx = q
2
x +a 2 2
(a tan θ) + a2
sec2 θ
Z
= dθ
sec θ
Z
= sec θ dθ

=
ln |sec θ + tan θ| + C1 .

x
√ q 
x 2

2
We have tan θ = a , and sec θ = tan θ + 1 = a + 1 = a1 x2 + a2 . We obtain
p
1 1 x
Z
√ dx = ln 2 2
x + a + + C1
x2 + a2 a a
p
= ln x + x2 + a2 − ln |a| + C1

√ 
= ln x + x2 + a2 + C ,

where C = − ln |a| + C1 .
7-140 Chapter 7 Techniques of Integration

Challenge Problems
2 2
88. We complete the square: 3x − x2 = 94 − x2 − 3x + 49 = 32 − x − 23 . Then


dx dx
Z Z
√ = q  2 .
3x − x 2 3 2
− x− 3
2 2

Let u = x − 23 , then du = dx. We substitute, use the result of problem 92, and obtain

dx du
Z Z
√ = q 
3x − x2 3 2
− u2
2
 
u
= sin−1 +C
(3/2)
!
−1 2 x − 23
= sin +C
3

= sin−1 2x−3

3 +C .

89. Let x = a sec θ, then dx = a sec θ tan θ dθ. We substitute and obtain
Z p Z q
x2 − a2 dx = (a sec θ)2 − a2 (a sec θ tan θ) dθ
Z
= a2 (tan θ)(sec θ tan θ) dθ
Z
= a2 tan2 θ sec θ dθ
Z
2
sec2 θ − 1 sec θ dθ

= a
Z
= a2 sec3 θ − sec θ dθ


 
2 1 1
= a sec θ tan θ + ln |sec θ + tan θ| − ln |sec θ + tan θ| + C
2 2
 
2 1 1
= a sec θ tan θ − ln |sec θ + tan θ| + C.
2 2
√ q 
x 2

We have sec θ = xa , and tan θ = sec2 θ − 1 = a − 1 = a1 x2 − a2 . We obtain
   p  
1 x 1 1 x 1 p 2
Z p
2

x2 − a2 dx = a 2 2
x − a − ln + 2
x − a + C
2 a a 2 a a
1 p 2 1 p 1
= x x − a2 − a ln x + x2 − a2 + a2 ln |a| + C
2
2 2 2
1
√ 1 2

= 2x x2 − a2 − a ln x + x2 − a2 + C .
2
7.3 Integration Using Trigonometric Substitution 7-141

90. Let u = sinh−1 x



a, so x = a sinh u, and dx = a cosh u du. We substitute and obtain

dx a cosh u
Z Z
√ = q du
2
x +a 2 2
(a sinh u) + a2
cosh u
Z
= p du
sinh2 u + 1
cosh u
Z
= du
cosh u
= u+C
x
= sinh−1 + C.
a

From the identity sinh−1 x = ln x + x2 + 1 we obtain


 
dx x
Z q
2
√ = ln + (x/a) + 1 + C
x2 + a2 a
 
x 1p 2 2
= ln + x +a +C
a a
 √ 
2 2
= ln x+ xa +a + C .

91. Let u = tan x, then du = sec2 x dx. We substitute and obtain

sec2 x du
Z Z
√ dx = √
2 2
u − 6u + 8
tan x − 6 tan x + 8
du
Z
= √
2
u − 6u + 9 − 1
du
Z
= q .
2
(u − 3) − 1

Let y = u − 3, then dy = du We substitute and obtain

sec2 x dy
Z Z
√ dx = p .
2
tan x − 6 tan x + 8 y2 − 1

Let y = sec θ, then dy = sec θ tan θ dθ. We substitute and obtain

sec2 x sec θ tan θ


Z Z
√ dx = √ dθ
tan2 x − 6 tan x + 8 sec2 θ − 1
sec θ tan θ
Z
= dθ
tan θ
Z
= sec θ dθ

= ln |sec θ + tan θ| + C.
√ p
We have sec θ = y, and tan θ = sec2 θ − 1 = y 2 − 1. We obtain

sec2 x
Z p
√ dx = ln

+ y 2 − 1 + C.
tan2 x − 6 tan x + 8
y
7-142 Chapter 7 Techniques of Integration

Since y = u − 3 = tan x − 3, we now have

sec2 x
Z q
2
√ dx = ln tan x − 3 + (tan x − 3) − 1 + C

2
tan x − 6 tan x + 8

= ln tan x − 3 + tan2 x − 6 tan x + 8 + C .

R
AP Practice Problems
R3√
1. To evaluate 0 9 − x2 dx, use the substitution x = 3 sin θ, − π2 ≤ θ ≤ π2 .
√ p p √
Then dx = 3 cos θ dθ and 9 − x2 = 9 − 9 sin2 θ = 3 1 − sin2 θ = 3 cos2 θ = 3 cos θ since
π π
−2 ≤ θ ≤ 2.

The lower limit of integration becomes θ = sin−1 0 = 0, and the upper limit of integration
becomes θ = sin−1 33 = π2 .
The integral becomes
Z 3 p Z π/2 Z π/2
2
9 − x dx = (3 cos θ)(3 cos θ dθ) = 9 cos2 θ dθ
0 0 0
π/2  π/2
1 9 1 9 h π
Z  i
9
=9 [1 + cos (2θ)]dθ = θ + sin (2θ) = + 0 − (0 + 0) = 4π .
0 2 2 2 0 2 2

The answer is C.
R √16−x2
2. To evaluate x2 dx, use the substitution x = 4 sin θ, − π2 ≤ θ ≤ π2 .
√ p p √
Then dx = 4 cos θ dθ and 16 − x2 = 16 − 16 sin2 θ = 4 1 − sin2 θ = 4 cos2 θ =
π π
4 cos θ since − 2 ≤ θ ≤ 2 .
The integral becomes
Z √
16 − x2 4 cos θ
Z Z Z
2
csc2 θ − 1 dθ = − cot θ − θ + C.

dx = 2 (4 cos θ dθ) = cot θ dθ =
x2 (4 sin θ)
r
√ √ 2
Since 16 − x2 = 4 cos θ, sec θ = √ 4 , tan θ = sec2 θ − 1 = √ 4 −1 =
16−x2 16−x2
√ √
2 16−x2
x
± √16−x 2
, and cot θ = ± 16−x
x . Since − π2 ≤ θ ≤ π
2, cot θ = x .

Since x = 4 sin θ, θ = sin−1 x4 .


Z √ √
16 − x2
− sin−1 x4 + C .
2
Therefore, 2
dx = −cot θ − θ + C = − 16−x
x
x
The answer is D.
R1 1 π π
3. To evaluate 0 (x2 +1) 3/2 dx, use the substitution x = tan θ, − 2 < θ < 2.
√ √ √
Then dx = sec2 θ dθ and x2 + 1 = tan2 θ + 1 = sec2 θ = sec θ since − π2 < θ < π
2.

The lower limit of integration becomes θ = tan−1 0 = 0, and the upper limit of integration
becomes θ = tan−1 1 = π4 .
7.3 Integration Using Trigonometric Substitution 7-143

R1 R π/4 R π/4 R π/4 √


1 1 1 π/4 2
So, 0 (x2 +1)3/2 dx = 0 sec3 θ sec2 θ dθ = 0 sec θ = 0 cos θ = [sin θ]0 = 2 .

The answer is B.
√ R4 √
x2 −4 x2 −4
4. Since y = x is nonnegative on [2, 4], A = 2 x dx is the area under the graph of

x2 −4
y= x from x = 2 to x = 4.
R √x2 −4
Use the substitution x = 2 sec θ (0 ≤ θ < π2 , π ≤ θ < 3π
2 ) to evaluate x dx.
√ √ √ √
Then dx = 2 tan θ sec θ dθ and x2 − 4 = 4 sec2 θ − 4 = 2 sec2 θ − 1 = 2 tan2 θ =
2 tan θ since 0 ≤ θ < π2 or π ≤ θ < 3π
2 .
2
The lower limit of integration becomes θ = sec−1 2 = 0, and the upper limit of integration
becomes θ = sec−1 42 = sec−1 2 = π3 .
4
√ Z π/3 
x2 − 4

2 tan θ
Z
Then A = dx = (2 tan θ sec θ dθ)
2 x 0 2 sec θ
Z π/3
=2 tan2 θ dθ
0
Z π/3
sec2 θ − 1 dθ

=2
0
π/3
= 2[tan θ − θ]0
h√ π i √
− (0 − 0) = 2 3 − π3 .

=2 3−
3

The answer is A.
1
tan θ, − π2 < θ < π √ dx
R
5. Use the substitution x = 3 2, to evaluate 1+9x2
.
√ q 2 √ √
Then dx = 13 sec2 θ dθ and 1 + 9x2 = 1+9 1
3 tan θ = 1 + tan2 θ = sec2 θ =
sec θ since − π2 < θ < π2 .
1 2 √
√ dx 3 sec θ dθ 1 1 1
R R R
Then 1+9x2
dx = sec θ = 3 sec θ dθ = 3 ln |sec θ + tan θ|+C = 3 ln 1 + 9x2 + 3x + C .

The answer is D.
√ R2 √
6. (a) Since f (x) = 4x2 + 1 is nonnegative on [−1, 2], A = −1 4x2 + 1 dx is the area
under the graph of f from x = −1 to x = 2.
R2 √
Use the substitution x = 21 tan θ, − π2 < θ < π2 , to evaluate −1 4x2 + 1 dx.
√ q √ √
Then dx = 21 sec2 θ dθ and 4x2 + 1 = 4 41 tan2 θ + 1 = tan2 θ + 1 = sec2 θ =


sec θ since − π2 < θ < π2 .


 
1 1
Z p Z Z
2
Then 2
4x + 1 dx = (sec θ) sec θ dθ = sec3 θ dθ.
2 2
Evaluate sec3 θ dθ using integration by parts.
R

Let u = sec θ and dv = sec2 θ dθ.


Then du = tan θ sec θ dx and v = sec2 θ dθ = tan θ.
R

Now sec3 θ dθ = sec θ tan θ − (tan θ)(tan θ sec θ dx) = tan θ sec θ − tan2 θ sec θ dθ.
R R R

Use the identity tan2 θ = sec2 θ − 1.


Z Z Z Z
sec3 θ dθ = tan θ sec θ − sec2 θ − 1 sec θ dθ = tan θ sec θ − sec3 θ dθ + sec θ dθ.

7-144 Chapter 7 Techniques of Integration

Add sec3 θ dθ to both sides.


R

Z Z
3
2 sec θ dθ = tan θ sec θ + sec θ dθ = tan θ sec θ + ln |sec θ + tan θ| + C

sec3 θ dθ = 12 [tan θ sec θ + ln |sec θ + tan θ|] + C.


R
and
1 1
Z p Z
So, 2
4x + 1 dx = sec3 θ dθ = [tan θ sec θ + ln |sec θ + tan θ|] + C.
2 4

From x = 12 tan θ, when x = −1, tan θ = −2 and sec θ = 5.

From x = 12 tan θ, when x = 2, tan θ = 4 and sec θ = 17.
2
1
Z
tan−1 (4)
p
Finally, A = 4x2 + 1 dx = [tan θ sec θ + ln |sec θ + tan θ|]tan−1 (−2)
−1 4
1 nh √ √ i h √ √ io
= 4 17 + ln 17 + 4 − −2 5 + ln 5−2
4
 √ √ √ √
= 14 4 17 + ln 17 + 4 + 2 5 − ln 5 − 2 .
 

(b) The region is pictured below. Rotate the region about the x-axis.

Using the method of disks, the volume is given by


Z 2 Z 2 p 2
2
V =π [f (x)] dx = π 4x2 + 1 dx
−1 1
Z 2
4x2 + 1 dx


1
2
4x3

=π +x
3 −1
   
32 −4
=π +2 − + (−1)
3 3
= 15π .
7.4 Integrands Containing ax2 + bx + c 7-145

(c) The region is shown below. Rotate the region about the y-axis.

Using the method of shells, the volume is given by


Z 1 Z 1 p
V = 2π xf (x) dx = 2π x 4x2 + 1 dx.
0 0

Now use the substitution u = 4x2 + 1. Then du = 8x dx and x dx = du


8 .
2
The lower limit of integration becomes u = 4 · 0 + 1 = 1, and the upper limit of
integration becomes u = 4 · 12 + 1 = 5.
Z 1 p
Then V = 2π x 4x2 + 1 dx
0
Z 1 p
= 2π 4x2 + 1(x dx)
0
Z 5
√ du
= 2π u
1 8
π 5√
Z
= u du
4 1
5
π u3/2

= 3
4 2 1

= π6 5 5 − 1 .


7.4 Integrands Containing ax2 + bx + c


Skill Building
2
1. We complete the square: x2 + 4x + 5 = (x + 2) + 1. Then we let u = x + 2, so du = dx,
and we obtain
dx dx
Z Z
2
= 2
x + 4x + 5 (x + 2) + 1
du
Z
= 2
u +1
= tan−1 u + C
= tan−1 (x + 2) + C .
7-146 Chapter 7 Techniques of Integration

2
2. We complete the square: x2 + 2x + 5 = (x + 1) + 4. Then we let u = x + 1, so du = dx,
and we obtain
dx dx
Z Z
= 2
x2 + 2x + 5 (x + 1) + 4
du
Z
=
u2 + 22
1 u
= tan−1 + C
2 2
1 −1 x+1

= 2 tan 2 +C .

2
3. We complete the square: x2 + 4x + 8 = (x + 2) + 4. Then we let u = x + 2, so du = dx,
and we obtain
dx dx
Z Z
2
= 2
x + 4x + 8 (x + 2) + 4
du
Z
=
u2 + 22
1 u
= tan−1 + C
2 2
1 −1 x+2

= 2 tan 2 +C .

2
4. We complete the square: x2 − 6x + 10 = (x − 3) + 1. Then we let u = x − 3, so du = dx,
and we obtain
dx dx
Z Z
= 2
x2 − 6x + 10 (x − 3) + 1
du
Z
= 2
u +1
= tan−1 u + C
= tan−1 (x − 3) + C .

2
5. We complete the square: 3 + 2x + 2x2 = 2 x + 12 + 25 . Then we let u = x + 1
2 to obtain

2 dx 2 dx
Z Z
2
= 2
3 + 2x + 2x 2 x + 21 + 25
du
Z
= 2
2u + 25
2

du
Z
=  √ 2
u2 + 25
 
2 u
= √ tan−1 √ +C
5 5/2
√ !
2 5 −1 2 x + 21
= tan √ +C
5 5
√  
= 2 5 5 tan−1 2x+1 √
5
+C .
7.4 Integrands Containing ax2 + bx + c 7-147

2
6. We complete the square: x2 + 6x + 10 = (x + 3) + 1. Then we let u = x + 3, so du = dx,
and we obtain
3 dx dx
Z Z
= 3 2
x2 + 6x + 10 (x + 3) + 1
du
Z
= 3
u2 + 1
= 3 tan−1 u + C
= 3 tan−1 (x + 3) + C .

7. We force the the derivative of the denominator to appear in the numerator. We then
2
complete the square with 2x2 + 2x + 3 = 2 x + 21 + 25 , and let u = x + 12 , so du = dx.
We obtain
Z 1 1
x dx 4 (4x + 2) − 2
Z
= dx
2x2 + 2x + 3 2x2 + 2x + 3
1 4x + 2 1 dx
Z Z
= dx −
4 2x2 + 2x + 3 2 2x2 + 2x + 3
1 2 1 dx
Z
= ln 2x + 2x + 3 − 2
4 2 2x + 2x + 3
1  1 dx
Z
2
= ln 2x + 2x + 3 − 2
4 2 2 x + 21 + 25
1  1 du
Z
= ln 2x2 + 2x + 3 −  √ 2
4 4
u2 + 25
 
1  1 2 u
= ln 2x2 + 2x + 3 − √ tan−1 √ +C
4 4 5 5/2
√ !
1 2
 5 −1 2 x + 12
= ln 2x + 2x + 3 − tan √ +C
4 10 5
 √  
= 14 ln 2x2 + 2x + 3 − 105 tan−1 2x+1 √
5
+C .

8. We force the the derivative of the denominator to appear in the numerator. We then
2
complete the square with x2 + 6x + 10 = (x + 3) + 1, and let u = x + 3, so du = dx. We
obtain
Z 3
3x dx 2 (2x + 6) − 9
Z
= dx
x2 + 6x + 10 x2 + 6x + 10
3 (2x + 6) dx
Z Z
= 2
dx − 9 2
2 x + 6x + 10 x + 6x + 10
3 2 dx
Z

= ln x + 6x + 10 − 9 2
2 (x + 3) + 1
3 du
Z
ln x2 + 6x + 10 − 9

=
2 u2 + 1
3
ln x2 + 6x + 10 − 9 tan−1 u + C

=
2
= 32 ln x2 + 6x + 10 − 9 tan−1 (x + 3) + C .

7-148 Chapter 7 Techniques of Integration

9. We complete the square: 8 + 2x − x2 = 9 − (x − 1)2 . Then we let u = x − 1 to obtain


dx dx
Z Z
√ = p
8 + 2x − x2 9 − (x − 1)2
du
Z
= √
3 − u2
2
u
= sin−1 +C
3
= sin−1 x−1

3 +C .

2
10. We complete the square: 5 − 4x − 2x2 = 7 − 2(x + 1) . Then we let u = x + 1, so du = dx,
and we obtain
dx dx
Z Z
√ = q
5 − 4x − 2x 2 2
7 − 2(x + 1)
du
Z
= √
7 − 2u2
1 du
Z
= √ r
2 √ 2
14
2 − u2
√  
2 u
= sin−1 √ +C
2 14/2
√ √ 
2 −1 14(x+1)
= 2 sin 7 +C .

2
11. We complete the square: 4x − x2 = 4 − (x − 2) . Then we let u = x − 2, so du = dx, and
we obtain
dx dx
Z Z
√ = q
4x − x2 2
4 − (x − 2)
du
Z
= √
22 − u2
u
= sin−1 +C
2
= sin−1 x−2

2 +C .

2
12. We complete the square: x2 − 6x − 10 = (x − 3) − 19. Then we let u = x − 3, so du = dx,
and we obtain
dx dx
Z Z
√ = q
x2 − 6x − 10 2
(x − 3) − 19
du
Z
= q √ 2
u2 − 19
p
= ln u + u2 − 19 + C (from problem 90, Section 7.3)

q
2
= ln (x − 3) + (x − 3) − 19 + C

= ln x − 3 + x2 − 6x − 10 + C .
7.4 Integrands Containing ax2 + bx + c 7-149

2
13. We complete the square: x2 + 2x + 2 = (x + 1) + 1. Then we let u = x + 1, so du = dx,
and we obtain
dx dx
Z Z
√ = q
(x + 1) x2 + 2x + 2 2
(x + 1) (x + 1) + 1
du
Z
= √
u u2 + 12

u2 + 1 − 1
= ln +C

u
q
2
(x + 1) + 1 − 1

= ln +C
x+1


2
= ln x +2x+2−1 +C .

x+1

2
14. We complete the square: x2 − 8x + 17 = (x − 4) + 1. Then we let u = x − 4, so du = dx,
and we obtain
dx dx
Z Z
√ = q
(x − 4) x2 − 8x + 17 (x − 4) (x − 4)2 + 1
du
Z
= √
u u2 + 12

u2 + 1 − 1
= ln +C

u
q
2
(x − 4) + 1 − 1

= ln +C
x−4


2
= ln x −8x+17−1 +C .

x−4

15. We complete the square: 24 − 2x − x2 = 25 − (x + 1)2 . Then we let u = x + 1, so du = dx,


and we obtain
dx dx
Z Z
√ = p
24 − 2x − x2 25 − (x + 1)2
du
Z
= √
5 − u2
2
u
= sin−1 +C
5
= sin−1 x+1

5 +C .
7-150 Chapter 7 Techniques of Integration

2
16. We complete the square: 9x2 + 6x + 10 = 9 x + 13 + 9. Then we let u = x + 1
3 to obtain

dx dx
Z Z
√ = q
2
9x + 6x + 10 2
9 x + 31 + 9
1 du
Z
= √
3 u + 12
2

1  p 
= ln u + u2 + 1 + C
3  
s
 2
1  1 1
= ln x + + x+ + 1 + C
3 3 3
r !
1 1 2 10
= ln x + + x2 + x + +C
3 3 3 9

= 13 ln 3x + 1 + 9x2 + 6x + 10 + C .


17. We force the the derivative of the denominator to appear in the numerator, and obtain
two integrals.
1
x −5 2 (2x − 2) − 4
Z Z
√ dx = √ dx
x2 − 2x + 5 x2 − 2x + 5
1 (2x − 2) dx
Z Z
= √ dx − 4 q .
2 x2 − 2x + 5 (x − 1)2 + 4

Now for the first integral let w = x2 − 2x + 5, so dw = (2x − 2) dx. In the second integral,
complete the square with x2 − 2x + 5 = (x − 1)2 + 4. ; and let u = x − 1, so du = dx. We
obtain
1 du
Z Z
= w−1/2 dw − 4 √
2 u + 22
2

1 √   p 
= 2 w − 4 ln u + u2 + 4 + C
2  
p q
2
= x2 − 2x + 5 − 4 ln x − 1 + (x − 1) + 4 + C
√ √ 
= x2 − 2x + 5 − 4 ln x − 1 + x2 − 2x + 5 + C .

18. We force the the derivative of the denominator to appear in the numerator. We then
2
complete the square with x2 − 4x + 3 = (x − 2) − 1, and let u = x − 2,so du = dx. We
obtain
7.4 Integrands Containing ax2 + bx + c 7-151

1
x +1 2 (2x − 4) + 3
Z Z
dx = dx
x2 − 4x + 3 x2 − 4x + 3
1 (2x − 4) dx
Z Z
= dx + 3 2
2 x2 − 4x + 3 (x − 2) − 1
1 2 du
Z

= ln x − 4x + 3 + 3
2 u2 − 1
 
1 2 1 1
= ln x − 4x + 3 + 3 ln |u − 1| − ln |u + 1| + C
2 2 2
 
1 2 1 1
= ln x − 4x + 3 + 3 ln |(x − 2) − 1| − ln |(x − 2) + 1| + C
2 2 2
1 3 3

= 2
2 ln x − 4x + 3 + 2 ln |x − 3| − 2 ln |x − 1| + C .

2
19. We complete the square: x2 − 2x + 5 = (x − 1) + 4, and let u = x − 1 to obtain
3 3
dx dx
Z Z
√ = q
2
x − 2x + 5 2
1 1 (x − 1) + 4
2
du
Z
= √
0 u + 22
2
h  p iu=2
= ln u + u2 + 4
 p  u=0 p 
= ln 2 + 22 + 4 − ln 0 + 02 + 4
 √ 
= ln 2 2 + 2 − ln 2
√ 
= ln 2 + 1 .

2
20. We complete the square: 2x − x2 = 1 − (x − 1) . Then we let u = x − 1 to obtain
1 1
x2 dx x2 dx
Z Z
√ = q
1/2 2x − x2 1/2 1 − (x − 1)2
0 2
(u + 1)
Z
= √ du
−1/2 1 − u2
0
u2 + 2u + 1
Z
= √ du
−1/2 1 − u2
0 Z 0 Z 0
u2 2u 1
Z
= √ du + √ du + √ du
1−u 2 1−u 2 1 − u2
−1/2 −1/2 −1/2
0 Z 0
u2 2u
Z
0
du + sin−1 u −1/2

= √ du + √
1−u 2 1−u 2
−1/2 −1/2
0 2 Z 0
u 2u π
Z
= √ du + √ du + .
−1/2 1 − u2 −1/2 1 − u 2 6
7-152 Chapter 7 Techniques of Integration

For the first integral we let u = sin θ, then we have


Z 0 Z 0
u2 sin2 θ
√ du = cos θ dθ
−1/2 1 − u2 −π/6 cos θ

1 0
Z
= (1 − cos (2θ)) dθ
2 −π/6
 0
1 1
= θ − sin 2θ
2 2 −π/6
1 1√
= π− 3.
12 8
And for the second integral we let w = 1 − u2 to obtain
Z 0 Z 1
2u
√ du = w−1/2 (− dw)
1 − u 2
−1/2 3/4
 √ 1
= −2 w 3/4

= 3 − 2.
And so we obtain
1
x2 dx 1√ √
  
1  π
Z
√ = π− 3 + 3−2 +
1/2 2x − x2 12 8 6
1 7

= 4π + 8 3 − 2 .

2
21. We complete the square: e2x + ex + 1 = ex + 12 + 43 . Then we let u = ex + 1
2 to obtain

ex dx ex dx
Z Z
√ = q
e2x + ex + 1 2
ex + 21 + 43
du
Z
= r   √ 2
3
u2 + 2
r !
3
= ln u + u2 + +C
4
 s 
2
x 1 x
1 3
= ln e + +
 e + + +C
2 2 4

1

ln ex +

= 2 + e2x + ex + 1 + C .

2
22. We complete the square: sin2 x + 4 sin x + 3 = (sin x + 2) − 1. Then we let u = sin x + 2
to obtain
7.4 Integrands Containing ax2 + bx + c 7-153

cos x dx cos x dx
Z Z
p = q
2 2
sin x + 4 sin x + 3 (sin x + 2) − 1
du
Z
= √
u2 − 1
p
= ln u + u2 − 1 + C

 q 
2
= ln sin x + 2 + (sin x + 2) − 1 + C
 p 
= ln sin x + 2 + sin2 x + 4 sin x + 3 + C .

23. We force the the derivative of the quadratic to appear in the numerator.

2x − 3 −(4 − 2x) + 1
Z Z
√ dx = √ dx
4x − x2 − 3 4x − x2 − 3
4 − 2x 1
Z Z
= − √ dx + q dx.
2
4x − x − 3 2
1 − (x − 2)

For the first integral let w = 4x − x2 − 3 so dw = (4 − 2x) dx. In the second integral,
2
complete the square with 4x − x2 − 3 = 1 − (x − 2) , and let u = x − 2, so du = dx. We
obtain
2x − 3 du
Z Z Z
√ dx = − w−1/2 dw + √
2
4x − x − 3 1 − u2
√  −1
= − 2 w + sin u + C

= −2 4x − x2 − 3 + sin−1 (x − 2) + C .

24. We force the the derivative of the quadratic to appear in the numerator.
1
x +3 2 (2x + 2) + 2
Z Z
√ dx = √ dx
2
x + 2x + 2 x2 + 2x + 2
1 2x + 2 dx
Z Z
= √ dx + 2 q .
2 x2 + 2x + 2 2
(x + 1) + 1

For the first integral we let w = x2 + 2x + 2 so dw = (2x + 2) dx. In the second integral,
we complete the square with x2 + 2x + 2 = (x + 1)2 + 1, and let u = x + 1, so du = dx.
We obtain
x +3 1 du
Z Z Z
√ dx = w−1/2 dw + 2 √
2
x + 2x + 2 2 u2 + 1
1 √   p 
= 2 w + 2 ln u + u2 + 1 + C
2  
p q
2
= x2 + 2x + 2 + 2 ln x + 1 + (x + 1) + 1 + C
√ √ 
= x2 + 2x + 2 + 2 ln x + 1 + x2 + 2x + 2 + C .
7-154 Chapter 7 Techniques of Integration

2
25. We complete the square: x2 − 2x + 10 = (x − 1) + 9. Then we let u = x − 1, so du = dx,
and obtain
dx dx
Z Z
3/2
= 3/2
2

(x − 2x + 10) 2
(x − 1) + 9
du
Z
= .
(u + 9)3/2
2

Let u = 3 tan θ, then du = 3 sec2 θ dθ. We substitute and obtain

dx 3 sec2 θ
Z Z
3/2
= 3/2 dθ
(x2 − 2x + 10)

2
(3 tan θ) + 9
sec2 θ
Z
= 3 3/2 dθ
9 tan2 θ + 9
1 sec2 θ
Z
= dθ
9 sec3 θ
1
Z
= cos θ dθ
9
1
= sin θ + C.
9
√ q 2 √
We have tan θ = u3 , so cot θ = u3 , and csc θ = 1 + cot2 θ = 1 + u3 = 1
u u2 + 9. So
sin θ = √uu2 +9 and we obtain

dx 1 u
Z
3/2
= √ +C
(x2 − 2x + 10) 9 u2 + 9
x−1
= q +C
2
9 (x − 1) + 9

= √ x−1 +C .
9 x2 −2x+10

2
26. We complete the square: x2 − 2x + 10 = (x − 1) + 9. Then we let u = x − 1 to obtain

dx dx
Z Z
√ = q
2
x − 2x + 10 (x − 1)2 + 9
du
Z
= √
u + 32
2
 p 
= ln u + u2 + 9 + C
 q 
2
= ln x − 1 + (x − 1) + 9 + C
√ 
= ln x − 1 + x2 − 2x + 10 + C .

2
27. We complete the square: x2 + 2x − 3 = (x + 1) − 4. Then we let u = x + 1, so du = dx,
and we obtain
7.4 Integrands Containing ax2 + bx + c 7-155

dx dx
Z Z
√ = q
2
x + 2x − 3 (x + 1)2 − 4
du
Z
= √
u2 − 4
p
= ln u + u2 − 4 + C

q
2
= ln x + 1 + (x + 1) − 4 + C


= ln x + 1 + x2 + 2x − 3 + C .

2
28. We complete the square: x2 − 4x − 1 = (x − 2) − 5. Then we let u = x − 2, so du = dx,
so we obtain
Z p Z q
2 2
x x − 4x − 1 dx = x (x − 2) − 5 dx
Z p
= (u + 2) u2 − 5 du
Z p Z p
= 2
u u − 5 du + 2 u2 − 5 du.

With the first integral, let w = u2 − 5, so dw = 2 du, and we obtain


1 √
Z p Z
u u2 − 5 du = w dw
2
 
1 2 3
= w2 + C
2 3
1 2 3
= u − 5 2 + C.
3
√ √
And for the second integral, let u = 5 sec θ, so du = 5 sec θ tan θ dθ, and we obtain
Z p Z p √
u2 − 5 du = 5 sec2 θ − 5 5 sec θ tan θ dθ
Z
= 5 sec θ tan2 θ dθ
Z
sec3 θ − sec θ dθ

= 5
 
1 1
= 5 sec θ tan θ − ln |sec θ + tan θ| + C
2 2
r 2 r !
u2

1 u u 1 u
= 5 √ − 1 − ln √ + − 1 + C

2 5 5 2 5 5
1 p 5 p
= u u2 − 5 − ln u2 − 5 + u + C.

2 2
7-156 Chapter 7 Techniques of Integration

So we obtain
 p 
1 2 1 5 p
Z p
3
x x2 − 4x − 1 dx = u − 5 2 + 2 u u2 − 5 − ln u2 − 5 + u + C

3 2 2
  23
2
(x − 2) − 5 q
2
= + (x − 2) (x − 2) − 5
3
q
2
−5 ln x − 2 + (x − 2) − 5 + C

3
(x2 −4x−1) 2 √ √
= 3 + (x − 2) x2 − 4x − 1 − 5 ln x − 2 + x2 − 4x − 1 + C .

2
29. We complete the square: 5 + 4x − x2 = 9 − (x − 2) . Then we let u = x − 2, so du = dx,
and we obtain
Z √
q
2
5 + 4x − x2
Z 9 − (x − 2)
dx = dx
x−2 x−2

9 − u2
Z
= du.
u
Let u = 3 sin θ, so du = 3 cos θ, and we have

Z √
q
2
5 + 4x − x2
Z 9 − (3 sin θ)
dx = (3 cos θ) dθ
x−2 3 sin θ
cos2 θ
Z
= 3 dθ
sin θ
1 − sin2 θ
Z
= 3 dθ
sin θ
Z
= 3 (csc θ − sin θ) dθ

= 3(− ln |csc θ + cot θ| + cos θ) + C


3 √9 − u2 1 p
!
= 3 − ln + + 9 − u2 + C

u u 3
q
9 − (x − 2)2 q

3
= −3 ln + + 9 − (x − 2)2 + C
x − 2 x − 2

√ √
= 3 ln |x − 2| − 3 ln 3 + 5 + 4x − x2 + 5 + 4x − x2 + C .

30. We complete the square: 5 + 4x − x2 = 9 − (x − 2)2 . Then we let u = x − 2, so du = dx,


and we obtain
Z p Z q
2
5 + 4x − x dx = 9 − (x − 2)2 dx
Z p
= 9 − u2 du.
7.4 Integrands Containing ax2 + bx + c 7-157

Let u = 3 sin θ, so du = 3 cos θ, and we obtain


Z p Z q
5 + 4x − x2 dx = 9 − (3 sin θ)2 (3 cos θ) dθ
Z
= 9 cos2 θ dθ
9
Z
= (1 + cos (2θ)) dθ
2
 
9 1
= θ + sin 2θ + C
2 2
9
= (θ + sin θ cos θ) + C
2
 u  u √ 9 − u2
!!
9
= sin−1 + +C
2 3 3 3
9 u 1 p
= sin−1 + u 9 − u2 + C
2  3 2

9 x−2 1
q
= sin −1
+ (x − 2) 9 − (x − 2)2 + C
2 3 2

= 92 sin−1 x−2 + 12 (x − 2) 5 + 4x − x2 + C .

3

31. We force the the derivative of the quadratic to appear in the numerator.
Z 1
x dx 2 (2x + 2) − 1
Z
√ = √ dx
2
x + 2x − 3 x2 + 2x − 3
1 2x + 2 dx
Z Z
= √ dx − q .
2 2
x + 2x − 3 2
(x + 1) − 4

In the first integral, let w = x2 + 2x − 3, so dw = (2x + 2) dx. In the second integral,


2
complete the square with x2 + 2x − 3 = (x + 1) − 4, and let u = x + 1, so du = dx. We
obtain
x dx 1 du
Z Z Z
√ = w−1/2 dw − √
2
x + 2x − 3 2 u2 − 4
1 √  p
= 2 w − ln u + u2 − 4 + C

2
q
2
p
= 2
x + 2x − 3 − ln x + 1 + (x + 1) − 4 + C

√ √
= x2 + 2x − 3 − ln x + 1 + x2 + 2x − 3 + C .

32. We force the the derivative of the quadratic to appear in the numerator.
Z 1
x dx 2 (2x − 4) + 2
Z
√ = √ dx
2
x − 4x + 3 x2 − 4x + 3
1 2x − 4 dx
Z Z
= √ dx + 2 q .
2 x2 − 4x + 3 (x − 2)2 − 1

In the first integral let w = x2 − 4x + 3, so dw = (2x − 4) dx. In the second integral,


complete the square with x2 − 4x + 3 = (x − 2)2 − 1, and let u = x − 2, so du = dx. We
obtain
7-158 Chapter 7 Techniques of Integration

x dx 1 du
Z Z Z
−1/2
√ = w dw + 2 √
2
x − 4x + 3 2 u2 − 1
1 √  p
= 2 w + 2 ln u + u2 − 1 + C

2
p q
x2 − 4x + 3 + 2 ln x − 2 + (x − 2)2 − 1 + C

=
√ √
= x2 − 4x + 3 + 2 ln x − 2 + x2 − 4x + 3 + C .

Applications and Extensions


33. Let u = x + h, then
dx du
Z Z
q = √ .
2
(x + h) + k u2 + k

We then apply the result of problem 87, section 7.3, to obtain


dx
Z h p i
q = ln u + u2 + k + C
(x + h)2 + k
q 
2
= ln (x + h) + k + x + h + C .

b 2 b2 b 2 2
34. We complete the square: ax2 + bx + c = a x + 2a − b −4ac
 
+ c − 4a = a x + 2a 4a . Then
b
let u = x + 2a , so du = dx, and we obtain

dx dx
Z Z
√ = q
2
ax + bx + c b 2 2
− b −4ac

a x + 2a 4a
du
Z
= q
2
au2 − b −4ac
4a
du
Z
= q
2 −4ac 
a u2 − b 4a 2

1 du
Z
2
= √ r 2 (using a > 0 and b − 4ac > 0)
a  √
2
b −4ac
u2 − 2a


v !2
u
1 u 2
b − 4ac
= √ ln u + tu2 − +C

a 2a

s
 2 2

1 b b b − 4ac
= √ ln x + + x+ − + C.
a 2a 2a 4a2
7.4 Integrands Containing ax2 + bx + c 7-159

And then we rewrite this and obtain


r
dx 1 b b c
Z
√ = √ ln x +
2
+ x + x+ +C
ax2 + bx + c a 2a a a

1 b 1 p
2

= √ ln x +
+√ ax + bx + c + C
a 2a a
√ √
ax + √b + ax2 + bx + c

1 2 a
= √ ln √ +C

a a
√ √

1 b p 1
= √ ln ax + √ + ax2 + bx + c − √ ln a + C
a 2 a a
√ √
= √1a ln ax2 + bx + c + ax + 2√b a + C .

Challenge Problems
35. We rewrite
s q
2
s
r 2 (a + x)
a+x (a + x) (a + x) (a + x) a+x
= = 2 2
= √ =√ .
a−x (a − x) (a + x) a −x 2
a −x 2 a2 − x2

Then we obtain
Z r
a+x a+x
Z
dx = √ dx
a−x a2 − x2
a x
Z Z
= √ dx + √ dx.
2
a −x 2 a − x2
2

Let u = a2 − x2 , so du = −2x dx. Then we have

− 12 du
Z r
a+x x
Z
dx = a sin−1 + √
a−x a u
−1 x √
= a sin − u+C
a

= a sin−1 xa − a2 − x2 + C .

R
AP Practice Problems
1
dx contains the quadratic expression x2 + 6x + 13.
R
1. The integrand of x2 +6x+13

So, we complete the square in the denominator.


1 1 1
Z Z Z
dx = dx = dx.
x2 + 6x + 13 (x2 + 6x + 9) + (13 − 9) (x + 3)2 + 4
1 1
R R
Now use the substitution u = x + 3. Then du = dx and x2 +6x+13 dx = (x+3)2 +4 dx =
R 1 1 −1 u 1 −1 x+3
u2 +4 du = 2 tan 2 + C = 2 tan 2 +C .

The answer is B.
7-160 Chapter 7 Techniques of Integration

R2 x
2. To evaluate −1 x2 +2x+10 dx, use a manipulation that causes the derivative of the denom-
inator to appear in the numerator.
Z 2
1 2 (2x + 2) − 2 1 2
Z 2
x 2x + 2 1
Z Z
2 + 2x + 10
dx = 2 + 2x + 10
dx = 2 + 2x + 10
dx − 2 + 2x + 10
dx.
−1 x 2 −1 x 2 −1 x −1 x
R2 2x+2
To evaluate −1 x2 +2x+10 dx, use the substitution u = x2 + 2x + 10. Then du = (2x + 2) dx.

The lower limit of integration becomes u = (−1)2 + 2(−1) + 10 = 9, and the upper limit
of integration becomes u = 22 + 2(2) + 10 = 18.
Z 2 Z 18
2x + 2 1
Then 2
dx = du = [ln |u|]18
9 = ln 18 − ln 9 = ln 2.
−1 x + 2x + 10 9 u
R2 1
The integrand of −1 x2 +2x+10 dx contains the quadratic expression x2 + 2x + 10.
So, we complete the square in the denominator.
Z 2 Z 2 Z 2
1 1 1
2
dx = 2
dx = 2
dx.
−1 x + 2x + 10 −1 (x + 2x + 1) + (10 − 1) −1 (x + 1) + 9

Now use the substitution u = x + 1. Then du = dx. The lower limit of integration becomes
u = −1 + 1 = 0, and the upper limit of integration becomes u = 2 + 1 = 3.
Z 2 Z 2 Z 3
1 1 1 1 h −1 u i3
Then 2
dx = 2
dx = 2
du = tan
−1 x + 2x + 10 −1 (x + 1) + 9 0 u +9 3 3 0
1 1 π
  π
= tan−1 1 − tan−1 0 =

−0 = .
3 3 4 12
Z 2
1 2
Z 2
x 2x + 2 1
Z
So, 2
dx = 2
dx − 2
dx = 12 ln 2 − 12
π
.
−1 x + 2x + 10 2 −1 x + 2x + 10 −1 x + 2x + 10

The answer is D.

7.5 Integration of Rational Functions Using Partial


Fractions; The Logistic Model
Concepts and Vocabulary
1. (a) less than
2. True
3. True
D
4. False, the term (x+1)4
is also needed

Skill Building
x2 +1
5. Apply polynomial division to x+1 .

x−1
(x + 1) x2 + 0x + 1
− (x2 + x)
−x + 1
− (−x − 1)
2
The quotient is x − 1 and the remainder is 2.
7.5 Integration of Rational Functions Using Partial Fractions; 7-161
The Logistic Model

Therefore,
x2 + 1 2
= x−1+ (x+1) .
x+1
x2 +4
6. Apply polynomial division to x−2 .

x+2
2
(x − 2) x + 0x + 4
−(x2 − 2x)
2x + 4
−(2x − 4)
8

The quotient is x + 2 and the remainder is 8.


Therefore,
x2 + 4 8
= x+2+ x−2 .
x−2
x3 +3x−4
7. Apply polynomial division to x−2 .

x2 + 2x + 7
(x − 2) x + 0x2 + 3x − 4
3

−(x3 − 2x2 )
2x2 + 3x
−(2x2 − 4x)
7x − 4
−(7x − 14)
10

The quotient is x2 + 2x + 7 and the remainder is 10.


Therefore,
x3 + 3x − 4 10
= x2 + 2x + 7 + x−2 .
x−2
x3 −3x2 +4
8. Apply polynomial division to x+3 .

x2 − 6x + 18
(x + 3) x − 3x2 + 0x + 4
3

−(x3 + 3x2 )
−6x2 + 0x
−(−6x2 − 18x)
18x + 4
−(18x + 54)
−50

The quotient is x2 − 6x + 18 and the remainder is −50.


Therefore,
x3 − 3x2 + 4 50
= x2 − 6x + 18 − x+3 .
x+3
7-162 Chapter 7 Techniques of Integration

2x3 +3x2 −17x−27


9. Apply polynomial division to x2 −9 .
2x + 3
(x2 − 9) 2x3 + 3x2 − 17x − 27
−(2x3 − 18x)
2
3x + x − 27
−(3x2 − 27)
x
The quotient is 2x + 3 and the remainder is x.
Therefore,
2x3 + 3x2 − 17x − 27 x
= 2x + 3 + x2 −9 .
x2 − 9
3x3 −2x2 −3x+2
10. Apply polynomial division to x2 −1 .
3x − 2
(x2 − 1) 3x3 − 2x2 − 3x + 2
−(3x3 − 3x)
−2x2 +2
−(−2x2 + 2)
0
The quotient is 3x − 2 and the remainder is 0.
Therefore,
3x3 − 2x2 − 3x + 2
= 3x − 2 .
x2 − 1
3x3 −2x2 −3x+2 x2 (3x−2)−(3x−2) (x2 −1)(3x−2)
Also, note that x2 −1 = x2 −1 = x2 −1 = 3x − 2 .
x4 −1 x4 −1
11. Apply polynomial division to x(x+4) = x2 +4x .

x2 − 4x + 16
2
(x + 4x) x + 0x + 0x2 + 0x − 1
4 3

−(x4 + 4x3 )
−4x3 + 0x2
−(−4x3 − 16x2 )
16x2 + 0x
−(16x2 + 64x)
−64x − 1
The quotient is x2 − 4x + 16 and the remainder is −64x − 1.
Therefore,
x4 − 1
= x2 − 4x + 16 − x64x+1
2 +4x .
x(x + 4)
x4 +x2 +1 x4 +x2 +1
12. Apply polynomial division to x(x−2) = x2 −2x .

x2 + 2x + 5
2
(x − 2x) x + 0x + x2 + 0x + 1
4 3

−(x4 − 2x3 )
2x3 + x2
−(2x3 − 4x2 )
5x2 + 0x
−(5x2 − 10x)
10x + 1
The quotient is x2 + 2x + 5 and the remainder is 10x + 1.
7.5 Integration of Rational Functions Using Partial Fractions; 7-163
The Logistic Model

Therefore,
x4 + x2 + 1 10x+1
= x2 + 2x + 5 + x(x−2) .
x(x − 2)
2x4 +x2 −2
13. Apply polynomial division to x2 +4 .

2x2 −7
2
(x + 4) 2x + 0x + x2 + 0x − 2
4 3

−(2x4 + 8x2 )
−7x2 −2
−(−7x2 − 28)
26

The quotient is 2x2 − 7 and the remainder is 26.


Therefore,
2x4 + x2 − 2 26
= 2x2 − 7 + x2 +4 .
x2 + 4
3x4 +x2
14. Apply polynomial division to x2 +9 .

3x2 − 26
2
(x + 9) 3x + 0x + x2 + 0x + 0
4 3

−(3x4 + 27x2 )
−26x2
−(−26x2 − 234)
234

The quotient is 3x2 − 26 and the remainder is 234.


Therefore,
3x4 + x2 234
= 3x2 − 26 + x2 +9 .
x2 + 9
15. We use long division to obtain
Z 2 Z  
x +1 2
dx = x−1+ dx
x+1 x+1
1 2
= 2x − x + 2 ln |x + 1| + C .

16. We use long division to obtain


Z 2 Z  
x +4 8
dx = x+2+ dx
x−2 x−2
1 2
= 2x + 2x + 8 ln |x − 2| + C .

17. We use long division to obtain


Z 3 Z  
x + 3x − 4 2 10
dx = x + 2x + 7 + dx
x−2 x−2
1 3
= 3x + x2 + 7x + 10 ln |x − 2| + C .
7-164 Chapter 7 Techniques of Integration

18. We use long division to obtain


Z 3
x − 3x2 + 4
Z  
2 50
dx = x − 6x + 18 − dx
x+3 x+3
1 3
= 3x − 3x2 + 18x − 50 ln |x + 3| + C .

2x3 +3x2 −17x−27 2x3 +3x2 −17x−27


R
19. To evaluate x2 −9 dx, apply polynomial division to x2 −9 .

2x + 3
(x2 − 9) 2x3 + 3x2 − 17x − 27
−(2x3 − 18x)
3x2 + x − 27
− (3x2 − 27)
x
The quotient is 2x + 3 and the remainder is x.
2x3 +3x2 −17x−27 x
So, x2 −9 = 2x + 3 + x2 −9 .
Therefore,
3x3 − 2x2 − 3x + 2
Z  
x
Z
1

dx = 2x + 3 + 2 dx = x2 + 3x + 2 ln x2 − 9 + C .
x2 − 1 x −9

3x3 −2x2 −3x+2 3x3 −2x2 −3x+2


R
20. To evaluate x2 −1 dx, apply polynomial division to x2 −1 .

3x − 2
(x2 − 1) 3x3 − 2x2 − 3x + 2
−(3x3 − 3x)
2
−2x +2
−(−2x2 + 2)
0
The quotient is 3x − 2 and the remainder is 0.
3x3 −2x2 −3x+2
So, x2 −1 = 3x − 2.
3x3 −2x2 −3x+2 x2 (3x−2)−(3x−2) (x2 −1)(3x−2)
Also, note that x2 −1 = x2 −1 = x2 −1 = 3x − 2.
Therefore,
3x3 − 2x2 − 3x + 2
Z Z
3 2
dx = (3x − 2) dx = 2x − 2x + C .
x2 − 1

21. We use partial fractions to obtain


1 A B
= +
(x − 2)(x + 1) x−2 x+1
1 = A(x + 1) + B(x − 2)

When x = −1 we obtain B = − 31 , and when x = 2 we have A = 13 . So we obtain


Z  
1 1/3 1/3
Z
dx = − dx
(x − 2)(x + 1) x−2 x+1
1 1
= 3 ln |x − 2| − 3 ln |x + 1| + C .
7.5 Integration of Rational Functions Using Partial Fractions; 7-165
The Logistic Model

22. We use partial fractions to obtain


1 A B
= +
(x + 4)(x − 1) x+4 x−1
1 = A(x − 1) + B(x + 4)

When x = 1 we obtain B = 51 , and when x = −4 we have A = − 15 . So we obtain


Z  
1 −1/5 1/5
Z
dx = + dx
(x + 4)(x − 1) x+4 x−1
= − 15 ln |x + 4| + 1
5 ln |x − 1| + C .

23. We use partial fractions to obtain


x A B
= +
(x − 1)(x − 2) x−1 x−2
x = A(x − 2) + B(x − 1)
When x = 2 we obtain B = 2, and when x = 1 we have A = −1. So we obtain
Z  
x dx 1 2
Z
= − + dx
(x − 1)(x − 2) x−1 x−2
= − ln |x − 1| + 2 ln |x − 2| + C .

24. We use partial fractions to obtain


3x A B
= +
(x + 2)(x − 4) x+2 x−4
3x = A(x − 4) + B(x + 2)
When x = 4 we obtain B = 2, and when x = −2 we have A = 1. So we obtain
Z  
3x dx 1 2
Z
= + dx
(x + 2)(x − 4) x+2 x−4
= ln |x + 2| + 2 ln |x − 4| + C .

25. We use partial fractions to obtain


x A B
= +
(3x − 2)(2x + 1) 3x − 2 2x + 1
x = A(2x + 1) + B(3x − 2)

When x = − 12 we obtain B = 17 , and when x = 23 we have A = 72 . So we obtain


Z  
x dx 2/7 1/7
Z
= + dx
(3x − 2)(2x + 1) 3x − 2 2x + 1
2 1
= 21 ln |3x − 2| + 14 ln |2x + 1| + C .

26. We use partial fractions to obtain


1 A B
= +
(2x + 3)(4x − 1) 2x + 3 4x − 1
1 = A(4x − 1) + B(2x + 3)
7-166 Chapter 7 Techniques of Integration

1
When x = 4 we obtain B = 72 , and when x = − 23 we have A = − 71 . So we obtain
Z  
dx −1/7 2/7
Z
= + dx
(2x + 3)(4x − 1) 2x + 3 4x − 1
1 1
= − 14 ln |2x + 3| + 14 ln |4x − 1| + C .

27. We use partial fractions to obtain


x−3 A B C
= + +
(x + 2)(x + 1)2 x + 2 x + 1 (x + 1)2
2
x−3 = A(x + 1) + B(x + 1)(x + 2) + C(x + 2)
When x = −1 we obtain C = −4, and when x = −2 we have A = −5. When x = 0 we
obtain B = 5. So we obtain
!
x−3 −5 5 −4
Z Z
2 dx = + +
x + 2 x + 1 (x + 1)2
dx
(x + 2)(x + 1)
−5 5 −4
Z Z Z
= dx + dx + 2 dx
x+2 x+1 (x + 1)
4
= −5 ln |x + 2| + 5 ln |x + 1| + x+1 +C .

28. We use partial fractions to obtain


x+1 A B C
= + + 2
x2 (x − 2) x−2 x x
x+1 = Ax2 + Bx(x − 2) + C(x − 2)
When x = 0 we obtain C = −1/2, and when x = 2 we have A = 3/4. When x = 1 we
obtain B = −3/4. So we obtain
Z  
x+1 3/4 −3/4 −1/2
Z
dx = + + dx
x2 (x − 2) x−2 x x2
3/4 −3/4 −1/2
Z Z Z
= dx + dx + dx
x−2 x x2
3 3 1
= 4 ln |x − 2| − 4 ln |x| + 2x +C .

29. We use partial fractions to obtain


x2 A B C
2 = + +
(x − 1) (x + 1) x + 1 x − 1 (x − 1)2
2
x2 = A(x − 1) + B(x − 1)(x + 1) + C(x + 1)
When x = 1 we obtain C = 1/2, and when x = −1 we have A = 1/4. When x = 0 we
obtain B = 3/4. So we obtain
!
x2 1/4 3/4 1/2
Z Z
dx = + + dx
(x − 1)2 (x + 1) x + 1 x − 1 (x − 1)2
1/4 3/4 1/2
Z Z Z
= dx + dx + 2 dx
x+1 x−1 (x − 1)
1 3 1
= 4 ln |x + 1| + 4 ln |x − 1| − 2(x−1) +C .
7.5 Integration of Rational Functions Using Partial Fractions; 7-167
The Logistic Model

30. We use partial fractions to obtain


x2 + x A B C
2 = + +
(x + 2)(x − 1) x + 2 x − 1 (x − 1)2
2
x2 + x = A(x − 1) + B(x − 1)(x + 2) + C(x + 2)
When x = 1 we obtain C = 2/3, and when x = −2 we have A = 2/9. When x = 0 we
obtain B = 7/9. So we obtain
!
x2 + x 2/9 7/9 2/3
Z Z
2 dx = + +
x + 2 x − 1 (x − 1)2
dx
(x + 2)(x − 1)
2/9 7/9 2/3
Z Z Z
= dx + dx + 2 dx
x+2 x−1 (x − 1)
2 7 2
= 9 ln |x + 2| + 9 ln |x − 1| − 3(x−1) +C .

31. We use partial fractions to obtain


1 A Bx + C
= + 2
x(x2 + 1) x x +1
2

1 = A x + 1 + (Bx + C)x
When x = 0 we obtain A = 1. So
= (1) x2 + 1 + (Bx + C)x

1
1 = (B + 1)x2 + Cx + 1
Equating coefficients, we obtain B = −1 and C = 0. We now have
Z  
dx 1 −x
Z
= + dx
x(x2 + 1) x x2 + 1
1 −x
Z Z
= dx + 2
dx
x x +1
= ln |x| − 12 ln x2 + 1 + C .


32. We use partial fractions to obtain


1 A Bx + C
= + 2
(x + 1)(x2 + 4) x+1 x +4
2

1 = A x + 4 + (Bx + C)(x + 1)
When x = −1 we obtain A = 1/5. So
= (1/5) x2 + 4 + (Bx + C)(x + 1)

1
   
1 2 4
1 = B+ x + (B + C)x + C +
5 5
Equating coefficients, we obtain B = −1/5 and C = 1/5. We now have
Z  
dx 1/5 −(1/5)x + 1/5
Z
= + dx
(x + 1)(x2 + 4) x+1 x2 + 4
1/5 −(1/5)x 1/5
Z Z Z
= dx + 2
dx + 2
dx
x+1 x +4 x +4
1
= 15 ln |x + 1| − 10 ln x2 + 4 + 101
tan−1 x2 + C .
 
7-168 Chapter 7 Techniques of Integration

33. We use partial fractions to obtain


x2 + 2x + 3 A Bx + C
= +
(x + 1)(x2 + 2x + 4) x + 1 x2 + 2x + 4
x2 + 2x + 3 = A x2 + 2x + 4 + (Bx + C)(x + 1)


When x = −1 we obtain A = 2/3. So

x2 + 2x + 3 = A x2 + 2x + 4 + (Bx + C)(x + 1)

     
2 2 4 8
x2 + 2x + 3 = B+ x + B+C + x+ C +
3 3 3
Equating coefficients, we obtain B = 1/3 and C = 1/3. We now have

x2 + 2x + 3
Z  
2/3 (1/3)x + 1/3
Z
dx = + dx
(x + 1)(x2 + 2x + 4) x+1 x2 + 2x + 4
2/3 1 2x + 2
Z Z
= dx + dx
x+1 6 x2 + 2x + 4
= 23 ln |x + 1| + 16 ln x2 + 2x + 4 + C .


34. We use partial fractions to obtain


x2 − 11x − 18 A Bx + C
+ 2 =
x(x2 + 3x + 3) x x + 3x + 3
x2 − 11x − 18 = A x2 + 3x + 3 + (Bx + C)x


When x = 0 we obtain A = −6. So


x2 − 11x − 18 = A x2 + 3x + 3 + (Bx + C)x


x2 − 11x − 18 = (B − 6)x2 + (C − 18)x − 18


Equating coefficients, we obtain B = 7 and C = 7. We now have
Z 2 Z  
x − 11x − 18 −6 7x + 7
dx = + dx
x(x2 + 3x + 3) x x2 + 3x + 3
Z 7 7
−6 2 (2x + 3) − 2
Z
= dx + 2
dx
x x + 3x + 3
−6 7 2x + 3 7 1
Z Z Z
= dx + dx −  √ 2 dx
x 2 x2 + 3x + 3 2 2
x + 23 + 23

!
7 2
 7 1 −1 x + 32
= −6 ln |x| + ln x + 3x + 3 − √ tan √ +C
2 2 3 3
2 2
√ √ 
3(2x+3)
−6 ln |x| + 27 ln x2 + 3x + 3 − 7 3 3 tan−1

= 3 +C .

35. We expand to obtain


2x + 1 2x 1
Z Z Z
2 dx = 2 dx + 2 dx
(x2 + 16) (x2 + 16) (x2 + 16)
1 1
Z
= − 2
+ 2 dx.
x + 16 (x2 + 16)
7.5 Integration of Rational Functions Using Partial Fractions; 7-169
The Logistic Model

Let x = 4 tan θ to obtain


2x + 1 1 4 sec2 θ
Z Z
2 2 dx = − x2 + 16 +
256 sec4 θ

(x + 16)
1 1
Z
= − 2 + cos2 θ dθ
x + 16 64
1 1
Z
= − 2 + (1 + cos 2θ) dθ
x + 16 128
 
1 1 1
= − 2 + θ + sin 2θ + C
x + 16 128 2
1 1 1
= − 2 + θ+ cos θ sin θ + C
x + 16 128 128
1 1 x 1 4 x
= − 2 + tan−1 + √ √ +C
x + 16 128 4 128 x2 + 16 x2 + 16
1 1
tan−1 x4 + 32(x2x+16) + C .

= − x2 +16 + 128

36. We use partial fractions to obtain

x2 + 2x + 3 Ax + B Cx + D
2 = +
(x2 + 4) x2 + 4 (x2 + 4)2
x2 + 2x + 3 (Ax + B) x2 + 4 + (Cx + D)

=
2
x + 2x + 3 = Ax3 + Bx2 + (4A + C)x + (4B + D).
Equating coefficients, we obtain A = 0, B = 1, C = 2, and D = −1, so
Z 2 !
x + 2x + 3 1 2x − 1
Z
2 dx = +
x2 + 4 (x2 + 4)2
dx
(x2 + 4)
1 2x 1
Z Z Z
= dx + 2 dx − 2 dx
x2 + 4 (x2 + 4) (x2 + 4)
1 x 1 1
Z
= tan−1 − 2 − 2 dx.
2 2 x +4 2
(x + 4)

Let x = 2 tan θ to obtain


Z 2
x + 2x + 3 1 −1 x 1 2 sec2 θ
  Z
dx = tan − − dθ
(x2 + 4)2 2 2 x2 + 4 16 sec4 θ
1 x 1 1
Z
= tan−1 − 2 − cos2 θ dθ
2 2 x +4 8
1 −1 x 1 1
  Z
= tan − 2 − (1 + cos 2θ) dθ
2 2 x + 4 16
 
1 x 1 1 1
= tan−1 − 2 − θ + sin 2θ + C
2 2 x + 4 16 2
1 x 1 1 1
= tan−1 − 2 − θ− cos θ sin θ + C
2 2 x + 4 16 16
1 x 1 1 x 1 2 x
= tan−1 − 2 − tan−1 − √ √ +C
2 2 x + 4 16 2 16 x2 + 4 x2 + 4
7
tan−1 x2 − x21+4 − 8(x2x+4) + C .

= 16
7-170 Chapter 7 Techniques of Integration

37. We use partial fractions to obtain

x3 Ax + B Cx + D Ex + F
3 = 2
+ 2 + 3
(x2 + 16) x + 16 2
(x + 16) (x2 + 16)
2
x3 (Ax + B) x2 + 16 + (Cx + D) x2 + 16 + Ex + F
 
=
3
x = Ax5 + Bx4 + (32A + C)x3 + (32B + D)x2
+ (256A + 16C + E)x + (256B + F + 16D).
Equating coefficients, we obtain A = 0, B = 0, C = 1, D = 0, E = −16, and F = 0. So
!
x3 dx x (−16)x
Z Z
3 = 2 + 3 dx
(x2 + 16) (x2 + 16) (x2 + 16)
x x
Z Z
= 2 dx − 16 3 dx
2
(x + 16) 2
(x + 16)
1 4
= − + +C .
2(x2 + 16) (x2 + 16)2

38. We use partial fractions to obtain

x2 Ax + B Cx + D Ex + F
3 = + 2 + 3
(x2 + 4) x2 + 4 2
(x + 4) (x2 + 4)
2
x2 = (Ax + B) x2 + 4 + (Cx + D) x2 + 4 + Ex + F


x2 = Ax5 + Bx4 + (8A + C)x3 + (8B + D)x2 + (16A + 4C + E)x + (16B + F + 4D).
Equating coefficients, we obtain A = 0, B = 0, C = 0, D = 1, E = 0, and F = −4. So
!
x2 dx 1 −4
Z Z
3 = 2 + 3 dx.
(x2 + 4) (x2 + 4) (x2 + 4)
7.5 Integration of Rational Functions Using Partial Fractions; 7-171
The Logistic Model

Let x = 2 tan θ to obtain


!
x2 dx 1 −4
Z Z
2 sec2 θ dθ

3 = 2 + 3
(x2 + 4) 4 tan2 θ + 4 4 tan2 θ + 4
 
Z  
1 −4
2 sec2 θ dθ

= 4
+ 6
16 sec θ 64 sec θ
Z  
1 1 1
sec2 θ dθ

= 4
− 6
8 sec θ sec θ
1
Z
cos2 θ − cos4 θ dθ

=
8
Z "  2 #
1 1 + cos (2θ) 1 + cos (2θ)
= − dθ
8 2 2
1 + cos (2θ) 1 + 2 cos (2θ) + cos2 (2θ)
Z  
1
= − dθ
8 2 4
1 cos2 (2θ)
Z  
1
= − dθ
8 4 4
Z  
1 1 + cos (4θ)
= 1− dθ
32 2
1
Z
= (1 − cos (4θ)) dθ
64
 
1 1
= θ − sin (4θ) + C
64 4
 
1 1
= θ − sin (2θ) cos (2θ) + C
64 2
1
θ − sin θ cos θ 2 cos2 θ − 1 + C

=
64 !!
 2
1 −1 x
  x 2 2
= tan −√ √ 2 √ −1 +C
64 2 x2 + 4 x2 + 4 x2 + 4

1 x x(x2 −4)
tan−1

= 64 2 + 32(x2 +4)2
+C .

39. We use partial fractions to obtain


x x A B
= = +
x2 + 2x − 3 (x + 3)(x − 1) x+3 x−1
x = A(x − 1) + B(x + 3)
When x = 1 we obtain B = 1/4, and when x = −3 we have A = 3/4. So we obtain
Z  
x dx 3/4 1/4
Z
= + dx
x2 + 2x − 3 x+3 x−1
3 1
= 4 ln |x + 3| + 4 ln |x − 1| + C .

40. We use partial fractions to obtain


x2 − x − 8 x2 − x − 8 A B C
= = + +
(x + 1)(x2 + 5x + 6) (x + 1)(x + 2)(x + 3) x+1 x+2 x+3
x2 − x − 8 = A(x + 2)(x + 3) + B(x + 1)(x + 3) + C(x + 1)(x + 2)
7-172 Chapter 7 Techniques of Integration

When x = −3 we obtain C = 2, when x = −2 we have B = 2, and when x = −1 we have


A = −3. So we obtain
x2 − x − 8
Z  
−3 2 2
Z
dx = + + dx
(x + 1)(x2 + 5x + 6) x+1 x+2 x+3
= −3 ln |x + 1| + 2 ln |x + 2| + 2 ln |x + 3| + C .

41. We use partial fractions to obtain

10x2 + 2x A B Cx + D
2 = + 2 + 2
(x − 1) (x2 + 2) x − 1 (x − 1) x +2
2 2
A(x − 1) x + 2 + B x2 + 2 + (Cx + D)(x − 1)
2
 
10x + 2x =

When x = 1 we obtain B = 4. We expand and obtain

10x2 + 2x = A(x − 1) x2 + 2 + 4 x2 + 2 + (Cx + D)(x − 1)2


 

10x2 + 2x = (A + C)x3 + (−A − 2C + D + 4)x2 + (2A + C − 2D)x + (−2A + D + 8)


Equating coefficients, we have
0 = A+C
10 = −A − 2C + D + 4
2 = 2A + C − 2D
0 = −2A + D + 8
We obtain A = −C, so 10 = −(−C) − 2C + D + 4 = D − C + 4 and D = 6 + C. Then
0 = −2A + D + 8 = −2(−C) + (6 + C) + 8 = 3C + 14, and C = −14/3. We now obtain
A = 14/3, and D = 4/3. So
!
10x2 + 2x 14/3 4 (−14/3)x + 4/3
Z Z
dx = + + dx
(x − 1)2 (x2 + 2) x − 1 (x − 1)2 x2 + 2
14/3 4 14 x 4 dx
Z Z Z Z
= dx + 2 dx − 3 2+2
dx + 2+2
x−1 (x − 1) x 3 x
 2√2 √ 
= 143 ln |x − 1| − 4
x−1 − 7
3 ln x2
+ 2 + 3 tan−1 2x
2 +C .

42. We use partial fractions to obtain


x+4 A B Cx + D
= + 2+ 2
x2 (x2 + 4) x x x +4
Ax x + 4 + B x2 + 4 + (Cx + D)x2
2
 
x+4 =

When x = 0 we obtain B = 1. We expand and obtain

x + 4 = Ax x2 + 4 + 1 x2 + 4 + (Cx + D)x2
 

x+4 = (A + C)x3 + (D + 1)x2 + 4Ax + 4


Equating coefficients, we have
0 = A+C
0 = D+1
1 = 4A
7.5 Integration of Rational Functions Using Partial Fractions; 7-173
The Logistic Model

We obtain A = 1/4, C = −1/4, and D = −1. So


Z  
x+4 1/4 1 (−1/4)x − 1
Z
dx = + 2+ dx
x2 (x2 + 4) x x x2 + 4
1/4 1 1 x dx
Z Z Z Z
= dx + 2
dx − 2
dx − 2
x x 4 x +4 x +4
1 1 1 2
 1 −1 x

= 4 ln |x| − x − 8 ln x + 4 − 2 tan 2 +C .

43. We use partial fractions to obtain


7x + 3 7x + 3 A B C
= = + +
x3 − 2x2 − 3x x(x + 1)(x − 3) x x+1 x−3
7x + 3 = A(x + 1)(x − 3) + Bx(x − 3) + Cx(x + 1)
When x = 3 we obtain C = 2, when x = −1, we get B = −1, and when x = 0, we have
A = −1. So
Z  
7x + 3 −1 −1 2
Z
dx = + + dx
x3 − 2x2 − 3x x x+1 x−3
−1 −1 2
Z Z Z
= dx + dx + dx
x x+1 x−3
= − ln |x| − ln |x + 1| + 2 ln |x − 3| + C .

44. We use partial fractions to obtain

(x + 1) x4 − x3 + x2 − x + 1

x5 + 1
=
x6 − x4 x4 (x − 1)(x + 1)
x4 − x3 + x2 − x + 1 A B C D E
= + 2+ 3+ 4 +
x4 (x − 1) x x x x x−1
x4 − x3 + x2 − x + 1 = Ax3 (x − 1) + Bx2 (x − 1) + Cx(x − 1) + D(x − 1) + Ex4
When x = 1 we obtain E = 1, and when x = 0, we have D = −1. So

x4 − x3 + x2 − x + 1 = Ax3 (x − 1) + Bx2 (x − 1) + Cx(x − 1) + (−1)(x − 1) + (1)x4


x4 − x3 + x2 − x + 1 = (A + 1)x4 + (B − A)x3 + (C − B)x2 + (−C − 1)x + 1.
Equating coefficients, we obtain A = 0, B = −1, and C = 0. We now integrate

x5 + 1
Z  
−1 −1 1
Z
dx = + 4 + dx
x6 − x4 x2 x x−1
−1 −1 1
Z Z Z
= dx + dx + dx
x2 x4 x−1
1 1
= x + 3x3 + ln |x − 1| + C .

45. We use partial fractions to obtain

x2 A B C
2 = + +
(x − 2)(x − 1) x − 2 x − 1 (x − 1)2
2
x2 = A(x − 1) + B(x − 2)(x − 1) + C(x − 2).
7-174 Chapter 7 Techniques of Integration

When x = 1 we obtain C = −1, and when x = 2, we have A = 4. So


2
x2 = 4(x − 1) + B(x − 2)(x − 1) + (−1)(x − 2)
x2 = (B + 4)x2 + (−3B − 9)x + (2B + 6).
Equating coefficients, we obtain B = −3. We now obtain
!
x2 4 −3 −1
Z Z
2 dx = + + dx
(x − 2)(x − 1) x − 2 x − 1 (x − 1)2
4 −3 −1
Z Z Z
= dx + dx + 2 dx
x−2 x−1 (x − 1)
1
= 4 ln |x − 2| − 3 ln |x − 1| + x−1 +C .

46. We use partial fractions to obtain

x2 + 1 A B C
= + +
(x + 3)(x − 1)2 x + 3 x − 1 (x − 1)2
2
x2 + 1 = A(x − 1) + B(x + 3)(x − 1) + C(x + 3).

When x = 1 we obtain C = 1/2, and when x = −3, we have A = 5/8. So


2
x2 + 1 = (5/8)(x − 1) + B(x + 3)(x − 1) + (1/2)(x + 3)
     
5 2 3 17
x2 + 1 = B+ x + 2B − x+ − 3B .
8 4 8

Equating coefficients, we obtain B = 3/8. We now obtain


!
x2 + 1 5/8 3/8 1/2
Z Z
dx = + + dx
(x + 3)(x − 1)2 x + 3 x − 1 (x − 1)2
5/8 3/8 1/2
Z Z Z
= dx + dx + 2 dx
x+3 x−1 (x − 1)
5 3 1
= 8 ln |x + 3| + 8 ln |x − 1| − 2(x−1) +C .

47. We use partial fractions to obtain


2x + 1 2x + 1 A Bx + C
= = + 2
x3 − 1 2
(x − 1)(x + x + 1) x−1 x +x+1
= A x2 + x + 1 + (Bx + C)(x − 1).

2x + 1

When x = 1 we obtain A = 1. So
(1) x2 + x + 1 + (Bx + C)(x − 1)

2x + 1 =
2x + 1 = (B + 1)x2 + (C − B + 1)x + (1 − C).
7.5 Integration of Rational Functions Using Partial Fractions; 7-175
The Logistic Model

Equating coefficients, we obtain B = −1, and C = 0. We now obtain


Z  
2x + 1 1 −x
Z
dx = + dx
x3 − 1 x − 1 x2 + x + 1
1 − 21 (2x + 1) + 12
Z Z
= dx + dx
x−1 x2 + x + 1
1 1 2x + 1 1 1
Z Z Z
= dx − 2
dx +  √ 2 dx
x−1 2 x +x+1 2 2
x + 21 + 23


 √ √ 
3(2x+1)
= ln |x − 1| − 12 ln x2 + x + 1 + 33 tan−1 3 +C .

48. We use partial fractions to obtain


1 1 A Bx + C
= = + 2
x3 −8 (x − 2)(x2
+ 2x + 4) x − 2 x + 2x + 4
2

1 = A x + 2x + 4 + (Bx + C)(x − 2).
1
When x = 2 we obtain A = 12 . So
 
1
x2 + 2x + 4 + (Bx + C)(x − 2)

1 =
12
     
1 2 1 1
1 = B+ x + C − 2B + x+ − 2C .
12 6 3
Equating coefficients, we obtain B = −1 −1
12 , and C = 3 . We now obtain
Z  1 1
− 12 x − 13

dx
Z
12
= + dx
x3 − 8 x − 2 x2 + 2x + 4
1 Z −1 1
24 (2x + 2) − 4
Z
12
= dx + 2
dx
x−2 x + 2x + 4
1
1 2x + 2 1 1
Z Z Z
12
= dx − dx − √ 2 dx
x−2 24 x2 + 2x + 4 4 2
(x + 1) + 3
√  √ 
1 1 3(x+1)
ln x2 + 2x + 4 − 123 tan−1

= 12 ln |x − 2| − 24 3 +C .

49. We use partial fractions to obtain


1 1 A B
= = +
x2 − 9 (x − 3)(x + 3) x−3 x+3
1 = A(x + 3) + B(x − 3).
When x = −3 we obtain B = − 61 , and when x = 3 we have A = 16 . So we obtain
Z 1 Z 1 
dx 1/6 −1/6
2
= + dx
0 x −9 0 x−3 x+3
 1
1 1
= ln |x − 3| − ln |x + 3|
6 6 0
 
1 1 1 1
= ln |1 − 3| − ln |1 + 3| − ln |0 − 3| − ln |0 + 3|
6 6 6 6
= − 61 ln 2 .
7-176 Chapter 7 Techniques of Integration

50. We use partial fractions to obtain


1 1 A B
= = +
x2 − 25 (x − 5)(x + 5) x−5 x+5
1 = A(x + 5) + B(x − 5).
−1 1
When x = −5 we obtain B = 10 , and when x = 5 we have A = 10 . So we obtain
4 4  1 −1 
dx
Z Z
10 10
= + dx
2 x2 − 25 2 x−5 x+5
 4
1 1
= ln |x − 5| − ln |x + 5|
10 10 2
 
1 1 1 1
= ln |4 − 5| − ln |4 + 5| − ln |2 − 5| − ln |2 + 5|
10 10 10 10
1 3
= 10 ln 7 − 10 ln 3 .

51. We use partial fractions to obtain


1 −1 A B
= = +
16 − x2 (x − 4)(x + 4) x−4 x+4
−1 = A(x + 4) + B(x − 4).

When x = −4 we obtain B = 18 , and when x = 4 we have A = −1


8 . So we obtain
3 3  
dx −1/8 1/8
Z Z
= + dx
−2 16 − x2 −2 x−4 x+4
 3
−1 1
= ln |x − 4| + ln |x + 4|
8 8 −2
 
−1 1 −1 1
= ln |3 − 4| + ln |3 + 4| − ln |−2 − 4| + ln |−2 + 4|
8 8 8 8
1 1 1
= ln 7 − ln 2 + ln 6 = 18 ln 21 .
8 8 8

52. We use partial fractions to obtain


1 −1 A B
= = +
9 − x2 (x − 3)(x + 3) x−3 x+3
−1 = A(x + 3) + B(x − 3).

When x = −3 we obtain B = 61 , and when x = 3 we have A = −1


6 . So we obtain
2 2  
dx −1/6 1/6
Z Z
= + dx
1 9 − x2 1 x−3 x+3
 2
−1 1
= ln |x − 3| + ln |x + 3|
6 6 1
 
−1 1 −1 1
= ln |2 − 3| + ln |2 + 3| − ln |1 − 3| + ln |1 + 3|
6 6 6 6
1 1
= 6 ln 5 − 6 ln 2 .
7.5 Integration of Rational Functions Using Partial Fractions; 7-177
The Logistic Model

53. Let x = sin θ, and substitute to obtain


cos θ dx dx
Z Z Z
dθ = = .
sin2 θ + sin θ − 6 x2 + x − 6 (x − 2)(x + 3)
We use partial fractions to obtain
1 A B
= +
(x − 2)(x + 3) x−2 x+3
1 = A(x + 3) + B(x − 2)

When x = −3 we obtain B = − 51 , and when x = 2 we have A = 15 . So we obtain


Z  
cos θ 1/5 −1/5
Z
dθ = + dx
sin2 θ + sin θ − 6 x−2 x+3
1 1
= ln |x − 2| − ln |x + 3| + C
5 5
1 1
= 5 ln (2 − sin θ) − 5 ln (sin θ + 3) + C .

54. Let u = cos x, and substitute to obtain


sin x −du −du
Z Z Z
2
dx = 2
= .
cos x − 2 cos x − 8 u − 2x − 8 (u − 4)(u + 2)
We use partial fractions to obtain
−1 A B
= +
(u − 4)(u + 2) u−4 u+2
−1 = A(u + 2) + B(u − 4)

When u = −2 we obtain B = 16 , and when u = 4 we have A = − 61 . So we obtain


Z  
sin x −1/6 1/6
Z
dx = + du
cos2 x − 2 cos x − 8 u−4 u+2
1 1
= − ln |u − 4| + ln |u + 2| + C
6 6
= − 61 ln (4 − cos x) + 1
6 ln (cos x + 2) + C .

55. Let x = cos θ, and substitute to obtain


sin θ −dx −dx
Z Z Z
3
dθ = 3
= .
cos θ + cos θ x +x x(x2 + 1)
We use partial fractions to obtain
−1 A Bx + C
= + 2
x(x2 + 1) x x +1
= A x2 + 1 + (Bx + C)x

−1

When x = 0 we obtain A = −1. So


−1 = (−1) x2 + 1 + (Bx + C)x


−1 = (B − 1)x2 + Cx − 1
7-178 Chapter 7 Techniques of Integration

Equating coefficients, we obtain B = 1 and C = 0. We now have


Z  
sin θ −1 x
Z
dθ = + 2 dx
cos3 θ + cos θ x x +1
−1 x
Z Z
= dx + dx
x x2 + 1
1
= − ln |x| + ln x2 + 1 + C

2
1
ln cos2 θ + 1 + C .

= − ln |cos θ| + 2

56. Let x = sin θ, and substitute to obtain


4 cos θ 4 dx 4dx
Z Z Z
dθ = = .
sin3 θ + 2 sin θ x3 + 2x x(x2 + 2)
We use partial fractions to obtain
4 A Bx + C
= + 2
x(x2 + 2) x x +2
2

4 = A x + 2 + (Bx + C)x
When x = 0 we obtain A = 2. So
2 x2 + 2 + (Bx + C)x

4 =
4 = (B + 2)x2 + Cx + 4
Equating coefficients, we obtain B = −2 and C = 0. We now have
Z  
4 cos θ 2 −2x
Z
dθ = + dx
cos3 θ + cos θ x x2 + 2
1 2x
Z Z
= 2 dx − dx
x x2 + 2
= 2 ln |x| − ln x2 + 2 + C


2 ln |sin θ| − ln sin2 θ + 2 + C .

=

57. Let x = et , and substitute to obtain


et dx dx
Z Z Z
dt = = .
e2t + et − 2 x2 + x − 2 (x − 1)(x + 2)
We use partial fractions to obtain
1 A B
= +
(x − 1)(x + 2) x−1 x+2
1 = A(x + 2) + B(x − 1)
When x = −2 we obtain B = − 31 , and when x = 1 we have A = 13 . So we obtain
et
Z  
1/3 −1/3
Z
dt = + dx
e2t + et − 2 x−1 x+2
1/3 −1/3
Z Z
= dx + dx
x−1 x+2
1 1
= ln |x − 1| − ln |x + 2| + C
3 3
1 1
= 3 ln |et − 1| − 3 ln (et + 2) + C .
7.5 Integration of Rational Functions Using Partial Fractions; 7-179
The Logistic Model

58. Let y = ex , and substitute to obtain


ex dy dy
Z Z Z
dx = = .
e2x + ex − 6 y2 + y − 6 (y − 2)(y + 3)

We use partial fractions to obtain


1 A B
= +
(y − 2)(y + 3) y−2 y+3
1 = A(y + 3) + B(y − 2)

When y = −3 we obtain B = − 51 , and when y = 2 we have A = 51 . So we obtain

ex
Z  
1/5 −1/5
Z
dx = + dy
e2x + ex − 6 y−2 y+3
1/5 −1/5
Z Z
= dy + dy
y−2 y+3
1 1
= ln |y − 2| − ln |y + 3| + C
5 5
1 1
= 5 ln |ex − 2| − 5 ln (ex + 3) + C .

59. Let y = ex , and substitute to obtain


ex dy dy
Z Z Z
dx = = .
e2x − 1 y2 − 1 (y − 1)(y + 1)

We use partial fractions to obtain


1 A B
= +
(y − 1)(y + 1) y−1 y+1
1 = A(y + 1) + B(y − 1)

When y = −1 we obtain B = − 21 , and when y = 1 we have A = 21 . So we obtain

ex
Z  
1/2 −1/2
Z
dx = + dy
e2x − 1 y−1 y+1
1/2 −1/2
Z Z
= dy + dy
y−1 y+1
1 1
= ln |y − 1| − ln |y + 1| + C
2 2
1 1
= 2 ln |ex − 1| − 2 ln (ex + 1) + C .

60. We rewrite the integral as


dx ex
Z Z
= dx.
e − e−x
x e2x − 1
Let y = ex , and substitute to obtain
dx dy dy
Z Z Z
x −x
dx = 2
= .
e −e y −1 (y − 1)(y + 1)
7-180 Chapter 7 Techniques of Integration

We use partial fractions to obtain


1 A B
= +
(y − 1)(y + 1) y−1 y+1
1 = A(y + 1) + B(y − 1)

When y = −1 we obtain B = − 21 , and when y = 1 we have A = 12 . So we obtain


Z  
dx 1/2 −1/2
Z
= + dy
ex − e−x y−1 y+1
1/2 −1/2
Z Z
= dy + dy
y−1 y+1
1 1
= ln |y − 1| − ln |y + 1| + C
2 2
1 1
= 2 ln |ex − 1| − 2 ln (ex + 1) + C .

61. We rewrite the integral as


dt et
Z Z
= dt.
e2t + 1 et (e2t + 1)

Let x = et , and substitute to obtain


dt dx
Z Z
= .
e2t + 1 x(x2 + 1)
We use partial fractions to obtain
1 A Bx + C
= + 2
x(x2 + 1) x x +1
2

1 = A x + 1 + (Bx + C)x

When x = 0 we obtain A = 1. So
(1) x2 + 1 + (Bx + C)x

1 =
1 = (B + 1)x2 + Cx + 1
Equating coefficients, we obtain B = −1 and C = 0. We now have
Z  
dt 1 −x
Z
= + dx
e2t + 1 x x2 + 1
1 −x
Z Z
= dx + dx
x x2 + 1
1
= ln |x| − ln x2 + 1 + C

2
 1
= ln e − ln e2t + 1 + C
t

2
1
= t − 2 ln e2t + 1 + C .


62. We rewrite the integral as


dt et
Z Z
= dt.
e3t + et e2t (e2t + 1)
7.5 Integration of Rational Functions Using Partial Fractions; 7-181
The Logistic Model

Let x = et , and substitute to obtain


dt dx
Z Z
= .
e3t + et x2 (x2 + 1)
We use partial fractions to obtain
1 A B Cx + D
= + 2+ 2
x2 (x2 + 1) x x x +1
= Ax x + 1 + B x2 + 1 + (Cx + D)x2
2
 
1

When x = 0 we obtain B = 1. We expand and obtain

1 = Ax x2 + 1 + (1) x2 + 1 + (Cx + D)x2


 

1 = (A + C)x3 + (D + 1)x2 + Ax + 1
Equating coefficients, we obtain A = 0, C = 0, and D = −1. So
Z  
dt 1 −1
Z
= + dx
e3t + et x2 x2 + 1
1 dx
Z Z
= 2
dx − 2
x x +1
1 −1
= − − tan x + C
x
= −e−t − tan−1 (et ) + C .

63. Let u = sin x − 1, and substitute to obtain


sin x cos x u+1
Z Z
2 dx = du
(sin x − 1) u2
Z  
1 −2
= +u du
u
1
= ln |u| − + C
u
1
= ln |sin x − 1| − sin x−1 +C .

64. Let u = cos x − 2, and substitute to obtain


cos x sin x − (u + 2)
Z Z
2 dx = du
(cos x − 2) u2
Z  
1
= − − 2u−2 du
u
2
= − ln |u| + + C
u
2
= − ln (2 − cos x) + cos x−2 +C .

65. Let u = sin x, and substitute to obtain


cos x du
Z Z
2 dx = .
2
sin x + 9 (u2 + 9)2
7-182 Chapter 7 Techniques of Integration

Let u = 3 tan θ, and substitute to obtain

cos x 3 sec2 θ dθ
Z Z
2 dx = 2
sin2 x + 9
 
(3 tan θ)2 + 9
1
Z
= cos2 θ dθ
27
1 1 + cos (2θ)
Z
= dθ
27 2
 
1 1
= θ + sin (2θ) + C
54 2
1 u 1
= tan−1 + sin θ cos θ + C
54 3 54
1 u 1 tan θ
= tan−1 + + C.
54 3 54 sec2 θ
√ q 
u 2

Since tan θ = u3 , sec θ = tan2 θ + 1 = 3 + 1 = 13 u2 + 9, and we have
u
cos x 1 u 1
Z
2 dx = tan−1 + √
3
2 + C
sin2 x + 9 54 3 54 1
u 2+9
3
1 u 1 u
= tan−1 + +C
54 3 18 u2 + 9
1 sin x 1 sin x
= 54 tan−1 3 + 18 sin2 x+9 +C .

66. Let u = cos x, and substitute to obtain


sin x −du
Z Z
2 dx = .
(cos2 x + 4) (u2 + 4)2

Let u = 2 tan θ, and substitute to obtain

sin x −2 sec2 θ dθ
Z Z
2 dx = 2
(cos2 x + 4)

2
(2 tan θ) + 4
−1
Z
= cos2 θ dθ
8
−1 1 + cos (2θ)
Z
= dθ
8 2
 
−1 1
= θ + sin (2θ) + C
16 2
−1 −1 u −1
= tan + sin θ cos θ + C
16 2 16
−1 u −1 tan θ
= tan−1 + + C.
16 2 16 sec2 θ
7.5 Integration of Rational Functions Using Partial Fractions; 7-183
The Logistic Model

√ q
u 2

Since tan θ = u2 , sec θ = tan2 θ + 1 = 1

2 +1= 2 u2 + 4, and we have
u
sin x −1 u −1
Z
2 dx = tan−1 + √
2
2 + C
(cos2 x + 4) 16 2 16 1
u 2 + 4
2
−1 u 1 u
= tan−1 − +C
16 2 8 u2 + 4
−1 cos x 1 cos x
= 16 tan−1 2 − 8 cos2 x+4 +C .

67. The area under the graph is given by


Z 5 Z 5
4 4
A= 2−4
dx = dx.
3 x 3 (x + 2)(x − 2)

We use partial fractions to obtain


4 A B
= +
(x + 2)(x − 2) x+2 x−2
4 = A(x − 2) + B(x + 2)
When x = 2 we obtain B = 1, and when x = −2 we have A = −1. So we obtain
Z 5 
−1 1
A = + dx
3 x+2 x−2
5
= [− ln (x + 2) + ln (x − 2)]3
= − ln (5 + 2) + ln (5 − 2) − (− ln (3 + 2) + ln (3 − 2))
= ln 3 + ln 5 − ln 7 = ln 15
7 .

68. The area under the graph is given by


6
x−4
Z
A= 2 dx.
4 (x + 3)

Let u = x + 3 to obtain
9
(u − 3) − 4
Z
A = du
7 u2
Z 9 
1 7
= − 2 du
7 u u
 9
7
= ln u +
u 7
 
7 7
= ln 9 + − ln 7 +
9 7
2
= ln 9 − ln 7 − 9 .

69. The area under the graph is given by


Z 2 Z 2
8 8
A= 3+1
dx = 2 − x + 1)
dx.
0 x 0 (x + 1)(x
7-184 Chapter 7 Techniques of Integration

We use partial fractions to obtain


8 A Bx + C
= +
(x + 1)(x2 − x + 1) x + 1 x2 − x + 1
A x2 − x + 1 + (Bx + C)(x + 1).

8 =

When x = −1 we obtain A = 8/3. We expand and obtain

8 = (8/3) x2 − x + 1 + (Bx + C)(x + 1)



     
8 2 8 8
8 = B+ x + B+C − x+ C + .
3 3 3

Equating coefficients, we have B = −8/3 and C = 16/3. We now have


Z 2 
8/3 (−8/3)x + 16/3
A = + dx
0 x+1 x2 − x + 1
 2 Z 2
8 (−4/3)(2x − 1)x + 4
= ln (x + 1) + dx
3 0 0 x2 − x + 1
4 2 2x − 1
Z 2
8 8 dx
Z
= ln (2 + 1) − ln (0 + 1) − dx + 4  √ 2
3 3 3 0 x2 − x + 1 2
x − 12 + 23
0


" √ √ !#2
8 4 2
2 2 3 −1 3(2x − 1)
= ln 3 − ln x − x + 1 0 + 4 tan
3 3 3 3
0
8 4
ln 3 − ln 22 − 2 + 1 − ln 02 − 0 + 1
 
=
3 " 3
√ √ ! √ √ !#
2 3 −1 3(2(2) − 1) 2 3 3(2(0) − 1)
+4 tan − tan−1
3 3 3 3

4 4 3
= 3 ln 3 + 3 π .

70. The volume is given by


5  2 5
4 16
Z Z
V = π 2 dx = π 2 2 dx.
3 x −4 3 (x − 2) (x + 2)

We use partial fractions to obtain


16 A B C D
2 2 = + + +
(x − 2) (x + 2) x − 2 (x − 2)2 x + 2 (x + 2)2
2 2 2 2
16 = A(x − 2)(x + 2) + B(x + 2) + C(x − 2) (x + 2) + D(x − 2) .
When x = −2 we obtain D = 1, and when x = 2 we have B = 1. We expand and obtain

16 = A(x − 2)(x + 2)2 + (1)(x + 2)2 + C(x − 2)2 (x + 2) + (1)(x − 2)2


16 = (A + C)x3 + (2A − 2C + 2)x2 + (−4A − 4C)x + (8C − 8A + 8).
7.5 Integration of Rational Functions Using Partial Fractions; 7-185
The Logistic Model

Equating coefficients, we have A + C = 0 and 2A − 2C + 2 = 0. So C = −A, and


2A − 2(−A) + 2 = 4A + 2 = 0. We obtain A = −1/2 and C = 1/2. We now have
Z 5 !
−1/2 1 1/2 1
V = π + + + dx
3 x − 2 (x − 2)2 x + 2 (x + 2)2
Z 5 Z 5 Z 5 Z 5 !
−1/2 1 1/2 1
= π dx + 2 dx + dx + 2 dx
3 x−2 3 (x − 2) 3 x+2 3 (x + 2)
 5  5  5  5 !
1 1 1 1
= π − ln (x − 2) + − + ln (x + 2) + −
2 3 x−2 3 2 3 x+2 3
      
1 1 1 1
= π − ln (5 − 2) − − ln (3 − 2) + − − −
2 2 5−2 3−2
   
1 1 1 1
+ ln (5 + 2) − ln (3 + 2) + − − −
2 2 5+2 3+2
= π 21 ln 7 − 12 ln 5 − 21 ln 3 + 105
76

.

71. The arc length of the graph of y = ln x from x = 1 to x = e is given by


Z e√ 2
s  2 s  2
Z e Z e
dy 1 x +1
L= 1+ dx = 1+ dx = dx.
1 dx 1 x 1 x

x2 +1
R
To evaluate x dx, use the substitution x = tan θ.
π
Since x > 0, 0 < θ < 2.
√ √ √ π
Then dx = sec2 θ dθ and x2 + 1 = tan2 θ + 1 = sec2 θ = sec θ since 0 < θ < 2.
π
The lower limit of integration becomes θ = tan−1 1 = 4, and the upper limit of integration
becomes θ = tan−1 e.
So,
Z √ 2
x +1 sec θ
Z
dx = sec2 θ dθ
x tan θ
Z
= (tan θ sec θ + csc θ) dθ

= sec θ − ln |cot θ + csc θ| + C.



Since 0 < θ < π2 and tan θ = x, sec θ = x2 + 1, cot θ = tan 1 1
θ = x,
√ q √
2
and csc θ = cot2 θ + 1 = x12 + 1 = xx+1 .
So,
Z √
x2 + 1
dx = sec θ − ln |cot θ + csc θ| + C
x
1 √x2 + 1

p
2
= x + 1 − ln +

+C

x x
1 + √x2 + 1

p
= x2 + 1 − ln + C.

x
7-186 Chapter 7 Techniques of Integration

Therefore,
e
√ "
1 + √x2 + 1 e
#
x2 + 1
Z p
L= dx = x2 + 1 − ln

x x

1
1
" √ !#
h√
p 1+ e +12  √ i
= e2 + 1 − ln − 2 − ln 1 + 2
e
hp  p  i h√  √ i
= e2 + 1 − ln 1 + e2 + 1 + ln(e) − 2 − ln 1 + 2
√ √  √ 
= 1 − 2 + e2 + 1 + ln 1+1+ √ 2
2
e +1
.

 
72. (a) The logistic model dW W
dt = −0.057W 1 − 14,656,248 is expressed in the form dW
dt =
W

kW 1 − M where k is the maximum growth rate and M is the carrying capacity.
Therefore, the carrying capacity is M = 14,656,248 farmers .
(b) The maximum yearly growth rate is k = −0.057 Since k < 0, we can also say that
the maximum yearly decay rate is k = 0.057 .
(c) At the inflection point, the number of farmers is one-half the carrying capacity, 12 M =
7,328,124 farmers .

73. (a) Since the rate of change of the number of people with the flu P with respect to
time t (in days) is proportional to the product of P and 50 − P , the differential
equation can be written as dPdt = CP (50 − P ). The differential equation can also
dP P
be written in the form dt = kP 1 − M where M = 50 is the carrying capac-
ity and k = 0.15 is the maximum growth rate. The differential equation becomes
dP P

dt = 0.15P 1 − 50 with P (0) = 1 .

(b) The initial number of people


 with the flu M
is P0 = 1. The solution to the differential
equation dP
dt = kP 1 − P
M is P (t) = 1+ae−kt
where k = 0.15, M = 50, and a =
M−P0 50−1 50
P0 = 1 = 49. Therefore, P (t) = 1+49e−0.15t .

(c) Find t so that P (t) = 25.


50
= 25
1 + 49e−0.15t
50
−1
e−0.15t = 25
49
 50
−1

25
−0.15t = ln
49
 
1 1 1
t=− ln = ln 49 ≈ 29.945 days .
0.15 49 0.15
7.5 Integration of Rational Functions Using Partial Fractions; 7-187
The Logistic Model

(d) Find t so that P (t) = 0.80(50).


50
= 40
1 + 49e−0.15t
50
−1
e−0.15t = 40
49
 
0.25
−0.15t = ln
49
 
1 0.25
t=− ln ≈ 35.187 days
0.15 49

74. (a) Since the rate of change of the fruit fly population P with respect to time t (in days) is
P
proportional to the product of P and 1 − 230 , the differential equation can be written
dP P
as dt = kP 1 − M . This is the logistic growth model dP P
 
dt = kP 1 − M where k is
the maximum growth rate and M is the carrying capacity. We are given the maximum
growth rate k = 0.37 and a carrying capacity of M = 230 fruit flies. The differential
equation becomes dP P

dt = 0.37P 1 − 230 with P (0) = 4 .

(b) The initial population size is P0 = 4 fruit flies. The solution to the differential equation
dP P M M−P0

dt = kP 1 − M is P (t) = 1+ae −kt where k = 0.37, M = 230, and a = P0 =
230−4 230
4 = 56.5. Therefore, P (t) = 1+56.5e−0.37t .
(c) Find t so that P (t) = 115.
230
= 115
1 + 56.5e−0.37t
230
−1
e−0.37t = 115
56.5
 230
115 − 1

−0.37t = ln
56.5
 230
−1

1
t=− ln 115 = 10.903 days .
0.37 56.5
(d) Find t so that P (t) = 180.
230
= 180
1 + 56.5e−0.37t
230
−1
e−0.37t = 180
56.5
 230
−1

−0.37t = ln 180
56.5
 230
180 − 1

1
t=− ln = 14.365 days .
0.37 56.5

75. (a) We test possible rational roots, and obtain that the zeros of q are −4, −2, and 3 .
(b) From part (a), we obtain q(x) = (x + 4) (x + 2)(x − 3).
(c) We use partial fractions to obtain
3x − 7 A B C
= + +
x3 + 3x2 − 10x − 24 x+2 x−3 x+4
3x − 7 = A(x − 3)(x + 4) + B(x + 2)(x + 4) + C(x + 2)(x − 3).
7-188 Chapter 7 Techniques of Integration

2
When x = −4 we obtain C = −19/14. When x = 3 we get B = 35 . And when
13
x = −2, we have A = 10 . We now obtain
Z  
3x − 7 13/10 2/35 −19/14
Z
dx = + + dx
x3 + 3x2 − 10x − 24 x+2 x−3 x+4
13/10 2/35 −19/14
Z Z Z
= dx + dx + dx
x+2 x−3 x+4
13 2 19
= 10 ln |x + 2| + 35 ln |x − 3| − 14 ln |x + 4| + C .

Challenge Problems
√ 1
76. Let u = x, du = √
2 x
dx, so dx = 2u du. We obtain

x dx u2
Z Z
√ = (2u) du
3+ x 3+u
u3
Z
= 2 du.
u+3
Let y = u + 3, then we have

x dx (y − 3)3
Z Z
√ = 2 dy
3+ x y
Z 3
y − 9y 2 + 27y − 27
= 2 dy
y
Z  
27
= 2 y 2 − 9y + 27 − dy
y
 
1 3 9 2
= 2 y − y + 27y − 27 ln |y| + C
3 2
2 3 2
= (u + 3) − 9(u + 3) + 54(u + 3) − 54 ln |u + 3| + C
3
2 √ 3 √ 2 √ √
= 3 ( x + 3) − 9( x + 3) + 54( x + 3) − 54 ln ( x + 3) + C .

√ 1
77. Let u = x, du = √
2 x
dx, so dx = 2u du. We obtain

dx 2u
Z Z
√ = du.
x+2 u+2

Let y = u + 2, then we have


dx y−2
Z Z
√ = 2 du
x+2 y
Z  
2
= 2 1− dy
y
= 2(y − 2 ln |y|) + C
= 2(u + 2) − 4 ln |u + 2| + C
√ √
= 2 x − 4 ln ( x + 2) + C .
7.5 Integration of Rational Functions Using Partial Fractions; 7-189
The Logistic Model

√ 1
78. Let u = 3
x, du = 3x2/3
dx, so dx = 3u2 du. We obtain
dx 3u2
Z Z
√ = du
x− x3
u3 − u
3u
Z
= du
u2 − 1
3u
Z
= du.
(u − 1)(u + 1)
We use partial fractions to obtain
3u A B
= +
(u − 1)(u + 1) u−1 u+1
3u = A(u + 1) + B(u − 1).
When u = −1 we obtain B = 3/2, and when u = 1 we have A = 3/2. We now obtain
Z  
dx 3/2 3/2
Z
√ = + du
x− 3x u−1 u+1
3 3
= ln |u − 1| + ln |u + 1| + C
2 2
√ √
= 2 ln | x − 1| + 23 ln | 3 x + 1| + C .
3 3

√ 1
79. Let u = 3
x, du = 3x2/3
dx, so dx = 3u2 du. We obtain
x dx u3
Z Z
3u2 du

√ =
3
x−1 u−1
u5
Z
= 3 du.
u−1
Let y = u − 1, then we have
x dx (y + 1)5
Z Z
√ = 3 dy
3
x−1 y
Z 5
y + 5y 4 + 10y 3 + 10y 2 + 5y + 1
= 3 dy
y
Z  
1
= 3 y 4 + 5y 3 + 10y 2 + 10y + 5 + dy
y
 
1 5 5 4 10 3 2
= 3 y + y + y + 5y + 5y + ln |y| + C
5 4 3
3 5 15 4 3 2
= (u − 1) + (u − 1) + 10(u − 1) + 15(u − 1) + 15(u − 1) + 3 ln |u − 1| + C
5 4
3 5 3 4 3
= u + u + u3 + u2 + 3u + 3 ln |u − 1| + C
5 4 2
3 5/3
x + x + x + 3 x2/3 + 3x1/3 + 3 ln x1/3 − 1 + C .
3 4/3

= 5 4 2

1
80. Let u = x1/6 , du = 6x5/6
dx, so dx = 6u5 du. We obtain
dx 6u5
Z Z
√ √ = du
x+ x3
u3 + u2
6u3
Z
= du.
u+1
7-190 Chapter 7 Techniques of Integration

Let y = u + 1, and substitute to obtain


3
dx 6(y − 1)
Z Z
√ √ = dy
x+ 3x y
6y 3 − 18y 2 + 18y − 6
Z
= dy
y
Z  
6
= 6y 2 − 18y + 18 − dy
y
= 2y 3 − 9y 2 + 18y − 6 ln |y| + C
3 2
= 2(u + 1) − 9(u + 1) + 18(u + 1) − 6 ln |u + 1| + C
= 2u3 − 3u2 + 6u − 6 ln |u + 1| + C
√ √
= 2 x − 3 3 x + 6x1/6 − 6 ln x1/6 + 1 + C .

1
81. Let u = x1/6 , du = 6x5/6
dx, so dx = 6u5 du. We obtain

dx 6u5
Z Z
√ √ = du
3 x− 3x 3u3 − u2
6u3
Z
= du.
3u − 1
Let y = 3u − 1, and substitute to obtain
y+1 3

dx 6 1
Z Z
3
√ √ = dy
3 x− 3x y 3
Z 2 3 2 2 2 2
9y + 3y + 3y + 9
= dy
3y
Z  
2 2 2 2 2
= y + y+ + dy
27 9 9 27y
2 3 1 2 2 2
= y + y + y+ ln |y| + C
81 9 9 27
2 3 1 2 2 2
= (3u − 1) + (3u − 1) + (3u − 1) + ln |3u − 1| + C
81 9 9 27
2 3 1 2 2 2
= u + u + u+ ln |3u − 1| + C
3 3 9 27
2 1/2
x + 1 x1/3 + 2 x1/6 + 2 ln 3x1/6 − 1 + C .

= 3 3 9 27

82. Let u = 2 + 3x, and substitute to obtain


1
dx du
Z Z
3
√ = √
3
2 + 3x 3
u
1
Z
= u−1/3 du
3
 
1 3 2/3
= u +C
3 2
1 2/3
= 2 (2 + 3x) +C .
7.5 Integration of Rational Functions Using Partial Fractions; 7-191
The Logistic Model

83. Let u = 1 + 2x, and substitute to obtain


1
dx du
Z Z
2
√ = √
4
1 + 2x 4
u
1
Z
= u−1/4 du
2
 
1 4 3/4
= u +C
2 3
2 3/4
= 3 (1 + 2x) +C .

84. Let u = 1 + x, and substitute to obtain

x dx (u − 1) du
Z Z
3/4
=
(1 + x) u3/4
Z  
= u1/4 − u−3/4 du
4 5/4
= u − 4u1/4 + C
5
4 5/4 1/4
= 5 (1 + x) − 4(1 + x) +C .

85. Let u = 1 + x, and substitute to obtain


dx du
Z Z
2/3
=
(1 + x) u2/3
Z
= u−2/3 du

= 3u1/3 + C
1/3
= 3(1 + x) +C .

√ 1
86. Let u = 3
x, du = 3x2/3
dx, so dx = 3u2 du. We obtain
Z √
x+1 u+1 2
3
Z
√ dx = 3u du.
3
x−1 u−1
7-192 Chapter 7 Techniques of Integration

Let y = u − 1, then we have


Z √
x+1 (y + 1) + 1
3
Z
2
√ dx = 3 (y + 1) dy
3
x−1 y
2
(y + 2) (y + 1)
Z
= 3 dy
y
Z 3
y + 4y 2 + 5y + 2
= 3 dy
y
 
2
Z
= 3 y 2 + 4y + 5 + dy
y
   
1 3 1 2
= 3 y +4 y + 5y + 2 ln |y| + C
3 2
   
1 3 1 2
= 3 (u − 1) + 4 (u − 1) + 5 (u − 1) + 2 ln |u − 1| + C
3 2
= u3 + 3u2 + 6u + 6 ln |u − 1| + C
√ √
= x + 3x2/3 + 6 3 x + 6 ln | 3 x − 1| + C .

1
87. Let u = x1/6 , du = 6x5/6
dx, so dx = 6u5 du. We obtain

dx 6u5
Z Z
√ √ 2 = 2 du
x(1 + 3 x) u3 (1 + u2 )
1 + u2 − 1
Z 
= 6 2 du
(1 + u2 )
!
1 1
Z
= 6 − 2 du
1 + u2 (1 + u2 )
1
Z
= 6 tan−1 u − 6 2 du.
(1 + u2 )

In the last integral, let u = tan θ, and substitute to obtain

1 sec2 θ
Z Z
2 du = dθ
(1 + u2 ) sec4 θ
Z
= cos2 θ dθ
1
Z
= (1 + cos (2θ)) dθ
2
 
1 1
= θ + sin 2θ + C
2 2
1 1
= θ + sin θ cos θ + C
2 2
1 −1 1 u 1
= tan u + √ √ +C
2 2 1 + u2 1 + u2
1 u
= tan−1 u + 2
+ C.
2 2(u + 1)
7.5 Integration of Rational Functions Using Partial Fractions; 7-193
The Logistic Model

We now obtain
 
dx 1 u
Z
√ √ 2 = 6 tan−1 u − 6 tan−1 u + +C
x(1 + 3 x) 2 2(u2 + 1)
3u
= 3 tan−1 u − 2 +C
u +1
x1/6
= 3 tan−1 x1/6 − 3 x1/3

+1
+C .

88. With the substitution z = tan x2 , sin x = 2z


1+z 2 , and dx = 2 dz
1+z 2 we obtain

dx 1 2 dz
Z Z
= 2z
1 − sin x 1− 1+z 2
1 + z2
dz
Z
= 2
1 − 2z + z 2
Z
= 2 (1 − z)−2 dz

= 2(1 − z)−1 + C
2
= 1−tan x +C .
2

89. With the substitution z = tan x2 , sin x = 2z


1+z 2 , and dx = 2 dz
1+z 2 we obtain

dx 1 2 dz
Z Z
= 2z
1 + sin x 1+ 1+z 2
1 + z2
dz
Z
= 2
1 + 2z + z 2
Z
−2
= 2 (1 + z) dz
−1
= −2(1 + z) +C
2
= − 1+tan x +C .
2

1−z 2
90. With the substitution z = tan x2 , cos x = 1+z 2 , and dx = 2 dz
1+z 2 we obtain

dx 1 2 dz
Z Z
= 1−z 2
1 − cos x 1− 1+z 2
1 + z2
dz
Z
=
z2
1
= − +C
z
1
= − +C
tan x2
= − cot x2 + C .
7-194 Chapter 7 Techniques of Integration

1−z 2
91. With the substitution z = tan x2 , cos x = 1+z 2 , and dx = 2 dz
1+z 2 we obtain

dx 1 2 dz
Z Z
= 1−z 2 1 + z 2
3 + 2 cos x 3 + 2 1+z 2

dz
Z
= 2 √ 2
+ z2 5
√ √ !
2 5 −1 5
= tan z +C
5 5
√ √ 
2 5 −1 5 x
= 5 tan 5 tan 2 +C .

1−z 2
92. With the substitution z = tan x2 , sin x = 2z
1+z 2 , cos x = 1+z 2 , and dx = 2 dz
1+z 2 we obtain

dx 1 2 dz
Z Z
= 1−z 2
sin x + cos x 2z
1+z 2 + 1+z 2
1 + z2
2 dz
Z
=
−z 2 + 2z + 1
−2 dz
Z
=
z 2 − 2z − 1
−2 dz
Z
= √  √  .
z− 1+ 2 z− 1− 2

We use partial fractions to obtain


−2 A B
√  √  = √ + √ 
z− 1+ 2 z− 1− 2 z− 1+ 2 z− 1− 2
  √    √ 
−2 = A z− 1− 2 +B z− 1+ 2 .
√ √ √ √
When z = 1 − 2 we obtain B = 2/2, and when z = 1 + 2 we have A = − 2/2. So
we obtain
√ √ !
dx − 2/2 2/2
Z Z
= √ + √  dx
sin x + cos x z− 1+ 2 z− 1− 2
√ √
2 √ 2 √
= − ln z − 1 − 2 + ln z − 1 + 2 + C
2 2
√ √ √ √
= − 22 ln tan x2 − 1 − 2 + 22 ln tan x2 − 1 + 2 + C .

1−z 2
93. With the substitution z = tan x2 , sin x = 2z
1+z 2 , cos x = 1+z 2 , and dx = 2 dz
1+z 2 we obtain

dx 1 2 dz
Z Z
=
1 − sin x + cos x 1− 2z
1+z 2 + 1−z 2
1+z 2
1 + z2
dz
Z
=
1−z
= − ln |1 − z|
− ln 1 − tan x2 + C .

=
7.5 Integration of Rational Functions Using Partial Fractions; 7-195
The Logistic Model

94. Let u = 3 + cos x, substitute to obtain


sin x dx −du
Z Z
=
3 + cos x u
= − ln |u| + C
= − ln (3 + cos x) + C.

95. With the substitution z = tan x2 , tan x = 2z


1−z 2 , and dx = 2 dz
1+z 2 we obtain
2
dx
Z Z
1+z 2
= 2z dz
tan x − 1 1−z 2 − 1
2 1 − z2
Z 
= dz
(1 + z 2 )(2z − (1 − z 2 ))
2 1 − z2
Z 
= dz.
(z 2 + 1)(z 2 + 2z − 1)

We use partial fractions to obtain

2 1 − z2

Az + B Cz + D
= + 2
(z 2 + 1)(z 2 + 2z − 1) z2 + 1 z + 2z − 1
−2z 2 + 2 = (Az + B) z 2 + 2z − 1 + (Cz + D) z 2 + 1
 

−2z 2 + 2 = (A + C)z 3 + (2A + B + D)z 2 + (2B − A + C)z + (D − B).


Equating coefficients, we have
A+C = 0
2A + B + D = −2
2B − A + C = 0
D−B = 2.
We solve this system and obtain A = −1, B = −1, C = 1, and D = 1. So
Z  
dx z+1 z+1
Z
= − dz
tan x − 1 z 2 + 2z − 1 z 2 + 1
1 2(z + 1) 1 2z 1
Z Z Z
= 2
− 2
dz − 2
dz
2 z + 2z − 1 2 z +1 z +1
1 2 1
ln z + 2z − 1 − ln z 2 + 1 − tan−1 z + C

=
2 2
1
2 x
+ 2 tan x − 1 − 1 ln tan2 x + 1 − x
 
= 2ln tan

2 2 2 2 2 +C .
7-196 Chapter 7 Techniques of Integration

96. With the substitution z = tan x2 , sin x = 2z


1+z 2 , tan x = 2z
1−z 2 , and dx = 2 dz
1+z 2 we obtain
2
dx
Z Z
1+z 2
= 2z 2z dz
tan x − sin x 1−z 2 − 1+z 2
2 1 − z2
Z 
= dz
2z((1 + z 2 ) − (1 − z 2 ))
2 1 − z2
Z 
= dz
4z 3
Z  
1 −3 1
= z − dz
2 z
 
1 1 −2
= − z − ln |z| + C
2 2
− 14 cot2 x2 − 21 ln tan x2 + C .

=

1+z 2
97. With the substitution z = tan x2 , sec x = 1−z 2 , tan x = 2z
1−z 2 , and dx = 2 dz
1+z 2 we obtain

1+z 2 2
sec x dx
Z Z
1−z 2 1+z 2
= 2z dz
tan x − 2 1−z 2 − 2
1
Z
= dz
z − (1 − z 2 )
1
Z
= dz
z2 + z − 1
1
Z
= 1
√ 1
√  dz.
z − − 21 5 − 12

z− 2 5− 2

We use partial fractions to obtain


1 A B
1
√ 1
√ = √ + √
z − − 21 5 − 12 1 1
z − − 2 5 − 12
1
  
z− 2 5− 2 z− 5− 2 2
1√ 1√
     
1 1
1 = A z− − 5− +B z− 5− .
2 2 2 2
√ √ √ √
When z = − 21 5 − 12 we obtain B = − 5/5, and when z = 21 5 − 21 we have A = 5/5.
So we obtain
√ √ !
sec x dx 5/5 − 5/5
Z Z
= √ + √  dz
tan x − 2 z − 12 5 − 12 z − − 21 5 − 21
√  √
1√ 1√
  
5 1 5 1
= ln z − 5− − ln z − − 5− +C
5 2 2 5 2 2

5
x 1
√ 1
√5 x 1
√ 1

= 5 ln tan 2 − 2 5 + 2 − 5 ln tan 2 + 2 5 + 2 + C .

7.5 Integration of Rational Functions Using Partial Fractions; 7-197
The Logistic Model

1−z 2
98. With the substitution z = tan x2 , sin x = 2z
1+z 2 , cot x = 2z , and dx = 2 dz
1+z 2 we obtain

1−z 2  
cot x dx 2
Z Z
2z
= 2z dz
1 + sin x 1 + 1+z 2 1 + z2
Z 1−z 2
z
= dz
1 + z2 +
2z
(1 − z)(1 + z)
Z
= 2 dz
z(1 + z)
1−z
Z
= dz.
z(1 + z)

We use partial fractions to obtain


1−z A B
= +
z(1 + z) z 1+z
1−z = A(1 + z) + Bz.
When z = −1, we have B = −2, and when z = 0, A = 1. We now obtain
Z  
cot x dx 1 −2
Z
= + dz
1 + sin x z 1+z
= ln |z| − 2 ln |1 + z| + C
= ln tan x2 − 2 ln 1 + tan x2 + C .

1+z 2
99. With the substitution z = tan x2 , sin x = 2z
1+z 2 , sec x = 1−z 2 , and dx = 2 dz
1+z 2 we obtain

1+z 2 2
sec x dx
Z Z
1−z 2 1+z 2
= 2z dz
1 + sin x 1 + 1+z 2

−2 1 + z 2
Z 
= 3 dz.
(z − 1)(z + 1)

We use partial fractions to obtain

−2 1 + z 2

A B C D
3 = + + 2 + 3
(z − 1)(z + 1) z − 1 z + 1 (z + 1) (z + 1)
3 2
−2 1 + z 2 = A(z + 1) + B(z − 1)(z + 1) + C(z − 1)(z + 1) + D(z − 1).


When z = −1, we have D = 2, and when z = 1, A = −1/2. We now obtain


3 2
−2z 2 − 2 = (−1/2)(z + 1) + B(z − 1)(z + 1) + C(z − 1)(z + 1) + 2(z − 1)
       
1 3 3 2 1 5
−2z 2 − 2 = B− z + B+C − z + −B z− B+C + .
2 2 2 2
7-198 Chapter 7 Techniques of Integration

Equating coefficients, we obtain B = 1/2 and C = −1. We now have


!
sec x dx −1/2 1/2 −1 2
Z Z
= + + + dz
1 + sin x z − 1 z + 1 (z + 1)2 (z + 1)3
−1/2 1/2 −1 2
Z Z Z Z
= dz + dz + 2 dz + 3 dz
z−1 z+1 (z + 1) (z + 1)
1 1 1 1
= − ln |z − 1| + ln |z + 1| + − +C
2 2 z + 1 (z + 1)2
= − 12 ln tan x2 − 1 + 12 ln tan x2 + 1 + tan 1x +1 − 1

2 + C .
2 (tan x2 +1)

100. With the substitution z = tan x2 , sin x = 2z


1+z 2 , and dx = 2 dz
1+z 2 we obtain

π/2 1
dx 1 2 dz
Z Z
= 2z
0 sin x + 1 0 1+z 2 + 1 1 + z2
1
dz
Z
= 2
0 1 + 2z + z 2
Z 1
−2
= 2 (1 + z) dz
0
h i1
−1
= −2(1 + z)
0
 
−1 −1
= −2(1 + 1) − −2(1 + 0)

= 1.

1+z 2
101. With the substitution z = tan x2 , csc x = 2z , tan x = 2z
1−z 2 , and dx = 2 dz
1+z 2 , we obtain

π/3 1+z 2 2
3/3
csc x
Z Z
2z 1+z 2
dx = 2z dz
π/4 3 + 4 tan x tan (π/8) 3 + 4 1−z 2

Z √3/3 1 2

z 1−z
= 2
dz
tan (π/8) 3(1 − z ) + 8z

Z 3/3 2
z −1
= dz.
tan (π/8) z(3z + 1)(z − 3)

We use partial fractions to obtain

z2 − 1 A B C
= + +
z(3z + 1)(z − 3) z 3z + 1 z − 3
z2 − 1 = A(3z + 1)(z − 3) + Bz(z − 3) + Cz(3z + 1).
7.5 Integration of Rational Functions Using Partial Fractions; 7-199
The Logistic Model

When z = 0, we have A = 1/3, when z = −1/3, B = −4/5, and when z = 3, C = 4/15.


We now obtain
Z π/3 Z √3/3  
csc x 1/3 −4/5 4/15
dx = + + dz
π/4 3 + 4 tan x tan (π/8) z 3z + 1 z − 3
 √3/3
1 4 4
= ln z − ln (3z + 1) + ln (3 − z)
3 15 15 tan (π/8)
√ √ ! √ !
1 3 4 3 4 3
= ln − ln 3 +1 + ln 3 −
3 3 15 3 15 3
 
1  π  4  π  4  π
− ln tan − ln 3 tan + 1 + ln 3 − tan .
3 8 15 8 15 8
√ √ √ √  √  √ 
= 13 ln 33 − 31 ln 2 − 1 − 15 4 4 4 4 3
  
ln 3 + 1 − 15 ln 4 − 2 + 15 ln 3 2 − 2 + 15 ln 3 − 3 .

1−z 2
102. With the substitution z = tan x2 , cos x = 1+z 2 , and dx = 2 dz
1+z 2 we obtain
2
π/2 1 1−z 2
cos x dx
Z Z
1+z 2 1+z 2
= 1−z 2
dz
0 2 − cos x 0 2 − 1+z 2
Z 1
2 1 − z2

= dz
0 (1 + z 2 )(2(1 + z 2 ) − (1 − z 2 ))
1
2 1 − z2
Z 
= dz.
0 (z 2 + 1)(3z 2 + 1)
We use partial fractions to obtain
2 1 − z2

Az + B Cz + D
= + 2
(z 2 + 1)(3z 2 + 1) z2 + 1 3z + 1
2
= (Az + B) 3z 2 + 1 + (Cz + D) z 2 + 1
  
2 1−z
2 − 2z 2 = (3A + C)z 3 + (3B + D)z 2 + (A + C)z + (B + D).
Equating coefficients, we obtain
3A + C = 0
3B + D = −2
A+C = 0
B+D = 2.
We solve this system, and obtain A = 0, B = −2, C = 0, and D = 4. So
Z π/2 Z 1 
cos x dx 2 4
= − 2 + dz
0 2 − cos x 0 z + 1 3z 2 + 1
Z 1
1 4 1 1
Z
= −2 2
dz +  √ 2 dz
0 z +1 3 0 2
z + 33
" √
√  1
#
1 4 3
= −2 tan−1 z 0 + tan−1
 
3z
3
0
√ √  4√3 √
−1 −1 4 3 −1

tan−1

= −2 tan 1 − −2 tan 0 + tan 3(1) − 3(0)
3 3

4 3
= 9 π − 21 π .
7-200 Chapter 7 Techniques of Integration

103. With the substitution z = tan x2 , tan x = 2z


1−z 2 , and dx = 2 dz
1+z 2 we obtain

π/4 tan (π/8) 2


4 dx 4 1+z
Z Z
2
= 2z dz
0 tan x + 1 0 1−z 2 +1
tan (π/8)
8 1 − z2
Z 
= dz
0 (1 + z 2 )(2z + (1 − z 2 ))
tan (π/8)
8 z2 − 1
Z 
= dz.
0 (z + 1)(z 2 − 2z − 1)
2

We use partial fractions to obtain

8 z2 − 1

Az + B Cz + D
= + 2
(z 2 + 1)(z 2 − 2z − 1) z2 + 1 z − 2z − 1
8 z 2 − 1 = (Az + B) z 2 − 2z − 1 + (Cz + D) z 2 + 1
  

8z 2 − 8 = (A + C)z 3 + (B − 2A + D)z 2 + (C − 2B − A)z + (D − B).


Equating coefficients, we obtain
A+C = 0
B − 2A + D = 8
−A − 2B + C = 0
−B + D = −8.
We solve this system, and obtain A = −4, B = 4, C = 4, and D = −4. We now have
π/2 tan (π/8)  
4 dx −4z + 4 4z − 4
Z Z
= + dz
0 tan x + 1 0 z2 + 1 z 2 − 2z − 1
Z tan (π/8) Z tan (π/8) Z tan (π/8)
2z 1 2z − 2
= −2 2+1
dz + 4 2+1
dz + 2 2 − 2z − 1
dz
0 z 0 z 0 z
tan (π/8) tan (π/8) tan (π/8)
−2 ln z 2 + 1 0 + 4 tan−1 z 0 + 2 ln 1 + 2z − z 2 0
  
=
−2 ln tan2 (π/8) + 1 − (−2) ln 02 + 1 + 4 tan−1 (tan (π/8)) − 4 tan−1 0
 
=
 
2
+2 ln 1 + 2 tan (π/8) − tan2 (π/8) − 2 ln 1 + 2(0) − (0)


√ √  √
 2     2 
1
= −2 ln 2 − 1 + 1 + 4 π + 2 ln 1 + 2 2 − 1 − 2−1
8
√ !
2 2−2 1
= 2 ln √ + π
2− 2 2
√  1
= 2 ln 2 + π
2
= ln 2 + π2 .
7.5 Integration of Rational Functions Using Partial Fractions; 7-201
The Logistic Model

1+z 2
104. With the substitution z = tan x2 , csc x = 2z , and dx = 2 dz
1+z 2 , we obtain

1 + z2 2
Z Z
csc x dx = dz
2z 1 + z 2
1
Z
= dz
z
= ln |z| + C
x
= ln tan + C

2
sin x

2
= ln +C
cos x2
q
1−cos x
2
= ln q +C
1+cos x
2
q
1−cos x
= ln 1+cos x +C .

105. We simplify as follows


s
r 2
1 − cos x (1 − cos x)
ln = ln
1 + cos x (1 + cos x)(1 − cos x)
1 − cos x
= ln √
1 − cos2 x
1 − cos x
= ln √ 2
sin x
1 − cos x
= ln
|sin x|

1 − cos x
= ln

sin x

1 cos x
= ln

sin x sin x
= ln |csc x − cot x| .

106. The domain of the function given by tanh−1 (x) is the interval (−1, 1), so the function
given by tanh−1 (x) is not an antiderivative of 1−x
1
2 on the interval [2, 3]. We use partial

fractions to obtain
1 −1 A B
2
= 2
= +
1−x x −1 x−1 x+1
−1 = A(x + 1) + B(x − 1).
7-202 Chapter 7 Techniques of Integration

When x = −1 we obtain B = 21 , and when x = 1 we have A = − 21 . So we obtain


Z 3 Z 3 
1 −1/2 1/2
2
dx = + dx
2 1−x 2 x−1 x+1
 3
1 1
= − ln |x − 1| + ln |x + 1|
2 2 2
 
1 1 1 1
= − ln |3 − 1| + ln |3 + 1| − − ln |2 − 1| + ln |2 + 1|
2 2 2 2
ln 2−ln 3
= 2 .

107. We simplify as follows


1 + sin x2

1 + tan x2

x
cos 2
ln =
ln sin x

1 − tan x2 1 − 2

cos x2

2 x
1 + sin 2
x x
sin 2 cos 2
= ln sin2 x

1 − sin x cos 2

x
2 2
x
1 + 1−cos

2
sin x
= ln 2
1−cos x

1 − sin2 x
2

1 + 1−cos x

sin x
= ln
1 − 1−cos
sin x
x

sin x + 1 − cos x
= ln
sin x − 1 + cos x

tan x + sec x − 1 sec x + tan x
= ln
tan x − sec x + 1 sec x + tan x

(tan x + sec x − 1)(sec x + tan x)
= ln
(tan x − sec x + 1)(sec x + tan x)

(tan x + sec x − 1)(sec x + tan x)
= ln 2 2

tan x sec x + tan x − sec x − sec x tan x + sec x + tan x

(tan x + sec x − 1)(sec x + tan x)
= ln
−1 + sec x + tan x
= ln |sec x + tan x| .
√ √
108. We factor 1 + x4 = x2 + 2x + 1 x2 − 2x + 1 , and use partial fractions to obtain
 

1 Ax + B Cx + D
= √ + √
1 + x4 x2 2
+ 2x + 1 x − 2x + 1
 √   √ 
1 = (Ax + B) x2 − 2x + 1 + (Cx + D) x2 + 2x + 1
 √ √   √ √ 
1 = (A + C)x3 + B + D − 2A + 2C x2 + A + C − 2B + 2D x + (B + D).
We solve the system
A+C = 0
√ √
B + D − 2A + 2C = 0
√ √
A + C − 2B + 2D = 0
B+D = 1
√ √
and obtain A = 2/4, B = 1/2, C = − 2/4, and D = 1/2. We now obtain
7.5 Integration of Rational Functions Using Partial Fractions; 7-203
The Logistic Model

√  √  !
1 2/4 x + 1/2 − 2/4 x + 1/2
Z Z
dx = √ + √ dx
1 + x4 x2 + 2x + 1 x2 − 2x + 1
 √ √ 
 √  √  
2/8 2x + 2 + 14 − 82 2x − 2 + 14
Z 
=  √ + √  dx
x2 + 2x + 1 x2 − 2x + 1
√ Z √
2 2x + 2 1 dx
Z
= √ dx + √ 2  √ 2
8 2
x + 2x + 1 4

x + 22 + 22
√ Z √
2 2x − 2 1 dx
Z
− √ dx + √ 2  √ 2
8 x2 − 2x + 1 4

x − 22 + 22


2
√ √
2
√ √
2
√ √
2

ln x2 + tan−1 ln x2 − tan−1
   
= 8 2x + 1 + 4 2x + 1 − 8 2x + 1 + 4 2x − 1 + C .

R
AP Practice Problems
x2 +6
1. Apply polynomial division to x+1 .

x−1
(x + 1) x2 + 0x + 6
−(x2 + x)
−x + 6
−(−x − 1)
7

The quotient is x − 1 and the remainder is 7.

x2 +6 7
R x2 +6
R 7

x2
Therefore, x+1 = x−1+ x+1 and x+1 dx = x−1+ x+1 dx = 2 − x + 7 ln |x + 1| + C .

The answer is C.
2x2
2. Apply polynomial division to 1+x2 .

2
2 2
(x + 1) 2x + 0x + 0
−(2x2 + 2)
−2

The quotient is 2 and the remainder is −2.


2x2 2
R 2x2 R 2

Therefore, 1+x 2 = 2 − 1+x2 and 1+x2 dx = 2− 1+x2 dx = 2x − 2 tan−1 x + C.
R1 2x2
1  π π
dx = 2x − 2 tan−1 x 0 = 2 − 2
 
So, 0 1+x2 4 − (0 − 0) = 2 − 2 .

The answer is B.
7-204 Chapter 7 Techniques of Integration

x4 +3x2 −2
3. Apply polynomial division to x2 +1 .

x2 +2
2
(x + 1) x + 0x + 3x2 + 0x − 2
4 3

−(x4 + x2 )
2x2 + 0x − 2
−(2x2 + 2)
−4

The quotient is x2 + 2 and the remainder is −4.


4 2 R x4 +3x2 −2 R 2 
Therefore, x +3x
2
x +1
−2
= x2
+ 2 − 2
4
x +1 and 2
x +1 dx = x +2− 4
x2 +1 dx =
3
x
3 + 2x − 4tan−1 x + C .

The answer is D.
R2
4. The average value of f (x) = x+2
x+1 over the closed interval [0, 2] is f¯ = 1
2−0
x+2
0 x+1 dx.

Use algebraic manipulation to rewrite x+2


x+1 in a form whose antiderivative is recognizable:
x+2 (x + 1) + 1 x+1 1 1
= = + =1+ .
x+1 x+1 x+1 x+1 x+1
Z 2
1 2
Z  
1 x+2 1 1 2
So, f¯ = dx = 1+ dx = (x + ln |x + 1|)0
2−0 0 x+1 2 0 x+1 2
1
= [(2 + ln 3) − (0 + ln 1)] = 1 + 12 ln 3 .
2
The answer is C.
5. To evaluate x212−9 dx, notice the integrand is a proper rational function in lowest terms.
R

Begin by factoring the denominator: x2 − 9 = (x + 3)(x − 3). Since the factors are linear
12 A B
and distinct, this is a Case 1 type integrand and can be written as (x+3)(x−3) = x+3 + x−3 .
Clear the fractions by multiplying both sides of the equation by (x + 3)(x − 3).

12 = A(x − 3) + B(x + 3).

Grouping like terms, 12 = (A + B)x + (−3A + 3B).


This is an identity in x, so the coefficients of like powers of x must be equal.
A+B =0
−3A + 3B = 12

This is a system of two equations containing two variables.


After solving this system, the solution is A = −2 and B = 2.
12 −2 2
R  −2 
and x212−9 dx = 2
R
So, (x+3)(x−3) = x+3 + x−3 x+3 + x−3 dx.


= −2 ln |x + 3| + 2 ln |x − 3| + C = 2 ln x−3
x+3 + C .

The answer is A.
7.5 Integration of Rational Functions Using Partial Fractions; 7-205
The Logistic Model

3x
R
6. To evaluate (x−2)(x+1) dx, notice the integrand is a proper rational function in lowest
terms.
Since the factors are linear and distinct, this is a Case 1 type integrand and can be written
3x A B
as (x−2)(x+1) = x−2 + x+1 .

Clear the fractions by multiplying both sides of the equation by (x − 2)(x + 1).

3x = A(x + 1) + B(x − 2).

Grouping like terms, 3x = (A + B)x + (A − 2B).


This is an identity in x, so the coefficients of like powers of x must be equal.
A+B =3
A − 2B = 0

This is a system of two equations containing two variables.


After solving this system, the solution is A = 2 and B = 1.
3x 2 1
R 3x
R 2 1

So, (x−2)(x+1) = x−2 + x+1 and (x−2)(x+1) dx = x−2 + x+1 dx =
2 ln |x − 2| + ln |x + 1| + C .

The answer is D.
R x+6
7. To evaluate x(x+2) dx, notice the integrand is a proper rational function in lowest terms.
Since the factors are linear and distinct, this is a Case 1 type integrand and can be written
x+6
as x(x+2) = A B
x + x+2 .

Clear the fractions by multiplying both sides of the equation by x(x + 2).

x + 6 = A(x + 2) + Bx

Grouping like terms, x + 6 = (A + B)x + 2A.


This is an identity in x, so the coefficients of like powers of x must be equal.
A+B =1
2A = 6

This is a system of two equations containing two variables.


Since 2A = 6, A = 3 and B = 1 − 3 = −2.
x+6
R 3 
= x3 − x+2
2
R x+6 2
So, x(x+2) and x(x+2) dx = x − x+2 dx = 3 ln |x| − 2 ln |x + 2| + C .

The answer is C.
R1
8. To evaluate 0 x22x−1
+3x+2 dx, notice the integrand is a proper rational function in lowest
terms. Begin by factoring the denominator: x2 + 3x + 2 = (x + 1)(x + 2). Since the
factors are linear and distinct, this is a Case 1 type integrand and can be written as
2x−1 A B
(x+1)(x+2) = x+1 + x+2 .

Clear the fractions by multiplying both sides of the equation by (x + 1)(x + 2).

2x − 1 = A(x + 2) + B(x + 1).

Grouping like terms, 2x − 1 = (A + B)x + (2A + B).


7-206 Chapter 7 Techniques of Integration

This is an identity in x, so the coefficients of like powers of x must be equal.


A+B =2
2A + B = −1
This is a system of two equations containing two variables.
After solving this system, the solution is A = −3 and B = 5.
2x−1 −3 5
R1 R 1  −3 
So, (x+1)(x+2) = x+1 + x+2 and 0 x22x−1
+3x+2 dx = 0 x+1 + 5
x+2 dx =

1
[−3 ln |x + 1| + 5 ln |x + 2|]0 = (−3 ln 2 + 5 ln 3) − (−3 ln 1 + 5 ln 2) = −8 ln 2 + 5 ln 3.
The answer is D.
9. Express the logistic model dP P dP P
 
dt = P 3 − 2000 in the form dt = kP 1 − M to find the
carrying capacity.
   
dP P P
=P 3− = 3P 1 − .
dt 2000 6000
Therefore, the carrying capacity is M = 6000 .
The answer is C.
10. Express the logistic model dP dP P

dt = 0.0005P (800 − P ) in the form dt = kP 1 − M to find
the carrying capacity M and the maximum population growth rate k.
   
dP P P
= 0.0005P (800 − P ) = 0.0005(800) 1 − = 0.4 1 − .
dt 800 800
(a) The carrying capacity is M = 800 .
(b) The maximum population growth rate is = 0.4 or 40% .
(c) At the inflection point, the size of the population is given by one-half the carrying
capacity. Therefore, the size of the population is 400 at the inflection point.
11. (a) The size of the insect population at time t follows a logistic growth model dPdt =
P

kP 1 − M where k is the maximum growth rate and M is the carrying capacity.
We are given the daily maximum growth rate of k = 0.20, a carrying capacity of
M = 600,000 insects, and an initial population size of 100 insects. The differential
 
equation becomes dPdt = 0.20P 1 − P
600,000 with P (0) = 100 insects.

(b) The initial population size is P0 = 100 insects.


The solution to the differential equation dP P M

dt = kP 1 − M is P (t) = 1+ae−kt where
k = 0.20, M = 600,000, and a = M−P P0
0
= 600,000−100
100 = 5999. Therefore,
600,000
P (t) = 1+5999e−0.20t .
(c) Find t so that P (t) = 100,000.
600,000
= 100,000
1 + 5999e−0.20t
600,000
100,000 − 1 5
e−0.20t = =
5999  5999
5
−0.20t = ln
5999
 
1 5
t=− ln = 35.449 days .
0.20 5999
The population of insects will exceed 100,000 on the 35th day.
7.6 Approximating Integrals: The Trapezoidal Rule, 7-207
Trapezoidal Sums, Simpson’s Rule

7.6 Approximating Integrals: The Trapezoidal Rule,


Trapezoidal Sums, Simpson’s Rule
Concepts and Vocabulary
1. True
2. True

Skill Building
6−0
3. When n = 3 we have ∆x = 3 = 2. So the Trapezoidal Rule provides the approximation
6
∆x
Z
f (x) dx ≈ [f (0) + 2f (2) + 2f (4) + f (6)]
0 2
2
= (6 + 2(3) + 2(3) + 4)
2
= 22 .
6−0
When n = 6 we have ∆x = 6 = 1. So the Trapezoidal Rule provides the approximation
6
∆x
Z
f (x) dx ≈ [f (0) + 2f (1) + 2f (2) + 2f (3) + 2f (4) + 2f (5) + f (6)]
0 2
1
= (6 + 2(3) + 2(3) + 2(4) + 2(3) + 2(2) + 4)
2
= 20 .

5−1
4. When n = 2 we have ∆x = 2 = 2. So the Trapezoidal Rule provides the approximation
5
∆x
Z
f (x) dx ≈ [f (1) + 2f (3) + f (5)]
1 2
2
= (0 + 2(3) + 6)
2
= 12 .
5−1
When n = 4 we have ∆x = 4 = 1. So the Trapezoidal Rule provides the approximation
5
∆x
Z
f (x) dx ≈ [f (1) + 2f (2) + 2f (3) + 2f (4) + f (5)]
1 2
1
= (0 + 2(2.5) + 2(3) + 2(6) + 6)
2
29
= 2 = 14.5 .

6−0
5. When n = 2 we have ∆x = 2 = 3. So Simpson’s Rule provides the approximation
6
∆x
Z
f (x) dx ≈ [f (0) + 4f (3) + f (6)]
0 3
3
= (6 + 4(4) + 4)
3
= 26 .
7-208 Chapter 7 Techniques of Integration

When n = 6 we have ∆x = 6−06 = 1. So Simpson’s Rule provides the approximation


Z 6
∆x
f (x) dx ≈ [f (0) + 4f (1) + 2f (2) + 4f (3) + 2f (4) + 4f (5) + f (6)]
0 3
1
= (6 + 4(3) + 2(3) + 4(4) + 2(3) + 4(2) + 4)
3
58
= 3 .

6. When n = 2 we have ∆x = 5−12 = 2. So Simpson’s Rule provides the approximation


Z 5
∆x
f (x) dx ≈ [f (1) + 4f (3) + f (5)]
1 3
2
= (0 + 4(3) + 6)
3
= 12 .
When n = 4 we have ∆x = 5−1
4 = 1. So Simpson’s Rule provides the approximation
Z 5
∆x
f (x) dx ≈ [f (1) + 4f (2) + 2f (3) + 4f (4) + f (5)]
1 3
1
= (0 + 4(2.5) + 2(3) + 4(6) + 6)
3
46
= 3 .

7. (a) With n = 3 we have ∆x = π−π/2


3 = π/6. So the Trapezoidal Rule provides the
approximation
Z π      
sin x ∆x  π  2π 5π
dx ≈ f + 2f + 2f + f (π)
π/2 x 2 2 3 6
π 2π 5π
π/6 sin 2 sin 3 sin 6
 
sin π
= π + 2 2π + 2 5π +
2 2 3
π
" √ # 6
π 2 3 3 6
= + + +0
12 π 2 π 5π
≈ 0.483 .
d sin x d2 sin x
= − x12 (sin x − x cos x), and dx − x13 x2 sin x − 2 sin x + 2x cos x .
  
(b) We have dx x 2 x =
The graph of |f ′′ (x)| = − x13 x2 sin x − 2 sin x + 2x cos x shows

 that the maximum
occurs at x = π, and so M = − π13 π 2 sin π − 2 sin π + 2π cos π = π22 . We obtain
3
(π − π/2) π22

Error ≤ 2 ≈ 0.007
12(3)

0.3

0.2

0.1 |f ''(x)|

0.0
1.6 1.8 2.0 2.2 2.4 2.6 2.8 3.0
7.6 Approximating Integrals: The Trapezoidal Rule, 7-209
Trapezoidal Sums, Simpson’s Rule

(c) We require
3 2

(π − π/2) π2 π
Error ≤ 2 = < 0.0001
12(n) 48n2
π 2
< n
48(0.0001)
π
r
< n
48(0.0001)
25. 58 < n
So we need n = 26 .

8. (a) With n = 3 we have ∆x = 2π−3π/2


3 = π/6. So the Trapezoidal Rule provides the
approximation
Z 2π        
cos x ∆x 3π 5π 11π
dx ≈ f + 2f + 2f + f (2π)
3π/2 x 2 2 3 6
π/6 cos 3π cos 5π cos 11π
 
2 3 6 cos (2π)
= 3π + 2 5π + 2 11π +
2 2 3

" √ # 6
π 3 6 3 1
= 0+ + +
12 5π 11 π 2π
≈ 0.1704 .
d cos x d2 cos x
= − x12 (cos 1 2
  
(b) We have dx x x + x sin x), and dx 2 x = x 3 2 cos x − x cos x + 2x sin x .

The graph of |f ′′ (x)| = x13 2 cos x − x2 cos x + 2x sin x shows that the maximum

M < 0.2. We obtain
(2π − 3π/2)3 (0.2)
Error ≤ 2 ≈ 0.0072
12(3)

0.3

0.2 |f ''(x)|

0.1

0.0
4.8 5.0 5.2 5.4 5.6 5.8 6.0 6.2

(c) We require
3
(2π − 3π/2) (0.2) π3
Error ≤ 2 = < 0.0001
12(n) 480n2
π3
< n2
0.0001(480)
s
π3
< n
0.0001(480)
25. 42 < n
So we need n = 26 .
7-210 Chapter 7 Techniques of Integration

9. (a) With n = 4 we have ∆x = 1−0 1


4 = 4 . So the Trapezoidal Rule provides the approxi-
mation
Z 1        
2 ∆x 1 1 3
e−x dx ≈ f (0) + 2f + 2f + 2f + f (1)
0 2 4 2 4
1/4 h −(0)2 1 2 1 2 3 2
i
+ 2e−( 4 ) + 2e−( 2 ) + 2e−( 4 ) + e−(1)
2
= e
2
1h 1 1 9
i
= 1 + 2e− 16 + 2e− 4 + 2e− 16 + e−1
8
≈ 0.743 .
   
d 2 2
d2 2 2 2
(b) We have dxe−x = −2xe−x , and dx 2 e−x = 4x2 e−x − 2e−x . The graph of
2

2
|f ′′ (x)| = 4x2 e−x − 2e−x shows that the maximum M = 2. We obtain

3
(1 − 0) (2) 1
Error ≤ 2 =
12(4) 96

2 |f ''(x)|

0
0.0 0.2 0.4 0.6 0.8 1.0

(c) We require
3
(1 − 0) (2) 1
Error ≤ 2 = < 0.0001
12(n) 6n2
1
< n2
6(0.0001)
s
1
< n
6(0.0001)
40. 82 < n

So we need n = 41 .

10. (a) With n = 4 we have ∆x = 1−0 1


4 = 4 . So the Trapezoidal Rule provides the approxi-
mation
Z 1        
2 ∆x 1 1 3
ex dx ≈ f (0) + 2f + 2f + 2f + f (1)
0 2 4 2 4
1/4 (0)2
h 2 2 2
i
+ 2e( 4 ) + 2e( 2 ) + 2e( 4 ) + e(1)
1 1 3 2
= e
2
1h 1 1 9
i
= 1 + 2e 16 + 2e 4 + 2e 16 + e
8
≈ 1.491 .
7.6 Approximating Integrals: The Trapezoidal Rule, 7-211
Trapezoidal Sums, Simpson’s Rule

 2  2
d 2
d2 2 2
(b) We have dx ex = 2xex , and dx 2 ex = 2ex + 4x2 ex . The graph of |f ′′ (x)| =
2
x 2 2 2
2e + 4x2 ex shows that the maximum M = 2e + 4(1) e1 ≈ 16. 31. We obtain

3
(1 − 0) (16. 31)
Error ≤ 2 ≈ 0.085
12(4)

20

10 |f ''(x)|

0
0.0 0.2 0.4 0.6 0.8 1.0

(c) We require

(1 − 0)3 (16. 31) 1. 359


Error ≤ 2 = < 0.0001
12(n) n2
1. 359
< n2
r 0.0001
1. 359
< n
0.0001
116. 6 < n

So we need n = 117 .

11. (a) With n = 4 we have ∆x = 0−(−1)


4 = 14 . So the Trapezoidal Rule provides the
approximation
Z 0        
dx ∆x −3 −1 −1
√ ≈ f (−1) + 2f + 2f + 2f + f (0)
−1 1 − x3 2 4 2 4

1/4  1 1 1
= q + 2q + 2q
2
1 − (−1)3 1 − (−3/4)3 1 − (−1/2)3

1 1
+ 2q +q 
3
1 − (−1/4) 1 − (0)3
1 1√ 16 √ 4√ 16 √
 
= 2+ 91 + 2+ 65 + 1
8 2 91 3 65
≈ 0.907 .
7-212 Chapter 7 Techniques of Integration

3x(5x3 +4)
   
d √ 1 3x2 d2 √ 1
(b) We have dx 1−x3
= 3 , and dx2 1−x3
= 5 . The graph of
2(1−x3 ) 2 4(1−x3 ) 2
3x (5x 3
+4)
|f ′′ (x)| = 5
shows that the maximum M ≤ 1. We obtain
4(1−x3 ) 2

3
(0 − (−1)) (1) 1
Error ≤ 2 =
12(4) 192

1.0

|f ''(x)| 0.5

0.0
-1.0 -0.8 -0.6 -0.4 -0.2 0.0

(c) We require

(0 − (−1))3 (1) 1
Error ≤ 2 = < 0.0001
12(n) 12n2
1
< n2
0.0001(12)
s
1
< n
0.0001(12)
28. 87 < n

So we need n = 29 .

12. (a) With n = 3 we have ∆x = 1−0 1


3 = 3 . So the Trapezoidal Rule provides the approxi-
mation
Z 1      
dx ∆x 1 2
√ ≈ f (0) + 2f + 2f + f (1)
0 1 + x3 2 3 3
 
1/3  1 1 1 1
= q + 2q  + 2q  +q

2 3 1 3 2 3
1 + (0) 1+ 3 1+ 3 1 + (1)3
3√ 6√ 1√
 
1
= 1+ 21 + 105 + 2
6 7 35 2
≈ 0.9046 .
7.6 Approximating Integrals: The Trapezoidal Rule, 7-213
Trapezoidal Sums, Simpson’s Rule

3x(5x3 −4)
   
d √ 1 −3x2 d2 √ 1
(b) We have dx 1+x3
= 3 , and dx2 1+x3
= 5 . The graph of
2(1+x3 ) 2 4(1+x3 ) 2
3x (5x 3
+4)
|f ′′ (x)| = 5
shows that the maximum M ≤ 1. We obtain
4(1−x3 ) 2

3
(1 − 0) (1)
Error ≤ 2 ≈ 0.0052
12(4)

1.0

0.5 |f ''(x)|

0.0
0.0 0.2 0.4 0.6 0.8 1.0

(c) We require

(1 − 0)3 (1) 1
Error ≤ 2 = < 0.0001
12(n) 12n2
1
< n2
0.0001(12)
s
1
< n
0.0001(12)
28. 87 < n

So we need n = 29 .
2−1
13. (a) With n = 4 we have ∆x = 4 = 14 . So Simpson’s Rule provides the approximation
2
ex
       
∆x 5 3 7
Z
dx ≈ f (1) + 4f + 2f + 4f + f (2)
1 x 3 4 2 4
1/4 e1 e5/4 e3/2 e7/4 e2
 
= +4 +2 +4 +
3 1 5/4 3/2 7/4 2
 
1 16 5 4 3 16 7 1
= e + e 4 + e 2 + e 4 + e2
12 5 3 7 2
≈ 3.059 .
7-214 Chapter 7 Techniques of Integration

d4 ex 1 x
 4 3 2
 4
(4)dx4 x = x5 e x − 4x + 12x − 24x + 24 . The graph of f (x) shows
(b) We have
M ≤ f (1) = 9e. We obtain
5
(2 − 1) (9e)
Error ≤ ≈ 5. 309 × 10−4
180(4)4

30

20
|f ''''(x)|
10

0
1.0 1.2 1.4 1.6 1.8 2.0

(c) We require
5
(2 − 1) (9e)
Error ≤ 4 < 0.0001
180(n)
9e
< n4
0.0001(180)
s
9e
4
< n
0.0001(180)
6. 072 < n

Since n must be even, we need n = 8 .

14. (a) With n = 4 we have ∆x = 2π−3π/2


4 = π8 . So Simpson’s Rule provides the approxima-
tion
Z 2π          
cos x ∆x 3π 13π 7π 15π
dx ≈ f + 4f + 2f + 4f + f (2π)
3π/2 x 3 2 8 4 8
 
π/8 cos (3π/2) cos (13π/8) cos (7π/4) cos (15π/8) cos (2π)
= +4 +2 +4 +
3 3π/2 13π/8 7π/4 15π/8 2π
≈ 0.1759 .
7.6 Approximating Integrals: The Trapezoidal Rule, 7-215
Trapezoidal Sums, Simpson’s Rule

d4 cos x
= x15 24 cos x − 12x2 cos x + x4 cos x − 4x3 sin x + 24x sin x . The
 
(b) We have dx 4 x

graph of f 4 (x) shows M ≤ 0.2. We obtain
5
(2π − 3π/2) (0.2)
Error ≤ 4 ≈ 4. 151 × 10−5
180(4)

0.2

0.1
|f ''''(x)|

0.0
4.8 5.0 5.2 5.4 5.6 5.8 6.0 6.2

(c) We require
5
(2π − 3π/2) (0.2) 1. 063 × 10−2
Error ≤ 4 = < 0.0001
180(n) n4
−2
1. 063 × 10
< n4
r 0.0001
−2
4 1. 063 × 10
< n
0.0001
3. 211 < n

Since n must be even, we need n = 4 .


1−0
15. (a) With n = 4 we have ∆x = 4 = 14 . So Simpson’s Rule provides the approximation
1        
∆x 1 1 3
Z
2
e−x dx ≈ f (0) + 4f + 2f + 4f + f (1)
0 3 4 2 4
1/4 −(0)2
h 2 2 2 2
i
= e + 4e−(1/4) + 2e−(1/2) + 4e−(3/4) + e−(1)
3
1 h 1 1 9
i
= 1 + 4e− 16 + 2e− 4 + 4e− 16 + e−1
12
≈ 0.747 .
7-216 Chapter 7 Techniques of Integration

 
d4 2 2
e−x = 4e−x 4x4 − 12x2 + 3 . The graph of f 4 (x) shows M ≤ 12.

(b) We have dx 4

We obtain
5
(1 − 0) (12)
Error ≤ 4 ≈ 2.604 × 10−4
180(4)

15

10 |f ''''(x)|

0
0.0 0.2 0.4 0.6 0.8 1.0

(c) We require

(1 − 0)5 (12) 1
Error ≤ 4 = < 0.0001
180(n) 15n4
1 4
< n
0.0001(15)
s
1
4
< n
0.0001(15)
5. 081 < n

We need n = 6 .
1−0
16. (a) With n = 4 we have ∆x = 4 = 41 . So Simpson’s Rule provides the approximation
1        
∆x 1 1 3
Z
2
ex dx ≈ f (0) + 4f + 2f + 4f + f (1)
0 3 4 2 4
1/4 (0)2
h 2 2 2 2
i
= e + 4e(1/4) + 2e(1/2) + 4e(3/4) + e(1)
3
1 h 1 1 9
i
= 1 + 4e 16 + 2e 4 + 4e 16 + e
12
≈ 1.464 .
7.6 Approximating Integrals: The Trapezoidal Rule, 7-217
Trapezoidal Sums, Simpson’s Rule

 2
d4 2
ex = 4ex 4x4 + 12x2 + 3 . The graph of f 4 (x) shows M = 76e ≈

(b) We have dx 4

206. 6. We obtain
5
(1 − 0) (76e)
Error ≤ 4 ≈ 4. 483 × 10−3
180(4)

200

100 |f ''''(x)|

0
0.0 0.2 0.4 0.6 0.8 1.0

(c) We require

(1 − 0)5 (76e) 1. 148


Error ≤ 4 ≈ 4 < 0.0001
180(n) (n)
1. 148
< n4
r 0.0001
4 1. 148
< n
0.0001
10. 35 < n

Since n must be even, we need n = 12 .


0−(−1)
17. (a) With n = 4 we have ∆x = 4 = 14 . So Simpson’s Rule provides the approximation
0        
dx ∆x −3 −1 −1
Z
√ ≈ f (−1) + 4f + 2f + 4f + f (0)
−1 1 − x3 3 4 2 4

1/4  1 1 1
= q + 4q + 2q
3 3 3
1 − (−1) 1 − (−3/4) 1 − (−1/2)3

1 1
+4 q +q 
3 3
1 − (−1/4) 1 − (0)
√ √ 4√ 32 √
 
1 1 32
= 2+ 91 + 2+ 65 + 1
12 2 91 3 65
≈ 0.910 .
7-218 Chapter 7 Techniques of Integration

135x2 (7x6 +40x3 +16)


 
d4 √ 1

(b) We have dx4 1−x3
= 9 . The graph of f 4 (x) shows M ≤ 15. We
16(1−x3 ) 2
obtain
5
(0 − (−1)) (15)
Error ≤ ≈ 3. 255 × 10−4
180(4)4

15

10
|f ''''(x)|
5

0
-1.0 -0.8 -0.6 -0.4 -0.2 0.0

(c) We require
5
(0 − (−1)) (15) 1
Error ≤ 4 = 4 < 0.0001
180(n) 12(n)
1
< n4
0.0001(12)
s
1
4
< n
0.0001(12)
5. 373 < n

We need n = 6 .
1−0
18. (a) With n = 4 we have ∆x = 4 = 41 . So Simpson’s Rule provides the approximation
1        
dx ∆x 1 1 3
Z
√ ≈ f (0) + 4f + 2f + 4f + f (1)
0 1 + x2 3 4 2 4

1/4  1 1 1
= q + 4q + 2q
3 2 2 2
1 + (0) 1 + (1/4) 1 + (1/2)

1 1
+4 q +q 
2 2
1 + (3/4) 1 + (1)
16 √ 4√ 16 1 √
 
1
= 1+ 17 + 5+ + 2
12 17 5 5 2
≈ 0.8814 .
7.6 Approximating Integrals: The Trapezoidal Rule, 7-219
Trapezoidal Sums, Simpson’s Rule

 
d4 √ 1 24x4 −72x2 +9

(b) We have dx4 1+x2
= 9 . The graph of f 4 (x) shows M = 9. We obtain
(x2 +1) 2
5
(1 − 0) (9) 1
Error ≤ 4 = ≈ 1. 953 × 10−4
180(4) 5120

10

5 |f ''''(x)|

0
0.0 0.2 0.4 0.6 0.8 1.0

(c) We require
5
(1 − 0) (9) 1
Error ≤ 4 = 4 < 0.0001
180(n) 20(n)
1
< n4
0.0001(20)
√4
500 < n
4.729 < n
We need n = 6 .
R2 2
19. (a) 1 dx
x = [ln x]1 = ln 2 − ln 1 = ln 2.
(b) With n = 5 we have ∆x = 2−1 1
5 = 5 . So the Trapezoidal Rule provides the approxi-
mation
Z 2          
dx ∆x 6 7 8 9
≈ f (1) + 2f + 2f + 2f + 2f + f (2)
1 x 2 5 5 5 5
 
1/5 1 1 1 1 1 1
= +2 +2 +2 +2 +
2 1 6/5 7/5 8/5 9/5 2
 
1 5 10 5 10 1
= 1+ + + + +
10 3 7 4 9 2
1753
=
2520
≈ 0.6956 .
(c) With n = 6 we have ∆x = 2−1 1
6 = 6 . So Simpson’s Rule provides the approximation
Z 2            
dx ∆x 7 4 3 5 11
≈ f (1) + 4f + 2f + 4f + 2f + 4f + f (2)
1 x 3 6 3 2 3 6
 
1/6 1 1 1 1 1 1 1
= +4 +2 +4 +2 +4 +
3 1 7/6 4/3 3/2 5/3 11/6 2
 
1 1 24 3 8 6 24 1
= + + + + + +
18 1 7 2 3 5 11 2
14 411
=
20 790
≈ 0.6932 .
7-220 Chapter 7 Techniques of Integration

R 4.4
20. The area is given by the integral 2 f (x) dx, which we approximate by Simpson’s Rule,
with ∆x = 0.4.
Z 4.4
∆x
f (x) dx ≈ [f (2.0) + 4f (2.4) + 2f (2.8) + 4f (3.2) + 2f (3.6) + 4f (4.0) + f (4.4)]
2 3
0.4
= [3.03 + 4(4.61) + 2(5.80) + 4(6.59) + 2(7.76) + 4(8.46) + 9.19]
3
≈ 15.73 .
r
R π/2 
dy
2 R π/2 q 2 R π/2 √
21. The arc length is given by 0 1+ dx dx = 0 1 + (cos x) dx = 0 1 + cos2 x dx.

(a) With n = 4 we have ∆x = (π/2)−0


4 = π8 . So Simpson’s Rule provides the approxima-
tion
Z π/2 p     π 
2
∆x π  π  3π
1 − cos x dx ≈ f (0) + 4f + 2f + 4f +f
0 3 8 4 8 2
p r r
π/8 π π
= 1 + cos2 0 + 4 1 + cos2 + 2 1 + cos2
3 8 4
r r #
3π π
+4 1 + cos2 + 1 + cos2
8 2
≈ 1.910 .

(π/2)−0 π
(b) With n = 3 we have ∆x = 3 = 6. So the Trapezoidal Rule provides the
approximation
π/2
∆x h π  π   π i
Z p
1 − cos2 x dx ≈ f (0) + 2f + 2f +f
0 2 6 3 2
 r r r 
π/6 p 2 2
π 2
π 2
π
= 1 + cos 0 + 2 1 + cos + 2 1 + cos + 1 + cos
2 6 3 2
≈ 1. 910 .
r
R4  2 R4q R4√
22. The arc length is given by 0
1+ dy
dx dx = 0
1 + (ex )2 dx = 0 1 + e2x dx.

4−0
(a) With n = 4 we have ∆x = 4 = 1. So Simpson’s Rule provides the approximation
4
∆x
Z p
1 + e2x dx ≈ [f (0) + 4f (1) + 2f (2) + 4f (3) + f (4)]
0 3
1 hp p p p p i
= 1 + e2(0) + 4 1 + e2(1) + 2 1 + e2(2) + 4 1 + e2(3) + 1 + e2(4)
3
≈ 54.32 .
7.6 Approximating Integrals: The Trapezoidal Rule, 7-221
Trapezoidal Sums, Simpson’s Rule

(b) With n = 8 we have ∆x = 4−0 1


8 = 2 . So the Trapezoidal Rule provides the approxi-
mation
Z 4p
∆x
1 + e2x dx ≈ [f (0) + 2f (1/2) + 2f (1) + 2f (3/2) + 2f (2)
0 2
+2f (5/2) + 2f (3) + 2f (7/2) + f (4)]
1 p q q
2( 12 )
+ 2 1 + e2( 2 )
p 3
2 2(0) 2(1)
= 1+e +2 1+e +2 1+e
2
q q 
+ 2 1 + e2(2) + 2 1 + e2( 2 ) + 2 1 + e2(3) + 2 1 + e2( 2 ) + 1 + e2(4)
p 5
p 7
p

≈ 55.17 .
R 2.5
23. The work is given by the integral 1 p dV , which we approximate by Simpson’s Rule, with
∆V = 0.25.
Z 2.5
∆V
p dV ≈ [f (1.0) + 4f (1.25) + 2f (1.5) + 4f (1.75) + 2f (2.0) + 4f (2.25) + f (2.5)]
1 3
0.25
= [68.7 + 4(55.0) + 2(45.8) + 4(39.3) + 2(34.4) + 4(30.5) + 27.5]
3
≈ 62.983 inch-pounds .

R 50
24. The work is given by the integral 0 F dx, which we approximate by the Trapezoidal Rule,
with ∆x = 5.
Z 50
∆x
F dx ≈ [f (0) + 2f (5) + 2f (10) + 2f (15) + 2f (20)
0 2
+2f (25) + f (30) + 2f (35) + 2f (40) + 2f (45) + f (50)]
5
= [100 + 2(80) + 2(66) + 2(56) + 2(50) + 2(45) + 2(40) + 2(36) + 2(33) + 2(30) + 28]
2
= 2500 .

R 150
25. The volume is given by the integral 0
S dx, which we approximate by the Trapezoidal
Rule, with ∆x = 25.
150
∆x
Z
S dx ≈ [f (0) + 2f (25) + 2f (50) + 2f (75) + 2f (100) + 2f (125) + f (150)]
0 2
25
= [105 + 2(118) + 2(142) + 2(120) + 2(110) + 2(90) + 78]
2
≈ 16, 787.5 m3 .

26. Let y represent


R 20 the vertical distance of the pond at position x. Then the area is given by
the integral 0 y dx, which we approximate by Simpson’s Rule, with ∆x = 5.
20
∆x
Z
y dx ≈ [f (0) + 4f (5) + 2f (10) + 4f (15) + f (20)]
0 3
5
= [0 + 4(12) + 2(19) + 4(13) + 2(13) + 0]
3
≈ 273.3 ft2 .
7-222 Chapter 7 Techniques of Integration

R 25
27. The volume is given by the integral 0 A dx. We first approximate by the Trapezoidal
Rule, with ∆x = 2.5.
Z 25
∆x
A dx ≈ [f (0) + 2f (2.5) + 2f (5.0) + 2f (7.5) + 2f (10) + 2f (12.5)
0 2
+2f (15.0) + 2f (17.5) + 2f (20) + 2f (22.5) + f (25)]
2.5
= [0 + 2(2510) + 2(3860) + 2(4870) + 2(5160) + 2(5590)
2
+2(5810) + 2(6210) + 2(6890) + 2(7680) + 8270]
≈ 131, 787.5 m3 .

We next approximate by Simpson’s Rule.


Z 25
∆x
A dx ≈ [f (0) + 4f (2.5) + 2f (5.0) + 4f (7.5) + 2f (10) + 4f (12.5)
0 3
+2f (15.0) + 4f (17.5) + 2f (20) + 4f (22.5) + f (25)]
2.5
= [0 + 4(2510) + 2(3860) + 4(4870) + 2(5160) + 4(5590)
3
+2(5810) + 4(6210) + 2(6890) + 4(7680) + 8270]
≈ 132, 625 m3 .

R 80
28. The area is given by the integral 0 y dx. We approximate by the Trapezoidal Rule, with
∆x = 10.
Z 80
∆x
y dx ≈ [f (0) + 2f (10) + 2f (20) + 2f (30) + 2f (40)+
0 2
+2f (50) + 2f (60) + 2f (70) + f (80)]
10
= [5 + 2(10) + 2(13.2) + 2(15)
2
+2(15.6) + 2(12) + 2(6) + 2(4) + 0]
≈ 783.0 m2 .
R8
29. The volume is given by the integral 2 πy 2 dx. We approximate by the Trapezoidal Rule,
with ∆x = 2.
Z 8
∆x
πy 2 dx ≈ [f (2) + 2f (4) + 2f (6) + f (8)]
2 2
2h 2 2 2 2
i
= π (1) + 2π(3) + 2π(3.5) + π (3)
2
≈ 164.934 .
R3
30. The distance traveled is given by the integral 0
v dt.

(a) We approximate by the Trapezoidal Rule, with ∆t = 0.5.


Z 3
∆t
v dt ≈ [f (0) + 2f (0.5) + 2f (1) + 2f (1.5) + 2f (2) + 2f (2.5) + f (3)]
0 2
0.5
= [5.1 + 2(5.3) + 2(5.6) + 2(6.1) + 2(6.8) + 2(6.7) + 6.5]
2
= 18.15 m .
7.6 Approximating Integrals: The Trapezoidal Rule, 7-223
Trapezoidal Sums, Simpson’s Rule

(b) We approximate by Simpson’s Rule, with ∆t = 0.5.


3
∆t
Z
v dt ≈ [f (0) + 4f (0.5) + 2f (1) + 4f (1.5) + 2f (2) + 4f (2.5) + f (3)]
0 3
0.5
= [5.1 + 4(5.3) + 2(5.6) + 4(6.1) + 2(6.8) + 4(6.7) + 6.5]
3
= 18. 13 m .
R1 2
31. The volume is given by 0 π sin−1 y dy.
1−0
(a) With n = 4 we have ∆y = 4 = 41 . Using the disk method, Simpson’s Rule provides
the approximation
1        
∆y 1 1 3
Z
2
π sin−1 y dy ≈ f (0) + 4f + 2f + 4f + f (1)
0 3 4 2 4
"  2 !  2 !
1/4 −1
2 −1 1 −1 1
= π sin 0 + 4 π sin + 2 π sin
3 4 2
 2 ! #
3 2
+ 4 π sin−1 + π sin−1 1
4
≈ 1.6095 .
π/2−0 π
Using the shell method, we have ∆x = 4 = 8. Simpson’s Rule provides the
approximation
Z π/2
2πx (1 − sin x) dx
0
    π 
∆x π  π  3π
≈ f (0) + 4f + 2f + 4f +f
3 8 4 8 2
π/8 h  π    π   π  π 
= 2π (0) (1 − sin (0)) + 4 2π 1 − sin + 2 2π 1 − sin
3     8 8 4 4
3π 3π π  π 
+ 4 2π 1 − sin + 2π 1 − sin
8 8 2 2
≈ 1.4709 .

1−0 1
(b) With n = 3 we have ∆y = 3 = 3. Using the disk method, Trapezoidal Rule
provides the approximation
Z 1 2
π sin−1 y dy
0
     
∆y 1 2
≈ f (0) + 2f + 2f + f (1)
2 3 3
"  2 !  2 ! #
1/3 −1
 −1 1 −1 2 −1
2
= π sin 0 + 2 π sin + 2 π sin + π sin 1
2 3 3
≈ 1.9705 .
7-224 Chapter 7 Techniques of Integration

π/2−0 π
Using the shell method, we have ∆x = 3 = 6. The Trapezoidal Rule provides
the approximation
π/2
∆x h π  π   π i
Z
2πx (1 − sin x) dx ≈ f (0) + 2f + 2f +f
0 2 6 3 2
π/6 h  π   π 
= 2π (0) (1 − sin (0)) + 2 2π 1 − sin
2    6   6 i
π  π  π π
+ 2 2π 1 − sin + 2π 1 − sin
3 3 2 2
≈ 1.3228 .

R8q 2
32. The arc length is given by 0 1 + (dy/dx) dx. We use implicit differentiation, and obtain

9x2 + 100y 2 = 900


dy
18x + 200y = 0
dx
dy 9x
= − .
dx 100y
 2  2
dy 9x 81x2 81x2 9x2
So dx = − 100y = 100(100y 2 ) = 100(900−9x2 ) = 100(100−x2 ) , and
s
8 8
9x2
Z q Z
1 + (dy/dx)2 dx = 1+ dx
0 0 100(100 − x2 )
8
r
1 10000 − 91x2
Z
= dx.
0 10 100 − x2
8−0
The Trapezoidal Rule, with n = 4 and ∆x = 4 = 2 is used.
8
∆x
Z q
2
1 + (dy/dx) dx ≈ [f (0) + 2f (2) + 2f (4) + 2f (6) + f (8)]
0 2
  "s 2
s
2
!
2 1 10000 − 91(0) 10000 − 91(2)
= 2 +2 2
2 10 100 − (0) 100 − (2)
s ! s ! s #
2 2 2
10000 − 91(4) 10000 − 91(6) 10000 − 91(8)
+2 2 +2 2 + 2
100 − (4) 100 − (6) 100 − (8)
≈ 8.148 .

33. (a) With n = 6 we have ∆x = π−0 π


6 = 6 . So the Trapezoidal Rule provides the approxi-
mation
Z π      
∆x π  π  π  2π 5π
f (x) dx ≈ f (0) + 2f + 2f + 2f + 2f + 2f + f (2π)
0 2 6 3 2 3 6

π/6 sin (π/6) sin (π/3) sin (π/2)
= 1+2 +2 +2
2 π/6 π/3 π/2

sin (2π/3) sin (5π/6) sin (2π)
+2 +2 +
2π/3 5π/6 2π
≈ 1.845 .
7.6 Approximating Integrals: The Trapezoidal Rule, 7-225
Trapezoidal Sums, Simpson’s Rule

(b) Simpson’s Rule provides the approximation


Z π      
∆x π  π  π  2π 5π
f (x) dx ≈ f (0) + 4f + 2f + 4f + 2f + 4f + f (2π)
0 3 6 3 2 3 6

π/6 sin (π/6) sin (π/3) sin (π/2)
= 1+4 +2 +4
3 π/6 π/3 π/2

sin (2π/3) sin (5π/6) sin (2π)
+2 +4 +
2π/3 5π/6 2π
≈ 1.852 .

34. (a) With n = 20 we have ∆x = 1−(−1)


20
1
= 10 . So the Trapezoidal Rule provides the
approximation
Z 1
2 ∆x
5e−x dx ≈ [f (−1) + 2f (−0.9) + · · · + 2f (0.9) + f (1)]
−1 2
1/10 h −(−1)2  2
  2
 2
i
= 5e + 2 5e−(−0.9) + · · · + 2 5e−(0.9) + 5e−(1)
2
≈ 7. 462 .

(b) Simpson’s Rule provides the approximation


Z 1
2 ∆x
5e−x dx ≈ [f (−1) + 4f (−0.9) + 2f (−0.8) + · · · + 4f (0.9) + f (1)]
−1 3
1 h    
2 2 2
= 10 5e−(−1) + 4 5e−(−0.9) + 2 5e−(−0.9)
3   i
2 2
+ · · · + 4 5e−(0.9) + 5e−(1)

≈ 7. 468 .
R1 2
(c) Using a CAS, we obtain −1 5e−x dx ≈ 7.468 .

Challenge Problems
35. Since Tn = 21 (Ln + Rn ) where Ln is the Riemann Sum using left endpoints, and Rn is the
Riemann Sum using right endpoints, we have
1
lim Tn = lim (Ln + Rn )
n→∞ n→∞ 2
1 1
= lim Ln + lim Rn
2 n→∞ 2 n→∞
1 b 1 b
Z Z
= f (x) dx + f (x) dx
2 a 2 a
Z b
= f (x) dx.
a

d4
36. Since f (4) (x) = dx 3 2

4 Ax + Bx + Cx + D = 0, we have M = 0, and the error obtained
using Simpson’s Rule satisfies
(b − a)5 (0)
Error ≤ = 0.
180n4
Rb
So an approximation using Simpson’s Rule, with any n, gives the exact value of a
f (x) dx.
7-226 Chapter 7 Techniques of Integration

R
AP Practice Problems
R4
1. Since y = x3 is nonnegative on [0, 4], 0 x3 dx is the area under the graph of y = x3 from
x = 0 to x = 4. Partition [0, 4] into four subintervals, each of equal width: [0, 1], [1, 2], [2, 3],
and [3, 4].
The width of each subinterval is ∆x = 1.
Now apply the Trapezoidal Rule:
Z 4
1 1 1 1
x3 dx ≈ [f (0) + f (1)]∆x + [f (1) + f (2)]∆x + [f (2) + f (3)]∆x + [f (3) + f (4)]∆x
0 2 2 2 2
1
= [f (0) + 2f (1) + 2f (2) + 2f (3) + f (4)]∆x
2
1
= 03 + 2 13 + 2 23 + 2 33 + 43 (1) = 68 .
   
2
The answer is B.
2. Partition [0, 30] into three subintervals, each of equal width: [0, 10], [10, 20], and [20, 30].
The width of each subinterval is ∆x = 10.
Now apply the Trapezoidal Rule:
Z 30
1 1 1
f (x) dx ≈ [f (0) + f (10)]∆x + [f (10) + f (20)]∆x + [f (20) + f (30)]∆x
0 2 2 2
1
= [f (0) + 2f (10) + 2f (20) + f (30)](10)
2
1
= [16 + 2(12) + 2(c) + 6](10)
2
= 230 + 10c.
R 30
Since the estimate of 0 f (x) dx using the Trapezoidal Rule is 310, then 230 + 10c = 310,
and c = 8 .
The answer is B.
3. Partition [−2, 6] into four subintervals, each of equal width: [−2, 0], [0, 2], [2, 4], and [4, 6].
The width of each subinterval is ∆x = 2.
Now apply the Trapezoidal Rule:
Z 6
2 1 1 1 1
ex dx ≈ [f (−2) + f (0)]∆x + [f (0) + f (2)]∆x + [f (2) + f (4)]∆x + [f (4) + f (6)]∆x
−2 2 2 2 2
1
= [f (−2) + 2f (0) + 2f (2) + 2f (4) + f (6)]∆x
2
1 4
= e + 2 · e0 + 2 · e4 + 2 · e16 + e36 (2)

2
= 2 + 3e4 + 2e16 + e36 .

The answer is A.
4. The
R 12 total volume V of water in the tank after 12 hours is given by the definite integral

0
V (t) dt.
R 12
Approximate 0 V ′ (t) dt using the Trapezoidal Rule with n = 4.
7.6 Approximating Integrals: The Trapezoidal Rule, 7-227
Trapezoidal Sums, Simpson’s Rule

Partition [0, 12] into four subintervals: [0, 2], [2, 5], [5, 10], and [10, 12].
∆t1 = 2 − 0 = 2, ∆t2 = 5 − 2 = 3, ∆t3 = 10 − 5 = 5, and ∆t4 = 12 − 10 = 2.
Now apply the Trapezoidal Rule:
Z 12
1 1 1
V ′ (t) dt ≈ [f (0) + f (2)]∆t1 + [f (2) + f (5)]∆t2 + [f (5) + f (10)]∆t3
0 2 2 2
1
+ [f (10) + f (12)]∆t4
2
1 1 1 1
= (16 + 12)(2) + (12 + 10)(3) + (10 + 12)(5) + (12 + 8)(2)
2 2 2 2
= 28 + 33 + 55 + 20 = 136 .
The answer is C.
4−0
5. (a) Partition the interval [0, 4] into four subintervals of equal width ∆x = 4 = 1.
The four subintervals are [0, 1], [1, 2], [2, 3], and [3, 4].
In each subinterval, choose ui as the left endpoint of the ith interval.
Then u1 = 0, u2 = 1, u3 = 2, and u4 = 3.
Z 4 4
1 X 1
dx ≈ ∆x
0 1+x
3
i=1
1 + u3i
= [f (0) + f (1) + f (2) + f (3)]∆x
 
1 1 1
= 1+ + + · 1 ≈ 1.647
2 9 28

(b) Partition the interval [0, 4] into four subintervals of equal width ∆x = 4−0
4 = 1.
The four subintervals are [0, 1], [1, 2], [2, 3], and [3, 4].
In each subinterval, choose ui as the right endpoint of the ith interval.
Then u1 = 1, u2 = 2, u3 = 3, and u4 = 4.
Z 4 4
1 X 1
dx ≈ ∆x
0 1+x
3
i=1
1 + u3i
= [f (1) + f (2) + f (3) + f (4)]∆x
 
1 1 1 1
= + + + · 1 ≈ 0.662
2 9 28 65

(c) Partition the interval [0, 4] into four subintervals of equal width ∆x = 4−0
4 = 1.
The four subintervals are [0, 1], [1, 2], [2, 3], and [3, 4].
Now apply the Trapezoidal Rule:
Z 4
1 1 1 1
3
dx ≈ [f (0) + f (1)]∆x + [f (1) + f (2)]∆x + [f (2) + f (3)]∆x
0 1 + x 2 2 2
1
+ [f (3) + f (4)]∆x
2
1
= [f (0) + 2f (1) + 2f (2) + 2f (3) + f (4)]∆x
2       
1 1 1 1 1
= 1+2 +2 +2 + (1)
2 2 9 28 65
≈ 1.155 .
7-228 Chapter 7 Techniques of Integration

7.7 Improper Integrals


Concepts and Vocabulary
1. (c), an improper
2. (b) diverges
3. False
4. True
5. False
Rt
6. lim− a f (x) dx
t→b

Skill Building

7. The integral is improper , since upper limit of integration is ∞.

8. The integral is not improper , since the function f (x) = x3 is continuous on the closed
interval [0, 5].
1
9. The integral is not improper , since the function f (x) = x−1 is continuous on the closed
interval [2, 3].
1
10. The integral is improper , since the function f (x) = x−1 is undefined, at the endpoint
x = 1.
1
11. The integral is improper , since the function f (x) = x is undefined, at the endpoint x = 0.

1
12. The integral is not improper , since the function f (x) = x2 +1 is continuous on the closed
interval [−1, 1].
x
13. The integral is improper , since the function f (x) = x2 −1 is undefined, at the endpoint
x = 1.
14. The integral is improper , since the upper limit of integration is ∞.

15. We evaluate
∞ b
dx dx
Z Z
= lim
1 x3 b→∞ 1 x3
b
1
= lim − 2
b→∞ 2x 1
" !#
1 1
= lim − 2 − − 2
b→∞ 2b 2(1)
 
1 1
= lim − 2
b→∞ 2 2b
1
= .
2
1
The improper integral converges to 2 .
7.7 Improper Integrals 7-229

16. We evaluate
−10 −10
dx dx
Z Z
= lim
−∞ x2 a→−∞ a x2
 −10
1
= lim −
a→−∞
a x
  
1 1
= lim − − −
a→−∞ −10 a
 
1 1
= lim +
a→−∞ a 10
1
= .
10
1
The improper integral converges to 10 .

17. We evaluate
Z ∞ Z b
2x
e dx = lim e2x dx
0 b→∞ 0
 b
1 2x
= lim e
b→∞ 2 0
 
1 2(b) 1 2(0)
= lim e − e
b→∞ 2 2
 
1 2b 1
= lim e −
b→∞ 2 2
= ∞.
The improper integral diverges .
18. We evaluate
Z ∞ Z b
−x
e dx = lim e−x dx
0 b→∞ 0
b
lim −e−x 0

=
b→∞
= lim −e−b − −e−0
 
b→∞
= lim 1 − e−b
 
b→∞
= 1.
The improper integral converges to 1 .
19. We evaluate
−1 −1
4 4
Z Z
dx = lim dx
−∞ x a→−∞ a x
−1
= lim [4 ln |x|]a
a→−∞
= lim [4 ln |−1| − 4 ln |a|]
a→−∞
= lim [−4 ln |a|]
a→−∞
= −∞.
The improper integral diverges .
7-230 Chapter 7 Techniques of Integration

20. We evaluate
∞ b
4 4
Z Z
dx = lim dx
1 x b→∞ 1 x
b
= lim [4 ln |x|]1
b→∞
= lim [4 ln |b| − 4 ln |1|]
b→∞
= lim [4 ln |b|]
b→∞
= ∞.

The improper integral diverges .

21. We evaluate
∞ b
dx dx
Z Z
4 = lim 4
3 (x − 1) b→∞ 3 (x − 1)
" #b
1
= lim − 3
b→∞ 3(x − 1) 3
" !#
1 1
= lim − 3 − − 3
b→∞ 3(b − 1) 3(3 − 1)
" #
1 1
= lim −
b→∞ 24 3(b − 1)3
1
= .
24
1
The improper integral converges to 24 .

22. We evaluate
0 0
dx dx
Z Z
4 = lim 4
−∞ (x − 1) a→−∞ a (x − 1)
" #0
1
= lim −
a→−∞ 3(x − 1)3 a
" !#
1 1
= lim − 3 − − 3
a→−∞ 3(0 − 1) 3(a − 1)
" #
1 1
= lim 3 +
a→−∞ 3(a − 1) 3
1
= .
3
1
The improper integral converges to 3 .
7.7 Improper Integrals 7-231

23. We evaluate
Z ∞ 0 Z b
dx dx dx
Z
2+4
= lim + lim
−∞ x a→−∞ a x2 + 4 b→∞ 0 x2 + 4
 0  b
1 x 1 x
= lim tan−1 + lim tan−1
a→−∞ 2 2 a b→∞ 2 2 0
   
1 0 1 a 1 b 1 0
= lim tan−1 − tan−1 + lim tan−1 − tan−1
a→−∞ 2 2 2 2 b→∞ 2 2 2 2
   
1 a 1 b
= lim − tan−1 + lim tan−1
a→−∞ 2 2 b→∞ 2 2
−1  π  1  π 
= − +
2 2 2 2
π
=
2
π
The improper integral converges to 2 .

24. We evaluate
Z ∞ 0 Z b
dx dx dx
Z
2
= lim + lim
−∞ x +1 a→−∞ a x2 + 1 2
b→∞ 0 x + 1
 −1 0 b
lim tan x a + lim tan−1 x 0

=
a→−∞ b→∞
lim tan−1 0 − tan−1 a + lim tan−1 b − tan−1 0
   
=
a→−∞ b→∞
lim − tan−1 a + lim tan−1 b
   
=
a→−∞ b→∞
 π  π 
= − − +
2 2
= π.
The improper integral converges to π .
25. We evaluate
1 1
dx dx
Z Z
= lim
0 x2 a→0+ a x
2
 1
1
= lim −
a→0+ x a
  
1 1
= lim − − −
a→0+ 1 a
 
1
= lim −1
a→0+ a
= ∞.

The improper integral diverges .


7-232 Chapter 7 Techniques of Integration

26. We evaluate
1 1
dx dx
Z Z
= lim
0 x3 a→0+ a x3
 1
1
= lim − 2
a→0+ 2x a
"  #
1 1
= lim − 2 − − 2a2
a→0+ 2(1)
 
1 1
= lim −
a→0+ 2a2 2
= ∞.

The improper integral diverges .

27. We evaluate
1 1
dx dx
Z Z
= lim
0 x a→0+ a x
= lim [ln |x|]1a
a→0+
= lim [ln |1| − ln |a|]
a→0+
= lim [− ln |a|]
a→0+
= ∞.

The improper integral diverges .

28. We evaluate
6 6
dx dx
Z Z
= lim+
4 x−4 a→4 a x−4
6
= lim [ln |x − 4|]a
a→4+
= lim [ln |6 − 4| − ln |a − 4|]
a→4+
= lim [ln 2 − ln |a − 4|]
a→4+
= ∞.

The improper integral diverges .

29. We evaluate
4 b
dx dx
Z Z
√ = lim− √
0 4−x b→4 0 4−x
 √ b
= lim −2 4 − x 0
b→4−
h √ √ i
= lim −2 4 − b − −2 4 − 0
b→4−
h √ i
= lim 4 − 2 4 − b
b→4−
= 4.

The improper integral converges to 4 .


7.7 Improper Integrals 7-233

30. Let u = 5 − x and substitute to obtain


Z 5 Z 0
x dx (5 − u) (−du)
√ = √
1 5−x 4 u
Z 4 
= 5u−1/2 − u1/2 du
0
Z 4  
= lim 5u−1/2 − u1/2 du
a→0+ a
4


2 3
= lim 10 u − u 2
a→0+ 3 a
√ √
  
2 3 2 3
= lim 10 4 − (4) − 10 a − a
2 2
a→0+ 3 3

 
2 3 44
= lim a 2 − 10 a +
a→0+ 3 3
44
= .
3
44
The improper integral converges to 3 .

31. We split the integral into two improper integrals:


Z 1 Z 0 Z 1
dx dx dx
√ = √ + √ .
−1
3
x −1
3
x 0
3
x
We consider
0 b
dx dx
Z Z
√ = lim− √
−1
3
x b→0 −1
3
x
 b
3 2
= lim x3
b→0− 2 −1
 
3 2 3 2
= lim b 3 − (−1) 3
b→0− 2 2
 
3 2 3
= lim b3 −
b→0− 2 2
3
= − .
2
And
1 1
dx dx
Z Z
√ = √ lim
0
3
x a
3
x a→0+
 1
3 2
= lim+ x 3
a→0 2 a
 
3 2 3 2
= lim (1) − (a)
3 3
a→0+ 2 2
 
3 3 2
= lim+ − a3
a→0 2 2
3
= .
2
R 1 dx 3 3
3 x converges to − 2 +
We conclude that the improper integral −1 √ 2 = 0.
7-234 Chapter 7 Techniques of Integration

32. We split the integral into two improper integrals:


Z 3 Z 2 Z 3
dx dx dx
2 = 2 + 2.
0 (x − 2) 0 (x − 2) 2 (x − 2)

We consider
2 b
dx dx
Z Z
2 = lim
0 (x − 2) (x − 2)2 b→2− 0
b 
1
= lim− −
b→2 x−2 0
  
1 1
= lim − − −
b→2− b−2 0−2
 
1 1
= lim− − −
b→2 b−2 2
= ∞.
R 3 dx
We conclude that the improper integral 0 (x−2) 2 diverges .

33. We evaluate
Z ∞ Z b
cos x dx = lim cos x dx
0 b→∞ 0
b
= lim [sin x]0
b→∞
= lim [sin b − sin 0]
b→∞
= lim [sin b] .
b→∞

Since this limit does not exist, the improper integral diverges .
34. We evaluate
Z ∞ Z b
sin (πx) dx = lim sin (πx) dx
0 b→∞ 0
 b
1
= lim − cos (πx)
b→∞ π 0
  
1 1
= lim − cos (πb) − − cos (π(0))
b→∞ π π
 
1 1
= lim − cos πb .
b→∞ π π
Since this limit does not exist, the improper integral diverges .
35. We evaluate
Z 0 Z 0
ex dx = lim ex dx
−∞ a→−∞ a
0
= lim [ex ]a
a→−∞

lim e0 − ea
 
=
a→−∞
= lim [1 − ea ]
a→−∞
= 1−0
= 1.
The improper integral converges to 1 .
7.7 Improper Integrals 7-235

36. We evaluate
Z 0 Z 0
−x
e dx = lim e−x dx
−∞ a→−∞ a
0
−e−x

= lim a
a→−∞
−0
− −e−a
 
= lim −e
a→−∞

lim −1 + e−a
 
=
a→−∞
= ∞.

The improper integral diverges .

37. Let u = x2 and substitute


Z π/2 Z π2 /4 1
x dx 2 du
2
=
0 sin x 0 sin u
Z π2 /4
1
= csc u du
2 0
Z 2
1 π /4
= lim+ csc u du
a→0 2 a
 π2 /4
1
= lim − ln |csc u + cot u|
a→0+ 2 a
π2 π 2
 
1 1
= lim − ln csc + cot + ln |csc a + cot a|
a→0+ 2 4 4 2
π2 π 2
 
1 1
= lim − ln csc + cot + ln |csc a + cot a|
a→0+ 2 4 4 2
= ∞.

The improper integral diverges .

38. We evaluate
1 1
ln x dx ln x dx
Z Z
= lim
0 x a→0+ a x
 1
1 2
= lim ln x
a→0+ 2 a
 
1 2 1
= lim ln 1 − ln2 a
a→0+ 2 2
 
1
= lim − ln2 a
a→0+ 2
= −∞

The improper integral diverges .


7-236 Chapter 7 Techniques of Integration

39. We evaluate
Z 1 b
dx dx
Z
2
= lim−
0 1−x b→1 0 (1 − x)(1 + x)
Z b 
1 1
= lim − dx
b→1− 0 2(x + 1) 2(x − 1)
 b
1 1
= lim ln (x + 1) − ln |x − 1|
b→1− 2 2 0
  
1 1 1 1
= lim ln (b + 1) − ln |b − 1| − ln (0 + 1) − ln |0 − 1|
b→1− 2 2 2 2
 
1 1
= lim ln (b + 1) − ln |b − 1|
b→1− 2 2
= ∞.
The improper integral diverges .

40. We evaluate
2 2
dx dx
Z Z
√ = lim √
1 x2 − 1 a→1+ a x2 − 1
h p i2
= lim ln x + x2 − 1 (by Formula 55)

a→1+ a
h p p i
= lim ln 2 + 22 − 1 − ln a + a2 − 1

a→1+
h √  p i
= lim ln 3 + 2 − ln a + a2 − 1

a→1+
√ 
= ln 3+2 .
√ 
The improper integral converges to ln 3+2 .
x
41. The function f (x) = √1−x 2
is continuous on [0, 1) but is not defined at x = 1, so
R1 x
0

1−x2
dx is an improper integral.
1 Z b
x
Z
−1/2
√ dx = lim− 1 − x2 x dx
1−x2 b→1
0 0
1 hp ib
= − lim 1 − x2
2 b→1− 0
1 p √ 
= − lim− 1 − b2 − 1 − 0 = 1.
2 b→1
R1
Therefore, √ x dx converges to 1 .
0 1−x2

1
R4
42. The function f (x) = √4−x is continuous on [0, 4) but is not defined at x = 4, so √1 dx
0 4−x
is an improper integral.
Z 4 Z b
1 −1/2
√ dx = lim (4 − x) dx
0 4−x b→4−
0
h ib
= −2 lim− (4 − x)1/2
b→4 0
√ √ 
= −2 lim− 4 − b − 4 − 0 = −2(0 − 2) = 4.
b→4
R4
Therefore, √1 dx converges to 4 .
0 4−x
7.7 Improper Integrals 7-237

43. We evaluate
Z π/4 Z b
tan (2x) dx = lim
π
tan (2x) dx
0 b→ 4 − 0
 b
1
= lim− ln |sec 2x|
b→ π
4
2 0
 
1
= lim ln |sec 2b|
b→ π
4
− 2
= ∞.
The improper integral diverges .
44. We evaluate
Z π/2 Z π/2
csc x dx = lim+ csc x dx
0 a→0 a
π/2
= lim [− ln |csc x + cot x|]a
a→0+
= lim (ln |csc a + cot a|)
a→0+
= ∞.
The improper integral diverges .
R ∞ x dx √
45. To evaluate 0 (x+1) 5/2 , use the substitution u = x + 1. Then x = u2 −1 and dx = 2u du.

The lower bound becomes u = 0 + 1 = 1.
For the upper bound, as x approaches ∞, u also approaches ∞.
The improper integral becomes
Z ∞ Z ∞ 2 Z ∞ 3  Z ∞ 
x dx (u − 1)2u du u −u 1 1
= =2 du = 2 − 4 du.
0 (x + 1)5/2 1 u5 1 u5 1 u2 u
By definition,
Z ∞  Z b   b
1 1 1 1 1 1
2 − du = 2 lim − du = 2 lim − +
1 u2 u4 b→∞ 1 u2 u4 b→∞ u 3u3 1
  
1 1 1 4
= 2 lim − + 3 − −1 + = .
b→∞ b 3b 3 3
R ∞ x dx 4
Therefore, 0 (x+1) 5/2 converges to 3 .

46. We evaluate
∞ b
dx dx
Z Z
√ = lim √
2 x x2 − 1 b→∞ 2
2 x x −1
 −1 b
= lim sec x 2
b→∞
lim sec−1 b − sec−1 2

=
b→∞
π π
= −
2 3
π
= .
6
π
The improper integral converges to 6 .
7-238 Chapter 7 Techniques of Integration

47. We split the integral into two improper integrals:


Z ∞ Z 0 Z ∞
dx dx dx
2
= 2
+ 2
.
−∞ x + 4x + 5 −∞ x + 4x + 5 0 x + 4x + 5
We consider
∞ b
dx dx
Z Z
2
= lim
0 x + 4x + 5 b→∞ 0 x2 + 4x + 5
b
dx
Z
= lim 2
b→∞ 0 (x + 2) + 1
 −1 b
= lim tan (x + 2) 0
b→∞
lim tan−1 (b + 2) − tan−1 (0 + 2)
 
=
b→∞
π
= − tan−1 2.
2
We now determine
Z 0 0
dx dx
Z
= lim
−∞ x2 + 4x + 5 a→−∞ a x2 + 4x + 5
0
dx
Z
= lim 2
a→−∞
a (x + 2) + 1
 −1 0
= lim tan (x + 2) a
a→−∞

lim tan−1 (0 + 2) − tan−1 (a + 2)


 
=
a→−∞
 π
= tan−1 2 − −
2
−1 π
= tan 2 + .
2
R∞ dx
= π2 − tan−1 2 +

We conclude that the improper integral converges, with −∞ x2 +4x+5
tan−1 2 + π2 = π .


48. We split the integral into two improper integrals:


Z ∞ Z 0 Z ∞
dx dx dx
x −x
= x −x
+ .
−∞ e + e −∞ e + e 0 e + e−x
x

We consider
∞ b
dx dx
Z Z
= lim
0 ex + e−x b→∞ 0 ex + e−x
b
ex dx
Z
= lim .
b→∞ 0 e2x +1
x
Let u = e , and substitute to obtain
Z ∞ eb
dx du
Z
x + e−x
= lim 2+1
0 e b→∞ 1 u
 −1 eb
= lim tan u 1
b→∞
= lim tan−1 eb − tan−1 1
 
b→∞
π π
= −
2 4
π
= .
4
7.7 Improper Integrals 7-239

We now determine
0 0
dx dx
Z Z
= lim
−∞ ex + e−x a→−∞ a ex + e−x
0
ex dx
Z
= lim .
a→−∞ a e2x +1
x
Let u = e , and substitute to obtain
Z 0 1
dx du
Z
x −x
= lim
−∞ e + e a→−∞ u2 + 1
ea
1
lim tan−1 u ea

=
a→−∞

lim tan−1 1 − tan−1 ea


 
=
a→−∞
π
= − (0)
4
π
= .
4
R∞ dx π
 π
 π
We conclude that the improper integral converges, with −∞ ex +e−x
= 4 + 4 = 2 .

49. We evaluate
2 2
dx dx
Z Z
√ = √
lim
−∞ 4−x 4−x
a→−∞
a
 √ 2
= lim −2 4 − x a
a→−∞
 √ √ 
= lim −2 4 − 2 − −2 4 − a
a→−∞
h √ √ i
= lim 2 4 − a − 2 2
a→−∞
= ∞.
The improper integral diverges .
50. We evaluate
1 Z 1
x dx x dx
Z
√ = lim √
−∞ 2−x a→−∞ a 2−x
Let u = 2 − x and substitute to obtain
Z 1 Z 1
x dx (2 − u)
√ = lim √ (−du)
−∞ 2−x a→−∞ 2−a u
Z 1  
= − lim 2u−1/2 − u1/2 du
a→−∞ 2−a
1


2 3
= − lim 4 u − u2
a→−∞ 3 2−a
√ √
   
2 3 2 3
= − lim 4 1 − (1) 2 − 4 2 − a − (2 − a) 2
a→−∞ 3 3

 
2 3 10
= − lim (2 − a) 2 − 4 2 − a +
a→−∞ 3 3

   
2 10
= − lim 2 − a (2 − a) − 4 +
a→−∞ 3 3
= −∞.
The improper integral diverges .
7-240 Chapter 7 Techniques of Integration

51. We evaluate
4 4
2x dx 2x dx
Z Z
√ = lim √
2
3
x2 − 4 a→2+ a
3
x2 − 4
2/3 4
 
3 2
= lim+ x −4
a→2 2 a
 
3 2 2/3 3 2 2/3
= lim+ 4 −4 − a −4
a→2 2 2
 
1/3 3 2/3
= lim+ 3(18) − a2 − 4
a→2 2

3
= 3 18 .

The improper integral converges to 3 3 18 .

52. We evaluate
π π
1 1
Z Z
dx = lim dx
0 1 − cos x a→0+ 1 − cos x
Z aπ
1 + cos x
= lim dx
a→0+ sin2 x
Z aπ
csc2 x + csc x cot x dx

= lim
a→0+ a
π
= lim [− cot x − csc x]a
a→0+
 π
cos x + 1
= lim −
a→0+ sin x a
 π
(cos x + 1)(cos x − 1)
= lim −
a→0+ sin x(cos x − 1) a
 2 π
sin x
= lim
a→0+ sin x(cos x − 1) a
 π
sin x
= lim
a→0+ cos x − 1 a
  
sin π sin a
= lim −
a→0+ cos π − 1 cos a − 1
 
sin a
= lim −
a→0+ cos a − 1
 
cos a + 1
= lim
a→0+ sin a
= ∞.

The improper integral diverges .

53. We split the integral into two improper integrals:


Z 1 Z 0 Z 1
dx dx dx
3
= 3
+ 3
.
−1 x −1 x 0 x
7.7 Improper Integrals 7-241

We consider
0 b
dx dx
Z Z
= lim
−1 x3 b→0− −1 x3
 b
1
= lim −
b→0− 2x2 −1
" !#
1 1
= lim − 2− − 2
b→0− 2b 2(−1)
 
1 1
= lim− − 2
b→0 2 2b
−∞. =
R1 dx
We conclude that the improper integral −1 x3 diverges .

54. We split the integral into two improper integrals:


2 1 2
dx dx dx
Z Z Z
= + .
0 x−1 0 x−1 1 x−1

We consider
1 b
dx dx
Z Z
= lim
0 x−1 b→1− 0 x−1
= lim [ln |x − 1|]b0
b→1−
= lim [ln |b − 1| − ln |0 − 1|]
b→1−
= lim [ln |b − 1|]
b→1−
−∞. =
R 2 dx
We conclude that the improper integral 0 x−1 diverges .

55. We split the integral into two improper integrals:


Z 2 Z 1 Z 2
dx dx dx
1/3
= 1/3
+ 1/3
.
0 (x − 1) 0 (x − 1) 1 (x − 1)

We consider
1 b
dx dx
Z Z
1/3
= lim− 1/3
0 (x − 1) b→1 0 (x − 1)
b
3 2
= lim−
(x − 1) 3
b→1 2 0
 
3 2 3 2
= lim− (b − 1) − (0 − 1)
3 3
b→1 2 2
 
3 2 3
= lim (b − 1) 3 −
b→1− 2 2
3
= − .
2
7-242 Chapter 7 Techniques of Integration

And
2 2
dx dx
Z Z
1/3
= lim
1 (x − 1) (x − 1)1/3 a→1+ a
2 
3 2
= lim+ (x − 1) 3
a→1 2 a
 
3 2 3 2
= lim (2 − 1) − (a − 1)
3 3
a→1+ 2 2
 
3 3 2
= lim+ − (a − 1) 3
a→1 2 2
3
= .
2
R2 dx 3 3
We conclude that the improper integral 0 (x−1) 1/3 converges to − 2 + 2 = 0.

56. We split the integral into two improper integrals:


Z 1 Z 0 Z 1
dx dx dx
5/3
= 5/3
+ 5/3
.
−1 x −1 x 0 x

We consider
0 b
dx dx
Z Z
= lim−
−1 x5/3 b→0 −1 x5/3
 b
3
= lim − 2
b→0− 2x 3 −1
" !#
3 3
= lim − 2 − − 2
b→0− 2b 3 2(−1) 3
 
3 3
= lim− − 2
b→0 2 2b 3
−∞. =
R1 dx
We conclude that the improper integral −1 x5/3
diverges .

57. We evaluate
2 b
dx dx
Z Z
3/4
= lim 3/4
1 (2 − x) b→2− 1 (2 − x)
h ib
1/4
= lim− −4(2 − x)
b→2 1
h  i
1/4 1/4
= lim− −4(2 − b) − −4(2 − 1)
b→2
h √
4
i
= lim− 4 − 4 2 − b
b→2
= 4.

The improper integral converges to 4 .


7.7 Improper Integrals 7-243

58. We evaluate
4 4
dx dx
Z Z
√ = lim √
0 8x − x2 a→0+ a 8x − x2
Z 4
dx
= lim+ q
a→0 a 16 − (4 − x)2
  4
4−x
= lim+ − sin−1
a→0 4
   a  
−1 4−4 4−a
= lim − sin − − sin−1
a→0+ 4 4
  
4−a
= lim sin−1
a→0+ 4
π
= .
2
π
The improper integral converges to 2 .

2x
59. The function f (x) = (x2 −1) 3/2 is continuous on (1, 3] but is not defined at x = 1, so
R 3 2x dx
1 (x2 −1)3/2 is an improper integral.

3 3
2x dx
Z Z
−3/2
3/2
= lim+ x2 − 1 (2x dx)
1 (x2 − 1) b→1 b
h −1/2 i3
= −2 lim x2 − 1
b→1+ b
 
1 1
= −2 lim+ √ −√
b→1 8 2
b −1
Since
3  
1 2x dx 1 1
Z
lim √ = ∞, 3/2
= −2 lim+ √ −√ = ∞.
b→1+ b2 − 1 1 (x2 − 1) b→1 8 b2 − 1

Therefore,
3
2x dx
Z
3/2
diverges .
1 (x2 − 1)

60. We evaluate
3 b
x dx x dx
Z Z
= lim
0 (9 − x2 )3/2 b→3− 0 (9 − x2 )3/2
h −1/2 ib
= lim 9 − x2
b→3− 0
h −1/2 i
2 −1/2
− 9 − 02

= lim− 9−b
b→3
 
−1/2 1
= lim− 9 − b2 −
b→3 3
= ∞.

The improper integral diverges .


7-244 Chapter 7 Techniques of Integration

61. We evaluate
Z ∞ Z b
−x2 2
xe dx = lim xe−x dx
0 b→∞ 0
 b
1 2
= lim − e−x
b→∞ 2 0
  
1 −b2 1 2
= lim − e − − e−0
b→∞ 2 2
 
1 1 −b 2
= lim − e
b→∞ 2 2
1
= .
2
1
The improper integral converges to 2 .

62. We evaluate, using formula 125,


Z ∞ Z b
e−x sin x dx = lim e−x sin x dx
0 b→∞ 0
 b
1 −x
= lim e (− sin x − cos x)
b→∞ 2 0
  
1 −b 1 −0
= lim e (− sin b − cos b) − e (− sin 0 − cos 0)
b→∞ 2 2
 
1 1 −b
= lim − e (cos b + sin b)
b→∞ 2 2
1
= .
2
1
The improper integral converges to 2 .

63. (a) We have


1 1 1
√ ≥√ = .
2
x −1 x2 x
R∞ 1 R∞
By the comparison test, since p = 1 and 1 x dx diverges, we conclude that 1 √ 1 dx
x2 −1
diverges .

64. (a) We have


2 2 2
√ >√ = .
2
x −4 x 2 x
R∞ 1 R∞
By the comparison test, since p = 1 and 2 x dx diverges, we conclude that 2 √ 2 dx
x2 −4
diverges .

65. (a) We have


1 + e−x 1
≥ .
x x
R∞ R∞ 1+e−x
By the comparison test, since p = 1 and 1 x1 dx diverges, we conclude that 1 x dx
diverges .
7.7 Improper Integrals 7-245

66. (a) For x ≥ 1, we have


1
e−x ≤
x
so
3e−x 3
≤ 2.
x x
R∞ 3
By the comparison test, since p = 2 and 1 x2 dx converges, we conclude that
R ∞ 3e−x
1 x dx converges .
(b) Using a CAS we obtain

3e−x
Z
dx ≈ 0.6582 .
1 x
67. (a) We have
sin2 x ≤ 1
so
sin2 x 1
≤ 2.
x2 x
R∞
By the comparison test, since p = 2 and 1 x12 dx converges, we conclude that
R ∞ sin2 x
1 x2 dx converges .
(b) Using a CAS we obtain

sin2 x
Z
dx ≈ 0.673 .
1 x2
68. (a) We have
cos2 x ≤ 1
so
cos2 x 1
≤ 2.
x2 x
R∞
By the comparison test, since p = 2 and 1 x12 dx converges, we conclude that
R ∞ cos2 x
1 x2 dx converges .
(b) Using a CAS we obtain

cos2 x
Z
dx ≈ 0.3265 .
1 x2
69. (a) We have
1 1 1
√ ≤ √ = 3/2 .
(x + 1) x x x x
R∞ 1
By the comparison test, since p = 32 and 1 x3/2 dx converges, we conclude that
R∞ 1 √
1 (x+1) x
dx converges .
(b) Using a CAS we obtain

1
Z
π
√ dx = 2 .
1 (x + 1) x
70. (a) We have
1 1 1
√ ≤ √ = 2.
x 1+x 2 x x2 x
R∞ 1
By the comparison test, since p = 2 and 1 x2 dx converges, we conclude that
R∞
√1
1 x 1+x2 dx converges .
(b) Using a CAS we obtain

1
Z
√ dx ≈ 0.8814 .
1 x 1 + x2
7-246 Chapter 7 Techniques of Integration

Applications and Extensions


R∞ 1 1

71. The area is given by the improper integral 0 x+1 − x+2 dx. We evaluate
∞   b  
1 1 1 1
Z Z
− dx = lim − dx
0 x+1 x+2 b→∞ 0 x+1 x+2
b
= lim [ln |x + 1| − ln |x + 2|]0
b→∞
= lim [ln |b + 1| − ln |b + 2| − (ln |0 + 1| − ln |0 + 2|)]
b→∞
= lim [ln 2 + ln |b + 1| − ln |b + 2|]
b→∞
 
b + 1
= lim ln 2 + ln
b→∞ b + 2
= ln 2 .
R∞ 1
72. The area is given by the improper integral 0 1+x 2 dx. We evaluate

Z ∞ Z b
1 1
2
dx = lim 2
dx
0 1+x b→∞ 0 1+x
b
= lim tan−1 x 0

b→∞
= lim tan−1 b − tan−1 0
 
b→∞
= lim tan−1 b
 
b→∞
π
= 2 .
R∞ 2
73. The volume is given by the improper integral 0 π(e−x ) dx. We evaluate
Z ∞ Z b
−x 2
πe−2x dx

π e dx = lim
0 b→∞ 0
b
1
= lim − πe−2x
b→∞ 2 0
  
1 −2b 1
= lim − πe − − πe−2(0)
b→∞ 2 2
 
1 1
= lim π − πe−2b
b→∞ 2 2
π
= 2 .

R ∞  1 2
74. The volume is given by the improper integral 1 π

x
dx. We evaluate
∞  2 b
1 1
Z Z
π √ dx = lim π dx
1 x b→∞ 1 x
b
= lim [π ln |x|]1
b→∞
= lim [π ln |b| − π ln |1|]
b→∞
= lim [π ln b]
b→∞
= ∞.
So the volume of the solid of revolution is not defined .
7.7 Improper Integrals 7-247

R∞ 3 R0 8a3
R ∞ 8a3
75. The area is given by the improper integral −∞ x28a +4a2 dx = −∞ x2 +4a2 dx+ 0 x2 +4a2 dx.
We evaluate first
Z 0 Z 0
8a3 8a3
2 2
dx = lim dx
−∞ x + 4a t→−∞ t x + 4a2
2
  x 0
3 1 −1

= lim 8a tan
t→−∞ 2a 2a
  t  
3 1 −1 0 3 1 −1 t
 
= lim 8a tan − 8a tan
t→−∞ 2a 2a 2a 2a
  
t
= lim −4a2 tan−1
t→−∞ 2a
2
 π 
= −4a −
2
= 2πa2 .
We next evaluate
Z ∞ t
8a3 8a3
Z
dx = lim dx
0 x + 4a2
2 t→∞ 0 x2
+ 4a2
  x  t
3 1 −1

= lim 8a tan
t→∞ 2a 2a
  0  
3 1 −1 t 3 1 −1 0
 
= lim 8a tan − 8a tan
t→∞ 2a 2a 2a 2a
  
2 −1 t
= lim 4a tan
t→∞ 2a
 π 
= 4a2
2
= 2πa2 .

So the area under the graph is 2πa2 + 2πa2 = 4πa2 .

76. (a) One can consider the total reaction as the rate of reaction times time. When the rate
is not constant, the total reaction is given by the integral of the rate of reaction for
t ≥ 0. This integral is equal to the area under the graph of y = r(t) on [0, ∞).
(b) We evaluate the improper integral
Z ∞ Z ∞
2
r(t) dt = te−t dt
0 0
Z b
2
= lim te−t dt
b→∞ 0
 b
1 2
= lim − e−t
b→∞ 2 0
  
1 −b2 1 −02
= lim − e − − e
b→∞ 2 2
 
1 1 −b2
= lim − e
b→∞ 2 2
1
= .
2
1
So the total reaction is 2 .
7-248 Chapter 7 Techniques of Integration

77. (a) We have R(t) = 100, and r = 0.08. So the present value of the asset is given by the
improper integral
Z ∞ Z b
−rt
R(t)e dt = lim 100e−(0.08)t dt
0 b→∞ 0
 b
100 −0.08t
= lim e
b→∞ −0.08 0
  
100 −0.08b 100 −0.08(0)
= lim e − e
b→∞ −0.08 −0.08
−0.08b
 
= lim 1250 − 1250e
b→∞
= 1250.
The present value of the asset is $1250.00.
(b) Now with R(t) = 1000 + 80t, and r = 0.07, the present value of the asset is given by
the improper integral
Z ∞ Z b
R(t)e−rt dt = lim (1000 + 80t)e−(0.07)t dt
0 b→∞ 0
" #
Z b Z b
= lim 1000 e−(0.07)t dt + 80 te−at dt
b→∞ 0 0
" !#b
1 1
= lim 1000 e−0.07t + 80 − e−(0.07)t ((0.07)t + 1)
b→∞ −0.07 (0.07)2
" ! 0
1 1
= lim 1000 e−0.07b + 80 − 2e
−(0.07)b
((0.07)b + 1)
b→∞ −0.07 (0.07)
!!#
1 −0.07(0) 1 −(0.07)(0)
− 1000 e + 80 − 2e ((0.07)(0) + 1)
−0.07 (0.07)
" !
1000 −0.07b 1 −(0.07)b
= lim e + 80 − e ((0.07)b + 1)
b→∞ −0.07 (0.07)2
!#
1000 80
− −
−0.07 (0.07)2
!
1000 80
= − −
−0.07 (0.07)2
≈ 30, 612.

The present value of the asset is approximately $30, 612 .


7.7 Improper Integrals 7-249

78. We evaluate the improper integral


Z ∞ Z b  2
Ri2 dt = lim R Ie−Rt/L dt
0 b→∞ 0
Z b
= lim RI 2 e−2Rt/L dt
b→∞ 0
b
RI 2 −2Rt/L

= lim e
b→∞ −2R/L 0
  
1 2 −2Rb/L 1
= lim − LI e − − LI 2 e−2R(0)/L
b→∞ 2 2
 
1 2 1 2 −2Rb/L
= lim LI − LI e
b→∞ 2 2
1 2
= 2 LI .

79. We evaluate the improper integral


2πN Ir ∞ dy 2πN Ir b
dy
Z Z
3/2
= lim 3/2
10 x (r2 + y 2 ) 10 b→∞ x (r2 + y 2 )
" #b
2πN Ir 1 y
= lim (by Formula 61)
10 b→∞ r2 r2 + y 2
p
x
 
2πN Ir 1 b 1 x
= lim √ − 2√
10 b→∞ r2 r2 + b2 r r2 + x2
 
2πN Ir 1 1 1 x
= lim  q − 2√ 
10 b→∞ r2 2 r r2 + x2
(r/b) + 1
 
2πN Ir 1 x
= 1 − √
10 r2 r2 + x2
 
πN I
= 5r 1 − √r2x+x2 .

80. We evaluate the improper integral by splitting into two integrals.


"Z #
rIm ∞ 0 Z ∞
dy rIm dy dy
Z
= +
10 −∞ (r2 + y 2 )3/2 10 −∞ (r2 + y 2 )
3/2
0 (r2 + y 2 )
3/2

We first evaluate
Z 0 0
dy dy
Z
3/2
= lim 3/2
−∞ (r2 + y2) a→−∞ a (r2 + y2)
" #0
1 y
= lim 2
p (by Formula 61)
a→−∞ r r + y2
2
a
 
1 0 1 a
= lim √ − 2√
a→−∞ r2 2
r +0 2 r r + a2
2
 
1
= lim  q 
a→−∞ 2
r2 1 + (r/a)
1
= .
r2
7-250 Chapter 7 Techniques of Integration

And we evaluate
Z ∞ b
dy dy
Z
3/2
= lim 3/2
0 (r2 + y2) b→∞ 0 (r2 + y2)
" #b
1 y
= lim (by Formula 61)
r2 r2 + y 2
p
b→∞
0
 
1 b 1 0
= lim 2
√ − 2√
b→∞ r 2
r +b 2 r r + 02
2
 
1
= lim  q 
b→∞ 2 2
r 1 + (r/b)
1
= .
r2
So
∞  
rIm dy rIm 1 1
Z
= +
10 −∞ (r2 + y 2 )
3/2 10 r2 r2
mI
= 5r .
R∞
81. We have W = 1 F (r) dr, where F is the force acting on an object. So we evaluate the
improper integral
Z ∞ Z b
GmM
F (r) dr = lim dr
1 b→∞ 1 r2
 b
GmM
= lim −
b→∞ r 1
  
GmM GmM
= lim − − −
b→∞ b 1
 
GmM
= lim GmM −
b→∞ b
= GmM .

R1
82. If α ≥ 0, then 0
xα dx converges as an ordinary definite integral. If −1 < α < 0, then
Z 1 Z 1
α
x dx = lim+ xα dx
0 a→0 a
1
xα+1

= lim
a→0+ α+1 a
1α+1 aα+1
 
= lim −
a→0+ α+1 α+1
1
, =
α+1
and the improper integral converges. If α = −1, then
Z 1 Z 1
xα dx = lim+ x−1 dx
0 a→0 a
1
= lim [ln |x|]a
a→0+
= lim [ln |1| − ln |a|]
a→0+
= ∞,
7.7 Improper Integrals 7-251

and the improper integral diverges. And if α < −1,


Z 1 Z 1
α
x dx = lim+ xα dx
0 a→0 a
1
xα+1

= lim
a→0+ α+1 a
1α+1 aα+1
 
= lim −
a→0+ α+1 α+1
= ∞,
R1
and the improper integral diverges. So 0
xα dx converges for −1 < α .
83. We evaluate the improper integral
Z ∞ Z b
xe−x dx = lim xe−x dx.
0 b→∞ 0

Let u = x, du = dx, and dv = e−x dx, v = −e−x , and use integration by parts.
Z ∞ " Z b #
−x −x b −x
  
xe dx = lim x −e 0
− −e dx
0 b→∞ 0
" #
Z b
= lim −be−b + e−x dx
b→∞ 0
h b i
lim −be−b + −e−x 0

=
b→∞
= lim −be−b + −e−b − −e−0
  
b→∞
= lim −be−b + 1 − e−b
 
b→∞

= 1.

84. We evaluate the improper integral


Z 1 Z 1
x ln x dx = lim+ x ln x dx.
0 a→0 a

Let u = ln x, du = x1 dx, and dv = x dx, v = 12 x2 , and use integration by parts.


Z 1 "  1 Z 1    #
1 2 1 2 1
x ln x dx = lim (ln x) x − x dx
0 a→0 + 2 a a 2 x
1 1
 
1
Z
= lim − a2 ln a − x dx
a→0+ 2 2 a
"  1 #
1 2 1 1 2
= lim − a ln a − x
a→0+ 2 2 2 a
  
1 2 1 1 1 2
= lim+ − a ln a − − a
a→0 2 2 2 2
 
1 2 1 2 1
= lim a − a ln a −
a→0+ 4 2 4
= − 14 .
7-252 Chapter 7 Techniques of Integration

85. We evaluate the improper integral


Z ∞ Z b
e−x cos x dx = lim e−x cos x dx.
0 b→∞ 0

We use integration by parts to obtain formula 126, and so


∞  b
1 −x
Z
e−x cos x dx = e (− cos x + sin x)
lim
0 b→∞ 2 0
  
1 −b 1 −0
= lim e (− cos b + sin b) − e (− cos 0 + sin 0)
b→∞ 2 2
 
1 1 −b
= lim − e (cos b − sin b)
b→∞ 2 2
1
= 2 .

86. We evaluate the improper integral


Z ∞ Z b
tan−1 x dx = lim tan−1 x dx.
0 b→∞ 0

1
Let u = tan−1 x, du = 1+x2 dx, and dv = dx, v = x, and use integration by parts.
"  #
∞ b 
1
Z Z
−1
 −1
b
tan x dx = lim x tan x 0− dx x
0 b→∞ 0 1 + x2
"  #
 b

−1 1 2
= lim b tan b − ln x + 1
b→∞ 2 0
 
1
lim b tan−1 b − ln b2 + 1

=
b→∞ 2
= ∞,

and the improper integral diverges .


R∞ dx dx
Rb
87. By definition, 0
= lim 3/2 .
(x2 +4)3/2 b→∞ 0 (x +4)
2

dx
use the substitution x = 2 tan θ, − π2 < θ < π
R
To evaluate (x2 +4)3/2
, 2.
√ √ √
Then dx = 2 sec2 θ dθ and x2 + 4 = 4 tan2 θ + 4 = 2 sec2 θ = 2 sec θ since − π2 < θ < π
2.

π−
When x = 0, 2 tan θ = 0 and so θ = 0. As x → ∞, tan θ → ∞ and so θ → 2 .

So,
∞ Z b Z b Z b
dx dx 1 1
Z
2
= lim = lim 2 sec θ dθ = lim cos θ dθ
0 (x2 + 4)3/2 b→∞ 0 (x2 + 4)3/2 b→π/2 0 8 sec3 θ 4 b→π/2 0
1 1 1 1
= lim [sin θ]b0 = lim (sin b − 0) = (1 − 0) = .
4 b→π/2 4 b→π/2 4 4
R∞ dx 1
Therefore, 0 (x2 +4)3/2
converges to 4 .
7.7 Improper Integrals 7-253

R∞ 1
Rb 1
88. By definition, 4 dx = lim dx.
(x2 −9)3/2 b→∞ 4 (x −9)
2 3/2

1 π
R
Use the substitution x = 3 sec θ to evaluate (x2 −9)3/2
dx. Since x ≥ 0, 0 ≤ θ < 2.

Then dx = 3 tan θ sec θ dθ and


3/2 3/2
(x2 − 9)3/2 = (9 sec2 θ − 9)3/2 = 9(sec2 θ − 1) = 9 tan2 θ = 27 tan3 θ.


Then
Z 

1 1 1 1
Z Z
dx = 3 (3 tan θ sec θ dθ) = · sec θ dθ
(x − 9)3/2
2 27 tan θ 9 tan2 θ
1 cos2 θ 1 1 cos θ 1 1 1
Z Z Z
= 2 · dθ = · dθ = cot θ · csc θ dθ = − csc θ + C.
9 sin θ cos θ 9 sin θ sin θ 9 9
√ q q
x2 −9
Since x = 3 sec θ, we have cos θ = x3 and sin θ = 1 − cos2 θ = 1 − x92 = x2 =

x2 −9
± x .

π x2 −9 √ x
Since 0 ≤ θ < 2, sin θ > 0. So, sin θ = x and csc θ = x2 −9
.

So,
1 1 x
Z
dx = − √ + C,
(x2 − 9)3/2 9 x2 − 9
and
Z ∞ b  b
1 1 1 1 x
Z
dx = − lim dx = − lim √
4 (x2 − 9)3/2 9 b→∞ 2
4 (x − 9)
3/2 9 b→∞ x2 − 9 4
    √
1 b 4 1 4 1 4 4 7 1
= − lim √ −√ =− 1− √ =− + √ = − .
9 b→∞ b2 − 9 7 9 7 9 9 7 63 9
R∞ √
1 4 7 1
Therefore, 4 (x2 −9)3/2
dx converges to 63 − 9 .
R∞ dx
Rb dx
89. By definition, 1
√ = lim √ .
(x+1) 2x+x2 b→∞ 1 (x+1) 2x+x
2

dx
begin by completing the square on the expression 2x + x2 .
R
To evaluate √
(x+1) 2x+x2
,

2
2x + x2 = x2 + 2x + 1 − 1 = (x + 1) − 1.

The integral becomes (x+1)dx √dx 2 .


R R

2x+x 2
=
(x+1) (x+1) −1

dx √dx
R R
Use the substitution u = x + 1. Then du = dx and √
(x+1) 2x+x2
= =
(x+1) (x+1)2 −1
√du
R
u u2 −1
= arcsec u + C = arcsec(x + 1) + C.

So,
∞ b
dx dx
Z Z
b
√ = lim √ = lim [arcsec(x + 1)]1
1 (x + 1) 2x + x2 b→∞ 1 (x + 1) 2x + x
2 b→∞
π π π
= lim [arcsec(b + 1) − arcsec(2)] = − = .
b→∞ 2 3 6
R∞ dx π
Therefore, 1

(x+1) 2x+x2
converges to 6 .
7-254 Chapter 7 Techniques of Integration

R∞ dx
Rb dx
90. By definition, 2
√ = lim √ .
(x+2)2 4x+x2 b→∞ 2 (x+2) 4x+x
2 2

dx
begin by completing the square on the expression 4x + x2 .
R
To evaluate √
(x+2)2 4x+x2
,

4x + x2 = x2 + 4x + 4 − 4 = (x + 2)2 − 4.

dx √dx
R R
The integral becomes √
(x+2)2 4x+x2
= .
(x+2)2 (x+2)2 −4

dx √dx
R R
Use the substitution u = x + 2. Then du = dx and √
(x+2)2 4x+x2
= =
(x+2)2 (x+2)2 −4
√du
R
u2 u2 −4
.

√du π
R
Use the substitution u = 2 sec θ to evaluate u2 u2 −4
. Since x ≥ 0, 0 ≤ θ < 2.

Then du = 2 tan θ sec θ dθ and


p p p p π
u2 − 4 = 4 sec2 θ − 4 = 2 sec2 θ − 1 = 2 tan2 θ = 2 tan θ since 0 ≤ θ < .
2
Then
Z  
du 2 tan θ sec θ dθ 1 1 1 1
Z Z Z
√ = 2
= dθ = cos θ dθ = sin θ + C.
u u2 − 4
2 (4 sec θ)(2 tan θ) 4 sec θ 4 4

2
√ q
2
q
u2 −4
Since u = 2 sec θ, we have cos θ = u and sin θ = 1 − cos2 θ = 1 − u2 = u2 =

u2 −4
± u .

π u2 −4
Since 0 ≤ θ < 2, sin θ > 0. So, sin θ = u .
So,
√ p √
dx du 1 u2 − 4 (x + 2)2 − 4 4x + x2
Z Z
√ = √ = +C= +C= +C
(x + 2)2 4x + x2 u2 u2 − 4 4 u 4(x + 2) 4(x + 2)

and
"√ #b "√ √ #
∞ b
dx dx 4x + x2 4b + b2 12
Z Z
√ = lim √ = lim = lim − .
2 (x + 2) 4x + x2
2 b→∞ 2 (x + 2) 4x + x2
2 b→∞ 4(x + 2) b→∞ 4(b + 2) 16
2

Note that
√ √ q
4

√
4b + b2 1 4b + b2
1
1 b + 1 1 0+1

1
b
lim = lim · 1 = lim  2
= = .
b→∞ 4(b + 2) 4 b→∞ b + 2 4 b→∞ 1+ b 4 1+0 4
b

So, "√

√ # √ √
dx 4b + b2 12 1 2 3 1 3
Z
√ = lim − = − = − .
2 (x + 2) 4x + x2
2 b→∞ 4(b + 2) 16 4 16 4 8
R∞ √
dx 1 3
Therefore, 2 (x+2)2

4x+x2
converges to 4 − 8 .
7.7 Improper Integrals 7-255

91. We evaluate the improper integral


Z ∞ Z b
sin x dx = lim sin x dx
0 b→∞ 0
b
= lim [− cos x]0
b→∞
= lim [− cos b − (− cos 0)]
b→∞
= lim [1 − cos b] .
b→∞
R∞
Since this limit does not exist, the improper integral 0 sin x dx diverges . We now
evaluate the improper integral
Z 0 Z 0
sin x dx = lim sin x dx
−∞ a→−∞ a
0
= lim [− cos x]a
a→−∞
= lim [− cos 0 − (− cos a)]
a→−∞
= lim [cos a − 1] .
a→−∞
R0
Since this limit does not exist, the improper integral −∞ sin x dx diverges . We now
determine
Z t
t
lim sin x dx = lim [cos x]−t
t→∞ −t t→∞

= lim [cos t − (cos (−t))]


t→∞
= lim [cos t − cos (t)]
t→∞
= lim [0]
t→∞

= 0.

1/π
92. If we let f (x) = x + 1+x2 , then
∞    b 
1/π 1/π
Z Z
x+ dx = lim x+ dx
0 1 + x2 b→∞ 0 1 + x2
 b
1 2 1 −1
= lim x + tan x
b→∞ 2 π
 0
1 2 1
= lim b + tan−1 b
b→∞ 2 π
= ∞,
so this integral diverges. And
Z 0  0 
1/π
Z
x dx = lim x+ dx
∞ a→−∞ a 1 + x2
 0
1 2 1 −1
= lim x + tan x
a→−∞ 2 π a
 
1 2 1 −1
= lim − a − tan a
a→−∞ 2 π
= −∞,
7-256 Chapter 7 Techniques of Integration

so this integral also diverges. But


t   t
1/π 1 2 1
Z
lim x+ dx = lim x + tan−1 x
t→∞ −t 1 + x2 t→∞ 2 π −t
  
1 2 1 −1 1 2 1 −1
= lim t + tan t − (−t) + tan (−t)
t→∞ 2 π 2 π
 
2
= lim tan−1 t
t→∞ π
= 1.

93. Since
1 1 1
√ ≥√ = √
2 + sin x 2+1 3
R∞ 1
R∞ 1
for all x, and since 0 3 dx diverges, by the Comparison Test, 0

2+sin x
dx diverges.

94. Since for x ≥ 2


ln x ln 2 ln 2
√ ≥ √ =
x2 − 1 x2 x
R∞ ln 2 ∞
√ln x dx diverges.
R
and 2 x dx diverges, by the Comparison Test, 2 x2 −1

95. (a) We evaluate the improper integral, and let u = xn , du = nxn−1 dx, dv = e−x dx, and
v = −e−x . We obtain
Z ∞ Z b
n −x
x e dx = lim xn e−x dx
0 b→∞
"0 Z b #
−x b
 n  n−1 −x

= lim x −e 0
− nx −e dx
b→∞ 0
" #
Z b
= lim −bn e−b + n xn−1 e−x dx
b→∞ 0
" Z #
b
lim −bn e−b + lim n−1 −x
 
= n x e dx
b→∞ b→∞ 0
"Z #
b
= 0 + n lim xn−1 e−x dx
b→∞ 0
Z ∞
= n xn−1 e−x dx.
0
7.7 Improper Integrals 7-257

(b) Using part (a) repeatedly, we obtain


Z ∞ Z ∞
n −x
x e dx = n xn−1 e−x dx
0 0
Z ∞
= n(n − 1) xn−2 e−x dx
0
= ··· Z ∞
= [n(n − 1) · · · 1] e−x dx
0
Z b
= (n!) lim e−x dx
b→∞ 0
b
= (n!) lim −e−x 0

b→∞

= (n!) lim −e−b − −e−0


 
b→∞
= (n!) lim 1 − e−b
 
b→∞
= n!.

96. We evaluate the improper integral, and obtain for p 6= 1


∞ b
dx dx
Z Z
p = lim p
e x(ln x) b→∞
e x(ln x)
 1−p b
ln x
= lim
b→∞ 1−p e
 1−p
b ln1−p e

ln
= lim −
b→∞ 1−p 1−p
 1−p 
ln b 1
= lim − .
b→∞ 1−p 1−p

If p > 1, then 1 − p < 0, and


∞  1−p 
dx ln b 1 1
Z
p = lim − = .
e x(ln x) b→∞ 1−p 1−p p−1

If p < 1, then 1 − p > 0, and


∞  1−p 
dx ln b 1
Z
= lim − = ∞.
e x(ln x)p b→∞ 1−p 1−p

And if p = 1, we have
∞ b
dx dx
Z Z
= lim
e x(ln x)p b→∞ e x ln x
= lim [ln (ln x)]be
b→∞
= lim [ln (ln b) − ln (ln e)]
b→∞
= lim [ln (ln b) − 0]
b→∞
= ∞.
R∞ dx
So e x(ln x)p converges if p > 1, and diverges if p ≤ 1.
7-258 Chapter 7 Techniques of Integration

97. We evaluate the improper integral, and obtain for p 6= 1


b b
dx dx
Z Z
p = lim p
a (x − a) t→a+ t (x − a)
" #b
1−p
(x − a)
= lim
t→a+ 1−p
t
" #
1−p 1−p
(b − a) (t − a)
= lim − .
t→a+ 1−p 1−p

If p > 1, then 1 − p < 0, and


" #
b 1−p 1−p
dx (b − a) (t − a)
Z
p = lim+ − = ∞.
a (x − a) t→a 1−p 1−p

If 0 < p < 1, then 1 − p > 0, and


" #
b 1−p 1−p 1−p
dx (b − a) (t − a) (b − a)
Z
p = lim − = .
a (x − a) t→a+ 1−p 1−p 1−p

And if p = 1, we have
b b
dx dx
Z Z
p = lim
a (x − a) t→a+ t x−a
b
= lim [ln |x − a|]t
t→a+
= lim [ln |b − a| − ln |t − a|]
t→a+
= ∞.
Rb dx
So a (x−a)p
converges if 0 < p < 1, and diverges if p ≥ 1.

98. We evaluate the improper integral, and obtain for p 6= 1


b t
dx dx
Z Z
p = lim p
a (b − x) t→b− a (b − x)
" #t
1−p
−(b − x)
= lim
t→b− 1−p
a
" !#
1−p 1−p
−(b − t) (b − a)
= lim − − .
t→b− 1−p 1−p

If p > 1, then 1 − p < 0, and


" !#
b 1−p 1−p
dx −(b − t) (b − a)
Z
p = lim− − − = ∞.
a (b − x) t→b 1−p 1−p

If 0 < p < 1, then 1 − p > 0, and


" !#
Z b 1−p 1−p 1−p
dx −(b − t) (b − a) (b − a)
p = lim− − − = .
a (b − x) t→b 1−p 1−p 1−p
7.7 Improper Integrals 7-259

And if p = 1, we have
b t
dx dx
Z Z
p = lim
a (b − x) t→b− a b−x
= lim [− ln |b − x|]ta
t→b−
= lim [− ln |b − t| − (− ln |b − a|)]
t→b−
= ∞.
Rb dx
So a (b−x)p converges if 0 < p < 1, and diverges if p ≥ 1.

1
R b dx 1
99. If p = 0, then (x−a)p= 1 for all x in (a, b], and so a (x−a) p = b − a. Likewise,
(b−a)p = 1
R b dx 1 1
for all x in [a, b), so a (b−x)p = 1. And if p < 0, then (x−a) p and
(b−a)p are continuous
R b dx R b dx
on [a, b], so the integrals a (x−a) p and
a (b−x)p are ordinary definite integrals, and hence
R 2 dx R2 h i2
converge. For example, 1 (x−1)−1 = 1 (x − 1) dx = 12 (x − 1)2 = 21 .
1
R∞ R∞
100. Suppose a g(x) dx diverges, then since g(x) ≥ 0, we know that a g(x) dx = ∞. If M is
any real number, then there exists b such that for every b ≤ t,
Z b Z b Z t
M≤ g(x) dx ≤ f (x) dx ≤ f (x) dx.
a a a
Rt R∞
So lim f (x) dx = ∞, by definition of an infinite limit. So a f (x) dx diverges.
t→∞ a R∞ R∞
Now suppose a f (x) dx converges. Then a g(x) dx must also converge, since from the
R∞
above, we otherwise would conclude that a f (x) dx diverges.
101. We evaluate, for s > 0,
Z ∞
L {f (x)} = e−sx f (x) dx
0
Z ∞
= e−sx x dx
0
Z b
= lim e−sx x dx
b→∞ 0
 b
1 −sx
= lim − 2 e (sx + 1)
b→∞ s 0
  
1 −bs 1 −(0)s
= lim − 2 e (bs + 1) − − 2 e ((0)s + 1)
b→∞ s s
 
1 1
= lim − 2 e−bs (bs + 1)
b→∞ s2 s
1
= s2 .
7-260 Chapter 7 Techniques of Integration

102. We evaluate, for s > 0,


Z ∞
L {f (x)} = e−sx f (x) dx
0
Z ∞
= e−sx cos x dx
0
Z b
= lim e−sx cos x dx
b→∞ 0
b
e−xs (sin x − s cos x)

= lim
b→∞ s2 + 1 0
e−bs (sin b − s cos b) e−(0)s (sin (0) − s cos (0))
 
= lim −
b→∞ s2 + 1 s2 + 1
 
s sin b − s cos b
= lim + e−bs
b→∞ s2 + 1 s2 + 1
s
= s2 +1 .

103. We evaluate, for s > 0,


Z ∞
L {f (x)} = e−sx f (x) dx
0
Z ∞
= e−sx sin x dx
0
Z b
= lim e−sx sin x dx
b→∞ 0
b
−e−sx (cos x + s sin x)

= lim
b→∞ s2 + 1 0
−e−bs (cos b + s sin b) −e−(0)s (cos (0) + s sin (0))
 
= lim −
b→∞ s2 + 1 s2 + 1
 
1 cos b + s sin b
= lim 2
− e−bs
b→∞ s + 1 s2 + 1
1
= s2 +1 .

104. We evaluate, for s > 1,


Z ∞
L {f (x)} = e−sx f (x) dx
0
Z ∞
= e−sx ex dx
0
Z b
= lim ex(1−s) dx
b→∞ 0
b
ex(1−s)

= lim
b→∞ 1−s 0
b(1−s)
e(0)(1−s)
 
e
= lim −
b→∞ 1−s 1−s
−b(s−1)
 
1 e
= lim −
b→∞ s − 1 s−1
1
= s−1 .
R∞
But for s ≤ 1, 0 e−sx ex dx diverges.
7.7 Improper Integrals 7-261

105. We evaluate, for s > a,


Z ∞
L {f (x)} = e−sx f (x) dx
0
Z ∞
= e−sx eax dx
0
Z b
= lim ex(a−s) dx
b→∞ 0
b
ex(a−s)

= lim
b→∞ a−s 0
eb(a−s) e0(a−s)
 
= lim −
b→∞ a−s a−s
e−b(s−a)
 
1
= lim −
b→∞ a−s a−s
1
= s−a .
R∞
But for s ≤ a, 0
e−sx ex dx diverges.
106. We evaluate, for s > 0,
Z ∞
L {f (x)} = e−sx f (x) dx
0
Z ∞
= e−sx (1) dx
0
Z b
= lim e−sx dx
b→∞ 0
 b
1
= lim − e−sx
b→∞ s 0
  
1 −bs 1
= lim − e − − e−(0)s
b→∞ s s
 
1 1 −bs
= lim − e
b→∞ s s
1
= s .

Challenge Problems
R1q 2
107. The arc length is given by 0 1 + (dy/dx) dx. We differentiate, and obtain

dy d p √ 
= x − x2 − sin−1 x
dx dx
1 −1/2 1 1
= x − x2 (1 − 2x) − q √
2 √ 22 x
1 − ( x)
1 − 2x 1
= √ − √
2 x−x 2 2 x − x2
−x
= √ .
x − x2
7-262 Chapter 7 Techniques of Integration

So
s  2
−x
q
2
1 + (dy/dx) = 1+ √
x − x2
r
x2
= 1+
x − x2
1
= √ .
1−x
We now evaluate the improper integral.
Z 1 Z b
1 −1/2
√ dx = lim− (1 − x) dx
0 1−x b→1 0
 √ b
= lim− −2 1 − x 0
b→1
h √ √ i
= lim− −2 1 − b − −2 1 − 0
b→1
h √ i
= lim 2 − 2 1 − b
b→1−

= 2.

108. We evaluate the improper integral


Z a Z a
(x−ex ) x
e dx = lim ex e−e dx.
−∞ x→−∞ x
x x
Let u = e , then du = e dx, and we obtain
Z a Z ea
x
e(x−e ) dx = lim e−u du
−∞ x→−∞ ex
 −u ea
= lim −e ex
x→−∞
h a
 x
i
= lim −e−e − −e−e
x→−∞
h x a
i
= lim e−e − e−e
x→−∞
a
= 1 − e−e .

109. We split the integral into two improper integrals:


Z ∞ Z 0 Z x
(x−ex ) x −ex x
e dx = lim e e dx + lim ex e−e dx.
−∞ x→−∞ x x→∞ 0
x x
For the first integral, let u = e , then du = e dx, and we obtain
Z 0 Z 1
x
lim ex e−e dx = lim e−u du
x→−∞ x x→−∞ ex
1
−e−u ex

= lim
x→−∞
h  x
i
= lim −e−1 − −e−e
x→−∞
h x i
= lim e−e − e−1
x→−∞
1
= 1− .
e
7.7 Improper Integrals 7-263

For the second integral, let u = ex , then du = ex dx, and we obtain


Z x Z ex
x
lim ex e−e dx = lim e−u du
x→∞ 0 x→∞ 1
ex
lim −e−u 1

=
x→∞
h x i
= lim −e−e − −e−1
x→∞
h x
i
= lim e−1 − e−e
x→∞
1
= .
e
So we have ∞  
1 1
Z
(x−ex )
e dx = 1 − + = 1.
−∞ e e
R ln 2 R∞
110. (a) We need to show that 0
e−x dx = ln 2
e−x dx. For the first integral we have
Z ln 2  −x ln 2
e−x dx = −e 0
0
−e− ln 2 − −e−0

=
1
= − +1
2
1
= .
2
The improper integral is determined next,
Z ∞ Z b
e−x dx = lim e−x dx
ln 2 b→∞ ln 2
b
lim −e−x ln 2

=
b→∞

lim −e−b − −e− ln 2


 
=
b→∞
 
1
= lim − e−b
b→∞ 2
1
= −0
2
1
= .
2
So the areas are equal.
(b) The volume obtained by rotating the first region about the x-axis is
Z ln 2 Z ln 2
−x 2
πe−2x dx

π e dx =
0 0
h π iln 2
= − e−2x
2 0
π −2(ln 2)  π −2(0) 
= − e − − e
2 2

= .
8
7-264 Chapter 7 Techniques of Integration

The volume obtained by rotating the second region about the x-axis is given by the
improper integral
Z ∞ Z b
−x 2
πe−2x dx

π e dx = lim
ln 2 b→∞ ln 2
h π ib
= lim − e−2x
b→∞ 2 ln 2
h π  π i
−2b
= lim − e − − e−2 ln 2
b→∞ 2 2
 
1 1 −2b
= lim π − πe
b→∞ 8 2
π
= .
8
So the volume of the first solid is three times the volume of the second solid.
111. Since a < b, f (x) ≥ 0 for all x. The integral of f is 0 outside of the interval [a, b], and so
we have Z ∞ Z b
1 1
f (x) dx = dx = (b − a) = 1.
−∞ a b−a b−a
We conclude that f is a probability density function.
112. We have f (x) ≥ 0 for all x. The integral of f is 0 for x < 0, and so we have
Z ∞ Z ∞
1 −x/a
f (x) dx = e dx
−∞ 0 a
Z b
1 −x/a
= lim e dx
b→∞ 0 a
h 1
ib
= lim −e− a x
b→∞ 0
h 1
 1
i
−a b
= lim −e − −e− a (0)
b→∞
h 1
i
= lim 1 − e− a b
b→∞
= 1.
We conclude that f is a probability density function.
113. We need to evaluate
∞ b
1
Z Z
xf (x) dx = x dx
−∞ a b−a
Z b
1
= x dx
b−a a
 
1 1 2 1 2
= b − a
b−a 2 2
(b − a)(b + a)
=
2(b − a)
a+b
= .
2
a+b
The mean of f is µ = 2 .
7.7 Improper Integrals 7-265

114. We need to evaluate


Z ∞ ∞
1 −x/a
Z
xf (x) dx = xe dx
−∞ 0 a
Z b
1 −x/a
= lim xe dx
b→∞ 0 a
h 1
ib
= lim −e− a x (a + x)
b→∞ 0
h 1
 1
i
−a b
= lim −e (a + b) − −e− a (0) (a + 0)
b→∞
h 1
i
= lim a − e− a b (a + b)
b→∞
= a.
The mean of f is µ = a .
115. We need to evaluate
Z ∞ b
 2
a+b 1
Z
2
(x − µ) f (x) dx = x− dx
−∞ a 2 b − a
Z b 2
1 a+b
= x− dx
b−a a 2
"  3 #b
1 1 a+b
= x−
b−a 3 2
a
"  3  3 #
1 1 a+b 1 a+b
= b− − a−
b−a 3 2 3 2
"  3  3 #
1 1 1 1 1 1 1
= b− a − a− b
b−a 3 2 2 3 2 2
1 h
3
i
= 2(b − a)
24(b − a)
2
(b − a)
= .
12
(b−a)2 b−a
The variance of f is σ 2 = 12 . The standard deviation is σ = √ .
2 3

116. We need to evaluate


Z ∞ ∞
1
Z
2 2
(x − µ) f (x) dx = (x − a) e−x/a dx
−∞ 0 a
Z b
1
= lim (x − a)2 e−x/a dx
b→∞ 0 a
h 1 ib
= lim −e− a x a2 + x2
b→∞ 0
h 1   −10 2 i
−a b 2
= lim −e a + b − −e a a + 02
2
b→∞
h 1 i
= lim a2 − e− a b a2 + b2
b→∞
= a2 .

The variance of f is σ 2 = a2 . The standard deviation is σ = a.


7-266 Chapter 7 Techniques of Integration

AP
R
Practice Problems
x+2 x+2
1. The function f (x) = x2 +4x−12 = (x+6)(x−2) is continuous on [0, 2) but is not defined at
R 2 x+2
x = 2, so 0 x2 +4x−12 dx is an improper integral.
2 b
x+2 x+2
Z Z
2
dx = lim dx.
0 x + 4x − 12 b→2− 0 x2 + 4x − 12
Now use the
R substitution uR= x2 +4x−12. So, du = (2x + 4) dx = 2(x + 2) dx, (x + 2) dx =
du x+2 1 du 1 1 2
2 , and x2 +4x−12 dx = u 2 = 2 ln |u| + C = 2 ln x + 4x − 12 + C.

Z 2 Z b
x+2 x+2
Therefore, 2
dx = lim 2
dx
0 x + 4x − 12 b→2 0 x + 4x − 12

b
 
1 2
= lim ln x + 4x − 12
b→2− 2 0
1  2 
= lim ln b + 4b − 12 − ln 12 .
2 b→2−
R2 x+2
Since lim− b2 + 4b − 12 = 0+ , lim− ln b2 + 4b − 12 = ∞ and so 0 x2 +4x−12 dx diverges .
b→2 b→2

The answer is D.
2. The function f (x) = x23 is continuous for x ≥ 1.
R∞ Rb
By definition, 1 x23 dx = lim 1 x23 dx.
b→∞

b b
2
Z Z
lim dx = 2 lim x−3 dx
b→∞ 1 x3 b→∞ 1
b
x−2

= 2 lim
b→∞ −2 1
 
1
= − lim −1
b→∞ b2

= −(0 − 1) = 1 .

The answer is B.
8x
3. The function f (x) = √
3
8−x2
is continuous for x ≥ 3.
R∞ 8x
R b 8x
By definition, 3

3 dx = lim √
3 dx.
8−x2 b→∞ 3 8−x2

b b
8x
Z Z
−1/3
lim √
3
= −4 lim 8 − x2 (−2x dx)
b→∞ 3 8 − x2 b→∞ 3
 b
3
= −4 lim (8 − x2 )2/3
b→∞ 3 2
h i
= −6 lim (8 − b2 )2/3 − 1 .
b→∞

s 2 h 2/3 i
2 2/3 3
Since lim (8 − b ) = lim (8 − b2 ) = ∞, −6 lim 8 − b2 − 1 = −∞ and so
b→∞ b→∞ b→∞
R ∞ 8x
3

3
8−x2
dx diverges .

The answer is D.
7.7 Improper Integrals 7-267

2
4. The function f (x) = xex is continuous for x ≤ 0.
R0 2 R0 2
By definition, −∞ xex dx = lim b xex dx.
b→−∞

0 0 h 2 i0
1 1 1
Z Z  
2 2 2
lim xex dx = lim ex (2x dx) = lim ex = lim e0 − eb .
b→−∞ b 2 b→−∞ b 2 b→−∞ b 2 b→−∞
2
 2
 R0 2
1
Since lim eb = ∞, lim
2 b→−∞ e0 − eb = −∞ and so −∞ xex dx diverges .
b→−∞

The answer is D.
1
5. The function f (x) = x (ln x)2
is continuous for x ≥ 2.
R∞ 1
Rb
1
By definition, 2 x (ln x)2
dx = lim 2 dx.
b→∞ 2 x (ln x)

To evaluate x (ln1 x)2 dx, use the substitution u = ln x. Then du = 1


R
x dx, dx = x du, and
−1
1
dx = xu1 2 (x du) = u−2 du = u−1 + C = − ln1x + C.
R R R
x (ln x)2

∞ b  b  
1 1 1 1 1
Z Z
So, 2 dx = b→∞
lim 2 dx = b→∞
lim − = − lim −
2 x (ln x) 2 x (ln x) ln x 2 b→∞ ln b ln 2
 
1
=− 0− = ln12 .
ln 2

The answer is A.
1
R1 1
6. The function f (x) = √4
x
is continuous on (0, 1] but is not defined at x = 0, so 0 √
4
x
dx is
an improper integral.
Z 1 Z 1  3/4 1 √
1 −1/4 x 4 
4
 4
3 = (1 − 0) = 4 .
√ dx = lim x dx = lim 3 = lim 1 − b 3
0
4
x b→0+ b b→0+
4 b
3 b→0+ 3

The answer is C.
1 2
R∞ 2 R∞ 
7. Using the method of disks, the volume is given by V = π 1 (y) dx = π 1 x2 dx =
R∞
π 1 x14 dx.
R∞ Rb
By definition, 1 x14 dx = lim 1 x14 dx.
b→∞

∞ b
1 1
Z Z
dx = lim dx
1 x4 b→∞ 1 x4
Z b
= lim x−4 dx
b→∞ 1
b
x−3

= lim
b→∞ −3 1
 
1 1
= − lim −1
3 b→∞ b3
1 1
= − (0 − 1) = .
3 3
R∞ 1 1 π
The volume is V = π 1 x4 dx = π · 3 = 3 .
7-268 Chapter 7 Techniques of Integration

7.8 Integration Using Tables and Computer


Algebra Systems
Skill Building
1. We use Integral 123,
eax
Z
eax cos bx dx = (a cos bx + b sin bx) + C,
a2 + b2
with a = 2 and b = 1. We obtain
e2x
Z
e2x cos x dx = (2 cos x + sin x) + C
22 + 12
1 2x
= 5 e (2 cos x + sin x) + C .

2. First let u = 2x + 3, so x = u−3 1


2 and 2 du = dx, and substitute to obtain

1
Z Z
u−3
e 5x+1
sin (2x + 3) dx = e5( 2 )+1 sin u du
2
1
Z
5 13
= e 2 u− 2 sin u du
2
1 − 13
Z
5
= e 2 e 2 u sin u du.
2
We now use Integral 122,
eax
Z
eax sin bx dx = (a sin bx − b cos bx) + C,
a2 + b2
5
with a = 2 and b = 1. We obtain
5 !
e2u

1 − 13 1 − 13 5
Z
5
e 2 e 2u sin u du = e 2 sin u − cos u +C
2 2 5 2 2

2 + 12
5 !
e 2 (2x+3)

1 − 13 5
= e 2 sin (2x + 3) − cos (2x + 3) +C
2 5 2 2

2 + 12
2 5x+1 5

= 29 e 2 sin (2x + 3) − cos (2x + 3) + C .

3. Use Integral 31 with a = 2, b = 3, and n = 4.

(2 + 3x)5 2 + 3x
 
2
Z
x(2 + 3x)4 dx = − +C
32 4+2 4+1
(2+3x)5 2+3x 2

= 9 6 − 5 +C
7.8 Integration Using Tables and Computer Algebra Systems 7-269

4. Use Integral 27 with a = 5 and b = 2.


x 5 1
Z
2 dx = 22 (5 + 2x) + 22 ln |5 + 2x| + C
(5 + 2x)
5 1
= 4(5+2x) + 4 ln |5 + 2x| + C

5. Use Integral 41 with a = 6 and b = 3.

x2 dx 2 
Z
8 · 62 − (4 · 6 · 3)x + 3 · 32 x2 6 + 3x + C
 √
√ = 2
6 + 3x 15 · 3
2
288 − 72x + 27x2 6 + 3x + C
√
=
135
2
32 − 8x + 3x2 6 + 3x + C
√
= 45

6. Use Integral 45 with a = 1 and b = 1.


Z √
1+x √ dx
Z
dx = 2 1 + x + √ .
x 1+x

Use Integral 43 with a = 1 > 0 and b = 1 to evaluate √dx


R
1+x
.

dx 1 + x − 1
Z
√ = ln √
+ C.
1+x 1 + x + 1
So, Z √
1+x √ √
dx = 2 1 + x + ln √1+x−1 +C

x 1+x+1

7. Use Integral 50 with a = 2. Then a2 = 4 and


Z √ 2
x +4 √ √
2

dx = x2 + 4 − 2 ln 2+ xx +4 + C

x

8. Use Integral 54 with a = 3. Then a2 = 9 and

x2 √ √
Z
dx = x2 x2 − 9 + 9

√ 2 ln x + x2 − 9 + C
2
x −9

9. Use Integral 59 with a = 2. Then a2 = 4 and


dx
Z
3/2
= − 4√xx2 −4 + C
2
(x − 4)

10. Use Integral 58 with a = 3. Then a2 = 9 and

x 3 · 34
Z
3/2 p p
x2 + 9 dx = 2x2 + 5 · 9 x2 + 9 + ln x + x2 + 9 + C

8 8

= x8 2x2 + 45 x2 + 9 + 243
√
8 ln x +
x2 + 9 + C
7-270 Chapter 7 Techniques of Integration

11. Use Integral 120 with m = 3 and n = 2.

x3+1 (ln x)2 2 x4 (ln x)2 1


Z Z Z
3 2 3 2−1
x (ln x) dx = − x (ln x) dx = − x3 ln x dx
3+1 3+1 4 2

Use Integral 117 with n = 3 to evaluate x3 ln x dx.


R

 3+1 
x4
  
x 1 1
Z
3
x ln x dx = ln x − +C = ln x − +C
3+1 3+1 4 4
So,

x4 (ln x)2 1 x4
  
1
Z
x4 (ln x)2 x4
x3 (ln x)2 dx = 1

− ln x − +C = 4 − 8 ln x − 4 +C
4 2 4 4

12. Use Integral 119.


dx
Z
= ln |ln x| + C
x ln x

13. Use Integral 47 with a = 4. Then a2 = 16 and


xp 2 16
Z p p
x2 − 16 dx = x − 16 − ln x + x2 − 16 + C

2 2
√ √
= x2 x2 − 16 − 8 ln x + x2 − 16 + C


14. Use Integral 50 with a = 3. Then a2 = 3 and
Z √ 2
x +3 √ √ √ √
2

dx = x2 + 3 − 3 ln 3+ xx +3 + C

x

15. Use Integral 67 with a = 6 . Then a2 = 6 and

x 3/2 (3 · 6)x p 3( 6)4 x
Z
3/2
6 − x2 dx = 6 − x2 + 6 − x2 + sin−1 √ + C
4 8 8 6
3/2 √ √
−1 x 6
= x4 6 − x2 + 94 x 6 − x2 + 27

2 sin 6 +C


16. Use Integral 66 with a = 10. Then a2 = 10 and
dx √
Z
2
√ dx = − 10−x
10x +C
x2 10 − x2

17. Use Integral 69 with a = 5.

52
 
x − 5p 5−x
Z p
2 2 −1
10x − x dx = (2 · 5)x − x + cos +C
2 2 5

= x−5 10x − x2 + 25 −1 5−x

2 2 cos 5 +C

18. Use Integral 76 with a = 3.


p
dx (2 · 3)x − x2 √
Z
2
√ =− + C = − 6x−x
3x +C
x 6x − x2 3x
7.8 Integration Using Tables and Computer Algebra Systems 7-271

19. Use Integral 95 with a = 3 and b = 8.

sin [(3 + 8)x] sin [(3 − 8)x] sin (11x) sin (−5x)
Z
cos (3x) cos (8x) dx = + +C = + +C
2(3 + 8) 2(3 − 8) 22 (−10)
sin (−5x) sin (5x)
Since sin (−5x) = − sin (5x), (−10) = 10 and the integral becomes
Z
sin (11x) sin (5x)
cos (3x) cos (8x) dx = 22 + 10 +C .

20. Use Integral 94 with m = 2 and n = 5.

sin [(2 + 5)x] sin [(2 − 5)x] sin (7x) sin (−3x)
Z
sin (2x) sin (5x) dx = − + +C = + +C
2(2 + 5) 2(2 − 5) 14 (−6)
sin (−3x) sin (3x)
Since sin (−3x) = − sin (3x), (−6) = 6 and the integral becomes
Z
sin (2x) sin (5x) dx = − sin14
(7x)
+ sin (3x)
6 +C

21. Use Integral 109. Z


x2 +1
x tan−1 x dx = 2 tan−1 x − x
2 +C

22. Use Integral 107.



Z
2x2 −1
x sin−1 x dx = 4 sin−1 x + x 1−x2
4 +C

23. Use Integral 117 with n = 4.


 4+1  
x 1
Z
4 x5 1

x ln x dx = ln x − +C = 5 ln x − 5 +C
4+1 4+1

24. Use Integral 116 with n = 3.


Z Z Z
(ln x)3 dx = x (ln x)3 − 3 (ln x)3−1 dx = x (ln x)3 − 3 (ln x)2 dx

Use Integral 116 with n = 2 to evaluate (ln x)2 dx.


R

Z  Z  Z
(ln x)3 dx = x (ln x)3 − 3 x (ln x)2 − 2 ln x dx + C = x (ln x)3 − 3x (ln x)2 + 6 ln x dx + C

R
Finally, use Integral 5 to evaluate ln x dx.
Z
(ln x)3 dx = x (ln x)3 − 3x (ln x)2 + 6(x ln x − x) + C

= x (ln x)3 − 3x (ln x)2 + 6x ln x − 6x + C

25. Use Integral 128. Z


sinh (2x)
sinh2 x dx = 4 − x
2 +C
7-272 Chapter 7 Techniques of Integration

26. Use Integral 130. Z


tanh2 x dx = x − tanh x + C

27. Use Integral 79 with n = 2 and a = 4.


p
dx (2 · 4)x − x2 2−1 dx
Z Z
√ = 2
+ √ +C
2
x 8x − x 2 4(1 − 2 · 2)x (2 · 2 − 1) · 4 x2−1 8x − x2

8x − x2 1 dx
Z
=− 2
+ √ +C
12x 12 x 8x − x2
dx
R
Use Integral 76 with a = 4 to evaluate x√8x−x 2.

p √
dx (2 · 4)x − x2 8x − x2
Z
√ =− +C =−
x 8x − x2 4x 4x
So, the integral becomes
√ " √ #
dx 8x − x2 1 8x − x2 √ √
Z
8x−x2 8x−x2
√ =− 2
+ − + C = − 12x2 − 48x +C
2
x 8x − x2 12x 12 4x

28. Use Integral 72 with a = 6.


Z √ p
12x − x2 2 (2 · 6)x − x2
 
−1 6−x
dx = − − cos +C
x2 x 6

2
= − 2 12x−x − cos−1 6−x

x 6 +C

29. Use Integral 36 with n = 2 and a = 2. Then a2 = 4 and


" #
dx 1 x dx
Z Z
2 = 2(2 − 1) · 4 2−1 + (2 · 2 − 3) 2−1 + C
(4 + x2 ) (4 + x2 ) (4 + x2 )
 
1 x dx
Z
= + +C
8 4 + x2 4 + x2
R dx
Use Integral 17 with a = 2 to evaluate 4+x 2.

dx 1 x
Z
2
= tan−1 + C
4+x 2 2
So,
dx
Z  
1 x 1 x
2 = 8 4+x2 + 2 tan−1 2 +C .
(4 + x2 )

30. Use Integral 37 recursively with a = 5 and n = 3. Then a2 = 25 and


" #
dx 1 x dx
Z Z
3 = 2(3 − 1) · 25 − 3−1 − (2 · 3 − 3) 3−1 + C
(x2 − 25) (x2 − 25) (x2 − 25)
" #
1 x dx
Z
= − −3
100 (x2 − 25)2 (x2 − 25)2
7.8 Integration Using Tables and Computer Algebra Systems 7-273

dx
R
Use Integral 37 with a = 5 and n = 2 to evaluate (x2 −25)2
.
" #
dx 1 x dx
Z Z
2 = 2(2 − 1) · 25 − 2−1 − (2 · 2 − 3) 2−1 + C
(x2 − 25) (x2 − 25) (x2 − 25)
 
1 x dx
Z
= − 2 − +C
50 x − 25 x2 − 25

So,
(   )
dx 1 x 1 x dx
Z Z
2 3 = 100 − 2 − 3 50 − x2 − 25 − x2 − 25
(x − 25) (x2 − 25)
" #
1 x 3x 3 dx
Z
= − + +
100 (x2 − 25)2 50(x2 − 25) 50 x2 − 25

dx 1
ln x−5 1
ln x−5
R
Using Integral 35 with a = 5, = x+5 + C = x+5 + C.

x2 −25 2·5 10

Finally,
" #
dx 1 x 3x 3 1 x − 5
Z
3 = 100 − 2 + 50(x2 − 25) + 50 · 10 ln x + 5 + C

(x2 − 25) (x2 − 25)
h i
1 x 3x 3 x−5
= 100 − (x2 −25) 2 + 50(x2 −25) + 500 ln x+5 + C

31. Use Integral 132. Z


x sinh x dx = x cosh x − sinh x + C

32. Using Integral 134 with n = 2,


Z Z
x2 sinh x dx = x2 cosh x − 2 x cosh x dx.

Using Integral 133, Z


x cosh x dx = x sinh x − cosh x + C.

So,
Z Z
x2 sinh x dx = x2 cosh x − 2 x cosh x dx

= x2 cosh x − 2(x sinh x − cosh x) + C


= x2 cosh x − 2x sinh x + 2 cosh x + C


33. Begin
p with the substitution
√ u = x + 1. Then x = u − 1, dx = du, and 4x + 5 =
4(u − 1) + 5 = 4u + 1.
The integral becomes
√ √
Z Z
(x + 1) 4x + 5 dx = u 4u + 1 du.
7-274 Chapter 7 Techniques of Integration

Use Integral 38 with a = 1 and b = 4.


√ 2 1
Z
u 4u + 1 du = (12u − 2)(1 + 4u)3/2 + C = (6u − 1)(1 + 4u)3/2 + C
15 · 16 60
So, using u = x + 1,
√ 1
Z
(x + 1) 4x + 5 dx = [6(x + 1) − 1][1 + 4(x + 1)]3/2 + C
60
1 3/2
= 60 (6x + 5)(4x + 5) +C

34. To evaluate [(2x+3)dx2 −1]3/2 , use the substitution u = 2x + 3. Then du = 2 dx, dx = du


R
2 ,
du
and [(2x+3)dx2 −1]3/2 = (u2 −1) 1 du
R R 2
R
3/2 = 2 .
(u2 −1)3/2

Use Integral 59 with a = 1. Then a = 1 and


du u
Z
3/2
= −√ + C.
2 2
u −1
(u − 1)

So,
 
dx 1 du 1 u
Z Z
= = −√ + C = − 12 √ 2x+32 + C
[(2x + 3)2 − 1]3/2 2 (u2 − 1)3/2 2 u2 − 1 (2x+3) −1

2
35. We complete the square, and write 3x − x2 = 94 − x − 23 . Then substitute u = x − 32 ,
split the integral, and use symmetry to obtain
Z 2 Z 2
x3 x3
√ dx = q 2 dx
1 3x − x2 1 9
− x− 3 4 2
1
3 3

u+
Z 2
= q 2 du
− 12 9 2
4 −u
1
u3 + 92 u2 + 27
4 u+
27
Z 2
8
= q du
− 12 9 2
4 −u
1 Z 12 9 2 27
u3 + 27 4 u 2u + 8
Z 2
= q du + q du
1 9 2 1 9 2
−2
4 − u −2
4 − u
Z 12
− 29 49 − u2 + 27
 
2
= 0+2 q du
9 2
0
4 − u
Z 12 r Z 12
9 1
= −9 − u2 du + 27 q du.
0 4 0 9
− u24
7.8 Integration Using Tables and Computer Algebra Systems 7-275

Now use Integrals 60 and 16 to get


" r #1/2 1/2
2 9
x3

u 9 u u
Z
−1
√ dx = −9 4
− u2 + sin 3 + 27 sin−1 3
1 3x − x2 2 4 2 2 0 2 0
 s !
1  2 1
 r
9 1 9 2 0 9 9 2 (0)
= −9  2 − + sin−1 2
− − 02 + sin−1 
2 4 2 8 3 2 4 4 3
!
1

−1 2 2 −1 2 (0)
+ 27 sin − sin
3 3
√ !  
2 9 −1 1 1
= −9 + sin − 0 + 27 sin−1 − 0
4 8 3 3

= 135
8 sin−1 1
3 − 9 2
4 .

36. We use Integral 57, √


dx x2 + a2
Z
√ =− + C,
x2 x2 + a2 a2 x

with a = 2. We obtain
Z e e
dx dx
Z
√ = √ 2 q
1 x x2 + 2
2
x2 x2 + 1 2
 q √ 2 e
x2 + 2
= − √ 2 
2 x
1
√ √ !
e2 + 2 12 + 2
= − − −
2e 2(1)
√ √
e2 +2 3
= − 2e + 2 .

37. (a) Using a CAS we obtain


Z
e2x
e2x cos x dx = 5 (sin x + 2 cos x) + C .

(b) We obtained in problem 1,


Z
e2x sin x dx = 1 2x
5 e (2 cos x + sin x) + C .

(c) Since
e2x 1
(sin x + 2 cos x) = e2x (2 cos x + sin x)
5 5
we see that the results are equivalent.
7-276 Chapter 7 Techniques of Integration

38. (a) Using a CAS we obtain


Z
e5x+1 sin (2x + 3) dx = − 29
1 5x+1
e (2 cos (2x + 3) − 5 sin (2x + 3)) + C .

(b) We obtained in problem 2,


Z
e2x sin x dx = 2 5x+1 5

29 e 2 sin (2x + 3) − cos (2x + 3) + C .

(c) Since
 
1 5x+1 2 5x+1 5
− e (2 cos (2x + 3) − 5 sin (2x + 3)) = − e cos (2x + 3) − sin (2x + 3)
29 29 2
 
2 5x+1 5
= e sin (2x + 3) − cos (2x + 3)
29 2
we see that the results are equivalent.
39. Refer to the integral in exercise 3. Using WoLframAlpha,
Z
x (2 + 3x)4 dx = 27 6 216 5 4 3 2
2 x + 5 x + 54x + 32x + 8x + constant

A screenshot is shown below.

40. Refer to the integral in exercise 4. Using WolframAlpha,


x
Z h i
1 5
dx = 4 2x+5 + log (2x + 5) + constant
(5 + 2x)2

where log (x) is the natural logarithm.


A screenshot is shown below.

41. Refer to the integral in exercise 5. Using WolframAlpha,


x2 √
Z
dx = 215x+2 3x2 − 8x + 32 + constant

√ √
3
6 + 3x
A screenshot is shown below.

42. Refer to the integral in exercise 6. Using WolframAlpha,


Z √
1+x √ √  √ 
dx = 2 x + 1 + log 1 − x + 1 − log 1 + x + 1 + constant
x
where log (x) is the natural logarithm.
7.8 Integration Using Tables and Computer Algebra Systems 7-277

A screenshot is shown below.

43. Refer to the integral in exercise 7. Using WolframAlpha,


Z √
4 + x2 √ √ 
dx = x2 + 4 − 2 log x2 + 4 + 2 + 2 log (x) + constant
x
where log (x) is the natural logarithm.
A screenshot is shown below.

44. Refer to the integral in exercise 8. Using WolframAlpha,

x2 √
Z
dx = 12 x2 − 9x + 9 log x2 − 9 + x + constant
√ 

−9 + x 2

where log (x) is the natural logarithm.


A screenshot is shown below.

45. Refer to the integral in exercise 9. Using WolframAlpha,


dx
Z
3/2
= − 4√xx2 −4 + constant
(−4 + x2 )
A screenshot is shown below.

46. Refer to the integral in exercise 10. Using WolframAlpha,


 √
Z
3/2
9 + x2 dx = 18 x x2 + 9 2x2 + 45 + 243 sinh−1 x
 
3 + constant

A screenshot is shown below.

47. Refer to the integral in exercise 11. Using WolframAlpha,


Z
x3 log2 (x) dx = 32 x 8 log2 (x) − 4 log (x) + 1 + constant
1 4
 

where log (x) is the natural logarithm.


7-278 Chapter 7 Techniques of Integration

A screenshot is shown below.

48. Refer to the integral in exercise 12. Using WolframAlpha,


1
Z
dx = log [log (x)] + C
x log (x)

where log (x) is the natural logarithm.


A screenshot is shown below.

49. Refer to the integral in exercise 13. Using WolframAlpha,


√ √
Z p
−16 + x2 dx = 12 x x2 − 16 − 8 log x2 − 16 + x + constant


where log (x) is the natural logarithm.


A screenshot is shown below.

50. Refer to the integral in exercise 14. Using WolframAlpha,


Z √
3 + x2 √ √ √ √  √
dx = x2 + 3 − 3 log 3 x2 + 3 + 3 + 3 log (x) + constant
x
where log (x) is the natural logarithm.
A screenshot is shown below.

51. Refer to the integral in exercise 15. Using WolframAlpha,



Z  
3/2 −1 √
6 − x2 dx = 272 sin x
6
− 41 x 6 − x2 (x2 − 15) + constant

A screenshot is shown below.

52. Refer to the integral in exercise 16. Using WolframAlpha,


dx √
Z
2
√ dx = − 10−x
10x + constant
2
x 10 − x 2
7.8 Integration Using Tables and Computer Algebra Systems 7-279

A screenshot is shown below.

53. Refer to the integral in exercise 17. Using WolframAlpha,


Z p √ √ √ √ √
−(x−10)x[ x−10(x−5) x−50 log ( x−10+ x)]
10x − x2 dx = √ √
2 x−10 x
+ constant

where log (x) is the natural logarithm.


A screenshot is shown below.

54. Refer to the integral in exercise 18. Using WolframAlpha,



1
Z
−(x−6)x
√ dx = − 3x + constant
x 6x − x2
A screenshot is shown below.

55. Refer to the integral in exercise 19. Using WolframAlpha,


Z
1 1
cos (3x) cos (8x) dx = 10 sin (5x) + 22 sin (11x) + constant .

A screenshot is shown below.

56. Refer to the integral in exercise 20. Using WolframAlpha,


Z
sin (2x) sin (5x) dx = 16 sin (3x) − 14
1
sin (7x) + constant

A screenshot is shown below.

57. Refer to the integral in exercise 21. Using WolframAlpha,


Z
x tan−1 x dx = 12 x2 + 1 tan−1 x − x + constant
  
7-280 Chapter 7 Techniques of Integration

A screenshot is shown below.

58. Refer to the integral in exercise 22. Using WolframAlpha,


Z
x sin−1 x dx = 14 1 − x2 x + (2x2 − 1) sin−1 (x) + constant
√ 

A screenshot is shown below.

59. Refer to the integral in exercise 23. Using WolframAlpha,


Z
x4 log (x) dx = 25
1 5
x [5 log (x) − 1] + constant

where log (x) is the natural logarithm.


A screenshot is shown below.

60. Refer to the integral in exercise 24. Using WolframAlpha,


Z
log3 (x) dx = x log3 (x) − 3 log2 (x) + 6 log (x) − 6 + constant
 

where log (x) is the natural logarithm.


A screenshot is shown below.

61. Refer to the integral in exercise 25. Using WolframAlpha,


Z
sinh2 (x) dx = 14 [sinh (2x) − 2x] + constant

A screenshot is shown below.

62. Refer to the integral in exercise 26. Using WolframAlpha,


Z
tanh2 x dx = x − tanh x + constant

A screenshot is shown below.


7.8 Integration Using Tables and Computer Algebra Systems 7-281

63. Refer to the integral in exercise 27. Using WolframAlpha,



1
Z
−(x−8)x(x+4)
√ dx = − 48x2 + constant
x2 8x − x2
A screenshot is shown below.

64. Refer to the integral in exercise 28. Using WolframAlpha,


Z √ √ √ √ √
12x − x2 2[x− x−12 x log ( x−12+ x)−12]
dx = √ + constant
x 2 −(x−12)x

where log (x) is the natural logarithm.


A screenshot is shown below.

65. Refer to the integral in exercise 29. Using WolframAlpha,

1
Z h
1 2x −1 x
i
2 dx = 16 x 2 +4 + tan 2 + constant
(4 + x2 )

A screenshot is shown below.

66. Refer to the integral in exercise 30. Using WolframAlpha,

10x (3x2 −125)


1 +3 log (5−x)−3 log (x+5)
Z
(x2 −25)2
3 dx = 50,000 + constant
(x2 − 25)

where log (x) is the natural logarithm.


A screenshot is shown below.

67. Refer to the integral in exercise 31. Using WolframAlpha,


Z
x sinh (x) dx = x cosh (x) − sinh (x) + constant
7-282 Chapter 7 Techniques of Integration

A screenshot is shown below.

68. Refer to the integral in exercise 32. Using WolframAlpha,


Z
x2 sinh x dx = x2 + 2 cosh x − 2x sinh (x) + constant


A screenshot is shown below.

69. Refer to the integral in exercise 33. Using WolframAlpha,



Z
(x + 1) 4x + 5 dx = 60 1
(4x + 5)3/2 (6x + 5) + constant

A screenshot is shown below.

70. Refer to the integral in exercise 34. Using WolframAlpha,


1
Z
3/2
dx = − 4√x2x+3
2 +3x+2 + constant
[−1 + (2x + 3)2 ]

A screenshot is shown below.

71. (a) Using a CAS we obtain


2 √
x3
Z
√ dx = 135
8 sin−1 1
3 − 9
4 2.
1 3x − x2

(b) We obtained in problem 15,


Z e
dx √
√ = 135
8 sin−1 1
3 − 9
4 2.
1 x2 x2 + 2

(c) Since
135 −1 1 9 √ 135 1 9√
sin − 2= sin−1 − 2
8 3 4 8 3 4
we see that the results are equivalent.
72. (a) Using a CAS we obtain
e √ √
dx
Z
1
√ = 2 3 − 12 e−1 e2 + 2 .
1 x2 x2 + 2
Chapter 7 Review Exercises 7-283

(b) We obtained in problem 16,


Z e √ √
dx 2
√ = − e2e+2 + 2
3
.
1 x2 2
x +2
(c) Since
1√ 1 p 1 p 1√
3 − e−1 e2 + 2 = − e−1 e2 + 2 + 3
2 2 2√ √ 2
e2 + 2 3
= − +
2e 2
we see that the results are equivalent.
73. Using a CAS we obtain Z p Z p
1 + x3 dx = x3 + 1 dx

and we conclude that this integral cannot be expressed using elementary functions.
74. Using a CAS we obtain
√ 2 √
Z
1 + sin x dx = (sin x − 1) sin x + 1 + C
cos x
and we conclude that this integral can be expressed using elementary functions.
75. Using a CAS we obtain
1√
Z
2
e−x dx = πerf(x) + C
2
and we conclude that this integral cannot be expressed using elementary functions.
76. Using a CAS we obtain
cos x
Z
dx = Ci(x) + C
x
and we conclude that this integral cannot be expressed using elementary functions.
77. Using a CAS we obtain
1
Z
x tan x dx = ix2 − idilog ieix − iπ tan−1 eix − idilog −ieix + C
  
2
and we conclude that this integral cannot be expressed using elementary functions.
78. Using a CAS we obtain
√ √ √ √
Z
 
1 + ex dx = 2 ex + 1 + ln ex + 1 − 1 − ln ex + 1 + 1 + C .

Chapter 7 Review Exercises


2
1. We complete the square: x2 + 4x + 20 = (x + 2) + 16. Then we let u = x + 2 to obtain
dx dx
Z Z
= 2
x2 + 4x + 20 (x + 2) + 42
du
Z
=
u + 42
2

1 u
= tan−1 + C
4 4
1 −1 x+2

= 4 tan 4 +C .
7-284 Chapter 7 Techniques of Integration

2. We rewrite the integral to obtain the derivative of the denominator, complete the square
2
with y 2 + y + 1 = y + 21 + 43 , and let u = y + 12 to obtain
1 1
y +1 2 (2y + 1) + 2
Z Z
2
dy = dy
y +y+1 y2 + y + 1
1 2y + 1 1 dx
Z Z
= dx +
2 y2 + y + 1 2 1 2 3

y+2 + 4
1  1 du
Z
= ln y 2 + y + 1 +  √ 2
2 2
u2 + 23
 
1  1 2 u
= ln y 2 + y + 1 + √ tan−1 √ +C
2 2 3 3/2
1 1 2y + 1
ln y 2 + y + 1 + √ tan−1 √

= +C
2 3 3
 √3 √
1 2 −1 3(2y+1)
= 2 ln y + y + 1 + 3 tan 3 +C .

3. Factor out sec φ tan φ. Then let u = sec φ, so du = sec φ tan φ dφ. We substitute and obtain
Z Z
sec3 φ tan φ dφ = sec2 φ sec φ tan φ dφ
Z
= u2 du
1 3
= u +C
3
1
= 3 sec3 φ + C .

4. We use the identity cot2 θ = csc2 θ − 1.


Z Z
2
csc2 θ − 1 csc θ dθ

cot θ csc θ dφ =
Z Z
= csc3 θ dθ − csc θ dθ
1 1
= − csc θ cot θ + ln |csc θ − cot θ|
2 2
− ln |csc θ − cot θ| + C (using Formulas 89 and 14)
= − 21 csc θ cot θ − 1
2 ln |csc θ − cot θ| + C .

5. Factor out sin φ and use the identity sin2 φ = 1 − cos2 φ.


Z Z
sin3 φ dφ = sin2 φ sin φ dφ
Z
1 − cos2 φ sin φ dφ.

=
Chapter 7 Review Exercises 7-285

Let u = cos φ, then du = − sin φ dφ, so sin φ dφ = −du. We substitute and obtain
Z Z
sin3 φ dφ = 1 − u2 (−du)


Z
u2 − 1 du

=
1 3
= u −u+C
3
1
= 3 cos3 φ − cos φ + C .

6. Let x = 2 sin θ, then dx = 2 cos θ dθ. We substitute and obtain

x2 (2 sin θ)2
Z Z
√ dx = q (2 cos θ) dθ
4 − x2 4 − (2 sin θ)2
sin2 θ
Z
= 8 p cos θ dθ
4 − 4 sin2 θ
sin2 θ
Z
= 4 cos θ dθ
cos θ
Z
= 4 sin2 θ dθ

1 − cos (2θ)
Z
= 4 dθ
2
Z
= 2 (1 − cos (2θ)) dθ
 
1
= 2 θ − sin 2θ + C
2
= 2θ − 2 sin θ cos θ + C.
p q
2 √
We have θ = sin−1 x2 , and cos θ = 1 − sin2 θ = 1 − (x/2) = 12 4 − x2 . We obtain


x2  x  1 p 
−1 x
Z  
√ dx = 2 sin −2 4−x +C2
4 − x2 2 2 2

= 2 sin−1 x2 − 21 x 4 − x2 + C .


7. Let u = x + 2, so du = dx, then substitute.


dx dx
Z Z
q = √
(x + 2)2 − 1 u2 − 1

Let u = sec θ, then du = sec θ tan θ dθ. We substitute and obtain


dx sec θ tan θ dθ
Z Z
q = √
2
(x + 2) − 1 sec2 θ − 1

sec θ tan θ dθ
Z
=
tan θ
Z
= sec θ dθ

= ln |sec θ + tan θ| + C.
7-286 Chapter 7 Techniques of Integration

√ √ q
2
We have sec θ = u = x + 2, so tan θ = sec2 θ − 1 = u2 − 1 = (x + 2) − 1. We obtain

dx
Z q
2
q = ln x + 2 + (x + 2) − 1 + C .
(x + 2)2 − 1

8. We use integration by parts with u = x and dv = sin (2x) dx. Then du = dx and v =
1 1
2 (− cos (2x)) = − 2 cos (2x). We obtain

π/4   π/4 Z π/4  


1 1
Z
x sin (2x) dx = x − cos (2x) − − cos (2x) dx
0 2 0 0 2
1 π/4
   
1 π π 1
    Z
= − cos 2 − − (0) cos (2(0)) + cos (2x) dx
2 4 4 2 2 0
 π/4
1 1
= 0+ sin 2x
2 2 0
  π  1  1 
1 1
= sin 2 − sin 2(0)
2 2 4 2 2
1
= 4 .

9. We use integration by parts with u = v and dw = csc2 w dw. Then du = dv and w =


− cot v. We obtain
Z Z
2
v csc v dv = v(− cot v) − (− cot v) dv
Z
= −v cot v + cot v dv

= −v cot v + ln |sin v| + C .

10. Factor out cos x and use the identity cos2 x = 1 − sin2 x.
Z Z
sin2 x cos3 x dx = sin2 x cos2 x cos x dx
Z
sin2 x 1 − sin2 x cos x dx.

=

Let u = sin x, then du = cos x dx. We substitute and obtain


Z Z
sin2 x cos3 x dx = u2 1 − u2 du


Z
u2 − u4 du

=
1 3 1 5
= u − u +C
3 5
= 1
3 sin3 x − 1
5 sin5 x + C .
Chapter 7 Review Exercises 7-287

11. Let x = 2 sin θ, then dx = 2 cos θ dθ. We substitute and obtain


Z Z  3/2
2 3/2 2

4−x dx = 4 − (2 sin θ) (2 cos θ) dθ
Z
3
= 2 (2 cos θ) cos θ dθ
Z
= 16 cos4 θ dθ
Z  2
1 + cos (2θ)
= 16 dθ
2
Z
1 + 2 cos (2θ) + cos2 (2θ) dθ

= 4

1 + cos (4θ)
Z
= 4(θ + sin (2θ)) + 4 dθ
2
1
= 4θ + 4 sin (2θ) + 2θ + sin (4θ) + C
2
= 6θ + 8 sin θ cos θ + sin (2θ) cos (2θ) + C
= 6θ + 8 sin θ cos θ + 2 sin θ cos θ 1 − 2 sin2 θ + C.


p q
2 √
We have θ = sin−1 x
, and cos θ = 1 − sin2 θ = 1 − (x/2) = 12 4 − x2 . We obtain

2
Z x  x  1 p 
2 3/2 −1

4−x dx = 6 sin +8 4−x 2
2 2 2

x 1p   x 2 
+2 4−x 2 1−2 +C
2 2 2
√ √
6 sin−1 x2 + 2x 4 − x2 + 14 x 4 − x2 2 − x2 + C .
 
=

12. We use partial fractions to obtain

3x2 + 1 3x2 + 1
=
x3 + 2x2 − 3x x(x + 3)(x − 1)
A B C
= + +
x x+3 x−1
3x2 + 1 = A(x + 3)(x − 1) + Bx(x − 1) + Cx(x + 3).

When x = 1 we obtain C = 1, when x = −3, we have B = 7/3, and when x = 0, we obtain


A = −1/3. So

3x2 + 1
Z  
−1/3 7/3 1
Z
dx = + + dx
x3 + 2x2 − 3x x x+3 x−1
−1/3 7/3 1
Z Z Z
= dx + dx + dx
x x+3 x−1
= − 13 ln |x| + 7
3 ln |x + 3| + ln |x − 1| + C .
7-288 Chapter 7 Techniques of Integration

13. Let u = et , so du = et dt. We substitute to obtain


e2t et
Z Z
dt = et dt
et − 2 et − 2
u
Z
= du
u−2
u−2+2
Z
= du
u−2
Z  
2
= 1+ du
u−2
= u + 2 ln |u − 2| + C
= et + 2 ln |et − 2| + C .

14. We rewrite the integral, and obtain


dy dy
Z Z
2
= 2.
5 + 4y + 4y 4 + (1 + 2y)
Let u = 1 + 2y, so du = 2 dy, and substitute:
1
dy 2 du
Z Z
=
5 + 4y + 4y 2 22 + u2
 
1 1 u
= tan−1 + C
2 2 2
1 1+2y
= 4 tan−1 2 +C .

15. We use partial fractions to obtain


x x
4
=
x − 16 (x2 − 4) (x2 + 4)
x
=
(x − 2) (x + 2) (x2 + 4)
A B Cx + D
= + + 2
x−2 x+2 x +4
so
x = A (x + 2) x2 + 4 + B (x − 2) x2 + 4 + (Cx + D) (x − 2) (x + 2) .
 

Let x = 2 to obtain A = 1/16, and x = −2 to get B = 1/16. We now have


1 1
(x + 2) x2 + 4 + (x − 2) x2 + 4 + (Cx + D) (x − 2) (x + 2)
 
x =
16
  16
 
1 3 2 1
x = C+ x + Dx + − 4C x − 4D.
8 2
Equating coefficients, we obtain C = −1/8 and D = 0. So
Z  
x dx 1/16 1/16 (−1/8) x
Z
= + + dx
x4 − 16 x−2 x+2 x2 + 4
1 1 1
ln |x − 2| + ln |x + 2| − ln x2 + 4 + C

=
16 16 16
1 2 1 2

= ln x − 4 −
ln x + 4 + C
16 16
2
1
= 16 ln xx2 −4
+4 + C .

Chapter 7 Review Exercises 7-289

16. Let y = x2 , so dy = 2x dx and xdx = 21 dy. We substitute and obtain


Z Z
2 2
x3 ex dx = x2 ex x dx
1
Z
= yey dy
2
1
Z
= yey dy.
2
We use integration by parts with u = y, du = dy, dv = ey dy, and v = ey . We obtain
 
1
Z Z
3 x2 y y
x e dx = ye − e dy
2
1
= (yey − ey ) + C
2
1 2 x2 2
= 2x e − 21 ex + C .

17. Let u = y + 1, so du = dy and y = u − 1. We substitute and obtain


2
y 2 dy (u − 1) du
Z Z
3 =
(y + 1) u3
Z 2
u − 2u + 1
= du
u3
Z  
1
= − 2u−2 + u−3 du
u
2 1
= ln |u| + − 2 + C
u 2u
2 1
= ln |y + 1| + y+1 − 2(y+1)2
+C .

18. Let x = 5 tan θ, then dx = 5 sec2 θ dθ. We substitute and obtain


dx 5 sec2 θ
Z Z
√ = q dθ
x2 x2 + 25 (5 tan θ)2 (5 tan θ)2 + 25
1 sec2 θ
Z
= √ dθ
5 tan2 θ 25 tan2 θ + 25
1 sec2 θ
Z
= dθ
25 tan2 θ sec θ
1 sec θ
Z
= dθ
25 tan2 θ
1 cos θ
Z
= dθ.
25 sin2 θ
Let u = sin θ, then du = cos θ dθ. We substitute and obtain
dx 1 1
Z Z
√ = du
x2 x2 + 25 25 u2
 
1 1
= − +C
25 u
1
= − csc θ + C.
25
7-290 Chapter 7 Techniques of Integration

√ q
5 2

We have tan θ = x5 , so cot θ = x5 , and csc θ = 1 + cot2 θ = 1 + 1

x = x x2 + 25. We
obtain
dx 1 1p 2
Z
√ = − x + 25 + C
2 2
x x + 25 25 x

x2 +25
= − 25x +C .

19. We use integration by parts with u = x and dv = sec2 x dx. Then du = dx and v = tan x.
We obtain
Z Z
x sec2 x dx = x tan x − tan x dx

= x tan x − ln |sec x| + C .

 
20. We complete the square: 16 + 4x − 2x2 = 2 8 + 2x − x2 = 2 9 − (x − 1)2 . Then we let


u = (x − 1) to obtain

dx dx
Z Z
√ = r 
16 + 4x − 2x2 2

2 9 − (x − 1)
1 du
Z
= √ √
2 3 − u2
2

1 u
= √ sin−1 +C
2 3

2 −1 x−1

= 2 sin 3 +C .

−1
21. We use integration by parts with u = ln (1 − y), du = 1−y dy, dv = dy, and v = y. We
obtain
Z  
−1
Z
ln (1 − y) dy = y ln (1 − y) − y dy
1−y
−y
Z
= y ln (1 − y) − dy
1−y
(1 − y) − 1
Z
= y ln (1 − y) − dy
1−y
Z  
1
= y ln (1 − y) − 1− dy
1−y
= y ln (1 − y) − y − ln (1 − y) + C .

22. We use partial fractions to obtain

x3 − 2x − 1 Ax + B Cx + D
2 = + 2
(x2 + 1) x2 + 1 (x2 + 1)
x3 − 2x − 1 = (Ax + B) x2 + 1 + Cx + D


x3 − 2x − 1 = Ax3 + Bx2 + (A + C)x + (B + D).


Chapter 7 Review Exercises 7-291

We equate coefficients, and obtain A = 1, B = 0, C = −3, and D = −1. We now have


Z 3 !
x − 2x − 1 x 3x + 1
Z
2 dx = −
x2 + 1 (x2 + 1)2
dx
(x2 + 1)
1 x 1
Z Z
ln x2 + 1 − 3

= 2 dx − 2 dx
2 (x2 + 1) (x2 + 1)
1 3 1
Z
ln x2 + 1 +

= − 2 dx.
2 2(x2 + 1) (x2 + 1)
For the remaining integral, we let x = tan θ, and obtain
1 sec2 θ
Z Z
2 dx = dθ
(x2 + 1) sec4 θ
Z
= cos2 θ dθ
1
Z
= (1 + cos (2θ)) dθ
2
 
1 1
= θ + sin 2θ + C
2 2
1
= (θ + sin θ cos θ) + C
2 
1 tan θ
= θ+ + C.
2 sec2 θ
√ √
Since x = tan θ, sec θ = tan2 θ + 1 = x2 + 1, and we have
1 1 1 x
Z
2 dx = tan−1 x + √ 2 + C
2
(x + 1) 2 2 2
x +1
1 1 x
= tan−1 x + + C.
2 2 x2 + 1
And so we obtain
Z 3
x − 2x − 1 1 3 1 1 x
ln x2 + 1 + tan−1 x −

2 dx = 2 2(x2 +1) − 2 2 x2 +1 +C .
(x2 + 1)

23. We use partial fractions to obtain


3x2 + 2 3x2 + 2
=
x3 − x2 x2 (x − 1)
A B C
= + 2+
x x x−1
3x2 + 2 = Ax(x − 1) + B(x − 1) + Cx2 .
Let x = 1 to obtain C = 5. Let x = 0 to get B = −2. Then
3x2 + 2 = Ax(x − 1) + (−2)(x − 1) + 5x2
3x2 + 2 = (A + 5)x2 + (−A − 2)x + 2.
We equate coefficients and determine A = −2. We now have
3x2 + 2
Z  
−2 −2 5
Z
dx = + 2 + dx
x3 − x2 x x x−1
2
= −2 ln |x| + x + 5 ln |x − 1| + C .
7-292 Chapter 7 Techniques of Integration

p p
24. Let y = 2/3 tan θ, then dy = 2/3 sec2 θ dθ. We substitute and obtain
p
dy 2/3 sec2 θ dθ
Z Z
p = r
2 + 3y 2 p 2
2+3 2/3 tan θ
Z p
2/3 sec2 θ dθ
= √
2
Z 2 + 2 tan θ
1 1
sec2 θ dθ

= √
3 sec θ
1
Z
= √ sec θ dθ
3
1
= √ ln |sec θ + tan θ| + C.
3
√ √ p
We have tan θ = 3/2y, and sec θ = 1 + tan2 θ = 1 + (3/2)y 2 = 22 2 + 3y 2 . We
p p
obtain

dy 1 2p
Z p
p = √ ln
2
2 + 3y + 3/2y + C

2 + 3y 2 3 2
√ √
1 2p 6
= √ ln 2 + 3y 2 + y + C

3 2 2

3
p √
= ln

4 + 6y 2+ 6y + C .
3

25. We use integration by parts with u = sin−1 x, du = √1−x


1
2
dx, dv = x2 dx, and v = 13 x3 .
We obtain
Z  
1 3 −1 1 3 1
Z
x2 sin−1 x dx = x sin x − x √ dx
3 3 1 − x2
1 3 −1 1 x2
Z
= x sin x − √ x dx.
3 3 1 − x2

Now let u = 1 − x2 , so du = −2x dx, x dx = − 21 du, and x2 = 1 − u. We substitute and


obtain
 
1 3 −1 1 1−u 1
Z Z
2 −1
x sin x dx = x sin x − √ − du
3 3 u 2
1 3 −1 1
Z  
= x sin x + u−1/2 − u1/2 du
3 6
1 √
 
1 3 −1 2
= x sin x + 2 u − u3/2 + C
3 6 3
√ 3/2
= 13 x3 sin−1 x + 31 1 − x2 − 91 1 − x2 +C .
Chapter 7 Review Exercises 7-293

4 4
26. Let x = 3 sec2 θ dθ. We substitute and obtain
tan θ, then dx = 3
s
 2  
4 4
Z p Z
16 + 9x2 dx = 16 + 9 tan θ sec2 θ dθ
3 3
4
Z p
= 16 + 16 tan2 θ sec2 θ dθ
3
4
Z
= (4 sec θ) sec2 θ dθ
3
16
Z
= sec3 θ dθ
3
 
16 1 1
= sec θ tan θ + ln |sec θ + tan θ| + C
3 2 2
8 8
= sec θ tan θ + ln |sec θ + tan θ| + C.
3 3
√ q 2 √
We have tan θ = 3x
4 , and sec θ = 1 + tan2 θ = 1 + 3x 4 = 41 16 + 9x2 . We obtain
  
8 1p 3x 8 1p 3x
Z p
16 + 9x2 dx = 16 + 9x2 + ln 16 + 9x2 + +C
3 4 4 3 4 4
1 p 8 p 8
= x 16 + 9x2 + ln 16 + 9x2 + 3x − ln 4 + C

2 3 3
1
√ 8

= 2
x 16 + 9x + ln 16 + 9x + 3x + C .
2
2 3

27. We use partial fractions to obtain


1 1
=
x2 + 2x 2(x + 2)
A B
= +
x x+2
1 = A(x + 2) + Bx.
When x = −2 we obtain B = −1/2, and when x = 0, we have A = 1/2. So
Z  
dx 1/2 −1/2
Z
= + dx
x2 + 2x x x+2
1 1
= ln |x| − ln |x + 2| + C
2 2

= 12 ln x+2
x
+C .

28. We use formula 105, with m = 4, n = 4, and we have


sin3 y cos5 y 4 − 1
Z Z
4 4
sin y cos y dy = − + sin4−2 y cos4 y dy
4+4 4+4
sin3 y cos5 y 3
Z
= − + sin2 y cos4 y dy.
8 8
With m = 2, n = 4 we have
sin3 y cos5 y 3 sin y cos5 y 2 − 1
Z  Z 
sin4 y cos4 y dy = − + − + sin2−2 y cos4 y dy
8 8 2+4 2+4
3 5 5
sin y cos y sin y cos y 1
Z
= − − + cos4 y dy.
8 16 16
7-294 Chapter 7 Techniques of Integration

We now can use formula 91, and obtain

sin3 y cos5 y sin y cos5 y


 
1 1 4−1
Z Z
4 4 3 2
sin y cos y dy = − − + cos y sin y + cos y dy
8 16 16 4 4
sin3 y cos5 y sin y cos5 y cos3 y sin y 3
Z
= − − + + cos2 y dy.
8 16 64 64
Finally, from formula 85, we have

sin3 y cos5 y sin y cos5 y cos3 y sin y


 
3 y sin 2y
Z
sin4 y cos4 y dy = − − + + + +C
8 16 64 64 2 4
3
y cos5 y sin y cos5 y cos3 y sin y
= − sin 8 − 16 + 64 + 3y
128 + 3 sin 2y
256 +C .

29. We use partial fractions to obtain


w−2 w−2
=
1 − w2 (1 − w)(1 + w)
A B
= +
1−w 1+w
w−2 = A(1 + w) + B(1 − w).
Let w = −1 to obtain B = −3/2, and let w = 1 to obtain A = −1/2. We now have
Z  
w−2 −1/2 −3/2
Z
dw = + dw
1 − w2 1−w 1+w
1 3
= 2 ln |1 − w| − 2 ln |1 + w| + C .

30. Let u = x2 − 4, so du = 2x dx, and x dx = 21 du. We substitute and obtain


 
x 1 1
Z Z
√ dx = √ du
x2 − 4 u 2
1
Z
= u−1/2 du
2
1 √ 
= 2 u +C
2

= x2 − 4 + C .
√ 1
31. Let u = x, so du = √ dx and √1 dx = 2 du. We substitute and obtain
2 x x

1 √
Z Z
√ cos2 x dx cos2 u (2) du

=
x
1 + cos (2u)
Z
= 2 du
2
Z
= (1 + cos (2u)) du
1
= u+ sin (2u) + C
2
= u + sin u cos u + C
√ √ √
= x + sin x cos x + C .
Chapter 7 Review Exercises 7-295

32. We use formula 96, and obtain


π
sin π2 − π x
 
π  sin 2 + π x
Z
sin x sin (πx) dx = − π + +C
2 π2 − π

2 2 2 +π

sin − π2 x
 
sin 2 x
= − + +C
3π −π
sin ( 3π
2 x) sin ( π
2 x)
= − 3π + π +C .

33. We use formula 98, and obtain

cos (1 + 2)x cos (1 − 2)x


Z
sin x cos (2x) dx = − − +C
2(1 + 2) 2(1 − 2)
1 1
= 2 cos x − 6 cos (3x) + C .

0 π
34. Let x = 2 cos θ, so dx = −2 sin θ dθ, and the limits of integration are θ = cos−1 2 = 2 and
θ = cos−1 12 = π3 . We substitute and obtain
1 π/3 2
x2 (2 cos θ)
Z Z
√ dx = q (−2 sin θ) dθ
0 4 − x2 π/2 4 − (2 cos θ)
2

π/3
cos2 θ sin θ
Z
= −8 dθ
π/2 2 sin θ
Z π/2
= 4 cos2 θ dθ
π/3

π/2
1 + cos 2θ
Z
= 4 dθ
π/3 2
Z π/2
= 2 (1 + cos 2θ) dθ
π/3
 π/2
1
= 2 θ + sin 2θ
2 π/3
    π 
π 1   π  π 1
= 2 + sin 2 − + sin 2
2 2 2 3 2 3
1√
  
1 1
= 2 π− π+ 3
2 3 4
1 1

= 3π − 2 3 .
7-296 Chapter 7 Techniques of Integration

1
35. Let u = 1 + x2 , then du = 2x dx, so x dx = du. The lower limit of integration is
2
2
√ 2
u = 1 + 0 = 1, and the upper limit becomes u = 1 + 3 = 4. We substitute to obtain

3 4 1
x dx du
Z Z
2
√ = √
0 1 + x2 1 u
4
1 −1/2
Z
= u du
1 2
h i4
= u1/2
1
= 41/2 − 11/2
= 1.

36. (a) Let u = 2x, so du = 2 dx. We substitute and obtain


cos2 2x 1
Z Z
−3
dx = cos2 u (sin u) du.
sin3 2x 2
We use Formula 105 with m = −3 and n = 2, and obtain
!
−3+1
cos2 2x 1 (sin u) cos2−1 u 2−1
Z Z
2−2 −3
dx = + cos u (sin u) du
sin3 2x 2 −3 + 2 −3 + 2
− cos u 1
Z
= − csc3 u du.
2 sin2 u 2
Next use Formula 89, and we have
cos2 2x
 
− cos u 1 1 1
Z
dx = − − csc u cot u + ln |csc u − cot u| + C
sin3 2x 2 sin2 u 2 2 2
1 1 1
= − cos 2x csc2 2x + csc 2x cot 2x − ln |csc 2x − cot 2x| + C.
2 4 4
(b) Using a CAS we obtain
cos2 2x
Z
3
dx = − 41 cos (2x)
sin2 (2x)
− 1
cos (2x) − 1
ln |csc (2x) − cot (2x)| + C .
sin3 2x 4 4

(c) We simplify the first two terms of the antiderivative in part (a),
1 1 1 1
− cos 2x csc2 2x + csc 2x cot 2x = − cos 2x csc2 2x + csc 2x (cos 2x csc 2x)
2 4 2 4
1 1
= − cos 2x csc 2x + csc2 2x cos 2x
2
2 4
1 2
= − cos (2x) csc (2x) .
4
And likewise, in part (b), we obtain
1 cos (2x) 1 − sin2 (2x)

1 cos3 (2x) 1 1
− − cos (2x) = − − cos (2x)
4 sin2 (2x) 4 4 2
sin (2x) 4
1 1 1
= − cos (2x) csc2 (2x) + cos (2x) − cos (2x)
4 4 4
1 2
= − cos (2x) csc (2x) .
4
And so adding the same logarithm terms, we see that our results above are equal.
Chapter 7 Review Exercises 7-297

1 1
37. We integrate by parts, with u = tan−1 x, du = 1+x n
2 dx, dv = x dx, and v = n+1 x
n+1
.
We obtain
  Z  
1 1 1
Z
xn tan−1 x dx = xn+1 tan−1 x − xn+1 dx
n+1 n+1 1 + x2
xn+1 1 xn+1
Z
= tan−1 x − dx.
n+1 n+1 1 + x2

1/2
38. We integrate by parts, with u = xn , du = nxn−1 dx, dv = (ax + b) dx, and v =
2 3/2
3a (ax + b) . We obtain
  Z  
2 2
Z
n 1/2 n 3/2 3/2
nxn−1 dx

x (ax + b) dx = (x ) (ax + b) − (ax + b)
3a 3a
2xn 2n
Z
3/2 1/2
xn−1 dx

= (ax + b) − (ax + b)(ax + b)
3a 3a
2xn 3/2
= (ax + b)
3a Z 
2n
Z
(ax)(ax + b)1/2 xn−1 dx + (b)(ax + b)1/2 xn−1 dx
 

3a
2xn 2n 2bn
Z Z
3/2 1/2
= (ax + b) − n
x (ax + b) dx − xn−1 (ax + b)1/2 dx.
3a 3 3a

We add 2n x (ax + b)1/2 dx to each side and obtain


R n
3

2xn
 Z
2n 2bn
Z
1/2 3/2 1/2
1+ xn (ax + b) dx = (ax + b) − xn−1 (ax + b) dx
3 3a 3a
2xn
 Z
2n + 3 2bn
Z
n 1/2 3/2 1/2
x (ax + b) dx = (ax + b) − xn−1 (ax + b) dx
3 3a 3a
3 2xn 3 2bn
Z Z
n 1/2 3/2 1/2
x (ax + b) dx = (ax + b) − xn−1 (ax + b) dx
2n + 3 3a 2n + 3 3a
2xn 2bn
Z
= (ax + b)3/2 − xn−1 (ax + b)1/2 dx
(2n + 3)a (2n + 3)a

39. We evaluate √
∞ b
e− x
Z Z
1/2
√ dx = lim x−1/2 e−x dx.
1 x b→∞ 1

Let u = x1/2 , then du = 21 x−1/2 dx. We substitute to obtain



∞ b1/2
e− x
Z Z
√ dx = lim 2e−u du
1 x b→∞ 1
b1/2
lim −2e−u 1

=
b→∞
h 1/2 i
= lim −2e−b − −2e−1
b→∞
h 1/2
i
= lim −2e−b + 2e−1
b→∞
2
= .
e
2
The improper integral converges to e .
7-298 Chapter 7 Techniques of Integration

40. We evaluate √
1 Z 1
sin x
Z  
√ dx = lim x−1/2 sin x1/2 dx.
0 x a→0+ a

Let u = x1/2
, then du = 21 x−1/2 dx. We substitute to obtain
1 √ 1
sin x
Z Z
√ dx = lim √
2 sin u du
0 x a→0+ a

= lim [−2 cos u]1√a


a→0+
 √ 
= lim+ −2 cos 1 − −2 cos a
a→0
 √ 
= lim+ −2 cos 1 + 2 cos a
a→0
= −2 cos 1 + 2(1)
= 2 − 2 cos 1.

The improper integral converges to 2 − 2 cos 1 .

41. We evaluate
1 b
x dx x dx
Z Z
√ = lim √ .
0 1 − x2 b→1− 0 1 − x2
2
Let u = 1 − x and substitute to obtain
1−b2
1
x dx − 21 du
Z Z
√ = lim √
0 1 − x2 b→1− 1 u
  Z 1−b2
1
= lim − u−1/2 du
b→1− 2 1
1−b2
1 √
 
= lim − 2 u
b→1− 2 1
 √ 1−b2
= lim − u 1
b→1−
h p  √ i
= lim − 1 − b2 − − 1
b→1−
= 1.

The improper integral converges to 1 .


42. We evaluate Z 0 Z 0
x
xe dx = lim xex dx.
−∞ a→−∞ a

We let u = x, du = dx, dv = ex dx, and v = ex and integrate by parts to obtain


Z 0  Z 0 
x x 0 x
xe dx = lim [xe ]a − e dx
−∞ a→−∞ a
 
0
= lim −aea − [ex ]a
a→−∞

lim −aea − e0 − ea

=
a→−∞
= −1.

The improper integral converges to −1 .


Chapter 7 Review Exercises 7-299

43. We evaluate
π/2 b
sin x
Z Z
dx = lim tan x dx
0 cos x b→(π/2)− 0
b
= lim [ln |sec x|]0
b→(π/2)−

= lim [ln |sec b| − ln |sec 0|]


b→(π/2)−

= lim [ln |sec b|]


b→(π/2)−
= ∞.
The improper integral diverges .
44. We evaluate √ Z b√

1 + x1/8 1 + x1/8
Z
dx = lim dx.
1 x3/4 b→∞ 1 x3/4
7
Let u = 1 + x1/8 , then du = 18 x−7/8 dx, so dx = 8(u − 1) du. We substitute and obtain
Z ∞√ Z 1+b1/8 √
1 + x1/8 u 7
3/4
dx = lim 6 8(u − 1) du
1 x b→∞ 2 (u − 1)
Z 1+b1/8

= lim 8 u (u − 1) du
b→∞ 2
Z 1+b1/8  
= lim 8 u3/2 − u1/2 du
b→∞ 2

16  25 3
1+b1/8
= lim 3u − 5u 2
b→∞ 15 2
  
16  52   3  16  5 3

1/8 1/8 2
= lim 3 1+b −5 1+b − 3(2) − 5(2)
2 2
b→∞ 15 15
 
16 3     16  5 3

1/8 2 1/8
= lim 1+b 3 1+b −5 − 3(2) − 5(2)
2 2
b→∞ 15 15
= ∞.
The improper integral diverges .
45. Since
1 + e−x 1

x x
R∞ 1
R∞ 1+e−x
for all x, and since 1 x dx diverges, by the Comparison Test, 1 x dx diverges .
46. Since
x x 1
3 ≤ = 2
(1 + x) x3 x
R∞ 1
R∞ x
for all x > 0, and since 0 x2 dx converges, by the Comparison Test, 0 (1+x)3
dx
converges .

47. We use integration by parts with u = x2 and dv = cos x dx. Then


Z Z
x2 cos x dx = x2 sin x − 2x sin x dx.

Then f (x) = x2 sin x .


7-300 Chapter 7 Techniques of Integration

Re 2
48. (a) The volume is given by the integral 1 π(ln x) dx. We use integration by parts with
2 2 ln x
u = (ln x) and dv = π dx. Then du = x dx and v = πx. We obtain
Z e ie Z e  
2
h
2 2 ln x
π(ln x) dx = (ln x) (πx) − (πx) dx
1 1 1 x
h i Z e
2 2
= (ln e) (πe) − (ln 1) (π(1)) − 2π ln x dx
1
Z e
= πe − 2π ln x dx.
1

1
We use integration by parts again with u = ln x and dv = dx. Then du = x dx and
v = x. We obtain
Z e  Z e   
2 e 1
π(ln x) dx = πe − 2π [(ln x)x]1 − x dx
1 1 x
 Z e 
= πe − 2π [(ln e)e − (ln 1)1] − dx
1
= πe − 2π(e − (e − 1))
= πe − 2π .
Re
(b) The volume is given by the integral 1 2πx ln x dx. We use integration by parts with
u = ln x and dv = 2πx dx. Then du = x1 dx and v = πx2 . We obtain
Z e Z e  
 2 e
 2
 1
2πx ln x dx = (ln x) πx 1 − πx dx
1 1 x
h  i Z e
2
 2
= (ln e) πe − (ln 1) π(1) −π x dx
1
 e
1
= πe2 − π x2
2 1
 
2 1 2 1 2
= πe − π e − (1)
2 2
1
= 2 πe
2
+ 21 π .

r
R π/2 
dy
2 R π/2 q 2 R π/2 √
49. The arc length is given by 0 1+ dx dx = 0 1 + (cos x) dx = 0 1 + cos2 x dx.

(π/2)−0 π
(a) With n = 3 we have ∆x = 3 = 6. So the Trapezoidal Rule provides the
approximation
π/2
∆x h π  π   π i
Z p
1 + cos2 x dx ≈ f (0) + 2f + 2f +f
0 2 6 3 2
 r r r 
π/6 p 2 2
π 2
π π
= 1 + cos 0 + 2 1 + cos + 2 1 + cos + 1 + cos2
2 6 3 2
≈ 1. 910 .
Chapter 7 Review Exercises 7-301

(b) With n = 4 we have ∆x = (π/2)−0


4 = π8 . So Simpson’s Rule provides the approxima-
tion
Z π/2 p     π 
2
∆x π  π  3π
1 + cos x dx ≈ f (0) + 4f + 2f + 4f +f
0 3 8 4 8 2
p r r
π/8 π π
= 1 + cos2 0 + 4 1 + cos2 + 2 1 + cos2
3 8 4
r r #
3π π
+4 1 + cos2 + 1 + cos2
8 2
≈ 1.910 .
R4
50. The distance traveled is given by the integral 1 v dt. We approximate by the Trapezoidal
Rule, with ∆t = 0.5.
4
∆t
Z
v dx ≈ [f (1) + 2f (1.5) + 2f (2) + 2f (2.5) + 2f (3) + 2f (3.5) + f (4)]
1 2
0.5
= [3 + 2(4.3) + 2(4.6) + 2(5.1) + 2(5.8) + 2(6.2) + 6.6]
2
= 15.4 meters .

51. The area is given by the improper integral


Z 1 Z 1
−2/3
x dx = lim+ x−2/3 dx
0 a→0 a
h i1
= lim 3x1/3
a→0+ a
h i
= lim 3(1) − 3(a)1/3
1/3
a→0+
= 3.

The area of the region is 3 .


52. The volume is given by the improper integral
Z 1  2 Z 1
π x−2/3 dx = lim+ πx−4/3 dx
0 a→0 a
h i1
= lim+ −3πx−1/3
a→0 a
h  i
−1/3 −1/3
= lim+ −3π(1) − −3π(a)
a→0
= ∞.

The improper integral diverges, so the volume does not exist .

53. Express the logistic model dP dP P



dt = 0.0024P (100 − P ) in the form dt = kP 1 − M where
M is the carrying capacity and k is the maximum population growth rate.
   
dP P P
= 0.0024P (100 − P ) = 0.0024 · 100P 1 − = 0.24P 1 −
dt 100 100
7-302 Chapter 7 Techniques of Integration

(a) The carrying capacity is M = 100 .


(b) The maximum population growth rate is k = 0.24 .
(c) At the inflection point, the size of the population is given by one-half the carrying
capacity. Therefore, the size of the population is 50 at the inflection point.
54. Partition [2, 10] into four subintervals [2, 3], [3, 5], [5, 8], and [8, 10].
The widths of the four intervals are
∆x1 = 3 − 2 = 1, ∆x2 = 5 − 3 = 2, ∆x3 = 8 − 5 = 3, and ∆x4 = 10 − 8 = 2.
Now apply the Trapezoidal Rule:
Z 10
1 1 1
f (x) dx ≈ [f (2) + f (3)]∆x1 + [f (3) + f (5)]∆x2 + [f (5) + f (8)]∆x3
2 2 2 2
1
+ [f (8) + f (10)]∆x4
2
1 1 1 1
= [0 + 5](1) + [5 + 2](2) + [2 + (−1)](3) + [−1 + 2](2)
2 2 2 2
5 14 3 2
= + + + = 12 .
2 2 2 2

AP
R
Review Problems
1. Use the identity cos2 θ = 12 [1 + cos (2θ)].
 
1 1 1
Z Z
cos2 x dx = [1 + cos (2x)] dx = x + sin (2x) + C = 1
2x + 1
4 sin (2x) + C
2 2 2

The answer is B.
x3 +2x2 −3x+5
R
2. Apply polynomial division to x+4 dx.
3 2
x +2x −3x+5
Apply polynomial division to x+4 .

x2 − 2x + 5
(x + 4) x3 + 2x2 − 3x + 5
−(x3 + 4x2 )
−2x2 − 3x
−(−2x2 − 8x)
5x + 5
−(5x + 20)
−15

The quotient is x2 − 2x + 5 and the remainder is −15.


x3 + 2x2 − 3x + 5 15
So, = x2 − 2x + 5 − .
x+4 x+4
Z 3
x + 2x2 − 3x + 5
Z  
2 15
Therefore, dx = x − 2x + 5 − dx
x+4 x+4
1 3
= 3x − x2 + 5x − 15 ln |x + 4| + C .

The answer is A.
3. Partition [0, 2] into five subintervals, each of equal width: [0, 0.4], [0.4, 0.8], [0.8, 1.2], [1.2, 1.6]
and [1.6, 2.0].
Chapter 7 Review Exercises 7-303

The width of each subinterval is ∆x = 0.4.


Now apply the Trapezoidal Rule:
Z 2
1 1 1
f (x) dx ≈ [f (0) + f (0.4)]∆x + [f (0.4) + f (0.8)]∆x + [f (0.8) + f (1.2)]∆x
0 2 2 2
1 1
+ [f (1.2) + f (1.6)]∆x + [f (1.6) + f (2.0)]∆x
2 2
1
= [f (0) + 2f (0.4) + 2f (0.8) + 2f (1.2) + 2f (1.6) + f (2.0)]∆x
2
1
= [3 + 2(4) + 2(4) + 2(6) + 2(8) + 10](0.4) = 57 5 .
2
The answer is C.
1
dx contains the quadratic expression x2 − 2x + 2.
R
4. The integrand of x2 −2x+2
So, we complete the square in the denominator.
1 1 1
Z Z Z
2
dx = 2
dx = dx.
x − 2x + 2 (x − 2x + 1) + (2 − 1) (x − 1)2 + 1
1 1
R R
Now use the substitution u = x − 1. Then du = dx and x2 −2x+2 dx = (x−1)2 +1 dx =
R 1 −1
u2 +1 du = tan u + C = tan−1 (x − 1) + C .

The answer is C.
5. Partition [0, 10] into four subintervals [0, 1], [1, 4], [4, 8], and [8, 10].
The widths of the four intervals are
∆x1 = 1 − 0 = 1, ∆x2 = 4 − 1 = 3, ∆x3 = 8 − 4 = 4, and ∆x4 = 10 − 8 = 2.
Now apply the Trapezoidal Rule:
Z 10
1 1 1
f (x) dx ≈ [f (0) + f (1)]∆x1 + [f (1) + f (4)]∆x2 + [f (4) + f (8)]∆x3
0 2 2 2
1
+ [f (8) + f (10)]∆x4
2
1 1 1 1
= [4 + 5](1) + [5 + 10](3) + [10 + 12](4) + [12 + 8](2)
2 2 2 2
9 45 88 40
= + + + = 91 .
2 2 2 2
The answer is D.
R
6. Evaluate x cos (πx) dx using integration by parts.
Let u = x and dv = cos (πx) dx.
Then du = dx and v = cos (πx) dx = π1 sin x.
R

  Z
1 1
Z
So x cos (πx) dx = x sin (πx) − sin (πx) dx
π π
 
x 1 x 1 1
Z
= sin (πx) − sin (πx) dx = sin (πx) − − cos (πx) + C
π π π π π
x 1
= π sin (πx) + π2 cos (πx) + C .

The answer is B.
7-304 Chapter 7 Techniques of Integration

x csc2 x dx using integration by parts.


R
7. Evaluate
Let u = x and dv = csc2 x dx.
Then du = dx and v = csc2 x dx = − cot x.
R
Z Z Z
2
Now x csc x dx = x (− cot x) − (− cot x)(dx) = −x cot x + cot x dx.

cotRx dx = cos x
R R
To evaluate R x dx,
sin use the substitution u = sin x. Then du = cos x dx,
and cot x dx = sin x dx = sin1 x (cos x dx) = u1 du = ln |u| + C = ln |sin x| + C.
cos x
R R
Z Z
So, x csc2 x dx = −x cot x + cot x dx = −x cot x + ln |sin x| + C .

The answer is D.
x
8. The function f (x) = (x2 +1)3 is continuous for x ≥ 1.
R∞ x
Rb x
By definition, 1 (x2 +1)3 dx = lim 3 dx.
b→∞ 1 (x +1)
2

∞ b
x x
Z Z
3 dx = lim dx
1 (x2 + 1) b→∞ 1 (x2 + 1)3
b
1
Z
−3
= lim x2 + 1 (2x dx)
2 b→∞ 1
"−2 #b
1 x2 + 1
= lim
2 b→∞ −2
1
" #
1 1 1
= − lim −
4 b→∞ (b2 + 1)2 4
 
1 1 1
=− 0− = 16 .
4 4
The answer is B.
9. The function f (x) = lnxx is continuous on (0, 10] but is not defined at x = 0.
R 10
So, 0 lnxx dx is an improper integral.
Z 10 Z 10
ln x ln x
dx = lim+ dx
0 x b→0 b x
To evaluate lnxx dx, use the substitution u = ln x. Then du = x1 dx, and
R

u2 (ln x)2
  Z
ln x 1
Z Z
dx = ln x dx = u du = +C = + C.
x x 2 2
Z 10 Z 10
ln x ln x
dx = lim+ dx
0 x b→0 b x
2 10
" #
(ln x)
= lim+
b→0 2
b
1 h i
= lim (ln 10) − (ln b)2 .
2
2 b→0+
 2
2 R 10
Since lim+ (ln b) = lim+ ln b = ∞, 0 lnxx dx diverges .
b→0 b→0

The answer is D.
Chapter 7 Review Exercises 7-305

2x+4
R
10. To evaluate (x−1)(x−3) dx, notice the integrand is a proper rational function in lowest
terms. Since the factors are linear and distinct, this is a Case 1 type integrand and can be
2x+4 A B
written as (x−1)(x−3) = x−1 + x−3 .

Clear the fractions by multiplying both sides of the equation by (x − 1)(x − 3).

2x + 4 = A (x − 3) + B(x − 1).

Grouping like terms, 2x + 4 = (A + B)x + (−3A − B).


This is an identity in x, so the coefficients of like powers of x must be equal.
A+B =2
−3A − B = 4

This is a system of two equations containing two variables.


After solving this system, the solution is A = −3 and B = 5.
2x+4 3 5
R 2x+4
R 3 5

So, (x−1)(x−3) = − x−1 + x−3 and (x−1)(x−3) dx = − x−1 + x−3 dx =
−3 ln |x − 1| + 5 ln |x − 3| + C .

The answer is B.
1
dx, use the substitution x = 4 sin θ, − π2 ≤ θ ≤ π
R
11. To evaluate x2 √16−x 2 2.
√ p p √
Then dx = 4 cos θ dθ and 16 − x2 = 16 − 16 sin2 θ = 4 1 − sin2 θ = 4 cos2 θ =
4 cos θ since − π2 < θ < π2 .
The integral becomes
1 1 1 1
Z Z Z
√ dx = 2 (4 cos θ dθ) = csc2 θ dθ = − cot θ + C.
2
x 16 − x 2 (4 sin θ) 4 cos θ 16 16
r
√ 4
√ 2
Since 16 − x2 = 4 cos θ, sec θ = √16−x , tan θ = sec 2θ−1 = √ 4 −1 =
2 16−x 2

x 16−x2
± √16−x 2
, and cot θ = ± x .

2
Since − π2 ≤ θ ≤ π2 , cot θ = 16−x
x .

1 1 1 16 − x2 √
Z
1
Therefore, √ dx = − cot θ + C = − + C = − 16x 16 − x2 + C .
x2 16 − x2 16 16 x
The answer is C.
12. The logistic model dP P dP P
 
dt = 0.1P 1 − 4000 is in the form dt = kP 1 − M where M is the
carrying capacity and k is the maximum population growth rate.

(d) The carrying capacity is M = 4000 .


(e) The maximum population growth rate is k = 0.1 .
(f) At the inflection point, the size of the population is given by one-half the carrying
capacity. Therefore, the size of the population is 2000 at the inflection point.
7-306 Chapter 7 Techniques of Integration

Practice AP
R
Exam, Part I
d d
1. For f (x) = e4x + sin (2x), f ′ (x) = e4x = 4e4x + 2 cos (2x).
   
dx (4x) + cos (2x) dx (2x)

So, f ′ (0) = 4e0 + 2 cos (0) = 4(1) + 2(1) = 6 .


The answer is D.
x2 −1 (x+1)(x−1)
2. Note that lim = lim = lim (x + 1) = 2.
x→1 x−1 x→1 x−1 x→1
  h i
(x2 −1)f (x) 2
Therefore, lim x−1 = lim xx−1−1
· lim f (x) = (2)(3) = 6 .
x→1 x→1 x→1

The answer is C.
3. Since lim f (x) 6= f (3), function f is discontinuous at x = 3. Statement (B) is TRUE.
x→3

Since lim f (x) = lim f (x), lim f (x) exists. Statement (C) is TRUE.
x→4− x→4+ x→4

Since lim f (x) = f (5), function f is continuous at x = 5. Statement (D) is TRUE.


x→5

Since lim− f (x) = 1 and 1 < f (2) < 2, lim− f (x) 6= f (2). Statement (A) is FALSE.
x→2 x→2

The answer is A.
4. For f (x) = x3 − 15x2 − 1800x + 2000, f ′ (x) = 3x2 − 30x − 1800 = 3(x − 30)(x + 20).
Note that f ′ (x) < 0 on the interval −20 < x < 30, and defined at x = −20 and x = 30.
Therefore, f is decreasing on the interval −20 ≤ x ≤ 30.
Use f ′′ (x) = 6x − 30 = 6(x − 5) to examine concavity.
Note that f ′′ (x) < 0 for x < 5. Therefore, f is concave down for x < 5.

So, f is both decreasing and concave down for −20 ≤ x < 5 .

The answer is B.
5. For y = e2x cos (3x),
   
d d 2x
f ′ (x) = e2x = e2x [−3 sin (3x)] + cos (3x) 2e2x

cos (3x) + cos (3x) e
dx dx
= e2x [−3 sin (3x) + 2 cos (3x)] = e2x [2 cos (3x) − 3 sin (3x)] .

The answer is B.
sin (7x)
6. Note that lim 2x = 7
lim sin7x
2 x→0
(7x)
.
x→0− −

Let t = 7x. As x → 0− , t → 0− .
sin (7x) 7 sin (7x) 7 sin t
Then lim− f (x) = lim− 2x = 2 lim 7x = 2 lim t = 27 (1) = 72 .
x→0 x→0 x→0− t→0−

Since k + 2 ln(x + ex+1 ) is continuous for all x ≥ 0,

lim+ f (x) = lim+ k + 2 ln(x + ex+1 ) = f (0) = k + 2 ln(0 + e1 ) = k + 2.


 
x→0 x→0

For f to be continuous at x = 0, choose k so that lim f (x) = lim f (x).


x→0− x→0+
Chapter 7 Review Exercises 7-307

7 3
So, 2 = k + 2. Therefore, k = 2 and lim f (x) = 27 .
x→0

Note that for k = 32 , f (0) = 7


2 = lim f (x).
x→0

3
Therefore, f is continuous at x = 0 for k = 2 .

The answer is C.
p h i4/5 p
7. Note that lim 5 (x − 3)4 = lim (x − 3) = 0 and f (3) = 5 (3 − 3)4 = 0.
x→3 x→3
p
Since lim 5 (x − 3)4 exists and is equal to f (3), function f is continuous at x = 3.
x→3

Note that f ′ (x) = 45 (x − 3)−1/5 = 5



5
4
x−3
. So, f ′ (3) is not defined.

Therefore, function f is not differentiable at x = 3 .


The answer is B.
sec x tan x + tan2 x dx
R R 
8. (sec x + tan x) tan x dx =

Use the identity tan2 x = sec2 x − 1.


Z Z
sec x tan x + sec2 x − 1 dx = sec x + tan x − x + C

(sec x + tan x) tan x dx =

The answer is C.

d 1/2
9. y ′ = 4x + 6etan x
dx
1 −1/2 d
= 4x + 6etan x 4x + 6etan x

2 dx
1 tan x −1/2
4 + 6etan x · sec2 x
 
= 4x + 6e
2
4 + 6etan x sec2 x
= √
2 4x + 6etan x
2 + 3etan x sec2 x
= √
4x + 6etan x

The answer is D.
10. For a function f that is continuous on the closed interval a ≤ x ≤ b and differentiable on
the open interval a < x < b, the Mean Value Theorem guarantees that f ′ (c) = f (b)−f
b−a
(a)

for at least one c between a and b. Applying the Mean Value Theorem to function f that
is continuous on 1 ≤ x ≤ 5 and differentiable on 1 < x < 5 with f (1) = 10 and f (5) = 50,
f ′ (c) = f (5)−f
5−1
(1)
= 50−10
4 = 10 for at least one c between 1 and 5.

The answer is B.
1 i 1
11. The ith term of the sum is ei/20 · = f (ui )∆x for f (x) = ex , ui = 20
20 , and ∆x = 20 .
 i−1 i 
The quantity ui is the right endpoint of the ith subinterval 20 , 20 for i = 1, 2, . . . , n.
7-308 Chapter 7 Techniques of Integration

Since the last term of the Riemann sum is e2 and the right endpoint of the nth subinterval
n n n
is 20 , therefore f 20 = en/20 = e2 , so 20 = 2, so n = 40. So, the interval [0, 2] is
partitioned into 40 subintervals of equal length
2 1
∆x = = .
40 20
40
1 h 1/20 i X R2
Therefore, e + e2/20 + · · · + e2 = f (ui )∆x = 0 ex dx.
20 i=1

The answer is B.
12. The region in the first quadrant bounded by the graph of y = 1 + e−2x and the line x = 3
is pictured below.

R3
Since y = 1 + e−2x is nonnegative on [0, 3], 1 + e−2x dx is the area under the graph of

0
y = 1 + e−2x from x = 0 to x = 3.
3    3    
1 −2x 1 −6 1 0
Z
−2x 7
− 21 e−6

A= 1+e dx = x + − e = 3− e − 0− e = 2
0 2 0 2 2

The answer is C.

13. Use the substitution
√ u = x. Then x = u2 and dx = 2u du. The lower√limit of integration
becomes u = 1 = 1, and the upper limit of integration becomes u = 4 = 2. Therefore,
Z 4 Z 2
1+x 1 + u2 R2 3
√ dx = 2u du = 2 1 u+u
1+u du .
1 1+ x 1 1+u

The answer is D.
d
14. Using the Chain Rule, dx f (f (x)) = f ′ (f (x)) · f ′ (x).
√ p
For f (x) = 25 − x2 , f (−3) = 25 − (−3)2 = 4 and

d
f (f (−3)) = f ′ (f (−3)) · f ′ (−3) = f ′ (4) · f ′ (−3).
dx
  
Since f ′ (x) = √ −x , d f (f (−3)) = f ′ (4) ·f ′ (−3) = √ −4 √ −(−3) −4 3
 
25−x2 dx 25−42 2
= 3 4 =
25−(−3)

−1 .

The answer is B.
Chapter 7 Review Exercises 7-309
Z 1 Z 0 Z 1 Z 0 Z 1
15. f (x) dx = f (x) dx + f (x) dx = e2x dx + e−4x dx
−1 −1 0 −1 0
 0  1
1 2x 1 1  1
+ − e−4x 1 − e−2 − e−4 − 1 = 3
− 21 e−2 − 41 e−4

= e = 4
2 −1 4 0 2 4

The answer is D.
f (x0 +h)−f (x0 )
16. Use the limit definition of the derivative of function f at x = x0 , f ′ (x0 ) = lim h .
h→0

Apply the definition to f (x) = sin (2x) at x0 = π4 .

sin 2( π4 + h) − sin (2 · π4 )
 
′ π sin ( π 2)
+ 2h) − sin ( π
 
f = lim = lim 2
h .
4 h→0 h h→0

The answer is B.
17. At x = 3, the graph of f crosses the x-axis. So, f (3) = 0.
At x = 3, the graph of f is decreasing. So, f ′ (3) < 0.
At x = 3, the graph of f is concave up. So, f ′′ (3) > 0.

Therefore, f ′ (3) < f (3) < f ′′ (3) .

The answer is C.
1 3 5
4x3 + 3x2 + 5 4x3 + 3x2 + 5 4+ +
 
x3 x x3 4+0+0 4
18. lim = lim · 1 = lim 1 1 = = 5 .
x→∞ 5x3 + x2 − x x→∞ 5x3 + x2 − x x→∞ 5 + − 5+0−0
x3 x x2

The answer is B.
19. Given v(3) = −3, the velocity of the particle at time t is given by
Z t Z t
t
v(t) = v(3) + a(x) dx = −3 + 3 dx = −3 + [3x]3 = −3 + (3t − 9) = 3t − 12.
3 3

The particle is at rest when v(t) = 3t − 12 = 0. So, the particle is at rest at t = 4.


Given x(2) = 5, the position of the particle at time t is given by
Z t Z t
x(t) = x(2) + v(x) dx = 5 + (3x − 12) dx.
2 2

4  4
3 2
Z
So, x(4) = 5 + (3x − 12) dx = 5 + x − 12x
2 2 2
= 5 + [(24 − 48) − (6 − 24)] = −1 .

The answer is C.
20. For the rectangle with two sides along the x-axis and y-axis and inscribed using the function
f (x) = e−0.2x , the width of the rectangle is x and the height is f (x) = e−0.2x . So, the area
as a function of x is given by A(x) = x · e−0.2x . To find x so that the area of the rectangle
is a maximum, set A′ (x) = 0 and solve for x.
Applying the Product Rule,
   
d −0.2x d
A′ (x) = x · + e−0.2x · x = x · −0.2e−0.2x + e−0.2x · (1) = e−0.2x (1 − 0.2x).

e
dx dx

Since e−0.2x > 0 for all x, A′ (x) = 0 when 1 − 0.2x = 0. So, A′ (x) = 0 when x = 5.
7-310 Chapter 7 Techniques of Integration

Note that A′ (x) > 0 for x < 5 and A′ (x) < 0 for x > 5. So, the area is a maximum for
x=5.
The answer is D.
x+2
21. For y = f (x) = 2x−6 ,find f ′ (x) by applying the Quotient Rule.
d  d 
(2x − 6) dx (x + 2) − (x + 2) dx (2x − 6)
 
d x+2
f ′ (x) = = 2
dx 2x − 6 (2x − 6)
(2x − 6)(1) − (x + 2)(2) −10
= 2 = 2.
(2x − 6) (2x − 6)
The slope of the tangent line to the graph of y at the point (4, 3) is mtan = f ′ (4) =
−10
(8−6)2
= − 52 .
−1
The slope of the normal line to the graph of y at the point (4, 3) is mnorm = f ′ (4) = 25 .

The equation of the normal line to the graph of y at the point (4, 3) is y − 3 = 52 (x − 4).

Multiplying both sides by 5, 5y − 15 = 2(x − 4) or 5y − 2x = 7 .

The answer is A.
2 2 2
d
22. For f (x) = ekx , f ′ (x) = ekx dx kx2 = 2kxekx


 
d kx2
(2kx) = (2kx)2 ekx + 2kekx = 2kekx 2kx2 + 1 .
2 2 2 2
d
and f ′′ (x) = 2kx dx + ekx dx
 
e

Since f has a point of inflection at x = ±2, f ′′ (±2) = 2ke4k (8k + 1) = 0.

Since k is nonzero, and e4k > 0 for all k, 8k + 1 = 0. Therefore, k = − 18 .

The answer is B.
23. Apply the Product Rule.
d 2 t  d t
 Z   Z  Z t  
d 2
t ln (u + 3) du = t2 ln (u + 3) du + ln (u + 3) du t
dt 2 dt 2 2 dt
Z t 
= t2 ln (t + 3) + ln (u + 3) du (2t)
2

2
Rt
= t ln (t + 3) + 2t 2
ln (u + 3) du

The answer is D.
dy dy
24. Rewrite the differential equation dx = y cos (2x) as y = cos (2x) dx.

Integrate both sides to obtain dy


R R
y = cos (2x) dx

or ln |y| = 0.5 sin (2x) + C for some constant C.


Applying the condition that y = 3 when x = 0 yields ln 3 = 0.5 sin (0) + C.
Thus, C = ln 3 and ln |y| = 0.5 sin (2x) + ln(3).
Solving for y yields y = ±e0.5 sin (2x)+ln(3) = ±e0.5 sin (2x) eln(3) = ±3e0.5 sin (2x) .

The condition y = 3 when x = 0 requires the positive sign. Therefore, y = 3e0.5 sin (2x) .

The answer is B.
Chapter 7 Review Exercises 7-311
Z 3 Z 3 Z 3 Z 3
25. [3f ′ (x) + g(x)g ′ (x) − 6] dx = 3 f ′ (x) dx + g(x)g ′ (x) dx − 6 dx
1 1 1 1
31n 2
o3
3
= 3[f (x)]1 + [g(x)] − 6[x]1
2 1
1n 2 2
o
= 3[f (3) − f (1)] + [g(3)] − [g(1)] − 6(3 − 1)
2
1 2
5 − 12 + 6(3 − 1)

= 3[4 − 2] +
2
= 6 + 12 − 12 = 6

The answer is A.
26. If x represents the number of words memorized at time t, then expression dx
dt represents the
rate at which a list of M words are memorized. We are given that the rate is proportion
to the product of the number of words memorized, M , and the number of words that have
dx
not been memorized, M − x. Therefore, the differential equation dt = kx(M − x) models
this situation.
The answer is C.
27. For f (x) = (x − 3)2 (x − 5),
   
d d
f ′ (x) = (x − 3)2 (x − 5) + (x − 5) (x − 3)2
dx dx
2
= (x − 3) + 2(x − 5)(x − 3)
= (x − 3)(3x − 13).
13
Note that f ′ (x) = 0 for x = 3 and x = 3 .

For x < 3, f ′ (x) > 0. So, the graph of f is increasing for x < 3.
13 13
For 3 < x < 3 , f ′ (x) < 0. So, the graph of f is decreasing for 3 < x < 3 .
13 13
For x > 3 , f ′ (x) > 0. So, the graph of f is increasing for x > 3 .
13
At x = 3 , the graph of f changes from decreasing to increasing.

13
Therefore, the function has a relative minimum at x = 3 .

The answer is D.
dy
28. Use implicit differentiation to find dx :

d 2 d
x + 3y 2 =

(12)
dx dx
dy
2x + 6y =0
dx
dy −2x x
= =− .
dx 6y 3y

dy (3)
So dx = − 3(1) = −1.
(x,y)=(3,1)
 
dy dy
d d 3y(1)−x 3 dx
3y dx (x)−x dx (3y) 3y−3x dx
2
   
d y d dy d x
Then dx2 = dx dx = dx − 3y = − (3y)2
=− 9y 2 =− 9y 2 =
dy
3x dx −3y
9y 2 .
7-312 Chapter 7 Techniques of Integration


d2 y 3(3)(−1)−3(1) −9−3
So dx2 (x,y)=(3,1) = 9(1)2
= 9 = − 34 .

The answer is B.
3x−sin (3x)
29. Since lim [3x − sin (3x)] = 0 and lim [1 − cos (2x)] = 0, lim is of indeterminate
x→0 x→0 x→0 1−cos (2x)
form 0
0 . Apply L’Hôpital’s Rule to evaluate lim 3x−sin (3x) .
x→0 1−cos (2x)

d
3x − sin (3x) dx [3x − sin (3x)] 3 − 3 cos (3x) 3 1 − cos (3x)
lim = lim d
= lim = − lim .
x→0 1 − cos (2x) −2 sin (2x) 2 x→∞ sin (2x)
dx [1 − cos (2x)]
x→0 x→0

1−cos (3x)
Since lim [1 − cos (3x)] = 0 and lim sin (2x) = 0, lim sin (2x) is of indeterminate form 00 .
x→0 x→0 x→0

1−cos (3x)
Apply L’Hôpital’s Rule to evaluate − 23 lim sin (2x) .
x→0

d
dx [1 − cos (3x)]
 
3 1 − cos (3x) 3 3 3 sin (3x) 3 3·0
− lim = − lim d
= − lim =− = 0
2 x→0 sin (2x) 2 x→0 dx [sin (2x)]
2 x→0 2 cos (2x) 2 2·1

The answer is B.
30. For graph I, the graph of f is concave down for 1 ≤ x ≤ 4. So, f ′′ (x) < 0 for 1 ≤ x ≤ 4.
The graph on the right could represent f ′′ (x) for function f on the left.
For graph II, the graph of f is concave up for 1 ≤ x ≤ 4. So, f ′′ (x) > 0 for 1 ≤ x ≤ 4.
The graph on the right could represent f ′′ (x) for function f on the left.
For graph III, the graph of f is concave down for 1 ≤ x < 52 and concave up for 52 < x ≤ 4.
So, f ′′ (x) < 0 for 1 ≤ x ≤ 25 and f ′′ (x) > 0 for 52 < x ≤ 4. The graph on the right could
represent f ′′ (x) for function f on the left.
The answer is D.

Practice AP
R
Exam, Part II
πt

31. Use a graphing calculator to examine the graph of g(t) = (t − 2) sin 2 on the interval
0 ≤ t ≤ 2.

Rx πt

Since g(t) < 0 for all 0 ≤ t ≤ 2, f (x) = 0
(t − 2) sin 2 dt < 0 for all 0 ≤ x ≤ 2.
Using the Fundamental Theorem,
Z x  
d πt  πx 
f ′ (x) = (t − 2) sin dt = (x − 2) sin < 0 for all 0 < x < 2.
dx 0 2 2
Finally, since the graph of g is increasing on the interval k < x < 2 for some k, f ′′ (x) =
g ′ (t) > 0 on the interval k < x < 2. Therefore, only statements I and II are true.
The answer is B.
Chapter 7 Review Exercises 7-313


32. For function f (x) = 2x2 + 3
x4 − 8 defined for x ≥ 0, find x so that f (x) = 10.
Using a graphing calculator, f (x) = 10 for x = 2.
32
Using a graphing calculator, f ′ (2) = 3 .

32 1 1 3
Since g is the inverse of f and f ′ (2) = 6= 0, then g ′ (10) = = = ≈ 0.0937 .
3 f ′ (2) ( 323 ) 32

The answer is D.
33. For f (x) = ex − xe − e, find the point where the graph of f crosses the x-axis.
Using a graphing calculator to solvef (x) = 0, x ≈ 3.4711.
Using a graphing calculator, f ′ (3.4711) ≈ 9.1063.
The slope of the normal line to the graph of f at x ≈ 3.4711 is
1 1
mnorm ≈ − f ′ (3.4711) ≈ − 9.1063 ≈ −0.110 .

The answer is B.
34. Functions f and g have perpendicular tangents for all x such that f ′ (x) · g ′ (x) = −1.
   
1
Since f ′ (x) = x+1 and g ′ (x) = − 2x1√x , find all x such that x+1
1
· − 2x1√x = −1.

Equivalently, find all x such that (x + 1)(2x x) = 1.

Using a graphing calculator, x ≈ 0.484 .


The answer is A.
35. Partition [0, 2] into five subintervals, each of equal width: [0, 0.4], [0.4, 0.8], [0.8, 1.2], [1.2, 1.6],
and [1.6, 2.0].
The width of each subinterval is ∆x = 0.4.
Now apply the Trapezoidal Rule:
Z 2
1 1
f (x) dx ≈ [f (0) + f (0.4)]∆x + [f (0.4) + f (0.8)]∆x
0 2 2
1 1 1
+ [f (0.8) + f (1.2)]∆x + [f (1.2) + f (1.6)]∆x + [f (1.6) + f (2.0)]∆x
2 2 2
1
= [f (0) + 2f (0.4) + 2f (0.8) + 2f (1.2) + 2f (1.6) + f (2.0)](0.4)
2
1
= [10 + 2(12) + 2(13) + 2(16) + 2(19) + 20](0.4)
2
= 30 .

The answer is B.
36. Given v(0) = 2, the velocity of the particle at time t is given by
Z t Z t
√  √ 
v(t) = v(0) + 3 − x dx = 2 + 3 − x dx.
0 0
R4 √ 20 26
The velocity at time t = 4 is v(4) = 2 + 0 (3 − x) dx = 2 + 3 = 3 .

The answer is C.
37. Using a graphing calculator, the region in the first quadrant enclosed by the graph of
y = tan x, y = 3 − x, and the y-axis is pictured below.
7-314 Chapter 7 Techniques of Integration

Using a graphing calculator, the graphs of y = tan x and y = 3 − x intersect at x ≈ 1.089.

The area of the region in the first quadrant enclosed by the graph of y = tan x, y = 3 − x,
R 1.089
and the y-axis is given by A ≈ 0 [(3 − x) − tan x] dx.
R 1.089
Using a graphing calculator, A ≈ 0 [(3 − x) − tan x] dx ≈ 1.905 .
The answer is A.
38. Let w(t) denote the weight of the animal after t days.
Rt Rt √
Then w(t) = w(0) + 0 r(x) dx = w(0) + 0 0.16e x dx.
R5 √
The number of pounds gained from t = 0 to t = 5 days is w(5) − w(0) = 0 0.16e x dx.
R5 √
Using a graphing calculator, w(5) − w(0) = 0 0.16e x dx ≈ 4.0 pounds .

The answer is B.
x ln x−x
39. The function g(x) = x+1 is defined for all x > 0.
x ln x−x ′
For g(x) = x+1 , find g (x) using the Quotient Rule.

d
 d 
′ (x + 1) dx (x ln x − x) − (x ln x − x) dx (x + 1)
g (x) =
(x + 1)2
1

(x + 1) x · x + ln x − 1 − (x ln x − x)(1) ln x + x
= 2
= .
(x + 1) (x + 1)2
ln x+x
To find a critical number for g(x), set g ′ (x) = (x+1)2 = 0.

Using a graphing calculator to solve ln x + x = 0, x ≈ 0.567 .


The answer is C.
40. The distance between a point (x, y) on the graph y = x2 + x and the point (3, 4) is given by
p p
d = (x − 3)2 + (y − 4)2 = (x − 3)2 + (x2 + x − 4)2 .

Since d > 0, finding x so that d is a minimum is equivalent to finding x so that d2 is a


minimum.
Let f (x) = d2 = (x − 3)2 + (x2 + x − 4)2 .
Then f ′ (x) = 2(x − 3) + 2(x2 + x − 4)(2x + 1).
To find the critical numbers of f , solve f ′ (x) = 2(x − 3) + 2(x2 + x − 4)(2x + 1) = 0.
Chapter 7 Review Exercises 7-315

Using a graphing calculator to solve 2(x − 3) + 2(x2 + x − 4)(2x + 1) = 0, x ≈ −2.137, −1,


and 1.637.
Note that f (−2.137) ≈ 28.854, f (−1) ≈ 32, and f (1.64) ≈ 1.958.

Therefore, the minimum distance is d ≈ 1.958 ≈ 1.399 .
The answer is B.
41. The perimeter of a regular hexagon is given by P = 6s, where s is the length of each side.
When the perimeter is P = 120 cm, the length of each side is s = 16 P = 16 (120 cm) = 20 cm.
dP cm
The perimeter is increasing at a constant rate of 30 cm/min. So, dt = 30 min .
The rate of increase in the area of the hexagon is
√ ! √  √ 

   
dA d 3 3 2 3 3 d 2 3 3 ds ds
= s = s = 2s =3 3 s .
dt dt 2 2 dt 2 dt dt

dP d
From P = 6s, dt = dt (6s) = 6 ds
dt .
ds 1 dP 1 cm cm

So, dt = 6 dt = 6 30 min = 5 min .

dA
√ √ cm2
= 3 3 s ds cm
 
Therefore, dt dt = 3 3(20 cm) 5 min ≈ 519.615 min .

The answer is B.
Rx √ 
42. For g(x) = 0 3f (t) + t3 + 1 dt,
Z 2 h p i Z 2 Z 2 p
g(2) = 3f (t) + t3 + 1 dt = 3 f (t) dt + t3 + 1 dt.
0 0 0
R2√
Using a graphing calculator, 0 t3 + 1 dt ≈ 3.241.
R2 R2 R2√
Since 0 f (t) dt = 10, g(2) = 3 0 f (t) dt + 0 t3 + 1 dt ≈ 3(10) + 3.241 = 33.241.
By Part 1 of the Fundamental Theorem of Calculus,
Z xh
d p i p
g ′ (x) = 3f (t) + t3 + 1 dt = 3f (x) + x3 + 1,
dx 0

and g ′ (2) = 3f (2) + 23 + 1 = 3(4) + 3 = 15.

Therefore, g(2) + g ′ (2) ≈ 33.241 + 15 = 48.241 .


The answer is C.
R2√
43. The average density across the length of the rod is f¯ = 1
2−0 0 10 − x3 dx.
R2√
Using a graphing calculator, 10 − x3 dx ≈ 5.584.0
R2√
Therefore, the average density is f¯ = 2−0
1
0 10 − x3 dx ≈ 21 (5.584) = 2.792 .

The answer is C.
7-316 Chapter 7 Techniques of Integration

R 10
44. The distance traveled by the object from t = 0 to t = 10 is given by 0 |v(t)| dt.

Since v(t) = e0.2t − 3, we find that v(t) ≤ 0 if 0 ≤ t ≤ 5 ln 3 and v(t) ≥ 0 if 5 ln 3 ≤ t ≤ 10.


Therefore,
Z 10 Z 5 ln 3 Z 10 Z 5 ln 3 Z 10
0.2t
e0.2t − 3 dt.
 
|v(t)| dt = [−v(t)] dt + v(t) dt = − e − 3 dt +
0 0 5 ln 3 0 5 ln 3
R 5 ln 3 R 10
e0.2t − 3 dt ≈ −6.4792 and e0.2t − 3 dt ≈
 
Using a graphing calculator, 0 5 ln 3
8.4245.
Therefore, the distance is
Z 5 ln 3 Z 10
0.2t
e0.2t − 3 dt ≈ −(−6.4792) + 8.4245 ≈ 14.904 .
 
− e − 3 dt +
0 5 ln 3

The answer is D.
45. Consider cross sections perpendicular to the x-axis of thickness dx.
The base of each triangle is [sin x − (1 − sin x)] = 2 sin x − 1.
The height of each triangle is 12 (2 sin x − 1).
2
The area of each triangle is 21 (base)(height) = 1 1
− 1) (2 sin x − 1) = 41 (2 sin x − 1) .
 
2 2 (2 sin x
2
The volume of each cross section is dV = (Area) dx = 41 (2 sin x − 1) dx.
To find the range for x, find the intersection of y = sin x and y = 1 − sin x.
sin x = 1 − sin x
2 sin x = 1
1
sin x =
2
π 5π
So, x = 6 and 6 .
R 5π/6 1 2 1
R 5π/6 2
Therefore, the volume of the solid is V = π/6 4 (2 sin x − 1) dx = 4 π/6 (2 sin x − 1) dx.
R 5π/6
Using a graphing calculator, V = 1
4 π/6
(2 sin x − 1)2 dx ≈ 0.272 .

The answer is B.

AP
R
Practice Exam
Part 2, Free Response
1. (a) Partition [0, 6] into four subintervals: [0, 1], [1, 3], [3, 5], and [5, 6].
∆x1 = 1 − 0 = 1, ∆x2 = 3 − 1 = 2, ∆x3 = 5 − 3 = 2, and ∆x4 = 6 − 5 = 1.
Chapter 7 Review Exercises 7-317

Now apply the Trapezoidal Rule:


1 6

1 1 1 1
Z
E(x) dx ≈ [E(0) + E(1)]∆x1 + [E(1) + E(3)]∆x2 + [E(3)
6 0 6 2 2 2

1
+ E(5)]∆x3 + [E(5) + E(6)]∆x4
2
1
= {[E(0) + E(1)](1) + [E(1) + E(3)](2) + [E(3) + E(5)](2)
12
+ [E(5) + E(6)](1)}
1
= [E(0) + 3E(1) + 4E(3) + 3E(5) + E(6)]
12
1
= [1.0 + 3(1.3) + 4(2.1) + 3(4.1) + 5.6]
12
= 2.6 thousand feet .
1
R6
(b) 6 0
E(x) dx = 2.6 thousand feet is
the average elevation of the trail along the entire length of the trail.
(c) For a function f that is continuous on the closed interval a ≤ x ≤ b and differentiable
on the open interval a < x < b, the Mean Value Theorem guarantees that f ′ (c) =
f (b)−f (a)
b−a for at least one c between a and b.
Since function E is differentiable on the interval 0 < x < 6, it is also continuous on the
interval. For the interval 3 < x < 5, the conditions of the Mean Value Theorem are
satisfied. Since E(3) = 2.1 and E(5) = 4.1, there E ′ (c) = E(5)−E(3)
5−3 = 4.1−2.1
2
ft
= 1 mile
for at least one c between 3 and 5. Therefore, there must be at least one distance x
for which the elevation increases at a rate of 1000 feet per mile.
(d) Since E ′ (x) > 0 on the interval 0 ≤ x ≤ 6, E is increasing on the interval 0 ≤ x ≤ 6.
Therefore, the Left Riemann sum is the lower sum and underapproximates the value
R6
of 61 0 E(x) dx.
2. (a) The region is pictured below.

To find the points of intersection of the curve y = x24x


+7 with the horizontal line y = 12 ,
4x 1 2

set x2 +7 = 2 and solve for x. Multiply both sides by 2 x + 7 to obtain

8x = x2 + 7
x2 − 8x + 7 = 0
(x − 1)(x − 7) = 0
The curve and the line intersect at x = 1 and 7.
7-318 Chapter 7 Techniques of Integration

R7 4x 1

The area bounded by the two curves is 1 x2 +7 − 2dx.
R7 4x

Using a graphing calculator, the area is 1 x2 +7 − 12 dx ≈ 0.892 .
(b) Use the method of rings to find the volume of the resulting solid when the region in
(a) is rotated about the line y = −2.
Z h 7 i
2 2
Volume = π (outer radius) − (inner radius) dx
1
Z 7 " 2  2 #
4x 1
=π 2+ 2 − 2+ dx ≈ 14.551
1 x +7 2

(c) Each section perpendicular to the x-axis is a rectangle with a height equal to five
1
times the length of the base. The base is x2 +7 − 2 . The height is 5 x24x
4x 1
+7 − 2 .
 2
The area of the rectangle is 5 x24x 1
+7 − 2 .
Multiply the area by the thickness of the section dx.
R7 2
The total volume is given by V = 5 1 x24x +7 − 1
2 dx ≈ 0.862 .

3. (a) The acceleration of the object is given by a(t) = v ′ (t).


Use the product rule to find v ′ (t).
      
d πt πt d −0.1t
a(t) = v ′ (t) = 10 e−0.1t cos + cos e
dt 4 4 dt
       
πt π πt  −0.1t
= 10 e−0.1t − sin

· + cos e (−0.1)
4 4 4
    
π πt πt
= −10e−0.1t sin + 0.1 cos
4 4 4
= −e−0.1t 5π πt πt
  
2 sin 4 + cos 4

(b) The position of the object at time t = 5 is given by


Z 5
x(5) = x(0) + v(t) dt
0
R5 πt
10e−0.1t cos

= 4+ 0 4 dt .

(c) Since both the velocity v(t) and acceleration a(t) are negative for 2 < t < 3.839, the
speed of the object is increasing on the interval 2 < t < 3.839 .

Since v(t) < 0 for 2 < t < 6, the object is moving to the left on the interval 2 < t < 6 .
(d) The distance traveled by the object on the interval 0 ≤ t ≤ 6 is
Z 6 R 6 −0.1t πt

d= |v(t)| dt = 10 0
e cos 4
dt .
0

R2
4. (a) g(2) = 0 f (t) dt is the area of the triangular region between f and the x-axis from
R2
x = 0 to x = 2. So, g(2) = 0 f (t) dt = 21 (2)(4) = 4 .
Chapter 7 Review Exercises 7-319

d
Rx
Using Part 1 of the Fundamental Theorem, g ′ (x) = dx 0 f (t) dt = f (x). So g ′ (2) =
f (2) = 4 .
Since g ′ (x) = f (x), g ′′ (x) = f ′ (x). The graph of f has a cusp at x = 2. The slope of
the line tangent to the graph of f at x = 2 is not defined. So, g ′′ (2) does not exist .
(b) Since g ′′ (x) = f ′ (x) > 0 for x < 2 and g ′′ (x) = f ′ (x) < 0 for x > 2, function g has a
point of inflection at x = 2 .
Rx d
Rx
(c) For h(x) = −2 f (t) dt, h′ (x) = dx −2 f (t) dt = f (x) = 0 when x = 5. So, h has a
critical number at x = 5. Since h′ (x) = f (x) > 0 for x < 5 and h′ (x) = f (x) < 0 for
x > 5, function h has a relative maximum at x = 5 .
R6
(d) Using properties of integrals, break up −2 f (t) dt.
Z 6 Z 0 Z 2 Z 4 Z 6
f (t) dt = f (t) dt + f (t) dt + f (t) dt + f (t) dt.
−2 −2 0 2 4
Z 2 Z 4 Z 6
h(6) = h(0) + f (t) dt + f (t) dt + f (t) dt.
0 2 4
R2 R6 26
From the graph, 0 f (t) dt = 4 and 4 f (t) dt = 0. Also, h(6) = 3 and h(0) = −2.
26
R4 R4 20
So, 3 = −2 + 4 + 2
f (t) dt + 0 and 2
f (t) dt = 3 .
R 4x √ R0√
5. (a) Since f (x) = 0 100 − t3 dt, g(0) = 0 100 − t3 dt = 0.
Using Part 1 of the Fundamental Theorem,
Z 4x p  
′ d 3
p
3
d p
g (x) = 100 − t dt = 100 − (4x) (4x) = 4 100 − 64x3 .
dx 0 dx
So, the slope of the line tangent to the graph of g at x = 0 is g ′ (0) = 40.
Using the point-slope form of the line, y − 0 = 40(x − 0).
The equation of the line tangent to the graph of g at x = 0 is y = 40x .

(b) Since g ′ (x) = 4 100 − 64x3 > 0 on the interval 0 ≤ x ≤ 1, function g is increasing
on the interval 0 ≤ x ≤ 1. So, g is one-to-one on the interval 0 ≤ x ≤ 1 and therefore
has an inverse on the interval 0 ≤ x ≤ 1.
(c) Since g(1) = A and h is the inverse of g, h′ (A) = g′1(1) = √ 1 = 1
24 .
4 100−64(13 )
′′ d d 1/2
g ′ (x) = 4 100 − 64x3

(d) g (x) =
dx
  dx  
1 −1/2 d
=4 100 − 64x3 (100 − 64x3 )
2 dx
−384x2
=√
100 − 64x3
Since g ′′ (x) < 0 on the interval 0 ≤ x ≤ 1, the graph of g is concave down on
the interval 0 ≤ x ≤ 1. Therefore, a trapezoidal sum used to estimate g(1) =
R4√
0 100 − t3 dt is an underestimation .
2
dy
6. (a) The right side of the differential equation dx = 2(y−1)

x
gives the slope of the line
tangent to the graph of y(x) at (x, y). For example, the slope of the line tangent to
2
the graph of y(x) at (1, 2) is 2(2−1)

1
= 2. Continuing in this manner, the slope is
calculated for each point indicated on the graph.
7-320 Chapter 7 Techniques of Integration

(x, y) Slope (x, y) Slope


(2, 2) √2
(1, 2) 2 2

(1, 1) 0 (2, 1) 0
(1, 0) 2 (2, 0) √2
2

The slope at each point is plotted on the graph below to produce the slope field.

2(2−1)2
(b) The line tangent to the graph of f at the point (1, 2) has slope m = √
1
= 2.
Use the point-slope form of a line to find the equation of the line.
y − 2 = 2(x − 1)
y − 2 = 2x − 2 or y = 2x

2
dy
(c) Rewrite the differential equation dx = 2(y−1)

x
dy
as (y−1) 2 =
√2 dx.
x
R dy R 2 1 √
Integrate both sides to obtain (y−1)2 = √
x
dx or − y−1 = 4 x + C for some
constant C.
Applying the condition that y = 2 when x = 1 yields −1 = 4 + C.
1 √
Thus, C = −5 and − y−1 = 4 x − 5.
1√
Solving for y yields y = 5−4 x
+1.

You might also like